You are on page 1of 311

Physics H7ABC

Welcome to the archival Web page for U.C. Berkeley's Physics H7ABC, Honors Physics for Scientists
and Engineers, Fall 1998, Spring 1999, and Fall 1999.
Instructor:
(Prof.) Mark Strovink. I have a research web page, a standardized U.C. Berkeley web page, and a
statement of research interests.

Physics H7A (Mechanics and Vibrations)


Problem set solutions initially composed by E.A. ("Ted") Baltz
Graduate Student Instructors: David Bacon and Elizabeth Wu

Physics H7B (Electromagnetism and Thermal Physics):


Most problem set solutions composed by Peter Pebler
Graduate Student Instructor: Robin Blume-Kohout

Physics H7C (Physical Optics and Modern Physics):


Problem set solutions composed by Graduate Student Instructor: Derek Kimball
Most documents linked on this page are in PDF format. They are typeset except where indicated. The
documents are intended to be displayed by Adobe Acrobat [Reader], version 4 or later (version 3 may
also work). (You may optimize Acrobat's rendering of equations by unchecking "Use Greek Text
Below:" on File-Preferences-General.)
You may right-click to download a single .pdf file (19.5 MB) that includes every displayable image on
this Web page.
Physics H7A (Mechanics and Vibrations)
Texts: Kleppner/Kolenkow, An Introduction to Mechanics; French, Vibrations and Waves.
General Information including schedules and rooms.
Course Outline including all reading assignments.
Questionnaire that was filled out by the students.
Typos in Kleppner & Kolenkow

Problem Set 1 Solution Set 1


Problem Set 2 Solution Set 2
Problem Set 3 Solution Set 3
Problem Set 4 Solution Set 4
Problem Set 5 Solution Set 5
Problem Set 6 Solution Set 6
Problem Set 7 Solution Set 7
Problem Set 8 Solution Set 8
Problem Set 9 Solution Set 9
Problem Set 10 Solution Set 10
Problem Set 11 Solution Set 11

http://d0lbln.lbl.gov/h7abc-web.htm (1 of 3) [1/5/2000 9:50:55 AM]


Physics H7ABC

Practice Exam 1 Solution to Practice Exam 1


Exam 1 Solution to Exam 1
Exam 2 Solution to Exam 2
Final Exam Solution to Final Exam
You may right-click to download a single .tar.gz archive (1.0 MB) that includes the source files required
to build every H7A file linked above.
Physics H7B (Electromagnetism and Thermal Physics)
Text: Purcell, Electricity and Magnetism.
General Information including schedules and rooms.
Course Outline including all reading assignments.
Special Relativity Notes* also used in H7C.

Problem Set 1 Solution Set 1


Problem Set 2 Solution Set 2
Problem Set 3 Solution Set 3
Problem Set 4 Solution Set 4
Problem Set 5 Solution Set 5
Problem Set 6 Solution Set 6
Problem Set 7 Solution Set 7
Problem Set 8 Solution Set 8
Problem Set 9 Solution Set 9
Problem Set 10 Solution Set 10
Problem Set 11 Solution Set 11
Problem Set 12 Solution Set 12
Problem Set 13 Solution Set 13

Midterm Solution to Midterm


Final Exam Solution to Final Exam*
* handwritten
You may right-click to download a single .tar.gz archive (20 MB) containing the source files required to
build every typeset H7B file linked above.
Physics H7C (Physical Optics and Modern Physics)
Texts: Fowles, Introduction to Modern Optics; Rohlf, Modern Physics from alpha to Z0.
General Information including schedules and rooms.
Course Outline including all reading assignments.
Note on H7C texts, required and supplementary.
Errata for Fowles, Introduction to Modern Optics, second edition.
Special Relativity Notes* also used in H7B.

http://d0lbln.lbl.gov/h7abc-web.htm (2 of 3) [1/5/2000 9:50:55 AM]


Physics H7ABC

Problem Set 1 Solution Set 1


Problem Set 2 Solution Set 2
Problem Set 3 Solution Set 3
Problem Set 4 Solution Set 4
Problem Set 5 Solution Set 5
Problem Set 6 Solution Set 6
Problem Set 7 Solution Set 7
Problem Set 8 Solution Set 8
Problem Set 9 Solution Set 9
Problem Set 10 Solution Set 10
Problem Set 11 Solution Set 11
Problem Set 12 Solution Set 12

Midterm 1 Solution to Midterm 1


Midterm 2 Solution to Midterm 2
Final Exam Solution to Final Exam
* handwritten
You may right-click to download a single .tar.gz archive (0.5 MB) containing the source files required to
build every typeset H7C file linked above.

http://d0lbln.lbl.gov/h7abc-web.htm (3 of 3) [1/5/2000 9:50:55 AM]


University of California, Berkeley
Physics H7A, Fall 1998 (Strovink)

General Information (1 Sep 98)

Instructors: Prof. Mark Strovink, 437 LeConte; (LBL) 486-7087; (home, before 10) 486-8079; (UC) 642-9685.
Email: strovink@lbl.gov . Web: http://d0lbln.lbl.gov/ . Office hours: M 3:15-4:15, Tu 10-11,
Th 10-11. Mr. David Bacon, 214 LeConte; (UC) 642-5430; (home, before 1 AM) 666-9867. Email:
dabacon@wco.com . Office hours: W 12-2. Ms. Elizabeth Wu, 214 LeConte; (UC) 642-5430. Email:
ecmwu@euler.berkeley.edu . Office hours in 262 LeConte: M 10-11. You may also get help in the 7A
Course Center, 262 LeConte.

Lectures: Tu-Th 11:10-12:30, 2 LeConte. Lecture attendance is essential, since not all of the course content can be
found in either of the course texts.

Labs: In the second week, in 270 LeConte, please enroll in one of only 3 special H7A lab sections [(A) #134, M 4-
6; (B) #241, Th 4-6; or (C) #312, F 8-10]. Section 134 is taught by Ms. Wu and Sections 241 and 312 are taught by
Mr. Bacon. If you can make more than one of these lab (and section, see below) slots, please attempt to enroll in the
earliest of these lab slots. Depending on crowding, you may be asked to move to a later lab. During "off" weeks not
requiring lab apparatus, your lab section will still meet, in (#134) 336 LeConte, (#241) 395 LeConte, or (#312) 335
LeConte.

Discussion Sections: Beginning in the second week, please enroll in the only one of the 1 hr H7A discussion
sections corresponding to your H7A lab section: (A) #134, W 1-2, 343 LeConte; (B) #241, Tu 4-5, 395 LeConte;
(C) #312, W 8-9, 385 LeConte. You are especially encouraged to attend discussion section regularly. There you
will learn techniques of problem solving, with particular application to the assigned exercises.

Texts (both required): Kleppner/Kolenkow, An Introduction to Mechanics, (McGraw-Hill, 1973). A.P. French,
Vibrations and Waves, Paper Edition (Norton, 1971).

Problem Sets: Twelve problem sets are assigned and graded, with solutions provided as part of the Syllabus. They
are due on Wednesday at 5 PM on weeks (including Thanksgiving) in which there is no exam, beginning in week 2.
Deposit problem sets in the box labeled "H7A" outside 201 LeConte. You are encouraged to attempt all the
problems. Students who do not do them find it almost impossible to learn the material and to succeed on the
examinations. Work independently. Credit for collective effort, which is easy to identify, will be divided among the
collectivists. Late papers will not be graded. Your lowest problem set score will be dropped, in lieu of due date
extensions for any reason.

Syllabus: H7A has two syllabus cards. The first card is mandatory; it will be collected at the time of the first in-
class examination. This card pays for the experiment descriptions and instructions that you will receive from your
GSI at the beginning of each laboratory. The second syllabus card is optional. It entitles you to pick up printed
solutions to the problem set assignments from Copy Central. These solutions will also be made available on the
Web. Both cards will be available for purchase at Copy Central beginning in the second week of class.

Exams: There will be two 80-minute in-class examinations and one 3-hour final examination. Before confirming
your enrollment in this class, please check that its final Exam Group 9 does not conflict with the Exam Group for any
other class in which you are enrolled. Please verify that you will be available for both in-class examinations (Th 24
Sep and Th 5 Nov, 11:10-12:30), and for the final examination, F 11 Dec, 5-8 PM. Except for unforeseeable
emergencies, it will not be possible for these in-class or final exams to be rescheduled. Passing H7A requires
passing the final exam.

Grading: 35% in-class examinations; 20% problem sets; 40% final exam; 5% lab. Grading is not "curved" -- it
does not depend on your performance relative to that of your H7A classmates. Rather it is based on comparing your
work to that of a generation of earlier lower division Berkeley physics students, with due allowance for educational
trends.
COURSE OUTLINE

Week Week Lecture Topic Problem Due Lab


No. of... chapter (K&K = Kleppner/Kolenkow, Intro. to Mechanics ) Set No. 5 PM on...
(French = Vibrations & Waves)
(Feynman = Lectures on Physics Vol. II)
K&K
1 24-Aug 1.1-8 Introduction; vectors, kinematics. none
(do experiment in lab=expt)
(have discussion in lab=disc)
2 31-Aug 1.9-2.3 Motion in polar coordinates; expt
Newton's laws; units. 1 Wed 2 Sep

3 7-Sep 2.4-2.5 LABOR DAY HOLIDAY disc


Application of Newton's laws; forces. 2 9-Sep

4 14-Sep 3.1-Note3.1 Momentum; center of mass. expt


3 16-Sep

5 21-Sep 4.1-6 Work; kinetic energy. disc


24-Sep EXAM 1 (covers PS 1-3)

6 28-Sep 4.7-14 Potential energy; nonconservative forces; disc


energy conservation; power; collisions. 4 30-Sep

7 5-Oct 6.1-7 Angular momentum; fixed axis rotation; disc


rotation with translation. 5 7-Oct

8 12-Oct 7.1-5 Vector angular momentum; conservation thereof. expt


8.1-8.4 Noninertial systems; fictitious forces. 6 14-Oct

9 19-Oct 8.5-Note8.2 Rotating coordinate systems; equivalence principle. expt


9.1-9.5 Central forces. 7 21-Oct

10 26-Oct 9.6-9.7 Planetary motion. disc


French 8 28-Oct
10-15,43-45,77-89 Damped forced harmonic oscillator.
11 2-Nov 62-70,92-96 Transient response. disc
5-Nov EXAM 2 (covers PS 1-8)

12 9-Nov 19-27,119-129 Coupled oscillator; beats. disc


161-170,189-196 Fourier expansion in normal modes. 9 11-Nov

13 16-Nov 201-209,213-215, Waves: travelling, sinusoidal, modulated; expt


228-234 phase and group velocity. 10 18-Nov

14 23-Nov 45-62,209-212 Longitudinal waves; sound. disc


170-178,274-279 Boundary reflections of waves; Doppler effect. 11 25-Nov
26,27-Nov THANKSGIVING HOLIDAY
15 30-Nov Feynman disc
II.2-1,2,3,4,5;II.40-1,2,3 Fluid statics and nonviscous dynamics. 12 Fri 4 Dec
LAST LECTURE (review)
16 7-Dec
11-Dec 5-8 PM FINAL EXAM (Group 9) (covers PS 1-12)

Physics H7A Fall 1998 (Strovink)


University of California, Berkeley

Physics H7A Fall 1998 (Strovink)

STUDENT QUESTIONNAIRE

Please complete this questionnaire if you are considering enrolling in


Physics H7A. Among other purposes, it will be used to make up the initial
class roll.

Are you qualified for this course? Your interest in Honors Physics is
already a good indication that you are. It is unlikely that the issue of
your eligibility will come into question. If it does, you will be
interviewed promptly by the instructor.

Last name__________________________First_______________________Initial_________

Registration No.___________________ Year (Freshman, etc.)_____________

SAT scores:

SAT I Quantitative______ SAT II Math 1______

SAT I Verbal______ SAT II Math 1C______

SAT II Math 2______

SAT II Physics______ SAT II Math 2C______

AP Calculus Exam: Grade______ AB BC (circle one)

AP Physics Exam: Grade______ B C (circle one)

Year of last math course_______ Where taken?________________________________

Course title and no._______________________________________________________

Course text_______________________________________ Grade received_______

Year of last physics course____ Where taken?________________________________

Course title and no._______________________________________________________

Course text_______________________________________ Grade received_______

List majors that you are considering at Berkeley_______________________________

___________________________________________________________________________

Why are you (might you be) interested in Honors in place of standard lower-

division physics at Berkeley?__________________________________________________

_______________________________________________________________________________

_______________________________________________________________________________

_______________________________________________________________________________

_______________________________________________________________________________

_______________________________________________________________________________
University of California, Berkeley
Physics H7A Fall 1998 (Strovink)
TYPOS IN KLEPPNER & KOLENKOW

This is a list of typos in Kleppner & Kolenkow


that have been brought to the attention of H7A
F98 sta by students in the course. Thanks to
them for pointing these out.
Page 205, Example 5.2:
In the equation below the dierential of f is,
replace the second dy with dx.
Page 276, Note 6.2:
In the equation following E = K + U , the rst
term 12 l2 2 should be multiplied by m.
Page 392, Equation 9.22:
The rst instance of x should be x2 instead.
University of California, Berkeley
Physics H7A Fall 1998 (Strovink)
PROBLEM SET 1

1. Specify the properties of two vectors a and b 5. K&K problem 1.6 Prove the law of sines us-
such that ing the cross product. It should only take a cou-
ple of lines. (Hint: Consider the area of a triangle
(a.) a + b = c and |a| + |b| = |c|.
formed by A, B, C, where A + B + C = 0.)
(b.) a + b = a b.
6. K&K problem 1.11 Let A be an arbitrary
(c.) a + b = c and |a|2 + |b|2 = |c|2 . vector and let n be a unit vector in some xed di-
(d.) |a + b| = |a b|. rection. Show that A = (A n) n+(n A)n.
(e.) |a + b| = |a| = |b|. 7. If the air velocity (velocity with respect to
the air) of an airplane is u, and the wind ve-
2. K&K problem 1.2 Find the cosine of locity with respect to the ground is w, then the
the angle between A = (3i + j + k) and B = ground velocity v of the airplane is
(2i 3j k).
3. The relation between Cartesian (x, y, z) and v = u + w.
spherical polar (r, , ) coordinates is:
An airplane les a straight course (with respect
x = r sin cos to the ground) from P to Q and then back to P ,
with air speed |u| which is always equal to a con-
y = r sin sin stant U0 , regardless of the wind. Find the time
z = r cos . required for one round trip, under the following
conditions:
Consider two points on a sphere of radius R:
(R, 1 , 1 ) and (R, 2 , 2 ). Use the dot product (a.) No wind.
to nd the cosine of the angle 12 between the (b.) Wind of speed W0 blowing from P to Q.
two vectors which point to the origin from these
(c.) Wind of speed W0 blowing perpendicular to
two points. You should obtain:
a line connecting P and Q.
cos 12 = cos 1 cos 2 + sin 1 sin 2 cos (1 2 ). (d.) Wind of speed W0 blowing at an angle
from a line connecting P and Q.
4. New York has North Latitude (= 90 ) (e.) Show that the round trip ying time is al-
= 41 and West Longitude (= 360 ) = 74 . ways least for part (a.).
Sydney has South Latitude (= 90 ) = 34 (f.) What happens to the answers to (b.)-(d.)
and East Longitude (= ) = 151 . Take the when W0 > U0 ? Interpret this limiting
earth to be a sphere of radius 6370 km; use the condition physically.
result of Problem 3.
8. A particle moves along the curve y = Ax2
(a.) Find the length in km of an imaginary such that its x position is given by x = Bt (t =
straight tunnel bored between New York time).
and Sydney.
(a.) Express the vector position r(t) of the par-
(b.) Find the distance of the shortest possible ticle in the form
low-altitude ight between the two cities.
(Hint: The great circle distance along the r(t) = if (t) + jg(t) [or xf (t) + yg(t)]
surface of a sphere is just R12 , where 12 is
the angle between the two points, measured where i and j [or x and y] are unit vectors,
in radians.) and f (t) and g(t) are functions of t.
(b.) Find the (vector) velocity v(t) as a function
of t.
(c.) Find the (vector) acceleration a(t) as a func-
tion of t.
(d.) Find the (scalar) speed |v(t)| as a function
of t.
(e.) Find the (vector) average velocity v(t0 )
between t = 0 and t = t0 where t0 is any
positive time.
9. Below are some measurements taken on
a stroboscopic photograph of a particle under-
going accelerated motion. The distance s is
measured from a xed point, but the zero of
time is set to coincide with the rst strobe ash:

time (sec) distance (m)


0 0.56
1 0.84
2 1.17
3 1.57
4 2.00
5 2.53
6 3.08
7 3.71
8 4.39

Plot a straight-line graph, based on these data,


to show that they are tted by the equation

s = a(t t0 )2 /2,

where a and t0 are constants, and extrapolate


the line to evaluate t0 .
1

University of California, Berkeley


Physics H7A Fall 1998 (Strovink)
SOLUTION TO PROBLEM SET 1
Composed and formatted by E.A. Baltz and M. Strovink; proofed by D. Bacon

1. You may remember the law of cosines from (b.) This part is simple. Just subtract the vec-
trigonometry. It will be useful for several parts tor a from both sides to see that b = b. The
of this problem, so we will state it here. If the only way that this can happen is if b = 0, the
lengths of the sides of a triangle are a, b, and c, zero vector.
and the angle opposite the side c is , then (c.) This part can also be done by the law of
c = a + b 2ab cos
2 2 2 cosines. Like part (a.), we have the following
two equations
(a.) When two vectors add up to a third vector, |c|2 = |a|2 + |b|2 + 2|a||b| cos
the three vectors form a triangle. If the angle
This is just the law of cosines again, where
between a and b is , then the angle opposite
is the angle between |a| and b|. The problem
the side formed by c is 180 .
states that
|c|2 = |a|2 + |b|2
Comparing this with the equation above, we nd
that cos = 0. This happens at = 90 . This
means that the vectors must be perpendicular to
each other.
(d.) Yet again, we can use the law of cosines. If
the angle between a and b is , then the angle
between a and b is 180 . The lengths of
The law of cosines then tells us that the sum and dierence are
|a + b|2 = |a|2 + |b|2 + 2|a||b| cos
|c|2 = |a|2 + |b|2 2|a||b| cos (180 )
|a b|2 = |a|2 + |b|2 2|a||b| cos
From trigonometry, remember that For these to be equal, we need cos = 0, which
happens when = 90 . Again, this means that
cos (180 ) = cos the vectors are perpendicular.
which gives (e.) Guess what? Yup, law of cosines. We know
that |a| = |b| = |a + b|. Adding a to b is going
|c|2 = |a|2 + |b|2 + 2|a||b| cos to look like two vectors stuck together to form
two sides of a triangle. If the angle between the
We know that |a| + |b| = |c|. Squaring this vectors is , the law of cosines gives
equation, we get
|a + b|2 = 2|a|2 + 2|a|2 cos
|c|2 = |a|2 + |b|2 + 2|a||b| where we have used the fact that a and b have
the same length. We also know that |a+b| = |a|.
If we compare this with the equation above, we Using this we get
can see that cos has to be equal to one. This
only happens when = 0 . What this means is |a|2 = 2|a|2 + 2|a|2 cos
that the two vectors are parallel to each other, Dividing by |a|2 , we nd a condition on the angle
and they point in the same direction. If the an-
1
gle between them were 180 , then they would be cos = = 120
parallel but point in opposite directions. 2
2

2. K&K problem 1.2 as we expected in the rst place. Now we calcu-


We can use the dot product, also known as late the dot product. Let x1 be the vector to the
the inner product, of two vectors here. Remem- rst point and x2 be the vector to the second
ber that point. We nd that
A B = |A||B| cos
x1 x2 = R2 (sin 1 sin 2 cos 1 cos 2
where is the angle between the vectors. We can
+ sin 1 sin 2 sin 1 sin 2 + cos 1 cos 2 )
use the formula for computing the dot product
from the vector components
This can be simplied if we remember the for-
A B = Ax Bx + Ay By + Az Bz mula for the cosine of a sum of two angles.

The vectors are given as follows: A = 3i + j + k cos( ) = cos cos sin sin
and B = 2i 3j k. Multiplying, we nd that
A B = 10. We need the lengths of A and B. Using this formula, we get the result
Remember that |A|2 = A A. This tells us that
|A|2 = 11 and |B|2 = 14. Dividing, we nd that
x1 x2 = R2 (sin 1 sin 2 cos(1 2 )
10 + cos 1 cos 2 )
cos = = 0.805
11 14
To get the angle we just divide by the lengths
3. Using the formulas on the problem set, we of each vector, which are both R. This gives the
can convert the points on the surface of the nal result.
sphere to Cartesian coordinates.
cos 12 = cos 1 cos 2
x1 = R sin 1 cos 1
+ sin 1 sin 2 cos(1 2 )
y1 = R sin 1 sin 1
z1 = R cos 1
4. This problem is an application of the results
x2 = R sin 2 cos 2
of problem 3.
y2 = R sin 2 sin 2
z2 = R cos 2 (a.) A straight tunnel between Sydney and New
York can be represented by the dierence of the
As in problem 2, we need to know the length of vectors pointing to their locations. To say it an-
these vectors in order to calculate the angle be- other way, the distance between the ends of two
tween them from the dot product. It is fairly vectors is the length of the dierence of the vec-
obvious that the lengths of the vectors are just tors. Adjusting for the fact that latitude and
R, because that is the radius of the sphere; we longitude are not quite the same as the coordi-
will show this explicitly. nates and , we nd the polar coordinates of
the cities.
|(R, , )|2 = R2 (sin2 cos2
+ sin2 sin2 + cos2 ) XNY = (6370 km, 49 , 286 )
XSydney = (6370 km, 124 , 151 )
Using the fact that sin2 + cos2 = 1, we get
  Converting to Cartesian coordinates (x, y, z) us-
|(R, , )|2 = R2 sin2 + cos2
ing the formulas from problem 3 we get
We can just repeat the previous step for now
and get XNY = (1325 km, 4621 km, 4179 km)
|(R, , )| = R XSydney = (4619 km, 2560 km, 3562 km)
3

The distance between New York and Sydney Let A be an arbitrary vector and let n be a
through the earth is just |XNY XSydney |. The unit vector in some xed direction. Show that
result of the calculation is
A = (A n) n + (n A) n
Distance = 12, 117 km
Form a triangle from the three vectors in this
equation. Let B = (A n) n and let C =
(b.) Using the result from problem 3 to calcu- (n A) n. Let the angle between A and
late the angle between Sydney and New York, n be . What this formula does is to break up
we nd that cos 12 = 0.809, thus 12 = 144.0 . the vector A into a piece parallel to n and a piece
To calculate the distance along the earths sur- perpendicular to n. B gives the parallel piece.
face we need to express this angle in radians. Its length is just |B| = |A| cos . The length of
The conversion formula is the perpendicular piece must then be |A| sin .

(radians) = (degrees)
180
Thus 12 = 2.513 radians. Multiplying this by
the radius of the earth, we get the great circle
distance between New York and Sydney:

Distance = 16, 010 km

5. K&K problem 1.6


Examining the vector C, we see that inside the
This question asks you to prove the law of
parentheses is a vector whose length is |A| sin
sines using the cross product. Let A, B, and C
and is perpendicular to n. This vector is then
be the lengths of the vectors making the three
crossed into n. Since it is perpendicular to n,
sides of the triangle. Let a, b and c be the an-
the length of the nal vector is |A| sin , which
gle opposite each of those sides. The law of sines
is what we want. Now we are just concerned
states that
with the direction. The rst cross product is
sin a sin b sin c perpendicular to the plane containing n and A.
= = The second cross product is perpendicular to the
A B C
rst, thus it is coplanar with n and A. It is
Remember that the length of the cross product also perpendicular to n. Thus it represents the
of two vectors is equal to the area of the paral- component of A that is perpendicular to n. Be
lelogram dened by them. Remember also that careful about the sign here.
the the length of the cross product is equal to
A useful vector identity that you will be see-
the product of the lengths times the sine of the
ing again is the so-called BAC-CAB rule. It is
angle between them: |A B| = |A||B| sin . We
an identity for the triple cross product.
have three vectors to play with in this problem,
and using the cross product we can compute the A(B C) = B(A C) C(A B)
area of the triangle from any two of them. We
nd that Its fairly obvious why this is called the BAC-
CAB rule. Using this rule, we see that
Area = AB sin c = BC sin a = AC sin b
n(n A) = n(n A) A(n n)
We just divide the whole thing by ABC and we
recover the law of sines. This immediately gives

6. K&K problem 1.11 A = (A n) n + (n A) n


4

Of course we havent derived the BAC-CAB rule (d.) Wind of speed W0 blowing at an angle
here. Its a mess. to the direction of travel. The plane again needs
7. The idea in all of the parts of this problem to cancel the component of the wind blowing in
is that the plane must oppose any perpendicular the perpendicular direction. The perpendicular
wind speed to maintain its straight path. If the component of the wind speed is W0 = W0 sin .
As in part (c.) the airspeed in the parallel direc-
wind is blowing with a speed v perpendicular to
tion can be computed
the path, the planes airspeed must be v per-
pendicular to the path. The airspeed is u, the 
wind speed relative to the ground is w, and the
U02 = U
2
+ U2 U = U02 W02 sin2
ground speed is v = u + w. |u| = U0 . Let the
total distance traveled be D.
In this case, the wind has a component along
(a.) No wind, w = 0 so u = v. T = D/U0 .
the direction of travel. This parallel component
(b.) Wind of speed W0 blowing parallel to the is W0 = W0 cos . On one leg of the trip, this
path. When the wind is going with the plane, adds to the ground velocity. On the other leg, it
v = W0 + U0 , when it opposes the plane, the subtracts. This gives us the following formula:
ground speed is v = U0 W0 . The time for the
rst leg is T1 = D/2(U0 + W0 ). The time for 
D 1
the second leg is T2 = D/2(U0 W0 ). The total T = 
2 U02 W02 sin2 + W0 cos
time is the sum
  
D 1 1 1
T = + +
2 U0 + W 0 U0 W 0 U02 W02 sin2 W0 cos
This can be simplied, and we get the nal an-
swer, which agrees with part (a.) when W0 = 0. This can be simplied considerably:
DU0
T = 
U02 W02 D U02 W02 sin2
T =
(c.) Wind of speed W0 blowing perpendicular to U02 W02
the path. This part is a little harder. The plane
will not be pointed straight along the path be- If you look at this carefully, you will realize that
cause it has to oppose the wind trying to blow it it reduces to the correct answer for parts (a.),
o course. The airspeed of the plane in the per- (b.), and (c.) with the proper values for W0 and
pendicular direction will be W0 , and we know . If = 90 , the wind blows perpendicular to
what the total airspeed is, so we can calculate the path and we get the result from part (c.). If
the airspeed along the path. = 0 , the wind blows parallel to the direction
 of travel and we recover the result from part (b.).
U02 = U2
+ U2 U = U02 W02
(e.) (f.) This part requires some calculus. We
The wind has no component along the path of need to do a minimization of a function. What
motion, so the airspeed in the parallel direction this part asks is to study the travel time as a
is the same as the ground speed in the parallel function of wind speed for an arbitrary angle .
direction. The ground speed is furthermore the We need to consider the result from part (d.) as
same on both legs of the trip. The nal answer a function of the wind speed:
again agrees with part (a.) when W0 = 0

D D U02 W02 sin2
T = T (W0 ) =
U02 W02 U02 W02
5

For now we are going to ignore the fact that it time, y = AB 2 t2 . In vector form, the position is
also depends on U0 and . Remember that func- then
tions have maxima and minima at places where r(t) = xBt + yAB 2 t2
the derivative vanishes, so we need to take the
derivative of T with respect to W0 : (b.) The vector velocity is obtained from the vec-
 tor position by dierentiating with respect to t
 2W U 2 W 2 sin2
d 0 0 0
d
T (W0 ) = D v(t) = r(t) = xB + y2AB 2 t
dW0 (U0 W0 )2
2 2
dt

W0 sin2

U02 W02 sin2 (U02 W02 ) (c.) The vector acceleration is obtained from
the vector velocity by again dierentiating with
The derivative is clearly zero when W0 = 0. In respect to t
this case the travel time T = D/U0 as in part
d
(a.). There is another case we have to worry a(t) = v(t) = y2AB 2
about though. We divide out what we can to dt
get an equation for another value where the
derivative vanishes (d.) The scalar speed is just the length
of the
velocity vector. Remember that |A| = A A.
U02 sin2 W02 sin2 = 2U02 2W02 sin2  
|v(t)| = v(t)v(t) = B 2 + 4A2 B 4 t2
This gives us the other point where the deriva-
tive is zero
(e.) The vector average velocity is the integral
2 sin2 of the velocity vector over a time interval, di-
W02 = U02 vided by the time interval. In general, the (time)
sin2
average of a quantity A is given by
Notice that this point always occurs when the
 t2
wind speed is greater than the air speed. No 1
progress can be made against the wind if this A
= A(t)dt
(t2 t1 ) t1
is the case, so the trip cannot occur. The -
nal possibility to consider is the case where the Applying this formula, we see the integral that
wind speed is the same as the air speed. Look- needs to be evaluated:
ing at the formula, the time taken is innite. 
The only possibility is that the minimum is at 1 t0
v(t0 )
= v(t)dt
W0 = 0. The nal piece of this problem is to t0 0

observe what happens to the time taken when 1 t0
W0 > U0 . For one thing, it becomes negative. = (xB + y2AB 2 t)dt
t0 0
In some circumstances it can even become imag-
inary. There is really no interpretation of this We could also use the fact that the integral of the
other than ask a stupid question, get a stupid velocity is the position to get a simpler looking
answer. The answer doesnt make sense be- formula for the average velocity
cause the question didnt make sense. The trip
cannot occur when W0 > U0 , so it is meaningless 1
v(t0 )
= (r(t0 ) r(0))
to ask how long it would take. t0
8. A particle moves along the curve y = Ax2
Evaluating this integral, we get an answer that
and its x position is given by x = Bt.
is not surprising
(a.) We can just plug the x equation into the y
equation to get the y position as a function of v(t0 )
= xB + yAB 2 t0
6

This is just (r(t0 ) r(0))/t0 ! The average veloc-


ity is just the distance traveled divided by the
time it took.
9. The idea behind this problem is to make a
graph of position vs. time data and show that
they t the equation s = a(t t0 )2 /2. In addi-
tion you are supposed to nd t0 . The way to
do this is to plot the square root of the distance
vs.
time,
 which will give a straight line graph:
s = (a/2)(t t0 ). The slope of this graph is
approximately 0.168, so we can use that to ex-
trapolate back to zero. We nd that the graph
reaches zero at about t = 4.45, so this means
that t0 = 4.45 to make the distance traveled
equal zero at t = 4.45.
University of California, Berkeley
Physics H7A Fall 1998 (Strovink)
PROBLEM SET 2

1. Calculate the following centripetal accel- 7. In the gure, two blocks are in contact on a
erations as fractions or multiples of g (= 9.8 table. The coecient of sliding friction between
m/sec2 ): the blocks and the table is . A force F is applied
to M as shown and the blocks begin to slide.
(a.) The acceleration toward the earths axis of a
Find the contact force between the two blocks.
person standing on the earth at 45 latitude.
Is it the same if the force is applied to m instead
(b.) The acceleration of the moon toward the of M ? Does the contact force depend on ?
earth.
(c.) The acceleration of an electron moving
around a proton at a speed of 2106 m/sec
in a circular orbit of radius 0.5 Angstroms
(1 Angstrom = 1010 m).
8. K&K problem 2.5 The Atwoods machine
(d.) The acceleration of a point on the rim of a
shown in....
bicycle wheel of 26 in diameter, traveling at
a constant speed of 25 mph. 9. In the gure, the pulley axle has no friction
and the pulley and cords have no mass. As the
2. K&K problem 1.17 A particle moves in a
system is studied for various values of the ex-
plane....
ternal applied upward force F, it is found that
3. K&K problem 1.20 A particle moves out- there are regimes (ranges of |F|) for which
ward along.... (i) Neither block moves.
4. At t=0 an object is released from rest at the (ii) Only the small block with mass m (m < M )
top of a tall building. At the time t0 a second moves.
object is dropped from the same point.
(iii) Both blocks move.
(a.) Ignoring air resistance, show that the time
at which the objects have a vertical separa- (a.) Find the values of |F| which dene the tran-
tion l is given by sitions between regimes (i) and (ii), and
between regimes (ii) and (iii).
l t0 (b.) Find the accelerations of the masses within
t= + .
gt0 2 the regimes (b) and (c), expressed as func-
tions of |F|, M , and m.
How do you interpret this result for l <
gt20 /2?
(b.) The above formula implies that there is an
optimum value of t0 such that the separa-
tion l reaches some specied value l0 at the
earliest possible value of t. Calculate this
optimum value of t0 and interpret the result.
5. K&K problem 1.21 A boy stands at the
peak....
10. K&K problem 2.6 In a concrete mixer....
6. K&K problem 2.1 A 5-kg mass moves under
the....
1

University of California, Berkeley


Physics H7A Fall 1998 (Strovink)
SOLUTION TO PROBLEM SET 2
Composed and formatted by E.A. Baltz and M. Strovink; proofed by D. Bacon

1. This problem is a simple application of the (c.) This time you are given the speed, not
formula for centripetal acceleration. An object the angular velocity, so we use the rst formula.
moving at speed v in a circular path of radius r The acceleration is 8.0 1022 m/sec2 . This is
has a centripetal acceleration directed inward: 8.2 1021 g!
v2
acentripetal = r (d.) A point on the rim of a wheel is moving
r at the same speed that the wheel is rolling. The
If we know instead the angular velocity of the radius is 13 inches, and the velocity is 25 mph,
object, we can use it instead. Remember that an so we need to convert these units. The ocial
object with angular velocity going on a circu- denition of the inch is 1 inch = 2.54 cm. This
lar path of radius r has speed v = r. Plugging gives 1610 meters per mile. The acceleration is
into the above formula, we get then 379 m/sec2 , which is 38.6g.

acentripetal = 2 rr 2. K&K problem 1.17

In plane polar coordinates the velocity and


acceleration are given by

v = rr + r
a = (r r2 )r + (r + 2r)

We know all of these things, so we can get a


formula for the magnitudes of v and a, given
that both the radial and angular velocities are
constant (r = = 0).
(a.) Standing at 45 latitude, the distance to the
axis of the earths rotation is just rearth / 2. This

is the radius to be used. The speed is just the an-
gular velocity times this radius. The angular ve- v = r2 + r2 2

locity is easy to guess, its 2/day. Using rearth =
a = r2 4 + 4r2 2
6370 km, the distance to the axis is 4504 km.
Plugging in these numbers, the acceleration

is 0.0238 m/sec2 . This is 2.4 103 g, a very Plugging into these, we nd that v = 52 m/sec
small acceleration compared to the acceleration and a = 20 m/sec2 .
of gravity.
(b.) This is the same sort of calculation, except 3. K&K problem 1.20
that the angular velocity is now 2/28 days. The
mean radius of the moons orbit is 3.84 108 m. The motion of a particle is given by r = A,
The centripetal acceleration is 0.0026 m/sec2 , = t2 /2 and A = (1/) m/rad. The sketch of
which is 2.6 104 g. this motion should look something like this:
2

4. Two objects are dropped from a building at


times t = 0 and t = t0 . The distance that the
rst has fallen as a function of time is just d1 =
gt2 /2. The distance that the second has fallen is
d2 = g(t t0 )2 /2. When t > t0 , the separation
between them is l = d1 d2 . Thus we get

1 2 1 2 1
l= gt g(t 2tt0 + t20 ) = gtt0 gt20
2 2 2

(b.) We plug the expression for into r to get Solving for t as a function of l, we get
r = At2 /2. From these we can get all of the
necessary derivatives. r = At, r = A, = t, l t0
t= +
and = . gt0 2

Using the equation for radial acceleration,


When l < gt20 /2 this time is negative. This does
we nd an expression for points where the radial
have a sensible interpretation, unlike the nega-
acceleration is zero.
tive time in the airplane problem of the previous
ar = r r2 = 0 r = r2 problem set. Think of the problem as if we
wanted to throw both objects at the same time,
Plugging in the expressions for the r and vari- but still have the initial conditions given. At t =
ables, we nd that A = A(t2 )2 /2. Using 0, when we drop the rst object, from where do
2 we have to throw the other object? The answer
that = t /2, we arrive at the result
the fact
= 1/ 2. is we want to throw upwards from below in such
a way that at time t = t0 , the ball has reached
(c.) For this part, we set the magnitudes of the
the peak of its path and is momentarily at rest
radial and tangential accelerations equal to each
at the point where the rst ball was dropped.
other and solve for the angle. Plugging into the
During the time between t = 0 and t = t0 , the
formulas for the two accelerations, we get
separation between the objects can be negative,
ar = r r2 meaning that the second one is below the rst.

a = r + 2r (b.) In this part we want to calculate the op-


timal value of t0 so that the separation reaches
Plugging into the various terms, we get some value l0 at the earliest time possible. In
    other words, we want to minimize the function
   
A 1 A3 t4  =  1 A2 t2 + 2A2 t2  t(t0 , l0 ). First we take the derivative and set it
 2  2 
to zero to nd local extrema.
Plugging in the expression for where we can, 
we get   dt 1 l0 2l0
1 22  = |5| = 2 = 0 t0 =
dt0 2 gt0 g
When the dust settles, there will be separate re-
sults depending on the value of . The nal Notice that this is just the time that it takes the
results are rst object to fall a distance l0 . The endpoints
here are t0 = 0, where the separation remains
1 33 5
< : = at l = 0 forever and t0 = , where the time to
2 4 reach a separation of l0 is also innity. Thus

1 33 + 5 this best time to drop it is at t0 = 2l0 /g. This
> : =
2 4 means that the best thing to do is to drop the
3

second object when the rst object has already Treating this as a function of , we can maxi-
fallen a distance l0 . mize the range by dierentiating with respect to
. Note that the endpoints in this problem are
5. K&K problem 1.21
not interesting. Throwing the ball straight up
This is another maximization problem. We ( = 90 ) and throwing it at an angle both
want to know the optimal angle to throw a ball result in the ball traveling no distance in the x
down a hill with slope angle . Splitting this direction.
into the x and y directions is the easiest way
to do the problem. First put the origin at the dx 2v 2
top of the hill. If the ball is thrown up at an = (cos (2) 2 cos sin tan )
d g
angle with speed v, the initial velocities are
2v 2
vx = v cos and vy = v sin . = (cos (2) sin (2) tan ) = 0
g

Solving for , we get

cos(2) = sin(2) tan cot(2) = tan

Remembering that cot = tan(/2 ), we see


the nal result:

Taking into account the acceleration of gravity,  


tan 2 = tan =
the positions are given by 2 4 2
x = vt cos
1 Note that on a level surface, when = 0, the
y = vt sin gt2 optimal angle is 45 , as you might already know.
2
We need to know where the ground is in these 6. K&K problem 2.1
coordinates. At a position x on the ground, the
y coordinate is given by yground = x tan . We This is the rst problem where you are asked
can now nd the time at which the ball hits the to consider the forces causing acceleration. The
ground. Plugging into the equation for distance force on a 5 kg mass is given by F = 4t2 x 3ty
traveled in y, we get Newtons. Apply Newtons second law of mo-
1 tion, namely F = ma, to get the acceleration,
yground = x tan = vt sin gt2 a = (4t2 /5)x (3t/5)ym/sec.
2
1
v sin gt = v cos tan (a.) We can get velocity from acceleration by
2 integrating
This gives the time at which the ball hits the
ground:  t
2v v(t) v(t0 ) = a(t )dt
t= (sin + cos tan ) t0
g
We now plug this time into the equation for
distance traveled in the x direction, giving the Plugging the acceleration we just determined
distance that the ball traveled: into this integral, and knowing that the velocity
at t = 0 is zero, we get the velocity as a function
2v 2
x= (cos sin + cos2 tan ) of time:
g
2v 2 1 4 3 3
= ( sin 2 + cos2 tan ) v(t) = t x t2 y m/sec
g 2 15 10
4

(b.) We get the position by integrating again: There are two unknowns here, ax and CM . We
 t need another equation. Luckily, there is another
r(t) r(t0 ) = v(t )dt block that we can write equations about. The
t0 small block, having mass m, is aected by four
Applying this to the result of part (a.), and re- forces. They are the contact force, the force of
membering that at t = 0 the mass is at the origin gravity, the normal force, and the force of fric-
so r(0) = 0, we get the position as a function of tion. The contact force is exactly opposite to
time: the contact force on the rst block. This is due
1 4 1 to Newtons third law of motion, which states
r(t) = t x t3 y m
15 10 that every force has an equal and opposite force.
(c.) Now all that is left is to take the cross prod- Thus Cm = CM and the contact force is CM x.
uct of the position with the velocity. We nd The force of gravity is just mgy, the normal
that force exactly opposes gravity as before, +mgy,
 and the force of friction is again mg. We
3 6 4 6 t6
rv = t + t z = z get the equation of motion for the second block,
150 150 150 noticing that acceleration of the second block is
the same as for the rst block because they are
7. This problem asks you to consider two blocks
moving together:
sliding on a table together. The larger block,
with mass M , has ve forces acting on it. They
are F x, the applied force, a contact force that CM
ax = g
I will call CM x, the force of gravity gy, the m
normal force N y, and the force of friction. Be-
cause there is no acceleration in the y direction, We are only concerned with CM here, so we can
we can easily nd that N = M g, so that there simplify the solution of these equations. Subtract
is no net force in the y direction. From the nor- the second equation from the rst to get
mal force we can determine the force of friction. 
An object that is sliding with friction along a F 1 1
CM + =0
surface is acted upon by a force opposing the di- M M m
rection of motion with magnitude N (= M g),
where is the coecient of sliding friction. We We can now solve for CM in terms of F :
now can write an expression for the acceleration
of the large block in the x direction 
m
Fx F CM CM = F
ax = = g M +m
M M M
If we follow the same procedure when the force
is acting on the second block, we get a very simi-
lar answer, but the mass in the numerator is the
larger mass

M
CM = F
M +m

This is a factor of M/m larger, which is what


we expect. The only force pushing on the sec-
ond block in each case is the contact force,
and the acceleration doesnt depend on which
side we push the combined system from. The
5

force of friction acts in proportion to the mass Solving for the acceleration,
in this case, so it does not aect this argu- 
ment. It can be thought of as a force that M m
a=g
acts on the combined system, not on the indi- M +m
vidual blocks, because it is proportional to the
The larger mass accelerates with magnitude a
mass. Notice that neither of these expressions
downward. The tension is found by plugging the
depend on the value of , which would indicate
acceleration into either of the starting equations
that the friction was aecting the contact force.
 
M m 2M m
T = Mg Mg =g
M +m M +m
8. K&K problem 2.5
This is the rst pulley problem, and it wont 9. Nope, this isnt the last pulley problem either.
be the last. The pulley is massless and friction- Again, the pulley and cords are massless and the
less, and supports two masses M and m by a pulley is frictionless. A force F is applied up-
massless rope connecting them. The rst thing ward, and various things will happen depending
to notice about this problem is that the ten- on what F is. The rst thing to notice is that the
sion in the rope must be the same on both sides pulley is massless. This means that the tensions
of the pulley. If the dierent sides had dier- on the two ropes must be equal, otherwise a nite
ent tensions, there would be a tendency to cause angular force would be applied to a massless ob-
an angular acceleration in the pulley. Since it ject, which again is unphysical. The second thing
is massless, this acceleration would be innite, to notice is that the upward force must exactly
which is unphysical, so the tensions must be balance the sum of the tensions. If this werent
equal. You will see this in more detail when you the case, there would be a net force applied to a
do rigid body motion later this term. massless object. This cant happen, so to balance
the forces we just need F = 2T . With these two
points in mind, we can do the rest of the problem.
(a.) The boundary between regimes (i) and (ii)
is where the lighter block lifts o the ground.
Consider the forces on this block. They are grav-
ity mg, the tension T , and the normal force N .
The equation of motion is ma = T + N mg. At
the boundary between regimes (i) and (ii) when
the block just barely can be lifted upward, the
normal force N is zero, but so is the accelera-
tion. This gives us T = mg. We know also that
The second thing to notice is that the more T = F/2, so the minimum force to lift the lighter
massive block will fall and the less massive block block is F = 2mg. This is just twice the weight
will rise, and their accelerations will be the same, of the lighter block, which we expect because the
but in opposite directions. This just means that force applied gets divided in half by the pulley.
the rope isnt stretching. For this problem I will The boundary between regimes (ii) and (iii) is
set down to be the positive direction. The found in a similar way. The equation of motion
equation for the larger mass is is T + N M g = M a. Again both a and N are
M a = M g T T = M (g a) zero at the transition point, so T = M g, which
gives the force F = 2M g. The nal results are
Using the expression for tension derived in the
above equation, the equation for the smaller regime(i) {F < 2mg}
mass is the following: regime(ii) {2mg < F < 2M g}
ma = mg + T m(a + g) = M (g a) regime(iii) {2M g < F }
6

(b.) Now we want to nd the accelerations


for regimes (ii) and (iii). This is easy be-
cause we have already determined the equations
of motion. For regime (ii), only the lighter
block accelerates. The equation of motion is
F/2 mg = ma. This gives the result

regime (ii) aM = 0
F
am = g
2m

In regime (iii), the equation of motion of the


rst block is the same, so we get the same result
for the acceleration. The equation of motion of
the larger block is F/2 M g = M a. These give
the results
F
regime (iii) aM = g
2M
F
am = g
2m

10. K&K problem 2.6


The cement mixers drum has a radius R.
We want to know how fast it can rotate so that
the material will not stick to the walls all of the
time. We just need to gure out at what speed
the drum can oppose gravity all of the time. For
a glob of material of mass m, the worst case is
at the top. To remain in contact with the drum,
at that point the glob must feel a downward
force from the drum that is positive. In addition
it feels the downward force mg due to gravity.
So the total downward force on it must be at
least mg. Now, what acceleration accompanies
this force? We dont want the glob to leave the
drum, so its radial velocity must remain equal
to zero. The only remaining possible downward
acceleration is the centripetal acceleration due
to the circular motion. This is 2 R. Equating
the mass m times this acceleration to the total
 force, we conclude that m R mg,
2
downward
or g/R. For the material to not always
stick, we need the nal result

g
<
R
University of California, Berkeley
Physics H7A Fall 1998 (Strovink)
PROBLEM SET 3

1. a package, it cuts into the package most


deeply as it passes around corners, where r
(a.) A clothesline is tied between two poles, 10
is least.)
m apart, in a way such that the sag is neg-
ligible. When a wet shirt with a mass of (c.) If the contact is not perfectly smooth, the
0.5 kg is hung at the middle of the line, the values of the tension at the two ends of the
midpoint is pulled down by 8 cm. What is arc can dier by a certain amount T be-
the tension in the clothesline? fore slipping occurs. The value of T is
equal to N , where is the coecient
(b.) A car is stranded in a ditch, but the driver of friction between string and rod. Deduce
has a length of rope. The driver knows that from this the exponential relation
he is not strong enough to pull the car out
directly. Instead, he ties the rope tightly T () = T0 exp ()
between the car and a tree that happens to
be 50 ft away; he then pushes transversely where T0 is the tension applied at one end
on the rope at its midpoint. If the midpoint of an arbitrary arc of string and T () is
of the rope is displaced transversely by 3 ft the tension at the other end.
when he pushes with a force of 500 N (50
kg), what force does this exert on the car? (d.) The above result expresses the possibility
If this were sucient to begin to move the of withstanding a large tension T in a rope
car, and the man pushed the rope another by wrapping the rope around a cylinder,
2 ft, how far would the car be shifted, as- a phenomenon that has been exploited by
suming that the rope does not stretch any time immemorial by sailors. Suppose, for
further? Does this seem like a practical example, that the value of in the contact
method of dealing with the situation? between a rope and a (cylindrical) bollard
on a dock is 0.2. For T0 = 100 lb applied
2. A string in tension T is in contact with a by the sailor, calulate the values of T cor-
circular rod (radius r) over an arc subtending a responding to 1, 2, 3, and 4 complete turns
small angle (see the gure). of the rope around the bollard. (It is inter-
esting to note that T is proportional to T0 .
This allows sailors to produce a big pull or
not, at will, by having a rope passing around
a continuously rotating motor-driven drum.
This arrangement can be described as a
force amplifier).
3. A popular demonstration of inertia involves
(a.) Show that the force with which the string
pulling the tablecloth out from beneath dishes
presses radially inward on the pulley (and
with which the table is set. Suppose a tablecloth
hence the normal force N with which the
just covers the area s2 of a square table. A dish
pulley pushes on the string) is equal to T .
is in the exact center of the table. The coe-
(b.) Hence show that the normal force per unit cient of sliding friction between the dish and the
length is equal to T /r. This is a sort of cloth is 1 , and that between the dish and the
pressure which, for a given value of T , gets table is 2 . A dinner guest withdraws the cloth
bigger as r decreases. (This helps to explain swiftly, but at a steady rate. Let the distance
why, when a string is tied tightly around the dish moves while in contact with the mov-
ing cloth be x1 and the distance it moves while This situation serves as a simple model of
in contact with the table be x2 . the standard view of interactions (repulsive, in
the present example) between elementary parti-
(a.) Solve for the maximum velocity v of the
cles.
dish in terms of x1 , 1 , and g.
8. Find the center of mass of a semicircular
(b.) Do the same in terms of x2 , 2 , and g.
hoop of radius R.
(c.) Show that in order for the dish just to re-
main on the table,

2
x1 = (s/2) .
1 + 2

(d.) Find the length of time during which the


dish and tablecloth are in contact under
conditions (c.).
(e.) A pitfall for the dinner guest is that the
dish may not slide at all, but instead merely
move with the cloth. How does she avoid
that?
4. A piece of string of length L, which can sup-
port a maximum tension T , is used to whirl a
particle of mass m in a circular path. What is
the maximum speed with which the particle may
be whirled if the circle is
(a.) horizontal;
(b.) vertical?
5. K&K problem 2.31 Find the frequency of
oscillation....
6. K&K problem 2.35 This problem involves...
A block of mass m slides....
7. Two skaters (A and B), both of mass 70 kg,
are approaching one another, each with a speed
of 1 m/sec. Skater A carries a bowling ball with
a mass of 10 kg. Both skaters can toss the ball at
5 m/sec relative to themselves. To avoid collision
they start tossing the ball back and forth when
they are 10 m apart. Is one toss enough? How
about two tosses, i.e. A gets the ball back? Plot
the entire incident on a time vs. displacement
graph, in which the positions of the skaters are
marked along the x axis, and the advance of
time is represented by the increasing value of the
y axis. (Mark the initial positions of the skaters
at x = + or 5 m, and include the space-time
record of the balls motion in the diagram.)
1

University of California, Berkeley


Physics H7A Fall 1998 (Strovink)
SOLUTION TO PROBLEM SET 3
Composed and formatted by E.A. Baltz and M. Strovink; proofed by D. Bacon

1. (a.) This problem is a simple force bal- will use sin = 0.12. We also need cos = 0.99.
ance. The component of the tension in the rope The force of tension is found by a force balance
pointing up must balance the force of gravity
pulling down on the shirt. We are going to 2T sin = 500 N T = 2083 N
ignore the eect of gravity on the rope. The
angle that the rope makes with the horizontal The force acting on the car is just the cosine of
is just tan = 8 cm/5 m = 0.016. The sine of the angle times this tension, assuming that we
this angle is very close to this value. In fact want the force directed towards the tree.
sin = 0.015998, so we will use sin = 0.016.
F = T cos = 2062 N
The equation for the force balance in the
vertical direction is just Now we want to nd how far the car is shifted.
The original
right triangle had sides 3 ft, 25
2T sin = mg ft, and 9 + 625 =25.18 ft. The rope will not
stretch anymore, so the hypotenuse of this tri-
This ensures that the force of tension balances angle will remain the same, 25.18 ft. The rope
gravity. There is a factor of two in front of the is pushed an additional 2 ft, so the short side
tension because the tension in each half of the now has length 5 ft. The long side must have
rope acts on the mass. Solving this equation length 634 25 = 24.68 ft. The car has moved
with the values given, the answer is twice this distance, since the are two triangles
with total hypotenuse 50 ft; only 0.63 ft of car
T = 153 N movement is produced by pushing the rope 2 ft.
This isnt a very practical way to get the car out
of the ditch.
2. (a.) In this problem we will again use the
fact that sin when  1. The angle
that the rope makes with the normal force is
/2, so the tension in the normal direction is
T sin(/2) T /2. There are two tensions
here, one from each side of the rope, and they
must balance the normal force. The normal force
on this section of string is then just

N = T

(b.) The length of rope that covers an angle


on a circular object is just r. of course
(b.) This part is similar to the rst. In this
is measured in radians. The normal force per
case the force being applied to the rope is a
length on the cylinder is N/ . Plugging in
man pushing on it, no gravity, but the method
the result for N from part (a.), and using
is the same. The force of 500 N must be bal-
= r, we nd
anced by tension in the rope. The angle tan =
3/25 = 0.12 is much larger, but the approxima- N T
tion sin tan is still very accurate, so we =
r
2

(c.) When the tension on the rope is not con- 3. A dish sits in the middle of a square table of
stant, there can be slipping. The force counter- side s. The coecient of friction between dish
ing this is friction. If the tension changes by an and tablecloth is 1 . The coecient of friction
amount T along a small section of rope, the between the dish and table is 2 . The tablecloth
frictional force must be equal to it. The fric- is rapidly pulled out from under the dish. The
tional force is N when it is just about to slip. dish moves a distance x1 while in contact with
We thus get T = N . Plugging in the re- the moving tablecloth, and a distance x2 while in
sults of (a.), we get T = T . We are going contact with the table. Let the mass of the dish
to promote this relation between small quanti- be m, but we will see that this doesnt matter.
ties to a dierential relation, so that we can get (a.) The tablecloth is being pulled out from
a dierential equation to solve. under the dish, so the dish is sliding on the
tablecloth. The frictional force tends to pull the
dT
dT = T d = T dish to the edge of the table because this op-
d poses the direction of the sliding. The normal
force of the dish on the table is just mg, so the
force of sliding friction is just 1 mg. Newtons
second law then tells us that

F = ma = m1 g a = 1 g

This is a constant acceleration, so we can easily


determine the amount of time that the dish is
This is a dierential equation that we can on the tablecloth and its maximum velocity. We
solve by direct integration. know the total distance traveled is x1 , so we get
the following equations for the time of contact
dT dT t1 and the maximum velocity v:
= T = d
d T 1
 T ()  1 gt21
dT  x1 = v = 1 gt1
= d 2
T0 T 0
These equations are easily solved for t and v:
These integrals are ones that you should memo- 
2x1 
rize if you havent yet. The result is t1 = v = 2x1 1 g
1 g
ln T () ln T0 = T () = T0 e
(b.) We do the same thing for the period when
the dish slides on the table. This time, the fric-
The tension increases exponentially, provided
tional force tends to slow the dish down. The
that the rope is about to slip.
frictional force is 2 mg, so the acceleration is
(d.) Here we are going to calculate some val- a = 2 g. The trip starts at the maximum ve-
ues for this amplication of force. = 0.2 and locity v, and ends with the dish at rest having
T0 = 100 lbs. The values of T for 1, 2, 3, and moved a distance x2 . We can again solve for the
4 complete turns are as follows. One complete travel time t2 and the maximum velocity v:
turn has angle 2. Exp(2) = 3.51, so the ten-
1
sion at the other end is 351 lbs. For two turns, x2 = 2 gt22 + vt2 v = 2 gt2
the angle is 4, so the amplication factor is 2
exp(4) = 12.35 and the tension at the other Again these equations are easily solved
end is 1235 lbs. For three complete turns, the 
tension is 4338 lbs. For four complete turns, the v  2x2
tension is 15240 lbs, almost 8 tons! t2 = v = 2x2 2 g t2 =
2 g 2 g
3

(c.) In part (c.), we derived two expressions 4. A string has length L and can support a ten-
for the maximum velocity v. If we equate these sion T . A mass m is spun around on the end of
we can get an expression relating the distances the string.
traveled x1 and x2 . (a.) The string is spun horizontally. The cen-
  tripetal acceleration is v 2 /L, so the force needed
2x1 1 g = 2x2 2 g 1 x1 = 2 x2 to supply this acceleration is mv 2 /L. In this
case, the only force to consider is the tension.
For the dish to remain on the table, we need The maximum velocity is given by
the total distance traveled to be less than half 
the length of the table, x1 + x2 s/2. We can mv 2 LT
=T v=
combine these equations to solve for the distance L m
x1 that the dish spends on the tablecloth. We
will consider the case where the dish stops at The above would be correct if gravity could
the edge of the table so the total distance trav- be ignored for part (a.). However, we know this
eled is s/2. From the previous equation we get cannot be the case, because then there would be
a relation between x1 and x2 no denition for the word horizontal. Grav-
ity must be present. If so, the string makes an
1 angle with the horizontal, and the radius of
x2 = x1
2 the spin is L cos . The centripetal acceleration
is v 2 /L cos . This must be provided entirely
Combining this with the previous equation, we by the tension in the rope in the radial direc-
get the nal answer tion, which is T cos . This gives a relationship
 between v and for a given T :
s 1  s
x1 + x2 = 1+ x1 = TL mv 2
2
s
2 2 v2 = cos2 cos2 =
2 m TL
x1 =
2 1 + 2 The tension must also oppose the force of grav-
ity downward, giving a second relation, which
will be easier to handle when squared
(d.) To nd the amount of time the dish is in
 mg 2
contact with the tablecloth,
 we just use the re- T sin = mg sin2 =
sult of part (a.), t1 = 2x1 /1 g. Plugging in T
our result, we get Adding these two equations, and using cos2 +
    sin2 = 1, we get a condition on the velocity
s 2 1
t1 = m 2 m2 g 2 2 LT mg 2 L
g 1 1 + 2 v + = 1 v =
TL L2 m T
The maximum velocity considering gravity is
(e.) It is possible that the dish wont slide at thus 
all. This is the case of static friction. This force LT mg 2 L
v=
must be overcome but a suciently hard tug. m T
The tablecloth must accelerate at the beginning
of the pull to get to its high, but constant veloc-
ity. If this acceleration is too low (a1 < static
1 g),
the force of static friction will be sucient to ac-
celerate the dish at the same rate, and the host
will not be pleased. If the initial acceleration is
high enough, the force of static friction cannot
impart the necessary acceleration to the dish,
and it begins to slide.
4

(b.) The string is spun vertically. In this case This relationship is usually written in a dierent
we must also consider gravity. Gravity directly way that is easier to remember. You can check
opposes the tension when the mass is at its low that it is correct:
point. At any other point, the tension is less.
At the low point, the two forces of tension and 1 1 1
= +
gravity oppose each other, so the tension must k k1 k2
be higher to provide the centripetal acceleration.
When the rope is at an angle to the vertical, The nal result for the frequency is
the centripetal force is provided by two sources, 
gravity and tension k1 k2
a =
2 m(k1 + k2 )
mv
= T () mg cos
L
T () is largest at the bottom of the path, when
= 0. Set T (0) = T , the largest allowed tension.
This gives the nal answer

mv 2 LT
= T mg v = Lg
L m

5. K&K problem 2.31


(b.) In this case the thing to notice is that the
Find the eective spring constants of these
displacements of the two springs must be equal,
two spring systems. Remember that the fre-
otherwise the support would be tilting.
quency of oscillation is = k/m.
(a.) Consider the point where the springs are F1 = k1 x F2 = k2 x
attached to each other. There shouldnt be any
force acting there because the small point is
The total force acting on the mass is just the sum
massless. We can write equations for the dis-
of these two forces F = F1 + F2 . We can easily
placements of the two springs x1 and x2 . This
nd the eective spring constant of the system:
spring force acts like a tension in that it pulls
from both ends.
F = kx = k1 x k2 x k = k1 + k2
F1 = k1 x1 = F2 = k2 x2 k1 x1 = k2 x2
The nal result for the frequency is
The total displacement of the mass is x1 + x2 ,
and the total force is just the spring force of the 
bottom spring F = k2 x2 . Applying the general k1 + k2
b =
relation F = kx, where k is the spring constant m
of the combined spring system and x = x1 + x2 ,
we get 6. K&K problem 2.35
k2 x2 (a.) In this problem we need to solve a dier-
F = k2 x2 = k(x1 + x2 ) k =
x1 + x2 ential equation. A block slides on a frictionless
Using the fact that x1 can be written in terms of table inside a xed ring of radius l. The ring has
x2 , we can remove the position dependence from a coecient of friction . We want to nd the ve-
the equation: locity as a function of time. At time t = 0 the ve-
locity is v0 . We will assume that the block moves
k2 k1 k2 in the circular path dened by the ring. This
k= k2
k=
k1 +1 k1 + k2 makes it eectively a one dimensional problem.
5

The nal result for the total angle traveled is

1  v0 
(t) = ln 1 + t
l

Notice that the total distance traveled is innite


if one waits for an innite time, even though the
There are two forces acting on the block in
velocity approaches zero as time increases. We
the plane of the table. They are the normal force
can of course write the x and y coordinates of
exerted by the ring and the frictional force. The
the block as functions of time:
normal force merely makes the block move in
the circular path that we assumed. We do need
to know it though, because we want to calcu- x(t) = l cos((t)) y(t) = l sin((t))
late the frictional force. We nd it in the usual
way, by requiring that it provide the centripetal
acceleration. 7. The two skaters each have mass 70 kg. Skater
2 2 A carries a 10 kg bowling ball. Initially each
v mv
acentripetal = N= skater is moving at 1 m/sec, and they are ap-
l l
proaching each other. They are going to try to
We can now write the equation of motion for the
avoid collision by throwing the bowling ball back
particle by Newtons second law
and forth.
dv mv 2 dv
m = N = = v2 This problem uses conservation of momen-
dt l dt l
tum. Skater A starts out with pA = (70 + 10)
This equation can be solved by direct integra- 1 = 80 kg-m/sec of momentum. Notice that we
tion, as you saw in problem 2: must include the momentum of the bowling ball
 v(t)  t in the momentum of skater A when he is carry-
dv dv dt
2
= 2
= ing it. This adds 10 kg-m/sec to skater As 70
v l v0 v 0 l
kg-m/sec, since the bowling ball has a mass of
1 1 t
= m = 10 kg. Skater B is going in the opposite di-
v0 v(t) l rection, so her momentum is negative, pB = 70
This result can be simplied to get K&Ks result kg-m/sec. Skater A throws the bowling ball to
 1 skater B in an attempt to stop the collision.
v0 t Since there are no external forces on the sys-
v(t) = v0 1 +
l tem consisting of skater A and the bowling ball,
the total momentum of these two objects is con-
(b.) Now that we know the velocity of the served. Throwing the bowling ball at 5 m/sec
block, nding the position is easy. It is eas- relative to the (initial) velocity of skater A gives
iest to describe the position in terms of the it a momentum of 60 kg-m/sec. This is because
angle on the circle. We can easily determine the bowling ball velocity is 6 m/sec when we add
the angular velocity as a function of time, be- the initial velocity of 1 m/sec. Skater A is left
cause (t) = v(t)/l. We know the velocity, so to with a momentum of pA = 20 kg-m/sec, so he
get the position we just integrate. Assume that hasnt reversed direction or stopped.
= 0 at t = 0, which also means x = l, y = 0
 t  t  Next we consider the system of skater B
v0 1 
(t) =  
(t )dt = dt and the bowling ball. Again there are no ex-
l 1 + v t  /l
0 0 0 ternal forces, so the momentum of skater B plus
1  v0  t the bowling ball is conserved. The total mo-
= ln 1 + t mentum is 60 kg-m/sec from the ball and 70
l 0
6

kg-m/sec from skater B. When skater B catches


the ball, she will then have all of this momen-
tum, pB = 10 kg-m/sec. After this exchange,
skater B has the ball, and the two skaters are
still approaching each other. One toss was not
enough. To summarize the rst toss

initial pA = 80 pB = 70
intermediate pA = 20 pB = 70 pball = 60
nal pA = 20 pB = 10
It is fairly obvious that the center of mass lies
The second toss will be enough to stop the col- on the line YCM = 0, or = 0. The center of
lision. We calculate the velocity of skater B mass is calculated by the following
(including the bowling ball): v = p/m = 0.125 
1
m/sec. Now skater B throws the bowling ball XCM = x dl
M
to skater A. The bowling balls velocity will be
5.125 m/sec, so its momentum will be 51.25 where is the linear mass density and dl is a dif-
kg-m/sec. This leaves skater B with pB = 41.25 ferential of length on the hoop. If the hoop has
kg-m/sec. This is in the opposite direction to mass M , the linear mass density is = M/R.
her initial motion. Skater A gets all of the We can use polar coordinates to integrate this.
momentum of the ball again, so pA = 31.25 Remember that x = R cos for points on the
kg-m/sec. This is also opposite to his initial di- hoop. The dierential of length on the hoop is
rection. So, after two tosses, the skaters are dl = Rd, so the integral we need to do is
moving away from each other and the collision
is averted. Plotting position versus time for the  /2
1 RM 2
two skaters, we get a graph like the following: XCM = cos d = R
M /2

8. This is an example of a center of mass calcu-


lation. Lets put the hoop on a polar coordinate
system so that it goes from = (/2, /2). In
cartesian coordinates this is on the right half-
plane.
University of California, Berkeley
Physics H7A Fall 1998 (Strovink)
PROBLEM SET 4

1. A very exible uniform chain of mass M and noring gravity, is given by


length L is suspended from one end so that it
hangs vertically, the lower end just touching the N
v = V ln .
surface of a table. The upper end is suddenly rN + n(1 r)
released so that the chain falls onto the table
and coils up in a small heap, each link coming (b.) Obtain a corresponding expression for the
to rest the instant that it strikes the table. Find additional velocity u gained from the second
the force exerted by the table on the chain at stage burn.
any instant, in terms of the weight of the chain
already on the table at that moment. (c.) Adding v and u, you have the payload ve-
locity w in terms of N , n, and r. Taking N
2. A supersonic plane of mass M has an air- and r as constants, nd the value of n for
speed v = 1000 m/sec. Its jet engine takes in 80 which w is a maximum.
kg of air per sec, mixes it with 30 kg of fuel per (d.) Show that the condition for w to be a max-
sec, and compresses the mixture so that it ig- imum corresponds to having equal gains of
nites. The resulting hot gasses leave the engine velocity in the two stages. Find the max-
with velocity 3000 m/sec relative to the plane. imum value of w, and verify that it makes
What thrust (force) does the engine deliver? sense for the limiting cases described by
r = 0 and r = 1.
3. K&K problem 3.13 A ski tow consists of....
(e.) Find an expression for the payload velocity
4. This problem is designed to illustrate the of a single-stage rocket with the same values
advantage that can be obtained by the use of of N , r, and V .
multiple-staged instead of single-staged rockets (f.) Suppose that it is desired to obtain a pay-
as launching vehicles. Suppose that the pay- load velocity of 10 km/sec, using rockets for
load (e.g. a space capsule) has mass m and is which V = 2.5 km/sec and r = 0.1. Show
mounted on a two-stage rocket (see gure). The that the job can be done with a two-stage
total mass both rockets fully fueled, plus the rocket but is impossible, however large the
payload is N m. The mass of the second-stage value of N , with a single-stage rocket.
rocket plus the payload, after rst-stage burnout
and separation, is nm. In each stage the ratio 5. A boat of mass M and length L is oating
of burnout mass (casing) to initial mass (cas- in the water, stationary; a man of mass m is sit-
ing plus fuel) is r, and the exhaust speed is V , ting at the bow. The man stands up, walks to
constant relative to the engine. the stern of the boat, and sits down again.
(a.) If the water is assumed to oer no resis-
tance at all to motion of the boat, how far
does the boat move as a result of the mans
trip from bow to stern?
(b.) More realistically, assume that the water of-
fers a viscous resistive force given by kv,
where k is a constant and v is the velocity
of the boat. Show that in this case one has
(a.) Show that the velocity v gained from the the remarkable result that the boat should
rst-stage burn, starting from rest and ig- eventually return to its initial position!
(c.) Consider the paradox presented by the fact
that, according to (b.), any nonzero value of
k, however, small, implies that the boat ends
up at its starting point, but a strictly zero
value of k implies that it ends up somewhere
else. How do you explain this discontinuous
jump in the nal position when the variation
of k can be imagined as continuous, down
to zero? For details, see D. Tilley, American
Journal of Physics Vol. 35, p. 546 (1967).
6. The Great Pyramid of Gizeh when rst
erected (it has since lost a certain amount of its
outermost layer) was about 150 m high and had a
square base of edge length 230 m. It is eectively
a solid block of stone of density about 2.5 g/cc.
(a.) What is the minimum amount of work re-
quired to assemble the pyramid, if the stone
is initially at ground level?
(b.) Assume that a slave employed in the con-
struction of the pyramid had a food in-
take of about 1500 Cal/day (1 Cal = 4182
joules). The Greek historian Herodotus re-
ported that the job took 100,000 slaves 20
years. What was the minimum eciency
of a slave (dened as work done divided by
energy consumed)?
7. A particle of mass m, at rest at t=0, is sub-
jected to a force f (t) whose magnitude at t=0
is F . This magnitude decreases linearly with
time, becoming zero at time t = T . The direc-
tion of the force remains unchanged. What is
the kinetic energy of the particle at time T ?
8. A wooden block of mass M , initially at rest
on a table with coecient of sliding friction , is
struck by a bullet of mass m and velocity v. The
bullet lodges in the center of the block. How far
does the block slide?
1

University of California, Berkeley


Physics H7A Fall 1998 (Strovink)
SOLUTION TO PROBLEM SET 4
Composed and formatted by E.A. Baltz and M. Strovink; proofed by D. Bacon

1. A chain of mass M and length L falls onto a 2. The airspeed of a plane is v = 1000 m/sec.
table. Initially, the chain is hanging so that its The engines take in 80 kg of air per second and
lower end just touches the table. The chain is mix it with 30 kg of fuel. The mixture is expelled
falling in gravity, so the velocity of a link that is after it ignites, and it is moving at a velocity of
falling is given by v = gt. The distance that the 3000 m/sec relative to the plane. We can calcu-
chain has fallen is given by x = gt2 /2. These two late the thrust of this engine by calculating the
facts tell us how much of the chain is on the table rate of change of momentum. The fuel is ejected
at a given time. The density of the chain is M/L, at a rate of 30 kg/sec, and it is given a velocity
so the mass of chain on the table is just M x/L: of 3000 m/sec relative to the plane. It started
at rest with respect to the plane, so it need to
1M 2
M (t) = gt be given the full velocity. The rate of change of
2 L momentum this corresponds to is
The rate at which the mass is falling on the table
is just
dp dm
d
M (t) =
M
gt =
M
v(t) = v = 30 3000 = 90, 000 kg m/sec2
dt L L dt dt
At time t, the free elements of chain are moving
with speed v(t). This is the velocity they have The air also contributes to the momentum. It
when they hit the table. The total rate at which is expelled at a rate of 80 kg/sec. Its velocity
momentum is being transferred is is already 1000 m/sec relative to the plane, so it
only needs to gain 2000 m/sec of velocity in the
dp d M 2 engine. The rate of change of momentum that
= v(t) M (t) = v (t)
dt dt L this corresponds to is
Writing this in terms of M (t), the mass on the
table at time t, we get the following: dp dm
= v = 80 2000 = 160, 000 kg m/sec2
dp dt dt
= 2M (t)g
dt
The total rate of momentum transferred to the
The rate of change of momentum should be fa-
exhaust by the planes engine is thus
miliar to you from Newtons second law which
states
d dp
F= p = 250, 000 kg m/sec2
dt dt
Thus the table must be exerting this force on the
chain to slow it down. Remember also that the This rate of momentum transfer is equal to the
table exerts a normal force on the chain which thrust of the engine:
is equal to the force of gravity
FN = M (t)g Fthrust = 250, 000 N
Thus the total force that the table exerts on the
chain is three times the weight of the chain on
the table: 3. K&K problem 3.13 This problem concerns the
total force that a ski lift must exert to lift skiers
F (t) = FN (t) + 2M (t)g = 3M (t)g to the top of a hill. There will be two parts to
2

the force. The rst is just the force necessary to the rst burn. So the rst burn velocity gain is
oppose the force of gravity on the skiers. The sec-
ond is the force required to accelerate the skier at N N
v = V ln = V ln .
the bottom from rest to the speed of the lift. The n + r(N n) N r + n(1 r)
rope is 100 meters long, and it is pulled at 1.5 me-
ters per second. On average, one skier uses the (b.) The method for this part is the same as for
tow rope every ve seconds. This means the tow part (a.) because the rst equation guarantees
rope travels 5 1.5 = 7.5 meters between skiers, that the velocity gain of a rocket is independent
so 100/7.5 = 13 13 skiers are on the rope on aver- of its initial velocity. Here the initial mass is
age. Each skier weighs 70 kg, so the average total the full mass of the second stage, nm. The nal
weight of the skiers who are on the rope is 933 kg. mass is nm minus the mass of fuel consumed in
The component of the force of gravity that must the second burn, which is (1 r)(nm m). This
be oset by the rope is determined by the angle yields m(1 + r(n 1)) for the nal mass, and a
of the slope, which is 20 . The component of the second burn velocity gain of
acceleration of gravity that is directed down the
n n
slope is just g sin 20 = 0.342g. Therefore the u = V ln = V ln .
force that the tow rope must exert to oset that 1 + r(n 1) nr + (1 r)
component of gravity is 933 g 0.342 = 3128
N. In addition, when a skier grabs the rope, he (c.) Here we optimize n with all other pa-
must be accelerated to the speed of the rope, rameters xed. We wish to maximize v + u.
1.5 m/sec. The change in momentum for the As V is xed, we choose equivalently to mini-
skier is 1.5 70 = 105 kg-m/sec. This change mize Q = ln (V /(v + u)) in order to simplify the
in momentum must be provided by the motor algebra. From (a.) and (b.) we have
once every ve seconds, which is how often skiers
N r + n(1 r) nr + (1 r)
use the lift. On average, this force is 105/5=21 Q= .
N. Therefore the total force that the lift must N n
provide is, on average, 3128+21=3149 N. Carrying out the division,
4. A two stage rocket carries a payload of mass n 1
m. The total mass of the rocket is N m, and Q = (r + (1 r))(r + (1 r)) .
N n
the mass of the second stage and payload is nm.
In each stage, the mass of the fuel is a fraction Multiplying,
(1-r) of the total, so the mass of the casing is a
fraction r of the total mass. The rst stage has (1 r)2 n 1
Q = r2 + + r(1 r)( + ) .
a mass (N n)m ,which is just the total minus N N n
the mass of the second stage.
Only the last term depends on n:
(a.) Since gravity can be ignored, the equation
for rocket motion derived in class reduces to d n 1
+ = 0, n = N .
dn N n
M0
v(t) v0 = V ln . (d.) For this value of n, the velocity gains from
M (t)
the rst and second burns are equal:
To determine the velocity gain v from the rst
N
burn, we need only to compute the mass of the v + u = 2u = 2V ln .
rocket at the end of the burn. The initial mass 1 + r( N 1)
is N m, while the mass of fuel burned by the rst
stage is (N m nm), the mass of the rst stage, (e.) A single stage rocket has the same values of
multiplied by (1r). The dierence m(n+r(N N, r, and V. The initial mass is N m, as in part
n)) of these two masses is the residual mass after (a.), and the nal mass is m + r(N m m), in
3

analogy to part (b.) with N substituted for n. this case zero. Centering the boat at x = 0, we
The nal velocity is can calculate the center of mass

N (L/2)m
v = V ln . XCM =
N r + (1 r) M +m
After the man is sitting at the bow, the center of
(f.) We want the nal velocity of the payload the boat will be at some position x, which means
to be v = 10 km/sec, and we have a rocket with that the man will be at a position x + (L/2).
exhaust velocity V = 2.5 km/sec and r = 0.1. However, the center of mass will be in the same
First lets see if this can be done with a single place.
stage rocket. Plugging into the result from part
M x + m(x + (L/2)) (L/2)m
(e.), we see that =
M +m M +m
N mL
10 = 2.5 ln x= .
0.9 + 0.1N M +m

We try to solve for the necessary N The boat has moved from its initial position.
(b.) This time, the water exerts a viscous force
e4 =
N F = kv on the boat. We can show that the
0.9 + 0.1N boat will always return to its original position.
5.46N + 49.1 = N Newtons second law gives the following equa-
N = 11.0 . tion. We want to use the total mass of the boat
plus the man, because we dont want the man
This answer doesnt make any sense, which accelerating relative to the boat
means that a single stage rocket cant do the  v(t)  t
dv k
job. Lets now look at the optimal two stage (m + M )v = kv = dt
rocket, using the result from part (d.): v0 v M +m t0

N This gives
10 = 5 ln  
0.1N + 0.9 N k
v(t) = v0 exp (t t0 )
M +m
Again we try to solve for N :
The distance traveled in this interval is just the
2 N integral of the velocity
e =
0.1N + 0.9 N
(M + m)v0  k

0.739N + 6.65 N = N x(t) = 1 e M +m (tt0 )
k
0.261 N = 6.65
N = 650 . Now we just need to nd the initial veloc-
ity of the boat. When the man starts moving,
This rocket indeed can be built. say he applies an impulse p. This is the
same impulse that the boat must receive, but
5. A boat of mass M and length L is oating at
in the opposite direction. Thus, the velocity
rest. A man of mass m is sitting at the stern. He
of the man is u = p/m and the velocity of
stands up, walks to the bow and sits down again.
the boat is v = p/M . This means that
(a.) There is no force from the water, there- the velocity of the man relative to the boat
fore the net force on the system is zero. The is u v = (m + M )p/M m. The man is
momentum of the system is conserved, and the now walking at constant speed relative to the
center of mass remains at the same velocity, in boat. We plug in the initial velocity of the boat
4

v = p/M to the solution of the dierential must specify the time interval. We choose the
equation and we nd the velocity of the boat interval from t = 0, just before any motion
  starts, to t = , at which time all motion must
p k have stopped due to eects of viscosity. At
v(t) = exp (t t0 )
M M +m both of those times the total momentum of the
boat+man system, whose rate of change is con-
At time = L/(u v) = LM m/(M + m)p, the trolled by Fext , is zero. Therefore the impulse in
man has reached the other end of the boat. The question, which is equal to the dierence P ()
velocity of the boat is P (0) of the boat+man system, must vanish.
 
p k The same impulse can also be written as
v( ) = exp
M M +m 

and it has traveled a distance 0= Fext dt


0
    
(M + m)p k dx
x( ) = 1 exp = k dt = k dx
kM M +m 0 dt 0
= k(x() x(0)) ,
He again applies an impulse, but this time it is
p. This gives the boat a change in velocity of
where x is the position of the boat. This proves
+p/M . The total velocity of the boat is now
that the boat returns to its original position.
  
p k (c.) The result of part (b.) says that any viscous
v( ) = 1 exp
M M +m force, no matter how small, results in the boat
returning to its original location. The result of
Using this as the initial velocity, we again solve part (a.) says that when there is no viscous force,
the dierential equation the boat moves some distance. Mathematically,
p   the dierence between the two results is due to
k k
v(t) = 1 e M +m e M +m t the order in which the limits are taken. In part
M (a.), the rst thing done is to take the limit
This is correct for all t > . We now calculate as k 0, no viscous force. Then the limit as
the total distance traveled in the second part of t is taken. If we look at the result of part
the trip. We take the nal time to be t = . (b.), we rst take the limit as t , then we
   consider what happens when there is no viscous
(M + m)p k force. This is an instance in which we cannot
x() = 1 exp reverse the order of taking limits. Denoting the
kM M +m
results from part (a.) and (b.) by capital letters,
This is exactly the opposite of the distance we see that
traveled in the rst part. Thus the boat will
eventually return to its starting point. lim lim A = lim lim B
t k0 k0 t

(b .) Here is a quick, elegant way to prove the
result of part (b.). It deserves full grading credit. So much for the reason why, mathematically, the
We do not mention only this method because, results of (a.) and (b.) are not the same. Phys-
as seen above, the problem is amenable to solu-
ically, they are not in conict. As the coecient
tion by systematic calculation as well as brilliant
k approaches zero in part (b.), the speed with
insight.
which the boat ultimately migrates back to its
Consider the impulse applied by the force original position approaches zero also. This can-
Fext of the water on the boat. To specify the not be distinguished by physical measurement
impulse, which is the time integral of Fext , we from the limiting case (a).
5

6. The Great Pyramid at Gizeh is h=150 m time. The direction remains the same. The
high and has a square base of side s = 230 m. It magnitude of the force is thus
has a density = 2.5 g/cc.  
t
(a.) If all the stone is initially at ground level, f (t) = F 1
T
it must be raised to its position in the pyramid.
The work required to this is The force acts on a particle of mass m initially
 at rest. The kinetic energy at t = T is just the
W = M ghcm = gz dV integral
 T    T  
The volume element is the area of the square at a t t
K= F 1 dx = F 1 v dt
height z times dz, the dierential of height. The 0 T 0 T
square has side s at z = 0 and side 0 at z = h.
We can nd v by applying Newtons second law,
The width of the square decreases linearly with
but once we have it, we dont need to do the
height, so the width and area at height z is given
integral because we know that K = mv 2 /2
by
 
 z  z 2 dv t
w(z) = s 1 A(z) = s2 1 f (t) = m =F 1
h h dt T

The volume element dV is given by dV = We can just directly integrate both sides with
A(z) dz. We can now perform the integral. respect to t, with limits t = 0 and t = T
Expanding the polynomial in z
F
 h  2 3
 v(T ) = T
z z 2m
W = gs2 z2 + 2 dz
0 h h We now have the answer
This is a simple integral to perform: 1 F 2T 2
  K=
8 m
2 2 1 2 1 1
W = gs h + = gs2 h2
2 3 4 12 8. Instantaneously after the collision of the bul-
Plugging in the values for these constants, we get let and block, after the bullet has come to rest
the amount of work required to erect the pyramid but before the frictional force on the block has
had time to slow it down more than an inn-
W = 2.43 1012 Joules tesimal amount, we can apply momentum con-
servation to the bullet-block collision. At that
time the total momentum of the block+bullet
(b.) The slaves employed in building this pyra-
system is (M + m)v0 , where v0 is the velocity of
mid consumed 1500 Calories per day, which is
the block+bullet system immediately after the
6.3 106 joules per day. With 100,000 slaves
collision. Momentum conservation requires that
working for 20 years, this is 730 million slave-
momentum to be equal to the initial momentum
days of work to build the pyramid. The total
mv of the bullet. Thus
energy the slaves spent is thus 4.6 1015 joules.
The eciency thus implied is low, " = 5.3104 . v0 =
mv
.
This does not necessarily reect a low intrinsic M +m
eciency, since the slaves undoubtedly expended
After the collision, the normal force on the
most of their energy on activities other than lift-
block+bullet system from the table is (M + m)g,
ing the stone blocks to their nal position.
giving rise to a frictional force
7. A force f(t) has magnitude F at t = 0, mag-
nitude 0 at t = T , and it decreases linearly with N = (M + m)g
6

on the sliding block+bullet system. This causes


a constant acceleration g of that system oppo-
site to its velocity.
Take t = 0 at the time of collision. After-
ward, the block+bullet systems velocity in the
horizontal direction will be v  (t) = v0 gt. It
will continue sliding until v  (t) = 0, at which
point the frictional force will disappear and it
will remain at rest. Solving, the time at which
the block-bullet system stops is

t = v0 /(g) .

The distance traveled in that time is

1 1 (v  )2
x = v0 t gt2 = v0 t = 0 .
2 2 2g

Plugging in the already deduced value for v0 ,


this distance is
 m 2 v 2
x= .
M + m 2g
University of California, Berkeley
Physics H7A Fall 1998 (Strovink)
PROBLEM SET 5

1. Mass M rests on a table, and mass m is


supported above it by a massless spring which
connects the two masses.

(a.) Find the minimum downward force that


must be exerted on m such that the entire
assembly will barely leave the table when
this force is suddenly removed.

(a.) Assuming a value of g independent of height,


(b.) Consider this problem in the time-reversed calculate how high the rocket would rise if
situation: Let the assembly be supported red directly upward from rest at (P ).
above the table by supports attached to m.
(b.) Let (Q) be a distance h vertically lower than
Lower the system until M barely touches
(P ), and suppose that the rocket is red at
the table and then release the supports.
(Q) after sliding down the frictionless chute.
How far will m drop before coming to a
What is the velocity of the rocket at (Q)
stop? Does knowledge of this distance help
just before the spring is released? Just after
you solve the original problem?
the spring is released?
(c.) To what height above (P ) will the rocket
(c.) Now that you have the answer, check it rise now? Is this higher than the earlier
against your intuition by (1) letting M be case? By how much?
zero and (2) letting m be zero. Especially
(d.) Remembering energy conservation, can you
in the second case, does the theoretical re-
answer a skeptic who claims that someone
sult agree with your common sense? If not,
has been cheated of some energy?
discuss possible sources of error.
3. K&K problem 4.7 A ring of mass M
hangs....
2. It has been claimed that a rocket would rise
4. Assume the moon to be a sphere of uniform
to a greater height if, instead of being ignited at
density with radius 1740 km and mass 7.31022
ground level (P ), it were ignited at a lower level
kg. Imagine that a straight smooth tunnel is
(Q) after it had been allowed to slide from rest
bored though the moon so as to connect any two
along a frictionless chute see the gure. To an-
points on its surface. The gravitational force on
alyze this claim, consider a simplied model in
an object by a uniform sphere is equal to the
which the body of the rocket is represented by
force that would be exerted by the fraction of the
a mass M , the fuel is represented by a mass m,
spheres mass which lies at smaller radius than
and the chemical energy released in the burn-
the object, as if that fraction were concentrated
ing of the fuel is represented by a compressed
at the center of the sphere.
spring between M and m which stores a denite
amount of potential energy, U , sucient to eject (a.) Show that the motion of objects along this
m. (This corresponds to instantaneous burning tunnel under the action of gravity would be
and ejection of all the fuel i.e. an explosion.) simple harmonic (neglect friction with the
Then proceed as follows: walls of the tunnel).
(b.) Calculate the period of oscillation.
(c.) Compare this with the period of a satel-
lite travelling around the moon in a circular
orbit at the moons surface.
5. K&K problem 4.23 A small ball of mass m
is placed on top....
6. In an historic piece of research, James Chad-
wick in 1932 obtained a value for the mass of the
neutron by studying elastic collisions of fast neu-
trons with nuclei of hydrogen and nitrogen. He
found that the maximum recoil velocity of hy-
drogen nuclei (initially stationary) was 3.3107
m/sec, and that the maximum recoil velocity of
nitrogen-14 nuclei was 4.7106 m/sec with an
uncertainty on the latter of 10%. What does
this tell you about:
(a.) The mass of the neutron (in amu)?
(b.) The initial velocity of the neutrons used?
(Take the uncertainty of the nitrogen measure-
ment into account. Take the mass of a hydrogen
nucleus as 1 amu and the mass of a nitrogen-14
nucleus as 14 amu.)
7. K&K problem 4.13 A commonly used po-
tential energy function....
1

University of California, Berkeley


Physics H7A Fall 1998 (Strovink)
SOLUTION TO PROBLEM SET 5
Composed and formatted by E.A. Baltz and M. Strovink; proofed by D. Bacon

1. A mass M rests on a table, and a mass m The expression under the square root is just
is supported on top of it by a massless spring (M + m)g, so the expression simplies a lot:
connecting it to M .
(a.) We want to nd the force F needed to F = (M + m)g
push down on the spring so that the whole sys-
tem will barely leave the table. We solve this We obviously want the plus sign.
by conservation of energy. The energy stored in (b.) This is a similar situation. The mass M is
a spring is given by kx2 /2 where x is the dis- dangling and barely touching the table. The dis-
placement from equilibrium of the spring. We placement of the spring just supports the weight
can measure the gravitational potential of the of the block so kx = M g. At the other end,
small mass relative to the equilibrium point of the small mass is momentarily at rest at some
the spring. Initially then, the spring is com- distance y from equilibrium. The energies are
pressed with a force F + mg which is just the
weight of the small mass plus the added force. M mg 2 M 2 g2
Hookes law tells us that x = (F + mg)/k. We Ei = +
k 2k
can now write the initial energy of the system

mg (F + mg)2 ky 2
Ei = (F + mg) + Ef = mgy +
k 2k 2

The rst term is the gravitational energy relative We equate these energies and solve for y.
to the equilibrium point of the spring, and the
second term is the energy stored in the spring. ky 2 M mg 2 M 2 g2
mgy + = +
We want the spring to be able to lift the mass 2 k 2k
M o the table. To do this it must apply a
force equal to M g, its weight. When the spring Using the quadratic formula again
is released, it will oscillate. At the top of the 
oscillation, there will be no kinetic energy. The ky = mg m2 g 2 + 2M mg 2 + M 2 g 2
displacement y of the spring must barely provide
the force to lift the lower block: ky = M g. The Again the discriminant is a perfect square, and
energy here is the following we want the positive value of y, so

M mg 2 M 2 g2 (2m + M )g
Ef = + y=
k 2k k
Conservation of energy tells us that these are
The total distance that the mass falls is x+y = d.
the same, so now we can solve for F .

mg (F + mg)2 M mg 2 M 2 g2 (M + m)g
(F + mg) + = + d=2
k
k 2k k 2k
Cancelling k and using the quadratic formula to This is just twice the displacement caused by the
solve for F + mg, force in part (a.), which makes sense, because
   the displacement upwards should be the same as
F + mg = mg m2 g 2 + 2M mg 2 + M 2 g 2 the displacement downwards.
2

(c.) (1) When M is zero, the necessary applied the answer to part (a.):
force is mg. This is just the weight of the small
mass. The spring will bounce back with the U 1
d=
same force, so this is what is needed to lift the M g 1 + M/m
whole assembly. The distance fallen makes sense
also, because the spring starts at its equilibrium (b.) This is a little more involved. The rocket
length. The mass wants to sit at mg/k below has gone around part of an oval track and is
this, to just support the weight. Thus it will now a distance h below where it started. The
oscillate down to 2mg/k below this point. gravitational energy (M + m)gh gets converted
to kinetic energy, so we get the velocity v0 of the
(2) When m is zero, the force needed to
rocket before the fuel is used:
move the assembly is M g; again this is the total
weight. The distance traveled by the end of the 1 
(M + m)gh = (M + m)v02 v0 = 2gh
spring in the second case is just 2M g/k. The 2
end of the spring is M g/k away from equilibrium Ignoring for the moment the gravitational en-
when it begins, so the total distance traveled by ergy, the energy of the rocket at this point is
the end is 2M g/k. While this seems to work out,
it does not necessarily agree with common sense: 1
Ei = U + (M + m)v02 = U + (M + m)gh
a massless spring would not seem to be able to 2
pull a massive block o the table by virtue of
The spring (fuel) imparts a change in veloc-
its own motion. However, we realize that, as the
ity v to M and u to m. As in part (a.),
spring mass approaches zero in this idealization,
instantaneous conservation of momentum gives
its maximum velocity approaches innity. This
M v = mu. After the spring is released, the
explains why the spring is still able to pull the
energy corresponding to Ei is
block o the table, defying our intuition.
2. 1 1
Ef = M (v0 + v)2 + m(v0 u)2
2 2
(a.) The rocket is red directly upward from the
ground. The initial energy is just U , the energy We know by conservation of energy that Ei =
in the fuel. After the fuel is spent, the fuel mass Ef , and we have M v = mu, so we can nd
m is moving down at speed u and the remaining v: 
rocket mass M is moving upwards at speed v. 2U
v =
Because momentum is conserved over this very M (1 + M/m)
short time, mu = M v. The energy of the sys-
The total velocity of M is now v = v0 + v:
tem is given by conservation of energy, and at

launch, all of the energy is kinetic:  2U
  v = 2gh +
1 1 M v2 M M (1 + M/m)
U = M v 2 + mu2 = 1+
2 2 2 m
(c.) We can easily nd the kinetic energy of
Now we want to consider the motion of the mass the remaining rocket, and, using energy conser-
M alone. Its kinetic energy is M v 2 /2, which we vation, the maximum height H to which it rises
can nd from the previous equation. above its current position:

U 1
KM = K= M (v0 + v)2 = M gH
1 + M/m 2

Using v0 = 2gh, we can solve for H:
Energy for the mass M is now conserved, so we
can just set KM = M gd, where d is the maxi- 2gh + v 2gh + v 2
mum height achieved by the rocket. This gives H=
2g
3

Plugging in the result for v, we arrive at the force is outward, at the top the normal force will
nal answer be negative. The radial equation of motion will
 be
U hU
H =h+ +
M g(1 + M/m) M g(1 + M/m) mac = N + mg cos N () = mg(2 3 cos )

This is an interesting result. The rst term Now we can go back to the force equation for the
just gets the rocket back to the height where it ring and use this result. The total force on the
started in the rst place. The second term gets ring will be zero, but the ring will just start to
it to the maximum height of the rocket in part move upwards when the thread is slack, which is
(a.). The fact that the third term is positive when T = 0. Using both of these facts, we get
means that the rocket actually ies higher in this an equation for
case. The gain in height is just the third term
 M g = 2mg cos (3 cos 2)
hU
H =
M g(1 + M/m) This is just a quadratic equation in cos . Multi-
plying it out, we can apply the quadratic formula
to get the answer
(d.) This result does not conict with energy
conservation, which says only that the total en- 
1 1 3M
ergy of the system is conserved. We have been cos = 1
neglecting what happens to the mass m, which 3 3 2m
will take away a smaller amount of energy in the
There is a small problem here in that the dis-
second case. If we looked at the total energy of
criminant can be negative, making the cosine of
both pieces, it would be conserved.
the angle complex. This of course is unphysical.
3. K&K problem 4.7. This problem is one in The problem is that for suciently small m, the
which both force and energy need to be consid- motion of the small masses is never important
ered. The forces on the ring are gravity, the enough to cause the tension in the rope to vanish,
tension in the thread T , and the normal forces so our calculation is wrong from the start. In-
due to the beads. The forces on the ring in the sisting that cos be real, we obtain the condition
vertical direction are
3
m> M
Fring = T M g 2N () cos 2
where is the angle of the beads position from Taking the positive root, the nal answer is
the top, and N () is taken to be positive out-  
ward. The two beads will move symmetrically. 1 1 3M
We now need to nd the normal force N (). = cos1 + 1
3 3 2m
First we determine the velocity of the bead from
conservation of energy. It yields the following:
4. We assume that the moon is a uniform sphere
1
E/2 = mgL(cos 1) + mv 2 () = 0 of mass M = 7.3 1022 kg and radius R = 1740
2 km. A straight, frictionless tunnel connects two
This gives the velocity, and thus the centripetal points on the surface. Given the mass and
acceleration ac , as a function of : radius, the density is just = 3M/4R3 . We
need to know the acceleration due to gravity at
v 2 () = 2gL(1 cos ) ac = 2g(1 cos ) a distance r from the center of the moon. This is
also straightforward. Recall that a spherical shell
The centripetal acceleration is provided by grav- of mass exerts no force on objects inside it, so at
ity and the normal force. Since a positive normal a radius r, the only force we need to consider is
4

due to the mass in the moon interior to radius r. (c.) A satellite traveling in a circular orbit
This is just the density times the volume interior must have centripetal acceleration provided by
to r, or M(r) = M r3 /R3 . The acceleration gravity, which means that
due to gravity is then just g(r) = GM(r)/r2 =
GM r/R3 . Thus the acceleration due to gravity v2 GM
increases linearly as one moves away from the = 2 = 2 R
R R
center of a uniform solid sphere.
(a.) In a spherical polar coordinate system with From the last equality we see that the angu-
its z axis at the moons north pole, assume lar frequency of a circular orbit of radius R
that the tunnel lies in a straight line between around the moon is the same as 0 above. Of
(r, 0 , ) = (R, 0 , 0) and (R, 0 , ), i.e. between course the period is the same as well.
two points at the same north latitude /2 0 5. K&K problem 4.23. Two balls of masses M
having the largest possible dierence in longi- and m are dropped from height h and collide
tude. This means that the distance along a great elastically. The small ball is on top of the larger
circle between the ends of the tunnel is 20 , while ball. Conservation of energy for the system gives
the distance from the center of the moon to the its speed v right before the balls hit the ground:
center of the tunnel is z0 = R cos 0 . Now assume
that the mass makes an angle ( 2 < < 2 ) 1 
with a line connecting the center of the moon (M + m)gh = (M + m)v 2 v = 2gh
2
and the center of the tunnel, i.e. with the z axis.
The distance of the mass from the center of the The ball M collides with the ground rst. In
tunnel is then x = z0 tan , while its distance order to conserve energy, it must still have
from the center of the moon is r = z0 / cos . We speed v instantaneously after it bounces from
now need to know the component Fx of the grav- the ground. Now it immediately collides with
itational force GM mr/R3 which lies in the (x) the small ball. (Think of this problem as if there
direction of the tunnel, which makes an angle were a very small gap between the two balls so
/2 with the radial direction. This is that the rst ball to hit the ground has a chance
to bounce before the second one hits it.) We con-
GM mr sider the elastic collision between the two balls,
Fx = cos (/2 )
R3 each moving at speed v towards the other.
GM m z0
= sin The easiest frame in which to study this
R3 cos
collision is a comoving (inertial) frame that is
GM m
= x instantaneously at rest with respect to the large
R3 ball M immediately after it has rebounded with

This is like the force from a Hookes law spring velocity v = 2gh from its elastic collision with
with eective spring constant ke = GM m/R3 , the ground. In this frame, M is instantaneously
yielding simple harmonic oscillation with reso- at rest, and m has (upward) velocity 2v. When
nant angular frequency the collision occurs, if m  M as stated in the
problem, M seems to m like a brick wall from
  which it bounces back elastically with the same
ke GM
0 = = speed. Thus, in the comoving frame immediately
m R3 after the collision, m has velocity +2v. Finally,
transforming back to the lab frame, m acquires
(b.) Plugging in values of M and R for the an extra velocity increment v, for a total of 3v.
moon, and using T = 2/0 , we get for the Since the height that m reaches is proportional
period of oscillation to the square of its velocity, this means that
m reaches nine times the height from which it
T = 6536 seconds = 109 minutes originally was dropped.
5

A less elegant approach considers the colli- Neglecting $ with respect to 3 or 1 in both
sion between M and m in the lab frame. Here quotients, the polynomial is
is it essential not to apply the approximation
m  M until near the end, since cancellations u2 2u 3 = (u 3)(u + 1)
occur which may make nonleading terms more
important than would initially be suspected. with the physical solution
In the lab frame, conservation of momentum u=3
gives 
Vm = 3v = 18gh
M v mv = M VM + mVm
Vm2
We also have conservation of energy through this h = = 9h
2g
collision. This condition gives
as before.
1 1 1
(M + m)v 2 = M VM
2
+ mVm2 6. This is a collision problem that has dierent
2 2 2
unknown quantities than those to which you are
These are two equations in the two unknowns accustomed, but it is still solvable. We have two
collisions to study, and the unknowns are the
Vm and VM , since we already know v = 2gh.
We are interested in Vm , which yields the de- neutron mass and the initial and nal speeds of
sired nal height Vm2 /2g of m, but we are not the neutrons. The initial speeds are the same, so
interested in VM . So we plan to eliminate VM by there are four unknowns in total. We have two
solving for it using the rst equation and then collisions, each of which yields two equations
substituting for it in the second. (one for momentum conservation, one for en-
ergy conservation since the collisions are elastic).
Before proceeding with this algebra, it is Therefore the system can be solved uniquely.
convenient to substitute The directions of the scattered neutrons rela-
tive to the incident directions do not represent
$ = m/M additional unknowns, since the maximum recoil
u = Vm /v velocities of the target nucleii will occur when
U = VM /v the collisions take place head-on, with the incom-
ing neutrons bouncing straight back. Thus we
so that all terms are dimensionless. The two can take this to be a one dimensional problem.
equations above become The equations are the following (the energy
equations have been multiplied by 2):
1 $ = U + $u
1 + $ = U 2 + $u2 mn v = mn v  + mH vH mn v 2 = mn v 2 + mH vH
2

mn v = mn v  + mN vN mn v 2 = mn v 2 + mN vN
2
Solving the rst equation for U ,
Solving these equations for mn and v requires
U = 1 $ $u careful algebra. We square the rst momentum
equation to get a relation between v  , mn , and v
Substituting this value for U in the second equa-
tion, (mn v mH vH )2
v 2 =
m2n
1 + $ = 1 2$ + $2 2$u + 2$2 u + $2 u2 + $u2
Now we plug this into the rst energy equation
0 = $(1 + $)u2 2$(1 $)u $(3 $)
1$ 3$ (mn v mH vH )2
0 = u2 2 u mn v 2 = 2
+ mH vH
1+$ 1+$ mn
6

Expanding, MeV/c2 , well within his experimental range.


The range of initial neutron velocity is given by
m2H vH
2
2mn mH vvH + mn mH vH
2
=0
v = 3.07 107 m/sec
Writing this as an equation for v, we get v+ = 2.82 107 m/sec
  v = 4.13 107 m/sec .
1 mH
v= 1+ vH
2 mn 7. K&K problem 4.13. The Lennard-Jones
potential is given by
This is fairly simple result. If we perform the

   r 6
same manipulations on the nitrogen equations, r0 12 0
we will get an analogous result U =$ 2
r r
 
1 mN
v= 1+ vN (a.) We nd the minimum of this potential by
2 mn dierentiating it with respect to r and setting
the results equal to zero:
We can now use these to solve for mn and v.

Equating the right hand sides, we get a single dU 12$  r0 12  r0 6
= =0
equation for the mass. dr r r r

mn vH + mH vH = mn vN + mN vN This is easy to solve:


mN vN mH vH  r 12  r 6
mn = . 0 0
vH v N = r = r0
r r
We can now use this to nd the initial velocity The depth of the potential well is just U (r0 ) =
of the neutrons: $. Thus the potential well has a depth $.

vH  mH (vH vN ) 
(b.) We nd the frequency of small oscillations
v= 1+ by making a Taylor expansion of the potential
2 mN vN mH vH
vH  mN vN mH vN 
about r = r0 . Read section 4.10 in K&K for
= more information on this. We can write the
2 mN vN mH vH potential as follows:
vH vN  mN mH   
= .
2 mN vN mH vH U (r) = U (r0 ) +
dU
(r r0 )
dr r=r0
We want to know the mass of the neutron in  
1 d2 U
amu, so we plug in mH = 1 and mN = 14 + (r r0 )2 +
(greater accuracy is unnecessary, since the recoil 2 dr2 r=r0
velocities are measured only to 10%). We also
We know that dU/dr = 0 at r = r0 , so we drop
look at both boundaries of the nitrogen veloc-
the middle term.
ity, calling these results m and v . Plugging  
in numbers, the values of mn are 1 d2 U
U (r) $ + (r r0 )2
2 dr2 r=r0
mn = 1.159 amu
m+ = 1.415 amu This is exactly the form of the potential of a mass
on a spring. We only have to identify the spring
m = 0.911 amu . constant. Remembering that Uspring = kx2 /2,
we make the identication
Chadwicks experimental work is seen to be re-  2 
liable; todays accepted value for the neutron d U
k=
mass is 1.008665 amu, or 938.27231 0.00028 dr2 r=r0
7

For the Lennard-Jones potential, we already


know the rst derivative, so we need to dieren-
tiate once more.

 
d2 U 12$ r0 12  r0 6
= 13 7
dr2 r2 r r

Plugging in r = r0 , we nd the eective spring


constant for this potential

72$
k=
r02

We now consider two identical masses m on the


ends of this spring. Their (coupled) equations
of motion are:

mr1 = k(r r0 ) mr2 = k(r r0 )

where r = r2 r1 is the distance between the


masses. Subtracting these two equations, we get

mr = 2k(r r0 )

The frequency of oscillation is then 2 = 2k/m.


(Note that we could have obtained the same
result by considering the two-mass system to
be a single mass of reduced mass mreduced =
m1 m2 /(m1 + m2 )). Plugging in the above value
for the eective spring constant k,

$
= 12 2
r0 m
University of California, Berkeley
Physics H7A Fall 1998 (Strovink)
PROBLEM SET 6

1. K&K problem 6.1 Show that if the total (vertical) component of force at the contact be-
linear momentum.... tween ladder and wall is negligible. The foot of
the ladder is placed 6 ft from the wall. The lad-
2. K&K problem 6.3 A ring of mass M and
der, with the mans weight on it, will slip if the
radius R lies....
tangential (horizontal) force at the contact be-
3. Expansion of the previous problem: tween the ladder and ground exceeds 80 lb. How
(a.) Let the azimuth of the bug on the ring be far up the ladder can the man safely climb?
that is, is zero when the bug starts
walking, and 360 when the bug makes one
revolution. Assume (for part (a.) only) that
the ring is xed. Calculate the angular mo-
mentum l of the bug about the pivot in the
previous problem, as a function of . Check
that your result is consistent with what you
used in the previous problem when was
180 .
(b.) Now assume that the bug is xed at some 6. K&K problem 6.8 Find the moment of
azimuth on the ring, but that the ring inertia of a uniform sphere....
itself is not xed, having angular velocity
7. K&K problem 6.14 A uniform stick of mass
about the pivot (opposite to the angular
M and length L is....
velocity of the bug when the bug was mov-
ing). Calculate the angular momentum l of 8. K&K problem 6.18 Find the period of a
the bug as a function of and . pendulum....
(c.) Now assume that neither the bug nor the
ring are xed. By requiring that the total
angular momentum l + l of the bug about
the pivot be balanced by the angular mo-
mentum of the ring, obtain an expression
for the angular velocity of the ring, as a
function of .
(d.) Get an integral expression for the angle
through which the ring rotates, as a function
of time, assuming that d/dt = = con-
stant. You need not evaluate the integral.
Note that the system is bootstrapping its
way around the pivot!
4. K&K problem 6.5 A 3,000-lb car is parked
on a....
5. A man begins to climb up a 12-ft ladder (see
gure). The man weighs 180 lb, and the ladder
20 lb. The wall against which the ladder rests
is very smooth, which means that the tangential
1

University of California, Berkeley


Physics H7A Fall 1998 (Strovink)
SOLUTION TO PROBLEM SET 6
Composed and formatted by E.A. Baltz and M. Strovink; proofed by D. Bacon

1. K&K problem 6.1 (b.) The proof for this part is identical if angu-
lar momentum is replaced by torque and linear
(a.) We know that the total linear momentum
momentum is replaced by force.
of the system is zero. (This would occur, for
example, if we were in the center of mass frame.) 2. K&K problem 6.3
 This problem and the next concern the same
P= pi = 0
i
system that of a bug walking along a hoop
that is free to pivot around a point on its edge.
Examine the total angular momentum of the The hoop lies at on a frictionless surface. The
system. The vector from the origin to point i is ring has mass M and radius R, and the bug has
denoted ri . The angular momentum in general mass m and walks on the ring with speed v.
depends on where the origin is:
The key idea in this problem is conservation
 of angular momentum. About the pivot there is
L= ri pi no net torque on the system, so the total angular
i
momentum about that point is conserved. The
ring starts at rest with the bug on the pivot, and
We now want to nd the angular momentum
the bug starts walking at speed v. Immediately
about a new origin whose position vector is R
after the bug starts walking, the total angular
in the current coordinate system. In this new
momentum measured about the pivot point con-
system, the position vector of point i becomes
tinues to be zero. The ring is not yet moving, so
ri R. Each point has its position changed by
it has no angular momentum; the bug has begun
the same amount. The new value of the angular
to move, but it is at r = 0, so it has no angular
momentum is
momentum yet.

Lnew = (ri R) pi
i

Expanding,
 
Lnew = ri pi R pi
i i

Because R is the same for all points, we can pull


it outside of the sum:
 
Lnew = ri pi R pi
i i

= ri pi R P We want to nd the angular velocity of
i the ring when the bug is opposite to the pivot.
The bug is moving at speed v on the ring, but
We know that P = 0, so we are done. the ring is also moving. The bug is at a distance
2R from the pivot, so the velocity of that por-
Lnew = L tion of the ring which is under the feet of the
2

bug is 2R. The total velocity of the bug is thus extending from the center of the hoop to the bug
v + 2R. Next we need to know the moment of and pivot, respectively. This is an isosceles tri-
inertia of the hoop. A hoop has moment of in- angle, with two equal sides of length R having
ertia I = M R2 about its center of mass. We use an angle between them. If we dene the az-
the parallel axis theorem to nd the moment of imuth of the bug on the hoop to be zero at the
inertia about a point on the edge. pivot, the angle is simply the azimuth of the
bug on the hoop. The length r of the third side
I = ICM + M d2 is found using the law of cosines:

The distance d from the center of mass to the r2 = R2 + R2 2R2 cos r2 = 2R2 (1 cos )
desired axis in this case is just R, so the moment
of inertia of the hoop about a point on the edge Using the trigonometric identity 1 cos =
is I = 2M R2 . We can now nd an expression for 2 sin2 2 , we can get a simple result for r:
the total angular momentum of the system. For

the hoop we use L = I and for the bug we use r = 2R sin
2
L = mvr sin . The angle between the position
vector and the velocity vector of the bug in this Now we know v and r. The only thing left to
case is simply /2, so sin is just 1. We now determine is the angle between the position and
write the angular momenta of the two pieces velocity vectors. The rst step is to nd the an-
gle between the position vector r of the bug and
Lbug = 2mR(v + 2R) Lhoop = 2M R2 the line from the center of the circle to the bug.
The isosceles triangle (like any triangle) has a
Since angular momentum about the pivot is con- total angle , and its central angle is . The re-
served throughout the motion, We know that maining two angles are equal, so they must be
Lbug + Lhoop = 0. This gives the following ex- ( )/2 each. Thus the three angles add up to
pression: radians. Because the velocity vector v is tan-
gent to the circle, the angle between r and v is
2mvR + 4mR2 + 2M R2 = 0 /2 minus this angle. Thus the angle between r
and v is /2. We can now get the angular mo-
We solve this equation for in terms of v and get mentum of the bug. Assuming that the ring is
xed, l = mvr sin from the bug alone, so
mv
=
M R + 2mR
l = 2mvR sin2
2
Note that the minus sign means that the hoop
rotates in a direction opposite to that in which (b.) In this part we assume that the ring is ro-
the bug moves. This makes sense because the tating with angular velocity , but that the bug
total angular momentum about the pivot point is xed on the ring. The velocity of the bug is
must vanish. just r, where r is the same as was calculated in
3. We now study the bug and hoop system in the previous part. The velocity in this case is al-
more detail. See the diagram in the previous ways perpendicular to its position vector. This
problem. can be seen by remembering that the bug isnt
moving on the ring, so it must be in uniform
(a.) In this part we assume that the ring is xed.
circular motion about the pivot, with a velocity
We want to calculate the angular momentum of
that is tangent to its present position. Therefore
the bug about the pivot point. The rst step is
the angular momentum l of the bug is simply
to nd the distance r between the bug and the
mvr, yielding
pivot. We can do this using the law of cosines.
Consider the triangle made by the line between
the bug and the pivot, and the two radial lines l = mr2 = 4mR2 sin2
2
3

(c.) We now allow both the bug and the ring 4. K&K problem 6.5
to move. The total angular momentum of the
bug is l + l from parts (a.) and (b.) respectively. A car of mass m is parked on a slope of angle
To this we must add the angular momentum of facing uphill. The center of mass is a distance d
the ring to get the total angular momentum of above the ground, and it is centered between the
the system. From problem 2. we know that the wheels, which are a distance l apart. We want
total angular momentum must be zero. The an- to nd the normal force exerted by the road on
gular momentum of the ring is I, so we get the the front and rear tires.
following equation. It is easiest to do this problem choosing the
origin as the point on the road directly below the

4mR2 sin2 + 2mvR sin2 + 2M R2 = 0 center of mass. About this point there are three
2 2 torques. The normal force on the front (Nf ) and
We solve this for in terms of . The result is rear (Nr ) set of wheels provides a torque, and
also gravity provides a torque mgd sin because

mv sin2 2
the car isnt horizontal. However, the forces of
= friction on the tires dont provide any torque be-
M R + 2mR sin2 2 cause they are in line with the direction to the
This agrees with the result of problem 2. when origin. The torque from the front wheels and the
the bug is at = , opposite to the pivot. torque due to gravity tend to want to ip the
car over backwards, while the torque on the rear
(d.) Finally, we want to nd an expression for wheels opposes this tendency. We want the sum
the angle through which the ring rotates. We of the torques to vanish, because the (static) car
know that is related by a simple dierential is not undergoing any acceleration, angular or
equation to the angular velocity of the hoop linear:
d
= l l
dt 0 = Nf + mgd sin Nr
2 2
but we want to express in terms of so that 2mgd
Nr N f = sin
we can use the fact that Rd/dt, the speed v of l
the bug with respect to the rim of the hoop, is
constant. We apply the chain rule to get We can get one more condition from the fact
that the car is not undergoing linear accelera-
d d d tion perpendicular to the road. This means that
= =
dt d dt the normal forces exactly cancel gravity:

Substituting d/dt = v/R, where v is constant,


we can write an integral for : Nr + Nf = mg cos


R mv sin2 ( /2) We can take the sum and dierence of these
= 2  d two equations to get expressions for Nf and Nr .
v o M R + 2mR sin ( /2)
These are
We can simplify this a little, but doing the inte-
1 d 
gral is hard, which is why you werent asked to
Nr = mg cos +
sin
evaluate it. Setting the initial bug azimuth 0 2 l
to zero and using the fact that d/dt = v/R is a 1 d 
Nf = mg cos sin
constant so that = vt/R, 2 l
 vt/R
sin2 ( /2) Plugging in = 30 , mg = 3000 lb, d = 2 ft, and
(t) = d
0 (M/m) + 2 sin2 ( /2) l = 8 ft, we get Nr =1674 lb and Nf =924 lb.
4

5. We will solve this problem symbolically and Requiring these three torques to sum to zero,
wait until the end to plug in numbers. This
is always good practice because it makes it a 0 = M + m + w
lot easier to check the units of the result and to H
explore whether the result is reasonable when the = M gh sin + mg sin Ffmax H cos
2
inputs have limiting values. We take M to be the
mass of the man (M g = 180 lb) and m to be the Solving for h,
mass of the ladder (mg = 20 lb). The length H
of the ladder is 12 ft, and its point of contact with Ffmax H cos mg H2 sin
h=
the wall is d = 6 ft from the wall. The angle that M g sin
max
it makes with the wall is = arcsin (d/H) = 30 . h Ff m
Finally, the force of friction on the ladder from = cot
H Mg 2M
the ground is Ff Ffmax , where Ffmax = 80 lb. (M + m) m
= cot
M 2M
There are ve forces to consider in this (M + m) cot (m/2)
problem. They are the two normal forces on =
the ladder, Ng from the ground and Nw from M
0.4(200) 3 10
the wall; the force Ff of friction at the base of =
the ladder; and the two forces of gravity, M g 180
on the man and mg on the ladder. This is = 0.7142
a torque balance problem, so choosing a good h = 8.571 ft
origin makes it a lot easier. With this choice of
the point of contact with ground, two of the ve 6. K&K problem 6.8
forces contribute no torque about that point.
Not bad! As a sanity check we evaluate , the Because of the spherical symmetry, we work in
coecient of friction between the ladder and the spherical polar coordinates. To nd the moment
ground. The normal force Nf from the oor is of inertia we need to evaluate the integral
equal and opposite to (M + m)g, the sum of the 
2
weights of the ladder and the man. We are given I = r dv
the maximum frictional force Ffmax , and we know
that Ffmax = N , so = Ffmax /((M + m)g) = where in these coordinates r = r sin is the
80/200 = 0.4, a reasonable value. perpendicular distance to the axis and dv =
r2 dr d(cos ) d is the element of volume. The
We now calculate the torques. To nd the integral to evaluate is thus
maximum height h to which the man can climb
R 1  2
without the ladder slipping, we assume that the
0
r4 dr 1 sin2 d(cos ) 0 d
ladder is about to slip. This means that the nor- I=M R  
r 2 dr 1 d(cos ) 2 d
mal force Nw from the wall is equal and opposite 0 1 0
to Ffmax , exactly countering the maximum force
of friction: since these two forces are the only where the denominator is the volume V of the
forces in the horizontal direction they must sum sphere, needed to evaluate its density = M/V .
to zero. The torque from the wall is then w = Substituting u r/R,
Ffmax H cos , where the minus sign indicates 1 1  2
that this torque pushes clockwise. The torque I 0
u4 du 1 sin2 d(cos ) 0 d
= 1 1  2
from the weight of the ladder is exerted at the M R2 u2 du 1 d(cos ) 0 d
0
midpoint of the ladder, its center of mass. The
value of this torque is m = mg H2 sin . Similarly, In both the numerator and the denominator, all
the torque exerted by the weight of the man, who three integrals have limits that do not depend
is a distance h up the ladder, is M = M gh sin . on the other variables, so each integral can be
5

evaluated independently. The integrals cancel, where the minus sign indicates that the accel-
and the u integrals have the ratio eration is downward. Finally we use Newtons
second law to nd the normal force at B. We
1/5 3 know the acceleration and we know the force of
=
1/3 5 gravity, so this is a simple equation
The integrand in the cos integral in the numer-
ator can be rewritten 3 1
N Mg = Mg N = Mg
4 4
sin2 d(cos ) = (1 cos2 )d(cos )
1 
= d(cos ) d cos3 where the positive direction is up, opposite to
3 the force of gravity.
Therefore the ratio of the integrals is
8. K&K problem 6.18
2
2 3 2
= We want to nd the equation of motion of
2 3 the pendulum to determine the frequency. We
Putting it all together, will use the torque equation = I. If we
choose the pivot point of the pendulum as the
3 2 2
I = M R2 = M R2 origin, only one force provides torque, the force
5 3 5 of gravity. It acts on the center of mass of the
pendulum, a distance lcm from the pivot point.
7. K&K problem 6.14 The magnitude of this force is just (M + m)g.
When the stick is released, there are two Thus the total torque is
forces acting on it, gravity at the midpoint, and
the normal force at the point B. We use the
= (M + m)glcm sin
point B as the origin, so the only torque about
this point is provided by gravity. At the mo-
ment of release, the stick is still horizontal, so where is the angular position of the pendulum.
the torque is Writing the torque equation and approximating
M gl sin , we get
B =
2
where the minus sign indicates that the torque I = (M + m)glcm
pulls clockwise. We know that the moment
of inertia of a thin stick about its endpoint is
I = M l2 /3, so we can easily nd the angular We recognize that this is the equation for a sim-
acceleration from = I. ple harmonic oscillator. The angular frequency
and period are thus
M gl 1 3g
= M l2 =
2 3 2l  
(M + m)glcm I
The vertical acceleration of the center of mass = T = 2
I (M + m)glcm
is given by the simple formula a = r, where
r is the distance between the center of mass
and point b, about which the stick is (instan- All that is left is to evaluate lcm and I. The
taneously) executing circular motion. (This is equation for the center of mass is easy to use.
analogous to the expression v = r.) Here this The center of mass of the rod is halfway along
distance is r = l/2. This gives the acceleration its length, and the disk is a distance l from the
of the center of mass: pivot, so
3 ml/2 + M l
a= g lcm =
4 M +m
6

This expression simplies the formula for the


period 
I
T = 2
(M + m/2)gl

In the rst case, where the disk is tied to the


rod, the moment of inertia is determined using
the parallel axis theorem. The disk is xed to
the rod, so, as the rod pivots, the disk must ro-
tate at the same angular velocity. The moment
of inertia of a stick about its end is ml2 /3. The
moment of inertia of a disk about its center is
M R2 /2. Because the center of mass is displaced
a distance l from the origin, the parallel axis
theorem tells us that the total moment of iner-
tia of the disk is M R2 /2 + M l2 . Thus the total
moment of inertia of the pendulum is

1 1
I = M R + M + m l2
2
2 3

This gives the period of oscillation



M R2 /2 + (M + m/3)l2
T = T = 2
(M + m/2)gl

In the second case, where the disk is free to rotate


on the rod, the moment of inertia is smaller.
Because the disk is not xed, it has no tendency
to rotate. Its eective moment of inertia about
the center of mass is thus zero. For our purpose
it is the same as a point mass a distance l away
from the origin. The new moment of inertia is

1
I = M + m l2
3

The period of oscillation in this case is also


smaller: 
(M + m/3)l
T = 2
(M + m/2)g
This is the same as our answer for the rst case
in the limit R 0.
University of California, Berkeley
Physics H7A Fall 1998 (Strovink)
PROBLEM SET 7

1. A trick cyclist rides his bike around a wall energy occurs.


of death in the form of a vertical cylinder (see
(c.) At what radial distance from the axis of ro-
figure). The maximum frictional force parallel
tation do the men experience the greatest
to the surface of the cylinder is equal to a frac-
centrifugal force as they make their way to
tion of the normal force exerted on the bike by
the center?
the wall. Assume that the cyclist and his bike
are small relative to the radius of the cylinder. 5. K&K problem 7.4 In an old-fashioned rolling
mill, grain....
6. K&K problem 7.5 When an automobile
rounds a curve....
7. K&K problem 8.2 A truck....
8. K&K problem 8.4 The center of mass....

(a.) At what minimum speed must the cyclist


go to avoid slipping down?
(b.) At what angle to the horizontal must he
be inclined at that minimum speed?
(c.) If =0.6 (typical of rubber tires on dry
roads) and the radius of the cylinder is 5
m, at what minimum speed must the cyclist
ride, and what angle does he make with the
horizontal?
2. K&K problem 6.24 Drum A of mass M and
radius R....
3. K&K problem 6.27 A yo-yo of mass M has
an axle....
4. Two men, each of mass 100 kg, stand at oppo-
site ends of the diameter of a rotating turntable
of mass 200 kg and radius 3 m. Initially the
turntable makes one revolution every 2 sec. The
two men make their way to the middle of the
turntable at equal rates.
(a.) Calculate the final rate of revolution and
the factor by which the kinetic energy of
rotation has been increased.
(b.) Analyze, at least qualitatively, the means
by which the increase of rotational kinetic
1

University of California, Berkeley


Physics H7A Fall 1998 (Strovink)
SOLUTION TO PROBLEM SET 7
Composed and formatted by E.A. Baltz and M. Strovink; proofed by D. Bacon

1. We will need to use ctitious forces to We choose the point of contact as the origin.
solve this problem easily. Fictitious forces are There are two torques, g caused by gravity and
never necessary, but they often simplify prob- c caused by the centrifugal force. Both act at
lems greatly. the center of mass. (This is important to note:
ctitious forces always act at the center of mass!)
We are assuming that the size of the cyclist l is
very small in comparison with the radius R, so

mlv 2
g = +mgl cos c = sin
R
We want to know the angle where the cyclist is
about to slip, so the normal force is mg/, equal
to the centripetal force mv 2 /R. Substituting for
mv 2 /R in the above equation,
(a.) We want to know the minimum speed the
cyclist needs not to slip down the side. The 1
mgl cos = mgl sin tan =
force of static friction must be mg to hold him
up, so we require that N = mg. The force of
static friction can be less than N , but we are So when the cyclist is about to slip, he rides at
setting it to the maximum to see what the limit an angle
is. Thus we need N > mg/. The only force = tan1
acting horizontally in the system is the normal
(c.) Taking = 0.6 and R = 5 meters, we nd
force, so it must entirely provide the centripetal
that the cyclist must ride at a speed of least 9.0
acceleration, which is v 2 /R. We thus obtain
meters per second, or 29.5 ft/sec, or 20.1 mph.
 On a road bike this is a mellow cruising speed.
mv 2 mg gR At this minimum speed, the angle the cyclist
v
R must make with the horizontal is 31 degrees. We
caution you not to try this at home; its tough
to get up to speed without crashing!
(b.) We now need to consider the ctitious
centrifugal force. The cyclist is in an acceler- 2. K&K problem 6.24
ating frame of reference because he is moving This problem is similar to many pulley problems
in a circle. To correctly apply Newtons second that you have seen before. We need to apply
law in the cyclists frame, we must introduce both Newtons second law and the torque equa-
the centrifugal force, which points outward with tion to solve it. We denote the (positive down-
magnitude mv 2 /R. In the frame of the cy- ward) acceleration of the falling mass as a. There
clist there are four forces: the normal force and are two forces on it in the vertical direction, ten-
the centrifugal force cancel each other, and the sion and gravity. Newtons second law requires
friction and gravity cancel each other. This is
required to insure that, by denition, the cyclist Mg T = Ma
isnt accelerating in his own frame of reference.
The problem is now reduced to a torque balance We now need to apply the torque equation to
to nd the angle at which the cyclist is stable. both drums. For each drum we choose its own
2

center as the origin. Each drum feels only one assumption of a pure downward motion. There-
torque, the torque from the tension. This has a fore the actual motion will be more complicated
magnitude = T R in both cases. Notice that than this problem asks you to assume.
both of these torques have the same sign, thus 3. K&K problem 6.27
the drums will tend to angularly accelerate in
the same direction. Writing the torque equation We need to apply both Newtons law and the
for each drum, with angular accelerations 1 for torque equation. The forces on the yo-yo hori-
the top drum and 2 for the bottom drum, zontally are the force F and the friction f . The
vertical forces are the normal force and grav-
T R = I1 T R = I2 ity, which immediately tell us that N = M g.
We want to nd the maximum force we can
where the moments of inertia of both disks are apply with the yo-yo not slipping. It is impor-
the same, I = M R2 /2. From these equations it tant to note that the force of friction, which
is easy to see that 1 = 2 = T R/I, so the stops the disk from slipping, is controlled by the
angular accelerations are both coecient s of static friction because the sur-
face of a rolling wheel is at rest with respect
2T to the ground. Since we are concerned with
= the maximum allowed force, we will consider
MR
the maximum allowed friction, which is s N .
We now need to nd a relation between a and . Newtons law gives us
The linear acceleration due to each disk is given
simply by a = R. There are two disks, both F s M g = M a
unwinding with the same angular acceleration ,
so the linear acceleration of the bottom one is The moment of inertia of the yo-yo is I =
just a = 2R. The previous equation becomes M R2 /2. Because we want the yo-yo to roll with-
out slipping, we can use a = R. The torque
4T Ma equation gives us
a= T =
M 4
1
Plugging this into the very rst equation that s M gR F b = I = M Ra
2
we got from Newtons law, we nd the initial ac-
celeration of the drum, assuming that it moves We want to solve these two equations for F , the
straight down, to be maximum allowed force. Eliminating a, we get

4 b
a= g F s M g = 2s M g 2F
5 R

Will the drum in fact move straight down? For Solving for F , we get
the moment assuming that the answer is yes,
consider a (downward accelerating but nonrotat- 3R
F = s M g
ing) frame with its origin at the (instantaneous) R + 2b
point of tangency between the lower drum and
Since R > b the applied force F is always larger
the tape. In this frame, the CM of the drum ex-
than the frictional force, so the yo-yo always
periences a downward force mg and an upward
accelerates to the right.
ctitious force 45 mg which does not quite com-
pensate it. Therefore it feels a net downward 4. We will solve this problem symbolically and
force 15 mg. About the chosen origin this force plug in numbers at the end. This is always a
causes a net (clockwise) torque, which causes good practice because it makes it a lot easier to
the lower drum to swing to the left like a pen- go back and check your work for correct dimen-
dulum bob in this frame. This contradicts our sions and reasonable results for limiting cases.
3

(a.) Let the disk have mass M and radius R, (b.) The extra kinetic energy comes from the
and the two men each have mass m. If the men work that the men must do against the (citious)
are momentarily at a radius r from the center of centrifugal force to make their way from the edge
the disk, the total moment of inertia is given by of the turntable to the center. This is the qualita-
tive statement which the problem requests. Op-
1
I(r) = M R2 + 2mr2 tionally, one can perform a quantitative analysis:
2
Each man pushes against the centrifugal
The initial angular velocity of the disk is 0 , so, force to get to the center. The work they do
when r = R, the initial angular momentum of is converted to rotational kinetic energy. The
the system is centrifugal force on each man is given by
 
1  2
L = I0 = M + 2m R2 0
2 2 1 + 4m/M
Fc = m((r)) r = m02 r
1 + 4mr2 /(M R2 )
There are no net torques acting on the system,
so L is conserved. We can use L conservation,
The work done is just Fc integrated from zero
I(r)(r) = I(R)0 , to obtain the angular veloc-
to R, doubled since each man does the same
ity of the system as a function of the radius of
amount of work.
the men:
 2 
M R2 + 4mR2 4m R
r dr
(r) = 0 W = 2m02 1+
M R2 + 4mr2 M (1 + 4mr
2
2
0 M R2 )
The nal angular velocity  is just the an-
gular velocity when the men reach the center, You can look this up in a table, or notice that
(r = 0). the top is proportional to the derivative of the
  bottom, so antidierentiating is not too hard:
 M + 4m 4m
= 0 = 1+ 0
M M  2  R
4m M R2 /8m
W = 2m02 1+ 2
Plugging in values, we nd that the nal angular M (1 + 4mr
M R2 )
2
0
velocity is 1.5 revolutions per second.
The factor by which the kinetic energy has Evaluating this, we get
increased is
Kf 1
I(0) 2  2  

= 12 2 M R2 02 4m 4m
Ki 2 I(R)0
W = 1+ 1+
4 M M
Evaluating this, we nd
1
 2 2 Multiplying this out,
Kf 4MR
2
1 + 4m
M 0
= 1 2 2 2
Ki 2
4 M R 0 mR 0
 
M R2 02 4m 16m2
W = +
Simplifying, 4 M M2

Kf 4m
=1+ Simplifying,
Ki M
4m
K = Ki 4m2 2 2
M W = mR2 02 + R 0
 M
For the masses in the problem, Kf /Ki is equal 4m
= 1+ mR2 02
to 3. The rotational kinetic energy is tripled. M
4

This is 4m/M times the initial kinetic energy, so, in magnitude. Since angular velocity is a vector,
as expected, W is equal to the kinetic energy we can add these separate components to obtain
gain K that we already calculated. the full angular velocity vector. In cylindrical
coordinates, with z pointing along the axis of
(c.) We want to nd where the maximum cen-
the orbit,
trifugal force is felt. This is just a maximization
problem. Dierentiate Fc with respect to r and R
= r + z
set it to zero, and also check the endpoints. b
 
dFc d (1 + 4m/M )2 where the leading minus sign tells us that the
= m02 r =0
dr dr (1 + 4mr2 /M R2 )2 radial component of is negative, i.e. the mill-
stone is rotating clockwise about its horizontal
This gives axle. To calculate the angular momentum, we
16mr2 1 choose as an origin the intersection of the cen-
1 =0 terline of the vertical shaft and the centerline
M R2 1 + 4mr2 /M R2
of the horizontal axle. Both the shaft and the
We can solve this for r. Set x = 4mr2 /M R2 . axle are parallel to mirror symmetry axes of the
Then 1 (4x/(1 + x)) = 0 so x = 1/3, and millstone; thus we expect that the component
of angular momentum due to will be parallel
4mr2 1 M to , and the component of angular momentum
= r=R
MR 2 3 12m due to will be parallel to . More quanti-
tatively, the component Lz along z is equal to
If we plug in the
mass values for this problem, we (I  + M R2 ), where I  = 14 M b2 is the moment
obtain r = R/ 6. Since the centrifugal force is
of inertia of a disk about a diameter and M R2
everywhere positive, and it is zero at the center,
is added to I  by use of the parallel axis theo-
this extremum must in fact be the maximum.
rem. Since Lz is constant, no torque is required
5. K&K problem 7.4 to maintain it and we dont need to consider it
further. To calculate the radial component Lr
of the angular momentum, we need I = M b2 /2,
the moment of inertia of a disk about its center:

1
Lr = M b2
2

where the minus sign again reminds us that the


millstone is rolling clockwise about its horizontal
axle. Remember that, in cylindrical coordinates,
the only unit vector which is constant is z; the
radial and azimuthal unit vectors depend on .
Even though the magnitude of Lr is constant,
its direction is changing. In a time increment dt,
the azimuth of the millstone axle with respect
to the shaft changes by an angular increment
Referring to the diagram, the stone orbits around
d = dt. This causes L to change by
the vertical shaft with orbiting angular velocity
. The velocity of the stones CM is thus
v = R. The stone is rolling without slipping on dL = Lr d
the at surface, so its rolling angular velocity is = Lr dt
v R 1
= = = M b2 dt
b b 2
5

The torque is thus the direction of its angular momentum would


not change as the car turns left or right. So
dL 1
= = M b2 the ywheel should be vertical with its spin
dt 2 axis pointing either sideways or forward. Decid-
ing between these alternatives requires a more
Finally we consider the forces on the sys- quantitative analysis.
tem. The vertical shaft exerts a force on the
horizontal axle, gravity pulls down on the mill-
stones CM, and the normal force pushes up on
the millstone. However, with respect to the cho-
sen origin, the rst of these forces can exert no
torque because it is applied at r = 0. The torque
due to gravity is in the + direction, and the
torque due to the normal force N of the at
surface on the millstone is in the direction.
Thus, in the + direction, we have
1
N R + M gR = = M b2
2
1 b2 Lets look at the car from the rear while it
M g+ = N is in motion with speed v. First well consider
2R
the car without any ywheel. Suppose that the
Substituting = R/b, car is in the process of turning to the left, tak-
  ing a turn of radius R0 . Adopt a reference frame
1 2 attached to the car, with an origin halfway be-
N = M g + b
2 tween the tires at the level of the road. It is easy
Of course, by Newtons third law, the contact to see why the act of turning causes the nor-
force exerted by the millstone upon the at sur- mal forces on the tires to become unbalanced.
face is equal and opposite to N . As advertised, The sum of the torques on the car must remain
the eective weight of the millstone for crushing zero if the car (assumed to have no suspension
grain is greater than M g; this increment rises system, so it doesnt lean) keeps all four tires
quadratically with the angular velocity. on the road. With respect to the origin chosen,
the forces of friction on both tires can exert no
What keeps the millstone from accelerat- torque, because these forces act directly toward
ing upward, since the upward normal force on it or away from the origin. Neither can the force of
is greater than the downward force of gravity? gravity exert a torque about this origin, for the
The force of the vertical shaft on the horizontal same reason. That leaves Nl and Nr , the nor-
axle, which we ignored in the torque equation mal forces on the left and right sets of tires, and
because it is applied at the origin, must push M v 2 /R0 , the ctitious centrifugal force which
downward, in alignment with gravity, with the pulls the CM to the right in this accelerating
value M b2 /2. frame. Let the CM be a distance d above the
Such millstones must have been in use be- road; let the right-left separation of the wheels
fore the time of Newton, so the benets of their be 2D. Along v, the sum of the torques is then
increased eective weight when rolling in a cir-
cle must have been discovered empirically rather M v2
Nl D + Nr D d=0
than logically. R0
6. K&K problem 7.5 Clearly Nr must exceed Nl if this equation is to
(a.) If the ywheel were horizontal with its spin be satised. This is the problem we are trying
axis pointing up, it would have little eect, since to solve with the ywheel.
6

The torques that we just considered were (a.) The acceleration of the truck is A, and
along v. If the ywheel is to help, its an- the mass and width of its rear door are M and
gular momentum L should be directed so that, w. The door starts fully open. The door can
when the car turns left, the ywheel produces be thought of as a series of thin sticks, pivoted
a torque on the car equal to +M v 2 d/R0 along about their ends. The moment of inertia of the
v. By Newtons third law, the torque of the door is thus
1
car on the ywheel should correspondingly be I = M w2
equal to +vM v 2 d/R0 . So, as the car turns left, 3
the change in L of the ywheel should be di- The easiest way to nd the angular velocity of
rected along +v. This will happen if the angular the door is to use work and energy. The rota-
momentum vector of the (vertical) ywheel is tional kinetic energy is given by
pointing to the right. This means that the y- 1 2
wheel should rotate in the opposite direction as K= I
2
the tires. For simplicity, well install it at height
d from the road so as not to perturb the CM. The work done by a torque on a system is given
by 
Its not necessary to reverse the ywheel di-
rection for right as opposed to left turns, because W = d
both the centrifugal force and the change in L
will correspondingly reverse direction. In this system there is one torque of interest.
We use the hinge of the door as the origin, so
(b.) Now that we have determined the ywheel the only torque comes from the ctitious force of
direction, we can calculate the desired magnitude acceleration. When the door has swung through
L of the ywheels angular momentum. We have an angle , this torque is given by

dL 1
= L = M v 2 d/R0 =
2
M Aw cos
dt
Note that w/2 is just the distance to the center
where = v/R0 is the angular velocity of the of mass. From this we can easily calculate the
car around the turn. Solving, work done from 0 to 90 degrees.
 /2
L = M vd 1 1
W = M Aw cos d = M Aw
0 2 2
For a disk-shaped ywheel of mass m and ra-
dius r, I = mr2 /2, and the ywheels angular Substituting the expression for rotational kinetic
velocity should be energy, we nd the angular velocity of the door
after it has swung through 90 :
2M vd
= 1 2 1 3A
mr2 I = M Aw =
2 2 w
This is independent of the turn radius R0 , which
is very nice. Weve achieved perfectly at cor- (b.) The force on the door needs to do two
nering for a turn of any radius! Unfortunately, things. It needs to accelerate the door at a rate
depends linearly on the velocity v of the car. A, and it needs to provide the centripetal ac-
So, unless we can come up with a quick easy way celeration to make the door rotate. At =90
of varying the kinetic energy of a big ywheel in degrees, the torque is zero, so the angular veloc-
concert with the square of the cars speed, were ity is not changing. Instantaneously, the door is
not going to get rich installing these devices as in uniform circular motion. The force required
high-performance vehicle options. to accelerate the door is just

7. K&K problem 8.2 FA = M A


7

The force required to provide the centripetal


acceleration is
w
Fc = M 2
2
The total force is the sum of these, and they act
in the same direction. We substitute the value
for from part (a.) to get
3 5
F = FA + Fc = M A + M A = M A
2 2
8. K&K problem 8.4
(a.) This is a torque balance problem. A car of
mass m has front and rear wheels separated by a
distance l, and its center of mass is midway be-
tween the wheels a distance d o the ground. If
the car accelerates at a rate A, it feels a ctitious
force acting on the center of mass. This tends to
lift the front wheels. When the front wheels are
about to lift o the ground, the normal force on
the front wheels, Nf is zero. This means that the
normal force on the back wheels Nb must equal
the weight of the car, Nb = mg. The simplest ori-
gin to use in this problem is the point on the road
directly under the center of mass. Here there are
only three torques, due to the two normal forces
and ctitious force. The torque from Nb exactly
balances the torque from the ctitious force when
the wheels are about to lift, so we have
1 1 l
Nb l = mAd = mgl A = g
2 2 2d
For the numbers given, A = 2g = 19.6 m/sec2 .
(b.) For deceleration at a rate g, again we sim-
ply apply torque balance about the same origin.
We also need the fact that
Nf + Nb = mg
Torque balance gives
1 1
Nf l Nb l mgd = 0
2 2
Substituting from the previous equation, we get
 
1 d
Nf = + mg
2 l
 
1 d
Nb = mg
2 l
Plugging in the numbers, we get Nf = 3mg/4 =
2400 lb, and Nb = mg/4 = 800 lb.
University of California, Berkeley
Physics H7A Fall 1998 (Strovink)
PROBLEM SET 8

1. K&K problem 8.5 Many applications.... motion that its angular momentum with respect
to the cloud is not changing. What attractive
2. K&K problem 8.11 A high speed hydro-
(central) force could account for such an orbit?
foil....
3. K&K problem 9.3 A particle moves....
4. K&K problem 9.4 For what values of n....
5. K&K problem 9.6 A particle of mass m....
6. K&K problem 9.12 A space vehicle is in
circular orbit....
7. A satellite of mass m is travelling at speed
V in a circular orbit of radius R under the
gravitational force of a xed mass at the origin.
a. Taking the potential energy to be zero at
innite radius, show that the total mechanical
energy of the satellite is mV 2 /2.
b. At a certain point B in the orbit (see g-
ure), the direction of motion of the satellite is
suddenly changed without any change in the
magnitude of the velocity. As a result, the
satellite goes into an elliptical orbit. Its closest
approach to the origin is now R/5. What is the
speed of the satellite at this distance, expressed
as a multiple of V ?
c. Through what angle (see gure) was the
velocity of the satellite turned at point B?

8. The commander of a spaceship that has shut


down its engines and is coasting near a strange-
appearing gas cloud notes that the ship is follow-
ing a path that will take it directly into the cloud
(see the gure). She also deduces from the ships
1

University of California, Berkeley


Physics H7A Fall 1998 (Strovink)
SOLUTION TO PROBLEM SET 8
Composed and formatted by E.A. Baltz and M. Strovink; proofed by D. Bacon

1. K&K problem 8.5 A hydrofoil moves with respect to the earths


surface at the equator with velocity v = 200
A gyroscope with mass M has angular veloc-
mi/hr directed along each of the four points of
ity s and moment of inertia Is . It pivots at
the compass. At rest with respect to the surface
one end, and the center of mass is a distance l
of the earth, the acceleration of gravity is g. We
from the pivot. The angular momentum of the
are asked to nd the eective gravitational ac-
gyroscope is thus
celeration g that is felt by a passenger (of mass
m) who is at rest with respect to the hydrofoil.
L = Is s
The (ctitious) Coriolus force on the pas-
The gyroscope undergoes an acceleration a per- senger, which is proportional to the passengers
pendicular to the spin axis. The ctitious force (vanishing) velocity in the hydrofoils frame,
will create a torque of magnitude must be zero if evaulated in this frame. The
(ctitious) centrifugal force on the passenger is
= M al Fcent = m  (  R)
= m(  (  R) R(   ))
The direction of this torque is perpendicular to
both the acceleration and the gyroscope axis = m(  (  R) + R 2 )
(down in the gure), causing the gyroscope axis where  (v) is the total angular velocity of the
to precess in the direction indicated by the an- passenger, due both to the rotation of the earth
gle . The magnitude of the angular momentum and to the motion of the hydrofoil; R = Rx is
will not change, but the direction will. Thus the a vector pointing from the earths center to the
gyroscope axis will rotate around the direction hydrofoil at the equator; and the bac cab rule
of acceleration. The rate at which this happens is applied to the rst line. The hydrofoils ve-
is , and locity has one of the four directions (E,W,N,S)
dL
= L = = (y, y, z, z), yielding an angular velocity
dt due to hydrofoil motion relative to the earths
This gives the following relation surface:
v
= (z, z, y, y)
M la(t) = Is s (t) R
To this one must add the earths angular velocity
Both the acceleration and the angular velocity = z
can depend on time. If we integrate both sides
of this equation, we can get a relation between in order to get the total angular velocity  =
the nal velocity and the total angle of rotation. + of the hydrofoil. Evidently  is perpen-
The integral of the acceleration is just the ve- dicular to R, so
locity and the integral of the angular velocity is Fcent (v) = xmR( (v))2
just the angle:
Therefore g points along x, i.e. toward the
Is s earths center, for all four directions (E,W,N,S)
M lv = Is s v = of v, as does g. Thus
Ml
g g g R(( 2 2 )
2. K&K problem 8.11 =
g g g
2

For these four directions, Since Ue is constant, d2 r/dt2 = 0, so if the par-


ticle acquires a nonzero radial velocity it will
 2 2 = 2 + 2 (E) continue with the same radial velocity. If the
= 2 + 2 (W) particle moves with uniform radial velocity vr ,
the following equations are satised
= 2 (N and S)
dr d L
Therefore = vr =
dt dt mr2 (t)
g R
= (2 2 ) (E) Solving the rst is easy: r(t) = r0 +vr t. Plugging
g g this result into the second equation, we nd
R
= (2 2 (W) d L
g =
R dt m(r0 + vr t)2
= ( 2 ) (N and S)
g
We can solve this equation by direct integration,
2 assuming that (0) = 0:
Using / = 0.1931 and R /g = 0.003432, we
calculate |g/g| = 0.001325 from the 2 term  t  
L dt L 1 1
and |g/g| = 0.000128 from the 2 term. Thus (t) = 2
=
0 m(r0 + vr t) mvr r0 r(t)
g/g = 0.001453 (E)
We replace L with 2mA to get the nal answer
= +0.001197 (W)
  
= 0.000128 (N and S) 1 2A 1 1
(t) =
vr m r 0 r(t)
3. K&K problem 9.3
4. K&K problem 9.4
A particle moves in a circle under the inuence
of an inverse cube law force. This means that the A particle moves in a circular orbit in the po-
potential is inverse squared and it is attractive. tential U = A/rn . We want to know for which
The eective potential is given by values of n the orbit is stable. The eective
potential is given by
L2 A
Ue (r) = 2
2 L2 A
2mr r Ue = n
2mr 2 r
The radial force is zero for a circular orbit, so
we can nd the radius. To nd the circular orbit radius we evaluate
dUe /dr = 0:
dUe L2 2A
0= = 3 + 3 dUe L2 nA
dr mr r = 3 + n+1
dr mr r
This shows that a circular orbit can have any ra-
dius, but there is only one possible magnitude of This gives the radius of the circular orbit r0
angular momentum, given by when we set it to zero.
nAm
L2 = 2Am r0n2 =
L2
Plugging this value of the angular momentum Since r0n2 must be a positive quantity for any
into the eective potential, we nd the peculiar value of n, and A > 0, this equation requires
result that
Ue = 0 n>0
3

We now look at the second derivative of the To nd the frequency of small radial oscillations,
eective potential at r = r0 . If it is positive, then we must evaluate the second derivative of the
it is a potential minimum and the orbit is stable. eective potential. Remember that for small
oscillations the eective spring constant k for
d2 Ue 3L2 n(n + 1)A radial motion is
2
= 4

dr mr rn+2 

1 3L 2
n(n + 1)A  d2 Ue 
k=
= 4
r m rn2
>0 dr2 r0

Since r is always greater than zero we can divide The eective potential is
it away. Substituting for rn2 at r = r0 ,
l2 1
3L 2
(n + 1)L 2 Ue = 2
+ Kr5
>0 n<2 2mr 5
m m
The second derivative is easily found
Putting both inequalities together,
d2 U 3l2
0<n<2 = + 4Kr3
dr2 mr4

Recall from the previous problem that when n = Plugging in l2 = m2 v 2 r2 = mKr07 , we get
2 the motion is barely unstable. When n = 0, U

is constant, so there is no attractive force, there- d2 U 
= 7Kr03
fore no circular orbit: this case is also unstable. dr2 r0
5. K&K problem 9.6
Substituting the value of r0 , we nd the eective
A particle moves in an attractive central force
spring constant k and the angular frequency of
Kr4 with angular momentum l. If it moves in
radial oscillation about the stable circular orbit:
a circular orbit with radius r0 , the central force
must provide exactly the necessary centripetal  l2 3/7
acceleration: k = m 2 = 7K
 mK
 2 3/14
mv 2 l2 l2 7K l
= Kr04 = r 7
0 = =
r0 mr03 mK m mK

Relative to r = 0, the energy of the orbit is 6. K&K problem 9.12


1 1 A spacecraft of mass m orbits the earth at a ra-
E= mv 2 + Kr05 dius r = 2Re . It will transfer to another circular
2 5
orbit with radius r = 4Re .
where we have integrated the force to get the
(a.) We know the radius of each orbit, so we can
potential. Plugging in v 2 = Kr05 /m, we get
easily nd the energies of the two orbits. The
1 1 7 energy of a bound orbit in a 1/r potential is
E= Kr05 + Kr05 = Kr5 given by
2 5 10 0
GM m
E=
Substituting the above value for r0 , we get the A
nal result for the energy (relative to r = 0): where A is the major axis of the elliptical (here
 5/7 the diameter of the circular) orbit. We can use
7 l2 this to nd the energies of the two orbits. The
E= K
10 mK values of A are simply 4Re for the rst and 8Re
4

for the second. The energy input needed to go Again this is due to the change in speed.
from one orbit to the other is at least 
  1 2 1 2 GM
GM m 1 1 GM m E = mv3 mv2 v3 =
E = = 2 2 4Re
Re 8 4 8Re
Finally we obtain the change in speed at point B
Plugging in the values given,   
1 1 GM
vB = = 726 m/sec
E = 2.34 1010 joules 2 6 Re

(b.) At point A, the rocket is red, putting Since the two velocities at point A are both tan-
the spacecraft in an elliptical orbit. The major gent to each other, and similarly for point B,
axis of this orbit is A = 6Re . To nd the initial the only changes in the velocities at either point
speed, we use the energy equation. The energy is are the changes in their magnitudes.
partly gravitational potential energy and partly 7. A satellite of mass m moves in a circular or-
kinetic energy: bit of radius R at speed v. It is inuenced by
the gravity of a xed mass at the origin.
1 GM m GM m
E= mv 2 =
2 0 2Re 4Re (a.) The mechanical energy of the satellite is
given by
Solving this equation for v0 , we nd the orbital 1 GM m
E = mV 2
speed  2 R
GM We know that gravity exactly provides the cen-
v0 =
2Re tripetal acceleration.
The energy of the elliptical orbit is given by GM m mV 2 GM
2
= =V2
GM m GM m R R R
E= E =
6Re 12Re The total energy is thus
The change in energy at point A is entirely due 1 1
E= mV 2 mV 2 = mV 2
to a change in speed. 2 2

1 2 1 2 2GM (b.) At a certain point on the orbit, the direction
E = mv1 mv0 v1 =
2 2 3Re of travel of the satellite changes. The magnitude
of the velocity does not change, so the total en-
The change in speed required at point A is thus ergy of the orbit doesnt change. We can now
   nd the kinetic energy at closest approach:
2 1 GM
vA = = 865 m/sec 1 1 5GM m
3 2 Re E = mV 2 = mv 2
2 2 R
1
We repeat this analysis at point B. Conservation = mv 5mV 2
2
of angular momentum gives 2
v = 3V

GM
2Re v1 = 4Re v2 v2 =
6Re (c.) The circular orbit has angular momentum
L1 = mRV , while the elliptical orbits angular
The energy of the new circular orbit is given by momentum, evaluated at the perigee, is
GM m GM m R 3
E= E = L2 = m 3V = L1
8Re 24Re 5 5
5

Therefore, just after the transition from circu- Finally, using the rst equation to eliminate
lar to elliptical orbit, a fraction 35 of the original cos ,
velocity must remain tangential, while 45 of it L2 32R5
F =
becomes radial (the squares of the two fractions 4mR3 r5
must add to unity according to Pythagoras). 8L2 R2
F =
Therefore the satellite turns through an angle mr5
4/5 Since L and R are constant, the unknown at-
= arctan = 53.1
3/5 tractive force depends on the inverse fth power
of the spaceships separation from the center of
8. A spaceship is moving on a circular path
the cloud, for this particular spaceship trajec-
that will take it directly through a gas cloud.
tory. The last equation is the desired result.
The angular momentum with respect to the gas
However, this is no simple force eld: its cou-
cloud is measured to be constant. We want to
pling to the spaceship is contrived to depend
know what attractive central force causes this.
quadratically both upon the spaceships angular
Immediately we notice that as the ship passes
momentum about the clouds center and upon
through the center of the cloud, it velocity must
the radius of its circular orbit.
become innite, because the angular momentum
l = mvr is conserved. If l were zero, the ship As an alternative to considering the cen-
could only fall straight into the cloud. tripetal force that must be supplied by F , one
can hypothesize that F is a conservative as well
We can express the circular trajectory of
as a central force. (At least it is clear from
the ship as a function of by inspection:
the fact that the spaceships orbit is closed that
r() = 2R cos (/2 < < /2) there can be no monotonic decrease or increase
Take to be the azimuth of the spaceship on in the total energy E). From the above equa-
the circle ( < < ), with 0 when = 0. tion for L, one readily sees that the ships speed
Consider the isosceles triangle with sides r, r, v = R = 2R is proportional to r2 . There-
and R. Requiring its angles to add up to , it is fore the ships kinetic energy K is proportional
easy to see that = 2. to r4 . If E is to be conserved, the potential en-
ergy U also must be proportional to r4 so that
We are given two denite facts. One is that it can cancel the r dependence of K; its radial
the ships angular momentum about the center derivative dU/dr = Fr must then be propor-
of the cloud tional to r5 . The constant of proportionality is
L = mr2 = 4mR2 cos2 easily veried to be the same as is given above.
is constant. (This expression conrms our pre-
vious observation that the ships velocity 2R
must be innite at the center of the cloud, where
cos = cos /2 = 0.) The second fact is that
the spaceship moves in a circle of radius R. The
centripetal force mR2 required to keep it in cir-
cular motion must be supplied by the component
along R of the unknown attractive force F :
F cos = mR2 = 4mR2
Using the previous equation for L to eliminate
from this equation,
L2
F cos = 4mR
16m2 R4 cos4
L2
F =
4mR3 cos5
University of California, Berkeley
Physics H7A Fall 1998 (Strovink)
PROBLEM SET 9

1. Stringed instruments are tuned (for compositions


in the key of C) so that this beat frequency is
a. Expand in Taylor series: f (x) = ln (1 x);
zero, producing a smoother tone. When a com-
f (x) = 1/(1 + x).
position in a dierent key is played, the stringed
b. Given two functions s(x) and c(x) such instrument can be retuned for that key, which
that ds/dx = c and dc/dx = s, prove that would be impossible for the piano.
s(x) + c(x) = [s(0) + c(0)]ex .
8. For an underdamped undriven harmonic os-
2. cillator with 0 / Q = 100, nd the number
a. French problem 1-4(b). of oscillations required to reduce the amplitude
of oscillation by the factor e 23.1.
b. French problem 1-9.
c. Prove DeMoivres theorem,

(cos + i sin )n = cos n + i sin n.

3. French problem 3-15.


4. At t = 0 a bullet of mass m and velocity
v0 strikes a motionless block of mass M which
is connected to a wall by a spring of constant
k. The block moves with coordinate x(t) (along
the direction of the bullet) on a frictionless ta-
ble next to the wall. The bullet embeds itself in
the block. If x(t) = [A exp (i(t + ))], with A
real, evaluate , A, and .
5. French problem 4-5.
6. French problem 4-8.
7. A piano has middle C = 256 Hz, and C-
above-middle-C = 512 Hz. The white keys of
its middle octave consist of middle C; D (1 step
above middle C); E (2 steps); F (2.5); G (3.5); A
(4.5); B (5.5); and C-above-middle-C (6 steps).
Each step causes the frequency to be multiplied
by a xed factor.
a. Find the frequencies of D, E, F, G, A, and
B.
b. If G were tuned to a perfect fth, its sec-
ond harmonic (2 the fundamental frequency)
would be the same as middle Cs third harmonic.
Find the beat frequency between the second har-
monic of G and the third harmonic of middle C.
(Pianos are tuned by listening for such beats.)
1

University of California, Berkeley


Physics H7A Fall 1998 (Strovink)
SOLUTION TO PROBLEM SET 9
Composed and formatted by E.A. Baltz and M. Strovink; proofed by D. Bacon

1. We recognize the sum as the Taylor series of ex .


(a.) The Taylor series for ln(1 x) is found as
follows: s(x) + c(x) = [s(0) + c(0)]ex


f (n) (0) n
f (x) = ln(1 x) = x 2.
n=0
n!
(a.) French problem 1-4(b).
The magnitude of a complex number a + ib,
where f (n) denotes the nth derivative of f . 2 2
where a and b are real, is just a + b . The
1 1 phase angle is equal to tan1 (b/a), where the
ln(1 x) = x x2 x3 + quadrant is determined by the
2 3 signs of botha
 and b. The rst vector
x n (2 + i 3) has length 7
ln(1 x) = and phase =tan1 ( 3/2) = 40.9 . The second
n
n=1 vector (2 i 3)2 is merely the square of the
complex conjugate of the rst vector. Therefore
We do the same for f (x) = 1/(1 + x). it has length 7 and 2 the phase, or 81.8 .
1 (b.) French problem 1-9.
= 1 x + x2 x3 +
1+x The value of ii is a little odd, but here goes. We
 need to know how to nd the log of a complex
1
= (1)n xn number:
1 + x n=0

ln z = ln |z|ei = ln |z| + i
(b.) We have two functions c(x) and s(x) related
as follows: There is an ambiguity here, because we can al-
ds dc
=c =s ways add an integer multiple of 2 to . Here
dx dx
we will choose not to do so, but simply take the
An easy way to approach this problem is to value of to be between and . Thus we
solve these dierential equations simultaneously. obtain
However, as is the case for most easy ways to
do things, the mathematics leading up to the so- 2
ii = ei ln i = ei /2
= e/2 = 0.2079
lution is somewhat advanced. Instead we will
use the Taylor series to solve it. Expanding
This means that paying 20 cents is bargain, but
around x = 0,
a very small one.
1
s(x) = s(0) + c(0)x + s(0)x2 + (c.) Prove (cos + i sin )n = (cos n + i sin n).
2 This is pretty trivial when we remember
1  n
c(x) = c(0) + s(0)x + c(0)x2 + (cos + i sin )n = ei = ein
2
Adding these two equations, we see that = cos n + i sin n


xn 3. French problem 3-15.
s(x) + c(x) = [s(0) + c(0)]
n=0
n! An oscillatory system loses energy according
2

to E = E0 et . We dene the Q value as two places in one oscillation. We want a place


Q 0 /. where it is zero and rising, because we know that
(a.) Middle C on a piano is played, and the en- x is increasing at the instant of contact. This is
ergy decreases to half of its initial value in one
= /2
second. The frequency is 256 Hz. The angular
frequency is this times 2, so 0 =1608.5/sec. The solution is now
We nd from
x(t) = [A exp(i(t /2)])
1
= (e/2 )2 = 0.693
2 We dierentiate to get the velocity, and evaluate
Lastly , the Q of the oscillator is this at t = 0.
v
Q = 1608.5/0.693 = 2321 v(0) = v = A sin(/2) = A A =

This gives the nal result for the amplitude
(b.) The note one octave above is struck (512
Hz). The decay time is the same, so the Q value mv0
A= 
is simply doubled: Q=4642. k(M + m)
(c.) A damped harmonic oscillator has mass
m = 0.1 kg, spring constant k = 0.9 N/m, and a 5. French 4-5.
damping constant b. The energy decays to 1/e (a.) A pendulum is forced by moving the point
in 4 seconds. This means that of support. The coordinate x gives the location
1 of the pendulum bob, and gives the location
= e4 = 0.25 sec1 of the point of support. The forces on the pen-
e
dulum are the damping force, which we must
b = m = 0.025 kg sec1
assume to be proportional to the absolute veloc-
 ity of the pendulum, and the force of gravity.
The natural frequency 0 = k/m = 3 Hz. Fi-
The force of gravity depends on the angle by
nally, the Q of the oscillator is Q = 0 / = 12.
which the pendulum is raised. This is propor-
4. At t = 0, the bullet collides inelastically with tional to the distance that the pendulum bob is
the block, so only the momentum is conserved. displaced from the point of support, x . This
The nal velocity of the block and bullet is given gives the equation of motion
by
d2 x dx mg
mv0 m = b (x )
mv0 = (M + m)v v = dt 2 dt l
M +m
Using 02 = g/l and = b/m, we put this in the
We now have the initial conditions for the os- standard form with as a forcing term.
cillation. The initial position is x(0) = 0 and
the initial velocity is 
v(0) = v. The frequency d2 x dx
is given as usual by k/mass, but the mass in 2
+ + 02 x = 02
dt dt
question is the total mass of the system:
 (b.) The motion of the point of support is given
k by (t) = 0 cos t. We use the formula for the
=
M +m amplitude of forced oscillation, but we note that
in this case, the equation is
The solution is given by x(t) = [A exp(i(t +
))]. The initial position tells us that cos = 0. d2 x dx
This is ambiguous because the cosine is zero in 2
+ + 02 x = 02 0 cos t
dt dt
3

The constant 02 0 takes the place of the F0 /m The solutions to this are
we normally see in this type of equation. The   
2 1 2 2 4 2 2
amplitude of the oscillation is thus given by = 20 + 12 0
2
02 0
A() =  2 Plugging in the numbers, the two frequencies are
(0 2 )2 + 2 2 = 3.147 sec1 and = 3.113 sec1 .
6. French 4-8.
At exact resonance, = 0 and the amplitude is
(a.) A mass is under the inuence of a viscous
0 force F = bv. Let = b/m as usual. The
A(0 ) = 0 = Q0
equation of motion is

Now we want to nd Q. We are given that the dv


+ v = 0
forcing amplitude is 0 = 1 mm. The length of dt
the pendulum is l = 1 m, so this gives 0 = 3.13
We can easily solve this equation by direct inte-
sec1 . We know that the amplitude falls o by
gration.
a factor of e after 50 swings, or 50 periods. We
v(t) = v0 et
know that A = A0 exp(t/2) so t = 2. t is 50
periods, or 100/, where is the frequency of We simply integrate this equation with respect
free oscillation to t to get the position.
 v0 t
= 02 + 2 /4 x(t) = C e

Then C is the integration constant that will allow us


t = 2 to t an initial condition.
100 (b.) A driving force F = F0 cos t is turned on.
=2
 We want to nd the steady state motion. We will
50 = 02 + 2 /4 use a complex exponential for the forcing term,
with the understanding that we take the real part
(50)2 2 = 02 + 2 /4 when were done. The new equation of motion is
Plugging in the numbers, we get = 0.0199. d2 x dx F0 it
This gives us Q = 157, and A = 15.7 cm. 2
+ = e
dt dt m
(c.) We want to nd the frequencies where the Assume a solution of the form
amplitude is half of the resonant value. We
merely solve x(t) = Aeit

02 0 0 where A is a complex number. Plugging this


 = 0
(02 2 )2 + 2 2 2 into the equation of motion, we see that

F0 F0 /m
This gives 2 A + iA = A=
m 2 + i
402 2 = (02 2 )2 + 2 2 We write the denominator as the product of
a magnitude and a phase. The magnitude is

Turning this into a quadratic equation for 2 , 4 + 2 2 . The denominator has a nega-
we get tive real part and a positive imaginary part,
so it is in the second quadrant with phase
0 = 4 + ( 2 202 ) 2 + 04 402 2 arctan (/). Since the numerator is real,
4

the phase of A is minus the phase of its de- Plugging this value of v0 into the rst equation,
nominator, or arctan (/) . According to
the notation of the problem, the phase of the F0 /m F0 /m
C= + 2 =0
oscillation is , so we nd 2
+ 2 + 2

= arctan (/) Collecting these results, the solution satisfying


both boundary conditions is
The amplitude of the oscillation is just the mag-
nitude of A, given by F0 /m t
x(t) = e
F0 /m 2 + 2
|A| =  F0 /m
4 + 2 2  cos(t tan1 (/))
4 + 2 2
The general solution to the problem is
v0 t 7. Middle C is 256 Hz, and C above it is double
x(t) = C e that frequency, or 512 Hz. The scale is divided

F0 /m into 6 whole steps, or 12 half steps. The note
 cos(t + tan1 (/)) after each half step is a constant multiple f of
4 + 2 2 the frequency of the previous note. When we
At t = 0, we want x = 0. At t = 0, the last term have gone up twelve half steps, the frequency
B in the general solution is will have doubled. The constant factor f is thus
given by f 12 = 2 or f = 21/12 .
F0 /m
B(0) =  cos(tan1 (/)) (a.) The frequencies in the scale are thus
4
+ 2 2
C=256, D=287.4, E=322.5, F=341.7, G=383.6,
F0 /m A=430.5, B=483.3, C=512 Hz. These are zero,
=  
4
+ 2 2 + 2
2 two, four, ve, seven, nine, eleven, and twelve
half steps above middle C, respectively.
F0 /m
= 2
+ 2 (b.) Middle Cs third harmonic is three times its
fundamental frequency, or 768 Hz. The second
Thus the condition x(0) = 0 gives us one equa- harmonic of G is 767.133 Hz. The beat frequency
tion for C and v0 : is always just the dierence between the two fre-
quencies. In this case the beat frequency is 0.867
v0 F0 /m
C= + 2 Hz, which is easily audible to the piano tuner.
+ 2
8. An undriven oscillator that is underdamped
The rst time derivative of B, evaluated at t = 0, has a Q of 100. We want to know how many os-
is cillations it takes to damp by a factor of e . This
F0 /m
B(0) =  sin(tan1 (/)) just means that t/2 = . Now we want to write
2 + 2 t in terms of the number of oscillations n. This
F0 /m takes n times the period, or t = 2n/0 . This
= 2
+ 2 gives n/0 = 1. Remember that Q = 0 /, so
n = Q = 100.
Then, requiring the rst time derivative of the
general solution to vanish at t = 0, the second
equation for v0 and C is
F0 /m
0 = 0 + v0 +
2 + 2
F0 /m
v0 = 2
+ 2
University of California, Berkeley
Physics H7A Fall 1998 (Strovink)
PROBLEM SET 10

1. French problem 5-6. and it is under tension S.


2. French problem 5-10. a. What is its period of oscillation T ?
3. French problem 5-14. b. At t = T /2, what is its shape?
4. A transverse wave on a string has: 7. A guitar string tuned to middle C (256 Hz)
at x=0, a time variation of the form cos t+ is plucked at exact center by giving it nonzero
transverse velocity in a very small region. What
sin t with a period of 102 sec.
tones (in Hz) will be heard in addition to 256 Hz?
a wavespeed of 10 m/sec.
the appearance of propagating to the left
(toward smaller x).
an amplitude of 0.01 m.
Suppose that the string is described by

y(x, t) = (A+ exp i(t kx)


+ A exp i(t + kx))

where A are complex constants.


a. Substitute numbers in place of all the vari-
ables (except x and t) in the above equation.
b. Compute the maximum transverse speed
and the maximum slope of the string.
5. If surface tension is included, the phase veloc-
ity of a surface wave on a liquid of mass density
and surface tension T is

2T g
vph = +
2

where is the wavelength and g is the accelera-


tion due to earths gravity.
a. Find the wavelength, frequency, and phase
velocity of ripples on water which advance with
minimum speed.
b. Find the group velocity of such a wave.
6. A non-dispersive string (/k = constant),
initially at rest, has length L (0 x L), and
mass per unit length . At t=0, its shape is

x 3x
y(x) = 3 sin + sin
L L
1

University of California, Berkeley


Physics H7A Fall 1998 (Strovink)
SOLUTION TO PROBLEM SET 10
Composed and formatted by E.A. Baltz and M. Strovink; proofed by D. Bacon

1. French 5-6. position xA = 0. This happens


when 0 is in
the second quadrant and 30 is in the third:
(a.) All three springs are identical, with constant
k. The equations of motion are
0 = 30
d2 xA 2

+ 202 xA 02 xB = 0 0 =
dt2 1+ 3

d2 xB However, at t = , mass B will not have re-


+ 202 xB 02 xA = 0 turned to its full original |displacement| since
dt2
| cos | < 1. Thus, even though mass A will be
We plug in a guessed solution, where the two
back in place, mass B will not, and the sys-
masses oscillate at the same frequency, but with
tem will not have returned to (plus or minus) its
dierent amplitudes A and B. This gives
original state. In fact, because the ratio of the
two normal frequencies is irrational, once both
202 2
2 A + 202 A 02 B = 0 B = A normal modes are excited the system can never
02 return to its original state.

02 2. French 5-10.
2 B + 202 B 02 A = 0 B = A The equations of motion for this double spring
202 2
system are as follows. The coordinate of the top
Equating these we see that mass is xA and the coordinate of the bottom
mass is xB .
(202 2 )2 04 = 0
d2 xA
+ 202 xA 02 xB = 0
Solving this quadratic equation, we nd that the dt2
two frequencies are 2 = 02 and 2 = 302 .
d2 xB
(b.) One mass is displaced by 5 cm. This excites + 02 xB 02 xA = 0
dt2
each normal mode equally, with amplitude 2.5
cm. To see this, rst excite the rst normal mode Plugging in the standard guess that both masses
with amplitude 2.5 cm. Now both masses are oscillate at the same frequency, but at ampli-
+2.5 cm from equilibrium. Now excite the sec- tudes A and B, we get the following equations.
ond normal mode, also with amplitude 2.5 cm.
This moves one mass forward 2.5 cm, and the (202 2 )A = 02 B
other back 2.5 cm. One is now 5cm from equilib-
rium, and the other is at its equilibrium position. (02 2 )B = 02 A
It is mass B that is displaced, so the masses obey Equating these, we get the quadratic equation

xA = 2.5 cos 0 t 2.5 cos 30 t 4 302 2 + 04 = 0

xB = 2.5 cos 0 t + 2.5 cos 30 t The solutions to this equation are

(c.) After a time such that cos 0 =
2 3 5
cos 30 , mass A returns to its equilibrium = 02
2
2

The amplitudes in these modes are easily found. Now we can nd the amplitude and frequency.
For + , we have The solution is

 3 2 5 2 
2
20 0 A+ = 02 B+ y(x, t) = A cos t + kx
2 2 0
4
1 5
B+ = A+ The amplitude is given as 0.01 m, and the period
2
is 102 sec. The frequency = 2/T = 200
Likewise for , sec1 . The speed of waves on the string is c =10
m/sec, and we know that = ck, so this tells us
1+ 5 that k = 20 m1 . We now have the nal result
B = A
2  
y(x, t) =  0.01ei(200t+20x/4)
3. French 5-14.
In the rst normal mode, the three = 0.01 cos(200t + 20x /4)
particles have
an amplitude ratio 2/2 : 1 : 2/2. The second
normal mode has amplitude ratios 1 : 0 : 1.
The (b.) We can now compute the maximum trans-
third normal
mode has amplitude ratios
2/2 : 1 : 2/2. verse speed and maximum slope. The transverse
speed is
4. A wave is described by
  dy
y(x, t) =  A+ ei(tkx) + A ei(t+kx) = 2 sin(200t + 20x /4)
dt
(a.) We know that the wave is moving to the The maximum value that the sine takes is 1,
left. This corresponds to the second exponen- so the maximum transverse speed is 2 = 6.28
tial. To see this, we note that a specic place m/sec. We can likewise nd the maximum slope
on the wave train always has the same value of
t kx. We want to see what happens when t
dy
increases. For the solution exp(i(t + kx)), we = 0.2 sin(200t + 20x /4)
see that as t increases, x must decrease to stay dx
on the same place in the wave. This is a left
moving wave. We thus note that A+ = 0. We The maximum slope is thus /5 = 0.628.
can now write the complex constant A = Aei , 5. The phase velocity of surface waves is given by
where A and are real.
   
y(x, t) =  Aei(t+kx+) 2T g kT g
vph = + = +
2 k
At x = 0 we know that the time dependence in
proportional to cos t + sin t. This tells us that (a.) Notice that at both zero and innite
wavenumber, the phase velocity is innite. To
cos(t + ) cos t + sin t
nd the minimum phase velocity, we dierentiate
vph with respect to k and set to zero.
Using the formula for the cosine of a sum

cos(t + ) = cos t cos sin t sin dvph T / g/k 2


=  =0
dk 2 kT / + g/k
For this to work we see that  
g T
cos = sin =
k= = 2
4 T g
3

The phase velocity at this wavenumber is term has frequency 2 = 3c/L. The peri-
ods of these two oscillations are T1 = 2L/c and
 1/4
4gT T2 = 2L/3c. The period of the total oscillation
vph = is the longer period, T1 . In one long period the

fast oscillation has had exactly three periods.
The frequency = vph k, which gives Thus, the period is
 1/4   1/4 
4gT g 4g 3 T =
2L
= 2L

= = c S
T T

(b.) The group velocity of this wave is given by (b.) After a time T /2, the rst term has gone

through a half period, and the second term has
d

gone through one and a half periods. In both


vgr =
dk
k=g/T cases, this just means that there is a minus sign
out front.
We know that = vph k, so
x 3x
 y(x, T /2) = 3 sin sin
L L
T k3
= + gk
7. A string of frequency 256 Hz is plucked
in the exact center. This means that the even
Taking the derivative with respect to k and eval- numbered modes are not exited at all. This is be-
uating at the wavenumber we found before cause the initial condition is symmetric around
d

3T k 2 / + g

the middle of the string, and the even numbered



= 
modes are antisymmetric around the middle of
dk k= g/T 2 T k 3 / + gk k= g/T the string. The odd numbered modes are also

2g  4gT 1/4 symmetric around the center of the string, so
= = they survive. These frequencies are
k

The slowest waves have the same phase and fn = (2n + 1)256 Hz, n = 0 . . .
group velocities. This is a general result. Look
at the equation for the frequency, and dierenti-
ate it to get the group velocity

d dvph
= vph k = vgr = vph + k
dk dk
We chose the phase velocity to be a minimum,
so vgr = vph .
6. A string has tension S and linear mass den-
sity . This
tells us the phase velocity of waves
on it, c = S/. The string has length L. At
t = 0, the strings shape is

x 3x
y(x, 0) = 3 sin + sin
L L

(a.) The frequencies of each of these is = ck,


so the rst term has 1 = c/L and the second
University of California, Berkeley
Physics H7A Fall 1998 (Strovink)
PROBLEM SET 11

1. French problem 6-9. where is some function of time t.


2. French problem 6-15(a). (a.) Prove that the uid density (x, y, z, t)
3. French problem 8-9. Note that 12 mv 2  = satises
1
3 y x =
2 kT for the sodium atoms in the vapor, where T x y t
is the temperature in degrees Kelvin ( K), and
k is Boltzmanns constant, 1.38 1023 J/ K. (b.) Show that the angular velocity of the uid
4. French problem 8-12. about the origin, evaluated at an arbitrary point,
is half of v evaluated at the same point.
5. A tank is lled with water to a height H. A
hole is punched in its wall a depth h below the (c.) If (t) = 0 = constant, prove that
surface of the water.
dv
a. Find the horizontal distance from the bot- = r02
dt
tom of the tank that the stream of water hits
the ground. where r = xx + yy, and dv/dt is the time rate
b. Could a hole punched at a dierent depth of change of the velocity of an element of uid
produce a stream with the same horizontal that is temporarily at (x, y, z) at time t.
range? If so, at what depth?
6. Consider the stagnant air at the front edge of
an airplane wing and the air rushing over a wing
surface at speed v. Find the greatest possible
value for v in streamline ow, using Bernoullis
equation and assuming that air is incompress-
ible. Take the density of air to be 1.2103
g/cm3 . Compare this numerical result with the
speed of sound, 340 m/sec.
7. Verify by explicit computation in a Cartesian
coordinate system that
(f (x, y, z)) = 0

8.
(a.) Consider the function f (x, y, z) x2 + y 2
z 2 . At the point (x, y, z) = (3, 4, 5), nd the di-
rection of a vector ds (of small xed length) such
that df /|ds| is a maximum.

(b.) Consider the surface z(x, y) = x2 + y 2 .
At the point (x, y, z) = (3, 4, 5), nd the direc-
tion of a vector du (of small xed length) which
is normal to this surface.
9. A uid has a velocity eld
v(x, y, z, t) = (yx xy)(t)
1

University of California, Berkeley


Physics H7A Fall 1998 (Strovink)
SOLUTION TO PROBLEM SET 11
Composed and formatted by E.A. Baltz and M. Strovink; proofed by D. Bacon

1. French 6-9. A1 = 10 . The last two equations become


(a.) The lowest resonant frequency of a room
is 50 Hz. All integer multiples of this frequency (L/4) = 5 2 cos (100t) + A2 cos (200t)
are also resonant. The lowest two modes are ex-
(3L/4) = 5 2 cos (100t) A2 cos (200t)
cited. These are 50 Hz (the fundamental) and
100 Hz (the [rst] harmonic). The amplitude is
As a trial solution, we assume that A2 is positive.
maximum at t = 0. The time interval t = 1/200
Then the maximum displacement of 10 at L/4
sec is one fourth of a period for the fundamental
is reached at t = 0; therefore A2 = (10 5 2)
and half a period for the harmonic. t = 1/100sec
. The maximum |displacement| at 3L/4, a neg-
is one half of a period for the fundamental and a
ative displacement in this case, is reached at
full period for the harmonic. These modes look
t = 1/100 sec, when the harmonic has changed
like
phase by a full 2, but the fundamental has
changed phase by only and has therefore be-
come negative. With the above values of A1 and
A2 , the displacement at 3L/4 is equal to -10 ,
in agreement with the problem.
2. French 6-15(a).
This is a bit messy, but bear with it. The
string has length L. Its initial conditions are
y(x, 0) = Ax(L x) and (y/t)t=0 = 0. We
write the solution as a Fourier series

  nx 
y(x, t) = An sin cos(n t n )
n=1
L

(b.) We can write the total displacement as We know the solution at t = 0. Dene Bn =
An cos n .
(x) = A1 sin (x/L) cos (100t)+
+ A2 sin (2x/L) cos (200t)
  nx 
y(x, 0) = Bn sin
n=1
L
where A1 and A2 are the unknown amplitudes
for the fundamental and harmonic mode, respec-
tively. In particular We can now solve for the Bn using Fouriers
trick:
(L/2) = A1 cos (100t) + 0  L  nx 
2
A1 Bn = Ax(L x) sin dx
(L/4) = cos (100t) + A2 cos (200t) L 0 L
2
A1 Integrating by parts once, notice that the surface
(3L/4) = cos (100t) A2 cos (200t)
2 term vanishes:
 L  nx 
Since the amplitude at L/2 is due only to the 2A
Bn = (L 2x) cos dx
fundamental, and is equal to 10 , we know that n 0 L
2

The L term integrates now. It always integrates The constant k = 1.38 1023 J/Kelvin is
to zero. Boltzmanns constant, and the appropriate mass
  nx  m 23mp is just
the mass of the sodium atom.
4A L
Bn = x cos dx Approximating v 2  vmax , this gives T
n 0 L
900 Kelvin. This eect can be used to measure
We integrate by parts again, and again the sur- the temperatures of objects in astronomy.
face term vanishes.
  nx  4. French 8-12.
4AL L The Doppler eect for a moving source and xed
Bn = 2 2 sin dx
n 0 L observer is given by
This integrates easily.
 nx L  0
4AL2 8AL2  () =
1 uv cos
Bn = 3 3 cos = 3 3
n L 0 n n odd
We see that Bn is zero for all even n. We (a.) We want to nd the Doppler eect for a
expected this because the initial condition is xed source and a moving observer at velocity
symmetric around the center of the string. Now u. We are going to do this dierently than it
we tackle the velocities. was done in French, in order to provide an al-
   nx 
y  ternative and perhaps easier way to think about
= A sin n sin(n ) = 0
t 
n it. We are going to go into the rest frame of the
t=0 n=1
L
wave crests, so the only thing moving will be the
This equation is satised if we simply set all of observer and the source. The velocities of the
the n = 0, so that sin n = 0 and cos n = 1. wave and the observer are
This also means that An = Bn . We now have
the full solution. We have rewritten the sum to
Vw = (v, 0) Vo = (u cos , u sin )
only include odd n.

 8AL2
y(x, t) = Transforming into the rest frame of the wave by
m=0
(2m + 1)3 3 subtracting (v, 0), the velocity of the observer is
 (2m + 1)x 
sin cos(2m+1 t)
L
Vo = (v u cos , u sin )
3. French 8-9.
This problem concerns a very important eect
The distance between wave crests is just L =
called Doppler broadening. Sodium atoms emit
v/0 , so the rate at which the observer crosses
light of 6000A. The observed light varies in a
the wave crests is just the x velocity divided by
small frequency range of (6000 0.02)A. This
this distance. This is the shifted frequency.
is caused by the thermal motion of the sodium
atoms. The Doppler eect tells us that
 u 
v 0.02  () = 0 1 + cos
= = = 3.33 106 v
c 6000
This gives the maximum velocity of the atoms
This trick doesnt work for light because light
vmax = 1000 m/sec. The thermal velocity is
doesnt have a rest frame. In relativity, there is
given by
a dierent Doppler formula that applies to both
1 3 mv 2  situations. It uses only the relative velocity of
mv 2  = kT T = the source and the observer.
2 2 3k
3

(b.) We want to know the approximate dier- The maximum velocity occurs when the pressure
ence between the two formulas. We Taylor ex- is zero. Plugging in P0 =105 N/m2 and = 1.2
pand the rst formula, assuming that the speed kg/m3 , we nd that v =408 m/sec. This is larger
is much less than the sound speed, u
v. than the speed of sound, vs = 340 m/sec.
 
u u2 7. The gradient f is equal to
() 0 1 + cos + 2 cos +
2
v v
f f f
This tells us the approximate dierence between f = x + y + z
x y z
the two Doppler shifted frequencies

u2 The x component of (f ) is the dier-


()  () 0 cos2 ence between the y and z partial derivatives,
v2
respectively, of the z and y components of f :
5. We can use Bernoullis equation to nd the
velocities of these two ows. f f
( (f ))x =
(a.) If we look right as the ow is leaving the y z z y
tank, we see that it must be at atmospheric pres-
sure. The stream is arbitrarily thin, so this is Interchanging the order of dierentiation in ei-
the only possibility. We then just use the grav- ther of the terms, this expression is seen to vanish
itational potential to nd the velocities. If the for well-behaved f . By cyclic permutation, the
hole were at the top of the tank, the velocity y and z components of (f ) vanish as well.
would be zero, so we use this as the constant.
8. (a.) When the point of observation (x, y, z)
1 is displaced incrementally by ds, where
P0 + g(H h) + v 2 = P0 + gH
2 
v(h) = 2gh ds = xdx + ydy + zdz
The time it takes to hit the ground is given by
the formula for constant acceleration. points in an arbitrary direction, the change df
in f (x, y, z) is given by the chain rule:
1 2 2(H h)
H h = gt t =
2 g f f f
df = dx + dy + dz
x y z
In this time, the water travels a horizontal dis-
tance vt, so the distance from the tank is
The right-hand side can be rewritten as the dot

d(h) = 4h(H h) product of

(b.) We notice that the previous formula says f f f


f = x + y + z
that water leaking from a depth h travels the x y z
same distance as water leaking from a depth
H h. and ds above:
6. We again apply Bernoullis equation. The
air at the leading edge is stagnant, and we df = f ds
will assume that it is at atmospheric pressure.
Bernoullis equation gives For a xed length |ds|, this dot product is great-
1 est when ds is parallel to f . Therefore, when
P0 = P + v 2 df /|ds| is a maximum, the direction of ds will be
2
4

along f . With f = x2 + y 2 z 2 , this direction xy plane and orthogonal to r . Thus the element
n is is in circular motion about the z axis (to which
x f f f
x + y y + z z r is perpendicular), with angular velocity
n =
|f |
x2x + y2y z2z |v|
=  = z = z(t)
|r |
2 x2 + y 2 + z 2
x6 + y8 z10 On the other hand
=
2 9 + 16 + 25  x
x3 + y4 z5 y 
= v = (t) z +
x y
5 2
= 2z(t)

(b.) The surface z(x, y) = x2 + y 2 can be
described as (c.) Suppose that the independent variables
(x, y, z, t) upon which a vector A(x, y, z, t) de-
0 = f (x, y, z) = x2 + y 2 z 2
pends change innitesimally, by (dx, dy, dz, dt).
This is the same f (x, y, z) as in part (a.). Sup- Then, by the chain rule, a component of A,
pose the point of observation (x, y, z) is displaced e.g. Ax , changes by an amount
innitesimally by dv, where dv is on the surface
f = 0. Then we would expect f not to change Ax Ax Ax Ax
dAx = dt + dx + dy + dz
at all. However, according to the results of part t x y z
(a.), dAz Ax Ax Ax Ax
= + vx + vy + vz
df = f dv dt t x y z
Ax  
Therefore df can vanish only if dv is perpendic- = + vx + vy + vz Ax
ular to f . Since dv can be any displacement t x y z
dAx  
which lies on the surface, this requires f to be = + v Ax
perpendicular to the surface. Therefore, the di- dt t
dA  
rection of the normal to the surface du in part = +v A
(b.) is the same as the direction n of ds in part dt t
(a.), the direction of maximum change in f .
(This is the convective derivative, yielding the
9. The uid velocity eld is total time rate of change of A.) In this problem
A = v itself, so
v(x, y, z, t) = (yx xy)(t)
dv  
(a.) The equation of continuity (conservation of = +v v
dt t
uid molecules) requires  
= 0 + 0 v x + vy (yx xy)
x y
+ (v) = 0  
t = 02 y +x (yx xy)
x y
Therefore
= 02 (yy xx)

= (yx xy) = 02 r
t
1
=y x
t x y

(b.) An element of uid at r = xx + yy has a


velocity v = (yx xy)(t) that is always in the
University of California, Berkeley
Physics H7A Fall 1998 (Strovink)
PRACTICE EXAMINATION 1

Directions. Do all three problems (weights are indicated). This is a closed-book closed-note exam
except for one 8 12 11 inch sheet containing any information you wish on both sides. You are free
to approach the proctor to ask questions but he or she will not give hints and will be obliged to
write your question and its answer on the board. You may use a calculator but you do not need one
roots, circular functions, etc., may be left unevaluated if you do not know them. Use a bluebook.
Do not use scratch paper otherwise you risk losing part credit. Cross out rather than erase any
work that you wish the grader to ignore. Justify what you do. Box or circle your answer.

1. (25 points). A 4 kg mass moves in the x-y d. What is the particles position at t=1 sec?
plane under the inuence of a constant force.
At t=0, the particles position, velocity, and e. What is the particles velocity when its ab-
acceleration are shown in the diagram. solute value (its speed) is at a minimum?

2. (40 points) A Millikan oil drop of mass m and


charge q moves between two horizontal capaci-
tor plates separated by a distance d. A battery
of voltage V is applied to the plates, so that
the electrical force on the drop is upward, of
magnitude qV /d. When it is moving, the drop
experiences an opposing drag force F = kv,
where v is its velocity and k is a constant.
a. For t < 0 the drop is observed to be ex-
actly stationary, despite the gravitational
This problem will be graded on answers only force that is exerted upon it. What is the
no part credit will be given. The answer to each voltage V in terms of the other constants?
question has three parts value (3 points), unit
b. At t = 0 the plates are shorted out (V =0),
(1 point), and direction (1 point). If for a par-
and remain shorted thereafter. Calculate
ticular question you believe the direction to be
a, the downward acceleration of the drop
undened or irrelevant, please leave it blank.
immediately after the plates are shorted.
Organize your answers in columns as shown:
c. As t the acceleration of the drop
part value unit direction becomes essentially zero. What is its down-
(a) ward velocity v then?
(b)
(c) d. For any time t > 0, write a dierential
(d) equation containing v, its rst time deriva-
(e) tive dv/dt, and constants.
a. What is the particles momentum at t=0? e. Find a solution for the downward accel-
eration a(t), valid for all t > 0. [Hint:
b. What is the force acting on the particle?
Dierentiate the answer to part (d.) with
c. What is the particles momentum at t=0.375 respect to time to get a simple dierential
sec? equation for a(t). Solve it by integration.
Use the result of part (b.) to determine the
constant of integration.]

3. (35 points) A wooden block of mass M ,


initially at rest on a horizontal table with coe-
cient of sliding friction , is struck by a bullet of
mass m and velocity v. The bullet lodges in the
center of the block. How far does the block slide?
1

University of California, Berkeley


Physics H7A Fall 1998 (Strovink)
SOLUTION TO PRACTICE EXAMINATION 1
Composed and formatted by E.A. Baltz and M. Strovink

1. For each of parts (a.) through (e.), specify moves between plates a distance d apart, and
magnitude, unit, and direction. Giving vector voltage V is applied. The electrical force on
components is sucient for magnitude and di- the drop is qV /d upwards. When the drop is
rection. The mass is 4 kg, the initial velocity moving, it encounters a drag force F = kv.
is 3 meters per second, and the initial accelera- (a.) At t < 0 the drop is stationary. The elec-
tion is 8 meters per second2 . The force acting trical force must balance the gravitational force,
on the particle is constant, so the acceleration is so mg = qV /d, giving the potential V
constant.
(a.) The initial momentum is p = mv = 12 kg mgd
V =
m/sec, in the +y direction. q
(b.) The force acting on the particle can be
found by F = ma, which is 32 kg m/sec2 , or (b.) The plates are shorted at t = 0, so there is
newtons, in the +x direction. no more electrical force. At this instant, the drop
isnt moving, so it feels only the gravitational
(c.) At t = 0.375 = 3/8 sec, the momentum force. Its acceleration is just the acceleration of
can be found easily because the acceleration is gravity, a = g downwards.
constant. The velocity in the +y direction isnt
changing because there is no force, but the +x (c.) As t , the acceleration goes to zero.
velocity is given by vx = at = 3 m/sec. The The velocity can be found by balancing the drag
momentum vector is thus (12,12) kg-m/sec. Al- force with the gravitational force, kv = mg, so
ternatively, we can write this as a magnitude the terminal velocity is
and aunit vector. The magnitude of this vector mg
is 12 2. The unit vector must have equal x and v(t ) =
k
y components, andits length must be one. This
gives p = (x + y)/ 2.
(d.) For t > 0 we can nd a dierential equa-
(d.) The position at t = 1 sec is given by the tion for the velocity. Newtons second law states
formulas for constant acceleration. In the y di- that F = dp/dt which in this case can be writ-
rection, there is no acceleration, so y is given ten F = m dv/dt. There are two forces, gravity
simply by y = y0 +v0y t = 6 m. In the x direction and the drag force. The equation we get is
there is a constant acceleration, so the position
is given by x = x0 + v0x t + at2 /2 = 8 m. The po- m
dv
= mg kv
sition vector is thus (8,6) m. Alternatively, the dt
length of this vector is 10 meters; its direction is
given by the unit vector r = (0.8x + 0.6y). (e.) Following the hint, we take d/dt of the
answer for (d.):
(e.) The y component of the velocity is constant
because there is no force in the y direction. The
d2 v dv
x component of  the velocity vanishes at t = 0. m 2
= k
dt dt
Therefore |v| = vx2 + vy2 can never be smaller
than it is at t = 0, when it is 3 m/sec in the y Substituting a = dv/dt:
direction.
da
2. An oil drop has mass m and charge q. It m = ka
dt
2

Rearranging: will remain at rest. Solving, the time at which


da k the block-bullet system stops is
= dt
a m
Integrating from 0 to t: t = v0 /(g) .

k The distance traveled in that time is


ln a(t) ln a(0) = t
m
1 1 (v  )2
Exponentiating: x = v0 t gt2 = v0 t = 0 .
2 2 2g
a(t) k
Plugging in the already deduced value for v0 ,
= e m t
a(0) this distance is
From part (b.), a(0) = g, so  m 2 v 2
x= .
k M + m 2g
a(t) = g e m t

The acceleration begins with value g and de-


creases exponentially with time constant equal
to m/k.
3. Instantaneously after the collision of the bul-
let and block, after the bullet has come to rest
but before the frictional force on the block has
had time to slow it down more than an inn-
tesimal amount, we can apply momentum con-
servation to the bullet-block collision. At that
time the total momentum of the block+bullet
system is (M + m)v0 , where v0 is the velocity of
the block+bullet system immediately after the
collision. Momentum conservation requires that
momentum to be equal to the initial momentum
mv of the bullet. Thus
mv
v0 = .
M +m
After the collision, the normal force on the
block+bullet system from the table is (M + m)g,
giving rise to a frictional force

N = (M + m)g

on the sliding block+bullet system. This causes


a constant acceleration g of that system oppo-
site to its velocity.
Take t = 0 at the time of collision. After-
ward, the block+bullet systems velocity in the
horizontal direction will be v  (t) = v0 gt. It
will continue sliding until v  (t) = 0, at which
point the frictional force will disappear and it
University of California, Berkeley
Physics H7A Fall 1998 (Strovink)
EXAMINATION 1

Directions. Do all four problems (weights are indicated). This is a closed-book closed-note exam
except for one 8 12 11 inch sheet containing any information you wish on both sides. You are free to
approach the proctor to ask questions but he or she will not give hints and will be obliged to write
your question and its answer on the board. Dont use a calculator, which you dont need roots,
circular functions, etc., may be left unevaluated if you do not know them. Use a bluebook. Do not
use scratch paper otherwise you risk losing part credit. Cross out rather than erase any work that
you wish the grader to ignore. Justify what you do. Box or circle your answer.

1. (20 points) You wish to y (at low altitude) ity v0 . What is the coecient of sliding
from Sapporo, Japan to Portland, OR. These friction between block and plane?
cities are both at 45 north latitude (halfway be-
b. (15 points) After the hanging block hits the
tween the equator and the north pole), and they
table, what is the distance s along the sur-
are separated by 90 in longitude (azimuth).
face of the plane along which the top block
Consider the earth to be a sphere of radius R.
continues to slide? (Assume that the plane
a. (10 points) You y a course that keeps you is long enough that the block does not fall
at constant latitude, i.e. you y due east. o. If you are unsure of your answer to part
Over what distance do you travel? (a.), you may leave as an undetermined
constant.)
b. (10 points) You y a great circle course
that takes you from Sapporo to Portland in
the shortest possible distance (without pen- 3. (35 points) To determine the dependence
etrating the earth, of course). What is that of the force of air resistance upon the speed of
distance? a slowly moving object, one starts with three
sheets of paper stacked together as they come
out of the package. One then crumples them
2. (30 points) A block of mass m slides on an in- against ones st as shown in the sketch.
clined plane with a slope of 5/12 (i.e. the slope
of the hypotenuse of a 5-12-13 triangle). A mass- Next one separates a single sheet from
less rope, guided by a massless pulley, connects the other two without changing their crumpled
the block to a second block of mass m/13, which (pseudo-conical) shapes. This yields two objects
is hanging freely above a lower table. with the same shape, but with dierent masses
m and 2m, where m is the mass of a single sheet.

Finally one releases these two objects simul-


taneously, and compares their motion in still air
under the inuence of gravity.
a. (15 points) The two-block system is ob- a. (5 points) Instantaneously after the two ob-
served to be moving with a constant veloc- jects are released, what is the ratio R of
the (downward) acceleration of the heavier
object to that of the lighter object?
b. (15 points) Very soon after being released,
the two objects reach terminal (constant)
velocity due to the eects of air resistance.
After a long time, one observes that the
object of mass 2m has dropped 2 times
farther than the object of mass m. Assum-
ing that the force of air resistance on these
objects is proportional to v , where v is the
velocity and is a constant exponent, what
is the value of ?
c. (15 points) Suppose that Mother Nature
were to turn gravity o while these ob-
jects are falling. If one were willing to wait
an arbitrarily long time, would they fall
an arbitrarily long distance, or would that
distance be bounded? Explain.

4. (15 points). An asymmetric barbell stands


vertically at rest on frictionless horizontal ice.
Mass M rests on the ice, and mass m is a dis-
tance h above it; the mass of the bar that rigidly
connects these two masses is negligible. The
dimensions of the masses can be neglected in
comparison to their separation h. Take x = 0 to
be the initial position of mass M .

Mass m is given an innitesimal tap in the


+x direction that produces a negligible momen-
tum, but does eventually cause the barbell to
topple over. You may assume that mass M re-
mains in contact with the ice throughout the
motion. When mass m hits the ice, at what hor-
izontal coordinate xM will mass M be located?
1

University of California, Berkeley


Physics H7A Fall 1998 (Strovink)
SOLUTION TO EXAMINATION 1

1. In spherical polar coordinates, take Sapporo the normal component of the gravitational force
and Portland to be at (r, , ) = (R, /4, 0) = rS on block m, or
and (R, /4, /2) = rP respectively.
N = mg cos .
(a.) Here the course is one quarter of a circle
with its center on the earths axis of rotation at The frictional force Ff then will be
a point above the earths
center. This circle has
radius R sin = R 2/2. The distance traveled, Ff = N = mg cos
s, is one quarter of its circumference:
opposite to the motion.

1 R 2 R 2 (a.) Along the motion, the system consisting
s= 2 = . of mass m plus mass m/13 experiences a force
4 2 4
due to gravity, consisting of the sum of the force
(b.) The Cartesian coordinates of rS and rP are mg sin on mass m and mg/13 on mass m/13. If
the velocity of the system is constant, i.e. there is
no acceleration, the gravitational and frictional
rS = R( 2/2, 0, 2/2)
forces must balance:
rP = R(0, 2/2, 2/2) .
mg cos = mg(sin + 1/13)
The angle between rS and rP is 5/13 + 1/13
= = 1/2 .
12/13
arccos (rS rP )
SP = (b.) After the hanging block hits the table,
R2
= arccos (1/2) = /3 . the (massless) rope goes limp and has no fur-
ther eect on the sliding block. Opposite to the
To calculate the minimum distance between Sap- direction of motion, the net force on m is
poro and Portland along the surface of the earth, f = mg cos mg sin
we bisect the earth using a plane that contains  1 12 5  mg
these two cities as well as the earths center. The = mg = ,
2 13 13 13
intersection of the earth with the bisecting plane
is a circle with its origin at the center of the producing a constant acceleration a = g/13 op-
earth, having a circumference 2R. Since both posite to the motion. The block decelerates for
rS and rP lie in this plane, the course consists a time t = v0 /a until it comes to rest. During
of the fraction SP /2 of this circumference. that time, the distance traveled is
Therefore the distance traveled is
1 v2 1 v02 1 v02 13 v02
s = v0 t at2 = 0 = = .
SP /3 R 2 a 2 a 2 a 2 g
s= 2R = 2R = .
2 2 3
3. Two crumpled paper objects have the same
(This is 0.333/0.354 94% of the length of the force (proportional to v ) due to air resistance,
due east course.) but dierent masses 2m and m.
2. Let = arctan (5/12) be the angle with which (a.) Immediately after the two objects are re-
the plane is inclined. Since there is no acceler- leased from rest, their velocity must still be
ation (or motion) perpendicular to that plane, negligible; otherwise they would have experi-
the normal force N on block m must be equal to enced innite acceleration. Likewise, the force of
2

air resistance, proportional to v , is negligible at which is arbitrarily large when t is arbitrarily


that time. So the only nonnegligible force act- large. Therefore the distance fallen will be un-
ing on them is the force of gravity, 2mg and mg bounded. [This result is similar to that obtained
respectively, yielding an acceleration g in either in Problem Set 3, Problem 6 (K&K 2.35).]
case. So the ratio of accelerations is R = 1. 4. Since the ice is horizontal and frictionless, it
(b.) After the objects reach terminal velocity cannot exert any force on the barbell in the x
(v2 and v1 respectively), and they no longer are direction, either as the result of a contact force
accelerating, the forces due to air resistance and or a frictional force. Therefore the x coordinate
gravity must cancel: xCM of the barbells center of mass, initially at
rest at x = 0, must remain at x = 0. When mass
2mg = Kv2 m hits the ice and the barbell is horizontal,
mg = Kv1 ,
M xM + m(xM + h)
0 = XCM =
where K is the unknown constant of proportion- M +m
ality. Taking the ratio of these two equations, m
xM = h .
M +m
v2 = 21/ v1 .

The long time after the objects are dropped


is very large compared to the very soon time
at which they reach terminal velocity. So, to an
excellent approximation, the distance they drop
during the long time is proportional to the
terminal velocity. Since mass 2m drops a factor

2 further,
v2 = 2v1 .
Comparing this to the previous equation,

=2.

(c.) After gravity is turned o, the force of


air resistance Kv 2 accelerates the falling object
opposite to its direction of motion:

dv
m = Kv 2 .
dt

Dividing through by v 2 , multiplying through by


dt, and integrating,

1 K
= t+C
v m
where C is a constant of integration that will
be negligible with respect to Kt/m when t is
suciently large. Therefore, asymptotically as
t , the downward velocity will be inversely
proportional to t. Integrating this statement, the
asymptotic distance traveled will increase as ln t,
University of California, Berkeley
Physics H7A Fall 1998 (Strovink)
EXAMINATION 2

Directions. Do all three problems (weights are indicated). This is a closed-book closed-note exam
except for one 8 12 11 inch sheet containing any information you wish on both sides. You are free to
approach the proctor to ask questions but he or she will not give hints and will be obliged to write
your question and its answer on the board. Dont use a calculator, which you dont need roots,
circular functions, etc., may be left unevaluated if you do not know them. Use a bluebook. Do not
use scratch paper otherwise you risk losing part credit. Cross out rather than erase any work that
you wish the grader to ignore. Justify what you do. Box or circle your answer.

1. (30 points) A cylinder of mass m and radius


R initially is at rest on frictionless ice. About its
C.M., the cylinders moment of inertia is mR2 /2.

a. (15 points) For this part, consider the bead


and the rod to be glued together. The
a. (15 points) The cylinder receives an instan- C.M. of the bead is observed to travel uni-
taneous horizontal tap at a point on its formly around a horizontal circle that is
circumference that is a vertical distance h centered below the rods pivot. The circles
above the ice. Immediately thereafter, it is radius is much smaller than H but much
observed to roll without slipping at a veloc- larger than the bead. With what angular
ity v0 , even though the ice is frictionless. velocity does the bead move on this path?
Calculate the value of h. b. (15 points) For this part, the bead is no
longer glued to the rod; instead it spins
b. (15 points) The cylinder continues to roll about the long axis of the rod with a con-
without slipping as long as the frictionless stant, large angular velocity 0 . About this
ice remains at. Eventually the ice slopes axis, the moment of inertia of the bead is I.
upward to form a hill as shown, all the Again, the C.M. of the bead is observed to
while staying frictionless. To what maxi- travel uniformly around a horizontal circle
mum height H will the cylinder rise? centered below the pivot. What restric-
tion(s) can be placed on the radius r of this
circle?
2. (30 points) A thin massless rod of length H
hangs freely pivoted from the ceiling. At its end
is a bead of mass m, negligible in size compared
to H, acted upon by gravity.
3. (40 points) A tiny pebble moves on the fric-
tionless inner surface of a vertical cone that has
a half-angle of 45 . It is observed to be in uni-
form circular motion with constant velocity v0 .
(You are not given the radius of this circle!)

a. (15 points) what is the angular frequency


of this uniform circular motion?
b. (15 points) The orbiting pebble is now
nudged so that its new orbit diers very
slightly from the original circle. The nudge
causes the orbit radius (the perpendicular
distance from the pebble to the cone axis) to
oscillate sinusoidally about its mean value.
(All the while the pebble remains in contact
with the inside surface of the cone.) Calcu-
late the angular frequency r of this small
radial oscillation.
c. (10 points) Is the perturbed orbit closed
that is, does the perturbed orbit ever repeat
itself? Explain.
1

University of California, Berkeley


Physics H7A Fall 1998 (Strovink)
SOLUTION TO EXAMINATION 2

1. we conclude that only Ktrans = 12 mv02 is avail-


able to be converted into potential energy mgH.
Therefore the maximum height is

v02
H=
2g
.
2.

(a.) The horizontal tap produces a (horizon-


tal) linear impulse F dt J. With respect
to the center of the cylinder,it also produces a
(clockwise) angular impulse dt = Jb, where
b = hR is the impact parameter of the horizon-
tal tap. Then, in terms of J, since the cylinder
is initially at rest,

mv0 = J
I0 = J(h R)
(a.) Let be the angle between the stick and
the vertical. This part can easily be worked
Substituting I = 12 mR2 , and imposing the con-
in the C.M. of the bead, where a centrifugal
dition v0 = R0 that the cylinder rolls without
force m2 r = m2 H sin balances the horizon-
slipping, these equations become
tal component mg tan of the tension mg/ cos
mv0 = J in the stick. Or it can be worked in the lab,
where the horizontal component of the tension
1 v0
mR2 = J(h R) supplies the necessary centripetal acceleration.
2 R Or, elegantly, the circular motion of the bead
These equations are mutually consistent only if can be considered to be the superposition of an
h R = R/2, or x pendulum and, delayed by one-quarter of a
period, a y pendulum with the same amplitude.
3 When the approximation  1, valid for part
h= R
2 (a.), is applied, cancels out, and any of these
approaches yields the usual result
(b.) Since the ice is frictionless, and the force 
of gravity acts on the C.M. of the cylinder, no g
=
torques about its axis can be exerted on the H
cylinder. Therefore its angular momentum and
angular velocity = 0 remain constant even (b.) Take the origin to be the pivot point. If the
on the hill. Considering that the kinetic energy bead is spinning with constant angular velocity
of a rigid body decomposes into Ktrans + Krot , 0 about the sticks axis, the vertical component
2

I0 cos of the spin angular momentum L re- where r is the perpendicular distance of the peb-
mains constant. But the horizontal component ble to the cone axis. (In the second term, we are
Lh = I0 sin of L precesses with angular ve- using the fact that, for a 45 cone with z = r, an
locity , as in a gyroscope. The torque Lh increase r causes an increase mgz = mgr
that is required to maintain this precession is in the true potential energy.) Then a circular
the torque due to gravity, mgr = mgH sin ; the orbit occurs when
stick cant supply this torque because its end
coincides with the origin. Then dUe 2l2
0= = + mg
dr 2mr3
I0 sin = mgH sin
The angular velocity of precession is Substituting l2 = mv0 r mr2 causes the r
dependence to cancel:
mgH
=
I0 2m2 v0
0= + mg
independent of . Because sin cancels out of 2m
the previous equation, there is no restriction on g
=
it and therefore no restriction on the orbit radius v0
r = H sin ; this circular motion can occur for
any orbit radius r H provided that that 0 is as before.
large enough to allow the spin angular momen- (b.) Proceeding with the eective potential
tum to dominate the other angular momenta in method, we obtain the eective spring constant
the problem. ke for radial motion by dierentiating Ue again
3. with respect to r:

d  l2 
ke = 3 + mg
dr mr
2
3l
=
mr4
3m2 r4 2
=
mr4
= 3m2

Thus 
ke g
r = = 3 = 3
m v0

(a.) This part of the problem can be done by bal- (c.) Since the ratio of r to is irrational, an
ancing forces; the normal force of the 45 cone integer number of orbital cycles cannot occur in
on the pebble has equal horizontal and vertical the same time interval as an integer number of
components mg. When the horizontal compo- radial cycles. Therefore the orbit never repeats
nent is equated to the centripetal acceleration itself and so is not closed.
mv0 , we obtain immediately
g
=
v0
Anticipating what will be needed for part (b.),
we can also solve part (a.) using the eective
potential
l2
Ue = + mgr
2mr2
1

University of California, Berkeley


Physics H7A Fall 1998 (Strovink)
FINAL EXAMINATION

Directions. Do all six problems (weights are indicated). This is a closed-book closed-note exam
except for three 8 12 11 inch sheets containing any information you wish on both sides. You are
free to approach the proctor to ask questions but he or she will not give hints and will be obliged
to write your question and its answer on the board. Use a bluebook. Do not use scratch paper
otherwise you risk losing part credit. Cross out rather than erase any work that you wish the grader
to ignore. Justify what you do. Box or circle your answer.

Problem 1. (30 points) (see gure). Find the maximum value of b for
Northern Canada has two peculiar features: ow- which the rocket will hit the planet.
ing to lack of roads, most surface transportation
occurs by train; and the principal fauna are tiny
black ies. Problem 3. (30 points)
For decades, scientists have been designing a
Consider the elastic (kinetic energy conserving) space colony in which thousands of people
head-on collision of a locomotive of mass M and could exist while orbiting the sun. People would
velocity V with a stationary black y of mass m. live on the inside curved surface of a large air-
You may make any reasonable approximation lled cylinder (length of order 10 km, radius R of
concerning the relative magnitude of M and m. order 1 km). The cylinder would rotate about its
a. (15 points) axis with an angular velocity such that earths
With what velocity v does the y recoil from the gravitational acceleration g would be simulated
locomotive? by the centrifugal force acting near that surface.
The curved surface would have dirt for farming,
b. (15 points) and also housing, factories, parks, hills, streams,
Assuming that the (coasting) locomotive has at a lake, etc. Sunlight would enter through one
frontal area A, and there are N black ies per end; it would be controlled by mirrors and shut-
cubic meter hovering over the track, apply the ters to simulate day and night. There would be
results of part (a.) to obtain a dierential equa- clouds and weather, etc.
tion for V (neglect air resistance). Solve it to
obtain V (t). a. (5 points)
Find the angular frequency of rotation.
Problem 2. (20 points)
b. (10 points)
Although many aspects of life in this colony
would resemble life on earth, one peculiar fea-
ture would be the large Coriolis acceleration.
When v (as seen by a colony inhabitant) is per-
pendicular to , the magnitude of the Coriolis
acceleration aC can be expressed as

v
aC = g
vC

A space probe is launched with initial velocity where vC is a characteristic velocity. Find
v0 and impact parameter b toward a very dis- vC appropriate to the surface inhabited by the
tant planet of radius R and very large mass Mp colonists.
2

c. (15 points) (along the same axis) is x. As usual, satises


For residents in the colony, north is dened to the wave equation
be in the direction of ; if a resident faces north
her right hand points east. A baseball pitcher, 2 1 2
new to the colony, res a ball toward the west + =0
x2 c2 t2
with velocity v at his target a distance D away.
If he were on the surface of the earth (where the
Coriolis force is negligible), the ball would hit its where c is the (phase and group) velocity of
target. In what direction (high, low, north, or sound waves in air.
south) does the ball miss its target? By what dis-
a. (3 points)
tance d does it miss (you may assume d  D)?
Keeping in mind that the pipe is closed at both
ends, write down the boundary conditions on
Problem 4. (30 points) (0, t) and (L, t).
A mass m is connected by a massless spring b. (12 points)
of stiness k = m02 to a point of support xs . The air inside the straight closed pipe is observed
When the spring is relaxed, and xs = 0, the mass to carry a standing sinusoidal sound wave. What
is at its equilibrium position x = 0. The mass is the lowest angular frequency s with which
moves only in the x direction, without friction. this wave can vibrate?

c. (3 points)
The ends of the pipe are now opened and the
pipe is bent into a hoop. The end at x = 0 is
welded to the end at x = L, so that the pipe
forms a continuously hollow circular torus (like
Suppose that the point of support is constrained a hula hoop) with a circumference equal to L.
by an external force to obey the following mo-
tion: xs = mA sin t, where A and are con-
stants, and is not necessarily equal to 0 . The
external force does not act directly on the mass,
but it nevertheless inuences the mass because
of the spring.
a. (15 points)
Find the particular solution xp (t) which would
vanish if A were zero.
Continue to consider sound waves that propa-
b. (15 points) gate along the (bent) axis of the pipe. As long
Find the solution x0 (t) which would be correct as the circumference of the hoop is much larger
if the mass were xed at its equilibrium position than the pipe thickness, which is the case here,
and released at t = 0. (x, t) satises the same wave equation as be-
fore. However, since the pipe is now bent into a
continuously hollow torus, x = 0 and x = L now
Problem 5. (30 points) describe the same coordinate along the pipes
Consider a thin cylindrical pipe of length L, axis. More generally, (x, t) and (x + L, t) de-
closed at both ends. The air inside the pipe can scribe the displacement from equilibrium of the
support longitudinal (sound) waves that propa- same molecule.
gate along the axis of the pipe. Let (x, t) be
the displacement (along the axis of the pipe) In light of the above, write down the relationship
of an air molecule whose equilibrium coordinate between (x, t) and (x + L, t).
3

d. (12 points) Note that, in spherical polar coordinates,


The air inside the bent pipe is observed to carry
a travelling sinusoidal sound wave. Keeping in 1 2
F= (r Fr ) + . . .
mind the result of part (c.), what is the lowest r2 r
angular frequency t that can characterize this
wave? What is the ratio of t to the result s
of part (b.)?

Problem 6. (30 points)


Nonviscous uid matter is in spherically sym-
metric, nonrelativistic ow toward a black hole
of mass M . Only the gravitational attraction
of the black hole itself (as opposed to the grav-
itational attraction of other uid elements) is
important to the uid motion. M is growing
slowly enough to be taken as constant.
a. (10 points)
Consider , the potential energy per unit mass
of uid due to the gravitational attraction of the
black hole. Starting from the standard formula
for the gravitational force between two point
objects, show that

GM
(r) =
r

where r is the distance from the black hole. Note


that in spherical polar coordinates

u
u = r + ...
r

(You may use this result for in subsequent


parts of the problem.)
b. (10 points)
Because M is taken as constant, the uid ow is
steady:
v
= =0
t t
where is the mass density. Also, the uid pres-
sure p is known not to vary either with position
or time.
Away from the black hole, determine the depen-
dence of uid |velocity| v upon r.
c. (10 points)
Away from the black hole, determine the depen-
dence of uid mass density upon r.
1

University of California, Berkeley


Physics H7A Fall 1998 (Strovink)
SOLUTION TO FINAL EXAMINATION

Problem 1. Problem 2.
At the instant that the probe barely grazes the
planet, it will have radius R and velocity vf
a.
directed tangentially to the planet. Angular
We consider this head-on collision in the center
momentum conservation requires
of mass. The center of mass velocity is
mv0 b = mvf R
M
V =V V b
M +m v f = v0
R
Using this approximation, in the C.M. the y Substituting for vf in the equation for energy
approaches the locomotive with speed V . Since conservation, we obtain
the collision is elastic, it bounces back with the
same speed. Transforming back to the lab, the 1 1 GMp m
mv 2 = mvf2
y has velocity 2 0 2 R
1 2 1 2 b2 GMp
v V + V = 2V v = v
2 0 2 0 R2 R
1 2  b2  GMp
v0 1 =
b. 2 R2 R
In each collision, the momentum 2mV that is b2 2GMp
1=
gained by the y is lost by the locomotive: R2 v02 R

P = M V = 2mV 2GMp
b=R 1+ 2
v0 R
V m
= 2
V M

In a time interval t, the volume swept out by Problem 3.


the front of the train is AV t; this volume con-
tains N AV t ies. So, for N AV t collisions,
a.
V m
= 2 N AV t mR 2 = mg
V M 
V m g
2
= 2N A t =
 V M R
dV m
= 2N A dt b.
V2 M
1 1 m v
= 2N A t 2v = aC = g
V V0 M vC
1 g
V (t) = vC =
2N A M t + V10
m 2
V0 g R
V (t) = m
vC =
1 + 2N AV0 M t 2 g
1
where V0 is the velocity at t = 0. vC = gR
2
2

c. be required otherwise, x0 (0) = 0 as well as


x0 (0) = 0. The general solution to the homoge-
FC = 2m( v) neous equation of motion (A = 0) is
 east
is north, and v is east; north  is down.
This is the direction in which the ball misses. xh (t) = C cos 0 t + D sin 0 t

g
aC = 2v = 2v The general solution to the full equation is ob-
R
tained by adding xh to xp . Applying initial
1
d = aC t2 conditions,
2 
1 g 2
= 2v t kA sin t
2 R x0 (t) = + C cos 0 t + D sin 0 t
02 2
D
t= x0 (0) = 0 C = 0
v
g D2 kA
x0 (0) = 0 0 = + 0 D
d=v
R v2 02 2
 kA
D2 g D=
d= 2
0 0 2
v R
0 sin t sin 0 t
x0 (t) = kA
(We ignore the centrifugal force on the ball, be- 0 (02 2 )
cause it is the same on the colony as on earth,
and the pitcher already compensates for it.) As a
sanity check, if D = 20 m and v = 40 m/sec (ap-
propriate to baseball), and R = 1000 m, we ob- Problem 5.
tain d 1 m. Indeed d is much smaller than D.
Nevertheless, from the standpoint of the pitcher,
the Coriolis force has a big eect on his control. a.

Problem 4. (x = 0, t) = (x = L, t) = 0
The equation of motion for x(t) is

mx = k(x xs ) = m02 (x xs ) b.
x = 02 x + 02 mA sin t  
(x, t) = sin kx 0 exp (it)
x + 02 x = kA sin t
sin kL = 0
a. kL = n, n = 1, 2, . . .
ck
try xp (t) = B sin t c
2 s =
( + 02 )B sin t = kA sin t L
kA
B= 2
0 2 c.
kA sin t
xp (t) = 2
0 2 (x, t) = (x + L, t)

b.
Because an innite force from the spring would d.
3

  c.
(x, t) =  0 exp (i(kx t)) In steady ow there can be no buildup of mass
exp(ikx) = exp(ik(x + L) density . Therefore the mass ow
1 = exp(ikL) 2
v (kg/m sec) 4r2 (m2 )
kL = 2n, n = 1, 2, . . .
ck through a spherical surface of radius r must be
2c independent of r. So, using the result of part
t = (a.),
L
t = 2s v r2
r3/2
More formally, but equally acceptably, one can
Problem 6. reach the same conclusion by applying the con-
tinuity equation
a.

Per unit mass of uid, the force f is + (v) = 0
t
GM
f = r and using the fact that for steady ow the rst
r2 term vanishes.
We seek a function (r) such that

= f

or equivalently, using spherical symmetry,



= fr dr

Clearly
GM
(r) =
r
satises either of these conditions.
b.
Since the ow is steady, we can use Bernoullis
equation (either along a streamline at constant
(, ), or, since the ow is irrotational, anywhere
outside the black hole):

1 2
v + p + = constant
2

Only the rst and third terms are not con-


stant, so they must have the same r dependence.
Therefore v 2 and have the same r dependence.
So
v r1/2
University of California, Berkeley
Physics H7B, Spring 1999 (Strovink)

General Information

Web site for this class: First link on http://d0lbln.lbl.gov/ .

Instructors: Prof. Mark Strovink, 437 LeConte; (LBL) 486-7087; (home, before 10) 486-8079; (UC) 642-9685.
Email: strovink@lbl.gov . Web: http://d0lbln.lbl.gov/ . Office hours: M 3:15-4:15, Th 10-11.
Mr. Robin Blume-Kohout, 208 LeConte; (UC) 642-5430. Email: rbk@socrates.berkeley.edu. Office
hours (to be held initially in 208 LeConte): M 1-2, W 3-4. You may also get help in the 7B Course Center, 206
LeConte.

Lectures: Tu-Th 11:10-12:30, 2 LeConte. Lecture attendance is essential, since not all of the course content can be
found in the course text or handouts.

Labs: In the second week, in 262 LeConte, please enroll in one of only 2 special H7B lab sections [(A) #401, F 2-4;
(B) #402, F 4-6]. Both sections are taught by Mr. Blume-Kohout. If you can make both of these lab (and section,
see below) slots, please attempt to enroll in the earlier of these lab slots. Depending on crowding, you may be asked
to move to the later lab. During "off" weeks not requiring lab apparatus, your lab section will still meet in the same
room, 262 LeConte.

Discussion Sections: Beginning in the second week, please enroll in the 1 hr H7B discussion section corresponding
to your H7B lab section: (A) #401, M 2-3, 385 LeConte; (B) #402, W 4-5, 343 LeConte. You are especially
encouraged to attend discussion section regularly. There you will learn techniques of problem solving, with
particular application to the assigned exercises.

Text (required): E.M. Purcell, Electricity and Magnetism (Berkeley Physics Course Volume 2), Second Edition
(McGraw-Hill, 1985). At the beginning of the semester we will also use Chapters 22 through 26 (pp 493-592) of
Resnick, Halliday and Krane, Physics (Volume 1), Fourth Edition (Wiley, 1992). The publisher has granted
permission to make it possible for students to purchase a Xerox copy of these pages from Copy Central.

Problem Sets: Thirteen problem sets are assigned and graded, with solutions provided on the web and at Copy
Central. They are due on Thursday at 5 PM on weeks in which there is no exam, beginning in week 2. Deposit
problem sets in the box labeled "H7B" outside 208 LeConte. You are encouraged to attempt all the problems.
Students who do not do them find it almost impossible to learn the material and to succeed on the examinations.
Discuss these problems with your classmates as well as with the teaching staff; however, when the time comes to
write up your solutions, work independently. Credit for collective writeups, which are easy to identify, will be
divided among the collectivists. Late papers will not be graded. Your lowest problem set score will be dropped, in
lieu of due date extensions for any reason.

Syllabus: H7B has one syllabus card, which is mandatory. It will be collected at the time of the midterm
examination. This card pays for the experiment descriptions and instructions that you will receive from your GSI at
the beginning of each laboratory. Also, we expect you also to have the opportunity to purchase at Copy Central a
copy of the above mentioned 100 pages of Resnick, Halliday and Krane. Finally, copies of solutions to the problem
sets will also be available for purchase at Copy Central. These solutions will also be available on the Web.

Exams: There will be one 80-minute midterm examination and one 3-hour final examination. Before confirming
your enrollment in this class, please check that its final Exam Group 12 does not conflict with the Exam Group for
any other class in which you are enrolled. Please verify that you will be available for the midterm examination (Th 4
Mar, 11:10-12:30), and for the final examination, W 19 May, 8-11 AM. Except for unforeseeable emergencies, it
will not be possible for the midterm or final exams to be rescheduled. Passing H7B requires passing the final exam.

Grading: 25% midterm; 25% problem sets; 45% final exam; 5% lab. Grading is not "curved" -- it does not depend
on your performance relative to that of your H7B classmates. Rather it is based on comparing your work to that of a
generation of earlier lower division Berkeley physics students, with due allowance for educational trends.
COURSE OUTLINE

Week Week Lecture Topic Problem Due Lab


No. of... chapter (RHK = Resnick/Halliday/Krane, Physics Vol. 1 ) Set No. 5 PM on...
(Purcell = Electricity and Magnetism )
(do experiment in lab="expt")
MARTIN LUTHER KING HOLIDAY (18-Jan) (have discussion in lab="disc")
1 18-Jan RHK (22), 25.7 Thermal expansion; heat transfer no lab
23.1-23.4 Kinetic definition of temperature

2 25-Jan 23.5-23.6 Energetics of an ideal gas; equipartition disc


24.1-24.4 Maxwellian distribution 1 Th 28-Jan

3 1-Feb 25.4-25.6 Heat capacities of an ideal gas; first law expt


26.1-26.4 Second law of thermodynamics 2 4-Feb

4 8-Feb RHK 26.5-26.9 Entropy disc


Purcell 1.1-1.8 Electric charge 3 11-Feb
PRESIDENTS' DAY HOLIDAY (15-Feb)
5 15-Feb 1.9-1.15 Electric fields disc
2.1-2.6 Electric potential 4 18-Feb

6 22-Feb 2.7-2.13 Gauss' law, Laplace's equation expt


3.1-3.4 Electic fields around conductors 5 25-Feb

7 1-Mar 3.5-3.8 Systems of conductors; capacitors disc


4-Mar 11:10 AM - 12:30 PM MIDTERM EXAMINATION (covers PS 1-5)

8 8-Mar 4.1-4.11 Electric currents expt


Appendix A Special relativity 6 11-Mar

9 15-Mar Appendix A Special relativity disc


5.1-5.5 Electric field in different frames of reference 7 18-Mar

22-Mar SPRING RECESS (22-26 Mar) disc


25-Mar

10 29-Mar 5.6-5.9 Fields of moving charges expt


6.1-6.2, 6.4-6.5 Magnetic fields 8 1-Apr

11 5-Apr 6.3, 6.6-6.9 Vector potential; magnetic field transformation disc


7.1-7.5 Faraday's law 9 8-Apr

12 12-Apr 7.6-7.10 Inductance expt


8.1-8.5 AC circuits 10 15-Apr

13 19-Apr 9.1-9.4; 9.6 Maxwell's equations disc


10.1-10.6 Electric dipoles 11 22-Apr

14 26-Apr 10.7-10.12 Electric fields in dielectric media disc


11.1-11.6 Magnetic dipoles (but not monopoles) 12 29-Apr

15 3-May 11.7-11.11 Magnetization disc


LAST LECTURE (review) 13 6-May

10-May INSTRUCTION ENDS (10-May) makeup

FINAL EXAMS BEGIN (14-May)


17-May
19-May 8-11 AM FINAL EXAM (Group 12) (covers PS 1-13)

Physics H7B Spring 1999 (Strovink)


University of California, Berkeley
Physics H7B Spring 1999 (Strovink)
PROBLEM SET 1

1. RHK problem 22.9 side b is longer by b. Show that if we neglect the


It is an everyday observation that hot and cold small quantity a b / ab (see the gure), then
objects cool down or warm up to the temperature
of their surroundings. If the temperature dier- A = 2AT .
ence T between an object and its surroundings
(T = Tobj Tsur ) is not too great, the rate of
cooling or warming of the object is proportional,
approximately, to this dierence; that is,

d T
= A(T ),
dt

where A is a constant. Ths minus sign appears


because T decreases with time if T is pos-
itive and increases if T is negative. This is
known as Newtons law of cooling. 4. RHK problem 25.47
(a) On what factors does A depend? What are The average rate at which heat ows out through
its dimensions? the surface of the Earth in North America is 54
(b) If at some instant t = 0 the temperature mW/m2 , and the average thermal conductivity
dierence is T0 , show that it is of the near surface rocks is 2.5 W/mK. Assum-
ing a surface temperature of 10 C, what should
be the temperature at a depth of 33 km (near
T = T0 exp (At)
the base of the crust)? Ignore the heat gener-
ated by radioactive elements in the crust; the
at a time t later. curvature of the Earth can also be ignored.
2. RHK problem 22.28 5. RHK problem 25.50
As a result of a temperature rise of 32 C, a A cylindrical silver rod of length 1.17 m and
bar with a crack at its center buckles upward, cross-sectional area 4.76 cm2 is insulated to pre-
as shown in the gure. If the xed distance vent heat loss through its surface. The ends are
L0 = 3.77 m and the coecient of linear ther- maintained at temperature dierence of 100 C
mal expansion is 25 106 per C, nd x, the by having one end in a water-ice mixture and
distance to which the center rises. the other in boiling water and steam.
(a) Find the rate (in W) at which heat is trans-
ferred along the rod.
(b) Calculate the rate (in kg/sec) at which ice
melts at the cold end.
Hints: The thermal conductivity of silver is 428
W/mK. The latent heat of fusion of water is
333 kJ/kg.
6. RHK problem 25.58
3. RHK problem 22.30 A container of water has been outdoors in cold
The area A of a rectangular plate is ab. Its coef- weather until a 5.0-cm-thick slab of ice has
cient of linear thermal expansion is . After a formed on its surface (see the gure). The air
temperature rise T , side a is longer by a and above the ice is at 10 C. Calculate the rate of
formation of ice (in centimeters per hour) on the
bottom surface of the ice slab. Take the thermal
conductivity and density of ice to be 1.7 W/mK
and 0.92 g/cm3 , respectively. Assume that no
heat ows through the walls of the tank.

7. RHK problem 23.16


A mercury-lled manometer with two unequal-
length arms of the same cross-sectional area is
sealed o with the same pressure p0 of perfect
gas in the two arms (see the gure). With the
temperature constant, an additional 10.0 cm3 of
mercury is admitted through the stopcock at the
bottom. The level on the left increases 6.00 cm
and that on the right increases 4.00 cm. Find
the original pressure p0 .
1

University of California, Berkeley


Physics H7B Spring 1999 (Strovink)
SOLUTION TO PROBLEM SET 1

1. RHK problem 22.9 equation, with dt substituted for dt,


It is an everyday observation that hot and cold
objects cool down or warm up to the temperature d T
of their surroundings. If the temperature dier- = A dt
T
ence T between an object and its surroundings  t  t
d T
(T = Tobj Tsur ) is not too great, the rate of = A dt
T
cooling or warming of the object is proportional,   0  0

approximately, to this dierence; that is, ln T (t) ln T (0) = At


 T (t) 
ln = At
T (0)
d T T (t)
= A(T ), = exp (At)
dt T (0)
T (t) = T0 exp (At) .

where A is a constant. Ths minus sign appears


because T decreases with time if T is pos- 2. RHK problem 22.28
itive and increases if T is negative. This is As a result of a temperature rise of 32 C , a
known as Newtons law of cooling. bar with a crack at its center buckles upward,
as shown in the gure. If the xed distance
L0 = 3.77 m and the coecient of linear ther-
(a) On what factors does A depend? What are mal expansion is 25 106 per C , nd x, the
its dimensions? distance to which the center rises.
Solution: The LHS (and therefore the RHS)
of the above equation have dimensions C /sec,
so A must have dimension sec1 . Suppose that
the heat owing between the object and its sur-
roundings is conducted by a thermal barrier
(i.e. a skin on the object that tends to in-
sulate it from its surroundings). Then, from
RHK Eq. 25.45, A should be proportional to the
thermal conductivity of that barrier and to its
area, and inversely proportional to the barriers
Solution: In Physics H7B, all problems in-
thickness.
volving numbers should be solved completely in
terms of algebraic symbols before any numbers
(b) If at some instant t = 0 the temperature are plugged in (otherwise it is much more di-
dierence is T0 , show that it is cult to give part credit). Let
L0 = xed distance = 3.77 m
x = distance to which the center rises
L = thermally expanded total length of the
T = T0 exp (At)
buckled bar (twice the hypotenuse of the right
triangle whose legs are x and L0 /2)
= coecient of linear thermal expansion =
at a time t later. 25 106 per C
Solution: Rearranging and solving the above T = temperature rise = 32 C
2

Then Then

A + A = (a + a)(b + b)
L = L0 + T = ab + a b + b a + a b
x2 = (L/2)2 (L0 /2)2 A = ab
L0  A + A A = ab + a b + b a + a b ab
x= (1 + T )2 1
2 A = a b + b a + a b
= 0.0754 m .  b a a b 
= ab + +
b a ab
 b a 
A ab +
b a
a b
= = T
a b
3. RHK problem 22.30
A A( T + T )
The area A of a rectangular plate is ab. Its coef-
cient of linear thermal expansion is . After a A
2 T .
temperature rise T , side a is longer by a and A
side b is longer by b. Show that if we neglect the
small quantity a b / ab (see the gure), then 4. RHK problem 25.47
The average rate at which heat ows out through
the surface of the Earth in North America is 54
mW/m2 , and the average thermal conductivity
A = 2AT . of the near surface rocks is 2.5 W/mK. Assum-
ing a surface temperature of 10 C, what should
be the temperature at a depth of 33 km (near
the base of the crust)? Ignore the heat gener-
ated by radioactive elements in the crust; the
curvature of the Earth can also be ignored.
Solution: Let
H/A = heat ow per unit area through Earths
surface = 54 103 W/m2
k = thermal conductivity of near surface rock =
2.5 W/mK
T0 = temperature at earths surface = 10 C
D = depth at which we wish to know the tem-
perature = 33 103 m
T = temperature at depth D
Then, using RHK Eq. 25.45,

H T
=k
Solution: Let A x
A = original area of rectangular plate T T0
=k
a = original width of plate D
b = original height of plate HD
= T T0
A + A = thermally expanded area of plate A k
a + a = thermally expanded width of plate HD
T0 + =T
b + b = thermally expanded height of plate A k
= coecient of linear thermal expansion 723 C = T .
3

5. RHK problem 25.50 6. RHK problem 25.58


A cylindrical silver rod of length 1.17 m and A container of water has been outdoors in cold
cross-sectional area 4.76 cm2 is insulated to pre- weather until a 5.0-cm-thick slab of ice has
vent heat loss through its surface. The ends are formed on its surface (see the gure). The air
maintained at temperature dierence of 100 C above the ice is at 10 C. Calculate the rate of
by having one end in a water-ice mixture and formation of ice (in centimeters per hour) on the
the other in boiling water and steam. bottom surface of the ice slab. Take the thermal
conductivity and density of ice to be 1.7 W/mK
(a) Find the rate (in W) at which heat is trans-
and 0.92 g/cm3 , respectively. Assume that no
ferred along the rod.
heat ows through the walls of the tank.
Solution: Let
L = length of cylindrical silver rod = 1.17 m
A = area of rod = 4.76 104 m2
k = thermal conductivity of silver = 428 W/mK
T = temperature dierence between ends of
rod = 100 C .
H = dQ/dt = rate at which heat is transferred
along the rod.
Then, using RHK Eq. 25.45

T
H = kA
x
T
= kA
L
= 17.4 W .

(b) Calculate the rate (in kg/sec) at which ice


melts at the cold end.
Solution: Let
Lf = latent heat of fusion of water = 333 103
J/kg
dm/dt = rate in kg/sec at which ice melts at the
cold end
Then, using RHK Eq. 25.7,

Q = Lf m Solution: Let
dQ dm A = area of slab of ice on waters surface
= Lf h = present thickness of slab = 0.05 m
dt dt
dm T = temperature of air above ice = 10 C
H = Lf T0 = temperature at which water freezes = 0 C
dt
k = thermal conductivity of ice = 1.7 W/mK
H dm
= = density of ice = 0.92 103 kg/m3
Lf dt
Lf = latent heat of fusion of water = 333 103
dm
5.23 105 kg/sec = . J/kg
dt H = dQ/dt = heat ow (in W) through the ice
dm/dt = rate of formation of ice (in kg/sec) on
Hints: The thermal conductivity of silver is 428 the bottom surface of the slab
W/mK. The latent heat of fusion of water is dh/dt = rate of change of ice thickness (in
333 kJ/kg. m/sec).
4

Then, using RHK Eqs. 25.45 and 25.7,

T
H = kA
x
T0 T
= kA
h
Q = Lf m
dQ dm
= Lf
dt dt
dm
H = Lf
dt
T0 T dm
kA = Lf
h dt
hA = m
dh 1 dm
=
dt A dt
1 dm
= Lf
ALf dt
kA T0 T
=
ALf h
k T0 T
=
Lf h
= 1.11 106 m/sec
= 0.400 cm/hr . Solution: Let
= density of Hg = 13.6 103 kg/m3
g = acceleration of gravity at earths surface =
9.81 m/sec2
L0 = initial height of gas in left arm of manome-
Note that the inverse dependence of dh/dt upon ter = 0.50 m
h requires h to increase only as the square root R0 = initial height of gas in right arm of
of the time t. Our numerical result for the rate manometer = 0.30 m
of growth of the ice thickness is valid only when L = nal height of gas in left arm of manometer
the ice has a particular thickness (5 cm). = 0.44 m
R = nal height of gas in right arm of manome-
ter = 0.26 m
7. RHK problem 23.16 A = cross-sectional area of each manometer arm
A mercury-lled manometer with two unequal- p0 = initial pressure in both arms of manometer
length arms of the same cross-sectional area is pL = nal pressure in left arm of manometer
sealed o with the same pressure p0 of perfect gas pR = nal pressure in right arm of manometer
in the two arms (see the gure). With the tem- NL = no. of gas molecules in left arm of manome-
perature held constant, an additional 10.0 cm3 ter
of mercury is admitted through the stopcock at NR = no. of gas molecules in right arm of
the bottom. The level on the left increases 6.00 manometer
cm and that on the right increases 4.00 cm. Find kB = Boltzmanns constant
the original pressure p0 . T = (constant) temperature
5

Applying the perfect gas law,

p0 AL0 = NL kB T
p0 AR0 = NR kB T
pL AL = NL kB T
pR AR = NR kB T
p0 L0 = pL L
p0 R0 = pR L
L0
p0 = pL
L
R0
p0 = pR
R
 R0 L0 
(I) pR pL = p0 .
R L

Using Archimedes principle (rst equation on


RHK page 387), the dierence (L0 L)(R0 R)
in nal height of Hg between the two arms is
proportional to the nal pressure dierence:
 
A(pR pL ) = gA (L0 L) (R0 R)
 
(II) pR pL = g (L0 L) (R0 R) .

Combining equations (I) and (II),

 R0 L0   
p0 = g (L0 L) (R0 R)
R L
(L0 L) (R0 R)
p0 = g
R0 /R L0 /L
= 1.526 105 Pa
= 1.506 atm .
University of California, Berkeley
Physics H7B Spring 1999 (Strovink)
PROBLEM SET 2

1. RHK problem 24.18


2. RHK problem 24.21
3. RHK problem 24.25
4. RHK problem 23.17
5. RHK problem 23.33
6. RHK problem 23.37
7. RHK problem 25.16
8. RHK problem 25.21
1

University of California, Berkeley


Physics H7B Spring 1999 (Strovink)
SOLUTION TO PROBLEM SET 2

1. RHK problem 24.18 (c)  v0 2 2 


Solution: For ease of notation, here we denote v Cv dv
the mean of any function f (v) of the speed v of v  = 0 v0
2

0
Cv 2 dv 
a gas molecule by 1 5
5 v0
= 1 3
 3 v0
f (v  )n(v  )dv  3 2
f (v) 0  = v
n(v  )dv  5 0
0 
vrms v 2 

where n(v) (called dN/dv in lecture) is the distri- 3
vrms = v0 .
bution of v. If this formula is used, n(v) does not 5
need to be normalized. With this notation, for (a) 
example, v v. Proceeding with the problem, v0
N Cv 2 dv 
0
 1 3
vrms v 2  (RHK Eq. 23.15) = Cv
3 0
0 (v v)2  3N
=C.
(v v)  = v  2vv + v 
2 2 2 v03
= v 2  v2v + v 2 3. RHK problem 24.25
= v  2v + v
2 2 2 Solution:
= v 2  v 2 n(E) E 1/2 exp (E/kT ) (RHK Eq. 24.27)

0 v 2  v 2 Erms E 2 
v 2 v 2   2 1/2
E E exp (E  /kT )dE 
 E  = 0 1/2
2
v v 2  0
E exp (E  /kT )dE 
v vrms . 1/kT
 5/2
E exp (E  )dE 
E  = 0 1/2
2
The equality occurs only when (v v)2  = 0, E exp (E  )dE 
0 2  
i.e. all the molecules have the average speed v.
d /d 2 0 E 1/2 exp (E  )dE 
= 
E 1/2 exp (E  )dE 
2. RHK problem 24.21  0
Solution: Using the notation introduced above, Z E 1/2 exp (E  )dE 
(b) 0
 v0  2  d2 Z/d 2
v Cv dv E 2  = .
v = 0 v0 Z
0
Cv 2 dv 
The remaining denite integral Z has dimension
1 4
4 v0 (energy)3/2 . Since the limits of the integral are
= 1 3
3 v0
not nite, the only available quantity with which
3 a dimensionful scale may be set is , which has
= v0 . dimension 1/energy. Therefore the integral must
4
2

be equal to 3/2 multiplied by some constant C: vesc = escape velocity at earths surface
 2   m = generic molecular mass
d /d 2 C 3/2 NA = Avogadro constant
E  =
2
C 3/2 = 6.022 1023 molecules/mole
 3  5  7/2 
2 2 C MHyd = molar mass of H2 = 0.0020 kg/mole
= (RHK Table 23.1)
C 3/2
15 2 MOxy = molar mass of O2 = 0.0320 kg/mole
= (RHK Table 23.1)
4
kB = Boltzmann constant = 1.38 1023 J/K
15 1 Hyd
Tesc (earth) = temperature ( K) at which rms
Erms =
4 H2 velocity is equal to escape velocity at earths

15 surface
= kT . Oxy
Tesc (earth) = temperature (( K) at which rms
4
O2 velocity is equal to escape velocity at earths
4. RHK problem 23.17 surface
Solution: In Physics H7B, all problems in- Hyd
Tesc (m) = temperature (( K) at which rms H2
volving numbers should be solved completely in velocity is equal to escape velocity at moons
terms of algebraic symbols before any numbers surface
are plugged in (otherwise it is much more di- Oxy
Tesc (m) = temperature (( K) at which rms O2
cult to give part credit). Let velocity is equal to escape velocity at moons
T = temperature of interstellar space = 2.7 K surface
M = molar mass of H2 = 0.0020 kg/mole (RHK Then
Table 23.1)
NA = Avogadro constant
= 6.022 1023 molecules/mole
m = mass of H2 molecule = M/NA
kB = Boltzmann constant = 1.38 1023 J/K
Then from RHK Eq. 23.20, 1 GMe m
2
mvesc =
1 3 2 Re
mv 2  = kB T 2GM
2 2 2
vesc =
e
3kB T Re
v  =
2

m = 2gRe
vrms v 2  1 3
 2
mvrms = kB Tesc
3kB T 2 2
= vrms = vesc (stated by problem)
m
 1 3
3kB NA T 2
mvesc = kB Tesc
= 2 2
M
1 3
= 183.5 m/sec . m 2gRe = kB Tesc
2 2
2mgRe
5. RHK problem 23.33 = Tesc .
3kB
Solution: Let
Re = radius of earth = 6.37 106 m
Rm = radius of moon = 1.74 106 m
GMe /Re2 = g = gravitational acceleration at
earths surface = 9.81 m/sec2
gm = gravitational acceleration at moons sur-
face = 0.16g We use this general result to evaluate each of
3

the four cases posed: under the line. Similarly, for path 2,

W = p dV
MHyd path
m=   
NA 8 2 2
2MHyd gRe = p dV p dV p dV
Hyd
Tesc (earth) = 2 8 2
3kB NA
= (12.5 kPa)(6 m3 ) + (5 kPa)(6 m3 ) 0
= 1.003 104 K
= 45 kJ .
Oxy 2MOxy gRe
Tesc (earth) =
3kB NA
7. RHK problem 25.16
= 1.604 105 K Solution: Let
Hyd 2MHyd gm Rm mv = (unknown) mass of vaporized material
Tesc (moon) =
3kB NA (ice), in kg
= 438 K mf = mass of fused material (ice) = 0.15 kg
2MOxy gm Rm Lv = latent heat of vaporization of water =
Oxy
Tesc (moon) = 2256 103 J/kg
3kB NA
Lf = latent heat of fusion of water = 333 103
= 7011 K . J/kg
c = specic heat capacity of water = 4190
J/kgC
At an altitude in the Earths atmosphere where Tv = temperature of steam = 100 C
the temperature is 1000 K, the preceding re- Tf = temperature of ice = 0 C
sults imply
 that the rms
velocity would be only T = nal temperature of steam-ice mixture =
a factor Tesc /T 10 below the escape ve- 50 C
locity; because of leakage out of the tail of the The fact that the container is thermally insu-
velocity distribution, little hydrogen would be lated means that the total heat Q transferred
expected to remain. For oxygen, the rms veloc- out of the steam molecules is transferred into
ity would be a factor 160 below the escape the ice molecules:
velocity, allowing that molecule to survive as an
atmospheric component. Q(lost by steam) = Q(gained by ice)
mv (Lv + c(Tv T )) = mf (Lf + c(T Tf ))
6. RHK problem 23.37 Lf + c(T Tf )
Solution: For path 1, the work W done on the mv = mf
Lv + c(Tv T )
gas is
= 0.033 kg .

W = p dV (RHK 23.24) 8. RHK problem 25.21
path Solution: Let
 8  8  2 Q = (unknown) heat transferred into sample
= p dV p dV p dV Ti = initial temperature = 6.6 K
2 8 8
Tf = nal temperature = 15 K
= (12.5 kPa)(6 m3 ) 0 + (20 kPa)(6 m3 ) m = mass of Al = 0.0012 kg
= 45 kJ , C = heat capacity per mole of Al
= coecient of T 3 in expression for C =
3.16 105 J/moleK4
where we have evaluated each straight-line seg- MAl = molar mass of Al = 0.0270 kg/mole
ment by reading p o the graph, multiplying (RHK Appendix D)
it by the dierence in V to compute the area c = heat capacity per kg of Al = C/MAl
4

With these denitions,


 Tf
Q=m c(T ) dT (RHK Eq. 25.4)
Ti
 Tf
m
= C(T ) dT
MAl Ti
C(T ) = T 3
 Tf
m
Q= T 3 dT
MAl Ti
m  4 
= Tf Ti4
4MAl
= 0.0171 J .
University of California, Berkeley
Physics H7B Spring 1999 (Strovink)
PROBLEM SET 3

1. RHK problem 25.27


2. RHK problem 25.34
3. RHK problem 25.37
4. RHK problem 25.43
5. RHK problem 26.16
6. RHK problem 26.19
7. RHK problem 26.23
8. RHK problem 26.27
1

University of California, Berkeley


Physics H7B Spring 1999 (Strovink)
SOLUTION TO PROBLEM SET 3

1. RHK problem 25.27 transferred between the gas and the ice water,
Solution: In Physics H7B, all problems in- but slowly enough to allow the pressure neverthe-
volving numbers should be solved completely in less to be dened (as it is in RHK Fig. 25b); and
terms of algebraic symbols before any numbers that slowly means slowly enough that the gas
are plugged in (otherwise it is much more di- and the ice water always have the same temper-
cult to give part credit). Let ature. If so, the quick compression of the gas
Q = (unknown) heat added to gas (J) would occur along an adiabat, while the slow
n = no. of moles of gas = 4.34 expansion would occur along an isotherm. Then
Cp = molar specic heat of gas at constant pres-  V2  V2  V1
sure (J/moleK) W = p dV p dV p dV .
V1 V2 V2
T = change in temperature of gas = 62.4 K
R = universal gas constant = 8.314 J/moleK Further assuming that the gas is ideal,
Eint = internal energy of gas (J) pV = p1 V1 (adiabat)
M = molecular weight of gas (kg/mole) pV = p1 V1 (isotherm)
v 2  = mean square velocity of gas molecules  V2  V1
(m2 /sec2 ) p1 V1 p1 V1
W =
dV 0 dV
(a) V1 V V2 V
Q = nCp T (RHK Eq. 25.17) 1  1 1  V1
= p1 V1 1 p1 V1 ln V
7 1 V21 V1 2
Cp = R (RHK Eq. 25.21)
2 
p1 V 1 V 11  V1
7 = 1 p1 V1 ln .
Q = nRT 1 V2 1 V2
2
= 7880 J . The above is correct, given the assumptions,
but it does not solve the problem; we are sup-
(b) plied neither the initial volume nor the number
of moles of gas. Instead we are told that m =
5 0.122 kg of ice in the surrounding ice water are
Eint = nRT (RHK Eq. 23.36)
2 melted in one cycle. The heat Q = Lf m re-
5 quired to melt this ice, where Lf = 333 kJ/kg is
Eint = nRT
2 the latent heat of fusion of water, must be trans-
= 5629 J . ferred from the gas (we call it Q because +Q
is dened to be the heat transferred to the gas).
(c) Around one cycle, the nal temperature of gas is
the same as the initial; its internal energy, which
1  3
n M v 2  = nRT (RHK Eq. 23.31) depends only on the temperature, can undergo
2 2 no net change. Therefore, around the cycle, the
1  3
n M v 2  = nRT work W done on the gas is given without any
2 2 assumptions by
= 3377 J .
Eint = 0
Eint = Q + W (1st Law)
2. RHK problem 25.34
W = Q
Solution: Plunging blindly ahead, we could
start by assuming that quickly means quickly = Lf m
enough so that a negligible amount of heat is = 40626 J .
2

3. RHK problem 25.37 (d)


Solution: Q
C (RHK Eq. 25.8)
(a) nT
 V2 6nRT1
=
W = p dV n(4T1 T1 )
V1
= 2R .
V p
= (from problem)
V1 p1 4. RHK problem 25.43
 V2
p1 Solution: This problem is overconstrained:
W = V dV that is, too many pieces of information are pro-
V1 V1
1 p1  2  vided. For example, TC need not have been
= V2 V12 supplied; it is uniquely determined by the facts
2 V1
1  V   that process BC is adiabatic; that VB = VA ; that
2 2
= p1 V1 1 pC = pA ; and that the gas is ideal monatomic.
2 V1 This is illustrated by the following calculation
p1 V1 = nRT1 (not required as part of the solution):
1  V  
2 2
W = nRT1 1 pB VB = pC VC (adiabatic)
2 V1
pB VA = pA VC
V2 = 2V1
pV = nRT
1
W = nRT1 (4 1) TB
2 p B = pA
3 TA
W = nRT1 .  nRT 
2 VA =
A
pA
(b)  nRT 
C
VC =
pA
Eint =
3
nRT (ideal monatomic gas) TB  nRTA   nRT 
C
2 pA = pA
TA pA pA
3
Eint = nR(T2 T1 ) TB
2 T = TC
TA A
nRT2 = p2 V2  T 1/
B
= (2p1 )(2V1 ) T C = TA
TA
= 4p1 V1 = 454.71497 K .
= 4nRT1
If we needed to get exact answers, we would
T2 T1 = 3T1 need to plug in this exact value of TC , rather
3 than the approximate value of 455 K supplied
Eint = nR(3T1 )
2 in the problem. To proceed further, we choose
9 not to use one known piece of information not
= nRT1 .
2 to use (as we did above) the specic relationship
between p, V , T , and for an adiabatic transi-
(c) tion. Because this choice is subjective and not
unique, when our solutions are expressed in al-
Eint = Q + W (1st Law)
gebraic symbols we expect them also not to be
Q = Eint W unique. However, as long as the exact value of
9  3  TC is plugged in, we expect any valid solution to
= nRT1 nRT1
2 2 yield the same numerical results.
= 6nRT1 . Let
3

TA = temperature at point A = 300 K Process CA:


TB = temperature at point B = 600 K
TC = temperature at point C = 454.71497 K
(see above discussion)
n = no. of moles of monatomic ideal gas = 1.00
R = universal gas constant = 8.314 J/moleK
pA = 1.013 105 Pa. 3
Eint = nR(TA TC )
Then 2
(a) 3
= nR(TC TA )
Process AB: 2
= 1929.45 J .
 VA
W = p dV
VC
= pVA + pVC (p = pA = pC )
3
Eint = nR(TB TA ) pV = nRT
2
= 3741 J . W = nRTA + nRTC
 VB = nR(TC TA )
W = p dV = 1286.30 J .
VA
VA Q = Eint W
= p dV 3
VA = nR(TC TA ) nR(TC TA )
2
=0. 5
Q = Eint W = nR(TC TA ) (= Cp T )
2
3
= nR(TB TA ) (= CV T ) = 3215.75 J .
2
= 3741 J .

Complete cycle:
Process BC:

Eint 0 (state variable) .


3
Eint = nR(TC TB ) 3
2 W = nR(TB TC ) + nR(TC TA )
3 2
= nR(TB TC ) 3 5
2 = nRTA nRTB + nRTC )
2 2
= 1812 J .
= 525.55 J .
Q = 0 (adiabatic) . 3 5
W = Eint Q Q = nR(TB TA ) nR(TC TA )
2 2
3 3 5
= nR(TB TC ) = nRTA + nRTB nRTC
2 2 2
= 1812 J . = 525.55 J .
4

(b) The inventor claims to have achieved a gure of


pB TB merit equal to
= (V xed)
pA TA
TB QH
pB = pA
TA F=
W
= 2.026 105 Pa . 20 kW
=
pC = pA 1.9 kW
= 10.526 .
= 1.013 105 Pa .
nRTA
VA = This is slightly larger than the Carnot gure
pA
of merit. Any reversible heat pump will have
VB = VA the same gure of merit as a Carnot engine.
nRTA The only other possibility would be that the in-
=
pA ventors heat pump is irreversible. For example,
= 0.0246 m3 . friction in the refrigerator could convert a certain
additional amount W  of work directly to heat
VC TC
= (p xed) in each cycle. In the best case, all of the heat
VA TA
from W  would be dumped into the hot rather
TC than the cold reservoir. Then W  would be added
VC = VA
TA both to the numerator and to the denominator of
= 0.0373 m3 . F, reducing its physical value further below the
value reported by the inventor. Therefore we are
5. RHK problem 26.16 forced to reject the inventors claim. (Neverthe-
Solution: Consider a Carnot engine operat- less, many patents indeed have been granted for
ing in reverse (as a refrigerator) between a cold processes that violate elementary physical laws.)
reservoir at temperature TL = 276 K and a hot
reservoir at TH = 308 K. Like all Carnot engines 6. RHK problem 26.19
it is characterized by the equality Solution: Again a Carnot engine is operated in
|QH | TH reverse between a hot reservoir at TH and a cold
= (RHK Eq. 26.9) . reservoir at TL . Again QH is negative and QL is
|QL | TL
positive, and, since the refrigerator is reversible,
For operation as a refrigerator, the heat QH
added to the gas by the hot reservoir is nega-
|QH | TH
tive. Conversely, QL is positive. The net heat = .
Q = QH + QL added to the gas over one com- |QL | TL
plete cycle is negative. Since the internal energy
Eint is a state function, over a complete cycle it Again Eint must be zero over a complete cy-
must be conserved. Therefore, in one complete cle, so that W = Q over the cycle.
cycle, Q must be balanced by the mechanical (a)
work W done on the gas. A gure of merit F
for a heat pump, the ratio of QH to W , is W = Q
QH = QH QL
F=  Q 
W H
QH = QL 1
= QL
QH QL T 
H
TH = QL 1
= TL
TH T L TH T L
= 9.625 . = QL .
TL
5

(b) Then
WE = Q1 + Q2
 Q2 
= Q1 1
Q1
 T2 
QL = Q1 1
K T1
W
QL WR = Q3 Q4
=  Q4 
QH QL
= Q3 1
1 Q3
= QH  T4 
QL 1 = Q3 1
1 T3
= WR
TH
TL 1 1=
WE
TL  
=
TH T L
. Q3 1 TT43
=  
Q1 1 TT21

Q3 1 T2
T1
=
(c) Q1 1 T4
T3

|Q3 | 1 T2
T1
= .
|Q1 | 1 T4
T3
TL = 260 K
8. RHK problem 26.27
TH = 298 K Solution: Let
TL Wab = work done on gas in stroke ab, etc.
K=
TH T L W = work done on gas in cycle
= 6.842 . Wby eng = work done by engine in cycle
p0 = smaller pressure = 1.01 105 Pa
p1 = larger pressure = 2p0
V0 = smaller volume = 0.0225 m3
V1 = larger volume = 2V0
Qabc = heat added to gas during pair of expan-
sion strokes
7. RHK problem 26.23 e = eciency of engine
Solution: Let eCarnot = eciency of Carnot engine operating
Q1 = heat transferred to gas in engine from between two temperatures with ratio p1 V1 /p0 V0
(hot) reservoir 1 (> 0) Then
Q2 = heat transferred to gas in engine from (a)
(cold) reservoir 1 (< 0)
W = Wab + Wbc + Wcd + Wda
Q1 = heat transferred to gas in refrigerator from
(hot) reservoir 3 (< 0) = 0 p1 (V1 V0 ) + 0 + p0 (V1 V0 )
Q1 = heat transferred to gas in engine from = (p1 p0 )(V1 V0 )
(cold) reservoir 4 (> 0) Wby eng = W
WE = mechanical work done on gas in engine
= (p1 p0 )(V1 V0 )
(< 0)
WR = mechanical work done on gas in refriger- = p0 V0
ator (> 0) = 22725 J .
6

(b)

Qabc = Qab + Qbc


= Eint (c) Eint (a) Wab Wbc
3
= nR(Tc Ta ) 0 + p1 (V1 V0 )
2
3 3
= p1 V1 p0 V0 + p1 V1 p 1 V0
2 2
5 3
= p1 V1 p 1 V0 p0 V0
2 2
13
= p0 V0
2
= 147713 J .

(c)
Wby eng
e
Qabc
(p1 p0 )(V1 V0 )
= 5 3
2 p1 V1 p 1 V0 2 p0 V0
2
=
13
= 0.1538 .
(d)
Tc T a
eCarnot =
Tc
p1 V1 p 0 V0
=
p1 V1
3
= .
4
University of California, Berkeley
Physics H7B Spring 1999 (Strovink)
PROBLEM SET 4

1. RHK problem 26.36


2. RHK problem 26.40
3. RHK problem 26.43
4. Purcell problem 1.5
5. Purcell problem 1.8
6. Purcell problem 1.14
7. Purcell problem 1.26
1

University of California, Berkeley


Physics H7B Spring 1999 (Strovink)
SOLUTION TO PROBLEM SET 4

1. RHK problem 26.36 (c)


Solution: Let
n = no. of moles of ideal monatomic gas = 1.00
R = universal gas constant = 8.314 J/moleK

Then
(a) dEint 0 (state function) .

Wabc = Wab + Wbc dS 0 (state function) .
 b
= p dV + 0
a
= p0 (4V0 V0 )
= 3p0 V0 .

(b)
2. RHK problem 26.40
Solution: Let
3 n = no. of moles of ideal diatomic gas = 1.00
Eint (b c) = nR(Tc Tb )
2 R = universal gas constant = 8.314 J/moleK
3 Then
= (pc Vc pb Vb )
2 (a)
3
= (8p0 V0 4p0 V0 )
2
= 6p0 V0 .
 c
Q pV = constant (isotherm)
Sbc =
T V1
b c p2 = p1
dEint V2
= Wbc p1
b T
 c =
3
.
dEint
= 0 pV = constant (adiabat)
b T
 V 
3 p3 = p1
1
Eint = nRT
V3
2 c
3 dT 7
Sbc = nR = (diatomic)
b 2 T 5
  1 7/5
3 Tc  p3 = p1
= nR ln 3
2 Tb
3 = 0.215 p1 .
= nR ln 2
2 T V 1 = constant (adiabat)
= 8.644 J/K .  V 1
1
T3 = T1
V3
 1 2/5
Note that the last result does not depend on p0
= T1
or V0 , even though the problem asks us to ex- 3
press it in terms of p0 and V0 . = 0.644 T1 .
2

(b) Q12 = Eint (1 2) W12


= 0 + p1 V1 ln 3
= 1.099 p1 V1 .
Q23 = Eint (2 3) W23
5
5   1 2/5 
Eint = nRT (diatomic)
2 = p1 V1 1 0
5 2 3
Eint (1 2) = nR(T2 T1 ) = 0.889 p1 V1 .
2
=0. Q31 0 (adiabat) .
5
Eint (2 3) = nR(T3 T2 )
2
5  1 2/5 
= nRT1 1
2 3  2
5   1 2/5  Q
= p1 V1 1 S12 =
2 3 1 T
= 0.889 p1 V1 . T = T1 (isotherm)

Q12
dEint 0 (state function) S12 =
T1
Eint (3 1) = Eint (1 2) p1 V1
= ln 3
T1
Eint (2 3)
5   1 2/5  = nR ln 3
= 0 + p1 V1 1 = 9.134 J/K .
2 3
= 0.889 p1 V1 . S31 0 (adiabat) .

dS 0 (state function)

S23 = S12 S31


= nR ln 3 0
= 9.134 J/K .
 2
W12 = p dV
1
 2
dV
= nRT1
1 V 3. RHK problem 26.43
V2 Solution: In Physics H7B, all problems in-
= nRT1 ln volving numbers should be solved completely in
V1
= p1 V1 ln 3 terms of algebraic symbols before any numbers
are plugged in (otherwise it is much more di-
= 1.099 p1 V1 . cult to give part credit). Let
 3
m1 = initial amount of water = 1.780 kg
W23 = p dV
2
(initial amount of ice is 0.262 kg)
=0. m2 = nal amount of water = (1.780 + 0.262)/2
= 1.021 kg
W31 = Eint (3 1) Q31 Lf = latent heat of fusion of water = 333000
5   1 2/5 
J/kg
= p1 V1 1 0
2 3 T0 = temperature of melting ice = 273 K
= 0.889 p1 V1 . Then
3

(a) 4. Purcell problem 1.5


Solution: Consider an element of charge dQ =
Q(m1 m2 ) = Lf (m2 m1 ) R d, where d is an element of azimuth around
 2 the semicircle (0 < < ), and = Q/R is the
Q
S(m1 m2 ) = charge per unit length (in esu/cm) around the
1 T
semicircle.
T T0
Construct a Cartesian coordinate system
Q(m1 m2 )
S(m1 m2 ) = with its origin at the center of the semicircle;
T0 choose x = R cos and y = R sin . Then the
Lf (m2 m1 ) symmetry about x = 0 and z = 0 requires the
=
T0 electric eld at the origin from the full semicircle
Lf (m1 m2 ) not to have any component in the x or z direc-
=
T0 tions. So the net electric eld must be parallel
= 925.8 J/K . to the y axis; it points toward y if the charge
Q is positive.
(b) At the origin, Coulombs law requires the
 above mentioned charge element dQ to create an
dS 0 (state function) element of electric eld dE which has a magni-
tude equal to dQ/R2 . However, only a fraction
S(m2 m1 ) = S(m1 m2 ) sin of that eld magnitude points in the y
Lf (m1 m2 ) direction. Therefore
=
T0
dQ
= 925.8 J/K . dEy = sin
R2
R d
(c) Here the change of entropy of the environ- = sin
R2
ment in this cycle is calculated assuming that Q
R d
the heat to melt the ice is supplied at a temper- = R 2 sin
ature T>0 which is greater than T0 , for example R
Q
by a Bunsen burner. Nevertheless, using the fact = 2 sin d
that the entropy of the environment is a state R 

variable, we calculate its change by making use Q
Ey = 2 sin d
of a hypothetical reversible process, S = Q/T : R 0
2 Q
 =
R2
dSicewater 0  2 Q 
 E = 0, ,0 .
Q(m1 m2 ) Q(m2 m1 ) R2
dSenviron =
T0 T>0
 L (m m )  5. Purcell problem 1.8
f 1 2
=
T0 Solution: Let a be the ionic spacing of the
Lf (m1 m2 ) one-dimensional crystal. Place the rst positive
ion at x = 0, two negative ions at x = a, two
T>0
1 1  more positive ions at x = 2a, etc. Consider
= Lf (m1 m2 ) Purcells Eq. 1.9:
T0 T>0
>0 N
 1   qj qk
U= .
dSuniverse > 0 . 2 j=1 rjk
k=j
4

This is a double sum. As the number N of ions 6. Purcell problem 1.14


approaches , the sum of the terms of the dou- Solution: This is similar to Purcells prob-
ble sum which involve any particular ion will be lem 1.5, discussed above, and we will use sim-
the same as the sum of the terms involving any ilar notation. Consider an element of charge
other particular ion (see the argument at the dQ = b d, where d is an element of azimuth
bottom of Purcells page 14). Thus the double around the circle (0 < < 2), and = Q/2b
sum reduces to a single sum: is the charge per unit length (in esu/cm) around
the circle.
N
1  q1 qk
U= N ,
2 r1k Construct a Cartesian coordinate system
k=2
with its origin at the center of the circle; choose z
where we have chosen to sum only the terms in- as the coordinate along the axis normal to plane
volving ion 1. Furthermore, since the string of of the circle. Consider a line drawn from dQ to
ions is symmetric about x = 0, we may consider a point (0, 0, z) on this axis. Dene to be the
in the single sum only the ions with x > 0, at angle that this line makes with the plane of the
the expense of multiplying the result by an extra circle. With these denitions, tan = z/b and
factor of 2: 2 < < 2 ; the distance from dQ to (0, 0, z) is
N b sec . Because the conguration is symmetric
1  q1 qk
U= 2N . about x = 0 and y = 0, on the z axis the electric
2 r1k eld must point in the z direction, away from
k=2; x>0
the plane of the ring if its charge Q is positive.
Here we evaluate r1k = a(k 1), and we use the
fact that the sign of q1 qk is equal to (1)k1 :
At the point (0, 0, z), Coulombs law re-
 q1 qkN quires the above mentioned charge element dQ
1 to create an element of electric eld dE which
U = 2N
2 a(k 1) has a magnitude equal to dQ/(b sec )2 . How-
k=2
N ever, only a fraction sin of that eld magnitude
N e  (1)k1
2
= points in the z direction. Therefore
a (k 1)
k=2
N 1
N e2  (1)j
= .
a j=1 j dQ
dEz = sin
b2
sec2
Taking N in the limit of the sum, b d
= 2 sin

b sec2
N e2  (1)j Q
U= 2b b d
a j=1 j = sin
b2 sec2
N e2 Q cos2 sin
= ln (1 + 1) = d
a 2b2

U e2 Q cos2 sin 2
= ln 2 , Ez = d
N a 2b2 0

where, following the hint, we have evaluated the Q cos2 sin


= .
sum by using the Taylor series expansion b2

 (b)j1
ln(1 + b) = .
j=1
j The problem thus reduces to nding the value of
5

which maximizes the product cos2 sin : We have seen that dEA,y exactly cancels dEB,y
u sin for any choice of ; therefore EC vanishes.
d  
0= u(1 u2 )
du
= 1 3u2

1
u=
3

1
= arcsin
3
  
1
z = b tan arcsin
3

1
=b .
2
7. Purcell problem 1.26
Solution: Place the origin of a Cartesian co-
ordinate system at the center of the semicircle,
with both parallel rods lying in the xy plane.
Orient the y coordinate so that the rods extend
to y = .
At point C, the origin of this coordinate
system, any electric eld can point only in along
the y direction, owing to the symmetry of the
problem about x = 0 and z = 0. Purcells gure
refers us to two elements of charge. The element
at point A has a value dQ = b d and gener-
ates an electric eld at the origin of magnitude
b d/b2 . Only a fraction sin of this eld points
in the y direction; thus
b d
dEA,y = 2 sin
b

= sin d .
b
The eld from the element of charge at point B is
slightly more complicated. This charge element
has value dQ = d|y|, where d|y| is an element of
length along the straight rod, and |y| = b tan .
Therefore dQ = b d tan = b sec2 d. This el-
ement of charge lies a distance b sec away from
the origin. Again, only a fraction sin of the
eld generated by this charge element points in
the +y direction. Putting it all together,
b sec2 d
dEB,y = + sin
b2 sec2

= + sin d .
b
University of California, Berkeley
Physics H7B Spring 1999 (Strovink)
PROBLEM SET 5

1. Purcell problem 1.16


2. Purcell problem 1.19
3. Purcell problem 1.29
4. Purcell problem 1.33
5. Purcell problem 2.1
6. Purcell problem 2.8
7. Purcell problem 2.19
8. Purcell problem 2.20
University of California, Berkeley
Physics H7B Spring 1999 (Strovink)
SOLUTION TO PROBLEM SET 5
Solutions by P. Pebler

1 Purcell 1.16 A sphere of radius a was lled with positive charge of uniform density . Then a
smaller sphere of radius a/2 was carved out, as shown, and left empty. What are the direction and
magnitude of the electric eld at points A and B?

The key is to consider the given distribution as a superposition of the two distributions at
right. The electric eld will be the sum of the contributions from these two spheres, which are easy
to evaluate. For points outside these spheres, we may treat them as point charges lying at their
centers. The charges are
 3
4 a a3
Q1 = () = ,
3 2 6
4
Q2 = a3 .
3
Consider the point A. The contribution from sphere 2 is zero since there is no eld at the center
of a spherical distribution. The point A lies outside the sphere 1.
Q1 2
EA = (y) = a y
(a/2)2 3
Point B lies outside both spheres.
Q2 Q1 4a3 2a3 34
EB = 2
(y) + 2
(y) = 2
y + 2
y = a y
a (3a/2) 3a 27a 27

2 Purcell 1.19 An innite plane has a uniform surface charge distribution on its surface.
Adjacent to it is an innite parallel layer of charge of thickness d and uniform volume charge
density . All charges are xed. Find the electric eld everywhere.

1
The contribution due to the surface charge has magnitude 2|| and points away from or
towards the surface depending on the sign of . To deal with the volume charge, we can treat it
as a stack of very thin layers of charge and treat these layers as surface charges. We could add up
all the contributions from these innitesimal layers by integrating. However, since the eld from
an innite plane of charge does not depend on how far away you are, the contribution from each
layer will be the same. So we will get the same answer by assuming the nite volume charge layer
to be a surface with surface charge t where t is the thickness of the layer in question. For x < 0,
everything pushes to the left.

E = 2(x) + 2 d(x) = 2( + d) x x<0

Likewise,

E = 2( + d) x x d.

For the region 0 < x < d, the volume layer is split into two. We can thing of the right side as a
single surface with surface charge (d x) pushing to the left, and the left side as a surface charge
x pushing to the right.

E = 2 x + 2 x x 2(d x) x = (2 + 4 x 2 d) x 0<x<d

If we wished to consider the plane x = 0, we could say that the surface charge contributes nothing.

E = 2 d x x=0

Notice that there is a discontinuity of 4 as we pass through zero. This is always the case for
idealized surface charges. There is no discontinuity at x = d however.

3 Purcell 1.29 A spherical shell of charge of radius a and surface charge density is missing a
small, approximately circular, piece of radius b  a. What is the direction and magnitude of the
eld at the midpoint of the aperture?

The picture above assumes for simplicity that > 0.


As a zero order approximation, we may consider the missing circle as innitesimal. The eld
at left can be viewed as a superposition of the two distributions at right. We temporarily ignore
points exactly at the surface. By considering the eld to the right or to the left we nd

E = 4 x + 2() x = 0 + 2()(x) = 2 x,

for the eld everywhere except at the surface. But for the distribution with the circle missing, there
can be no discontinuity when passing through the hole, so the eld directly at the surface is also
2 x.
This should be a good approximation when b  a. But for a nite missing piece, this will not
be the exact answer even at the center. To nd the contributions of higher order in b/a we can
integrate. This is actually not too bad.

2
From symmetry considerations, we know the eld is radial in the center of the aperture.
da R2 2 d
dEr = cos = cos(90 /2)
r2 2R2 (1 cos )
sin sin 2 (1 + cos ) (1 + cos )
= 2 d =
d = cos d
sin 2 cos 2 2
  
o
Er = cos d = 2 1 sin
o 2 2
We can take the initial angle o to be b/R.
   3 
b 1 b
Er = 2 1 + ...
2R 3! 2R

4 Purcell 1.33 Imagine a sphere of radius a lled with negative charge 2e of uniform density.
Imbed in this jelly of negative charge two protons and assume that in spite of their presence the
negative charge remains uniform. Where must the protons be located so that the force on each of
them is zero?

The forces on the protons from each other will be equal and opposite. Therefore, the forces on
them from the negative charge distribution must be equal and opposite also. This requires that
they lie on a line through the center and are equidistant from the center. The force on each proton
at radius r from the negative charge will be proportional to the amount of negative charge lying
inside a sphere of radius r. For purposes of nding the electric eld, we may treat all of this charge
as if it were a point charge sitting in the center. We ignore all negative charge outside the radius
of the proton positions. The negative charge inside the radius r is
r3
q= 2e.
a3
The force on the right proton must be zero.
 
e2 e(2er3 /a3 ) e2 r3 a
F= x + x = 18 3 x = 0 r=
(2r)2 r2 (2r)2 a 2

5 Purcell 2.1 The vector function


Ex = 6xy Ey = 3x2 3y 2 Ez = 0

3
represents a possible electrostatic eld. Calculate the line integral of E from the point (0, 0, 0) to the
point (x1 , y1 , 0) along the path which runs straight from (0, 0, 0) to (x1 , 0, 0) and thence to (x1 , y1 , 0).
Make a similar calculation for the path which runs along the other two sides of the rectangle, via the
point (0, y1 , 0). Now you have the a potential function (x, y, z). Take the gradient of this function
and see that you get back the components of the given eld.

We take the rst path in two parts. While moving along the x axis we have ds = dx x so that
E ds = Ex dx and while moving up parallel to the y axis we have ds = dy y and E ds = Ey .
  x1  y1  x1  y1
E ds = Ex dx + Ey dy = 6xy dx + (3x2 3y 2 ) dy
0 0 0 0

When integrating along the x axis, y has the constant value y = 0 which we plug in to the rst
integral. Along the second part of the path, x = x1 .
  y1
E ds = 0 + (3x21 3y 2 ) dy = 3x21 y1 y13
0

We do the same thing along the second path.


  y1  x1
2 2
E ds = (3(0) 3y ) dy + 6xy1 dx = y13 + 3x21 y1
0 0

(x, y, z) = 3x2 y y 3


(x, y, z) = (3x2 y y 3 ) x + (3x2 y y 3 ) y + (3x2 y y 3 ) z = 6xy x + (3x2 3y 2 ) y
x y z

6 Purcell 2.8 Consider an innitely long cylinder of radius a and uniform charge density . Use
Gausss law to nd the electric eld. Find the potential as a function of r, both inside and outside
the cylinder, taking = 0 at r = 0.

Our Gaussian surface both inside and outside the cylinder will be a cylinder of length L. Inside
we have

E da = Er 2rL = 4Qenc = 4r2 L,

E = 2 r r r < a.

Outside,

E da = Er 2rL = 4Qenc = 4a2 L,

4
2a2
E= r r a.
r
To nd the potential, we integrate radially outward from the center so that ds = dr r.
  r
(r) = (0) E ds = 0 2 r dr = r2 ra
0

For points outside the cylinder,


 r
2a2 r
(r) = (a) dr = a2 2 a2 ln r > a.
a r a

7 Purcell 2.19 Two metal spheres of radii R1 and R2 are quite far apart compared with these
radii. Given a total amount of charge Q, how should it be divided so as to make the potential energy
of the resulting charge distribution as small as possible? Assume that any charge put on one of
the spheres distributes itself uniformly over the sphere. Show that with that division the potential
dierence between the spheres is zero.
Because the spheres are far apart, the energy will be essentially due to the energy of each sphere.
We may assume that the charge on each sphere is uniformly distributed if the other sphere is very
far away. To nd this energy we can use the standard formula adapted to surface charge,

1
U= da.
2
The potential just outside a uniformly charged sphere is q/r and because the potential is contin-
uous, this is also the potential at the surface. Then,

1 q q 2 1 q2
U= r sin dd = .
2 4r2 r 2 r
Breaking up the charge into q and Q q,
q2 (Q q)2
U= + .
2R1 2R2
If the minimum energy is obtained with q = qo ,
dU qo (Q qo )
(qo ) = = 0,
dq R1 R2

qo Q qo
= .
R1 R2

But these are just the potentials at both spheres.

8 Purcell 2.20 As a distribution of electric charge, the gold nucleus can be described as a sphere of
radius 6 1013 cm with a charge Q = 79e distributed fairly uniformly through its interior. What
is the potential o at the center of the nucleus, expressed in megavolts?
For a uniformly charged sphere of radius a,
Q
E= r r > a,
r2

5
Qr
E= r r a.
a3
Since the potential is zero at innity, the potential at any point P is
 P
(P ) = E ds.

We can make the path of integration come radial straight in. If the point P has r < a,
 r  a  r
Q Qr Q Qr2 Q
(r) = Er dr = dr dr = 3 +
r2 a a2 a 2a 2a
In SI units,
1 3Q 3 79(1.6 1019 C)
(0) = = = 28.4 megavolts.
4!o 2a 4(8.85 1012 C 2 /N m2 )2(6 1915 m)

6
University of California, Berkeley
Physics H7B Spring 1999 (Strovink)
PROBLEM SET 6

1. Purcell problem 2.27


2. Purcell problem 2.29
3. Purcell problem 2.30
4. Purcell problem 3.1
5. Purcell problem 3.9
6. Purcell problem 3.17
7. Purcell problem 3.23
8. Purcell problem 3.24
University of California, Berkeley
Physics H7B Spring 1999 (Strovink)
SOLUTION TO PROBLEM SET 6
Solutions by P. Pebler

1 Purcell 2.27 The electrostatic potential at a point on the edge of a disc of radius r and uniform
charge density is = 4r. Calculate the energy stored in the electric eld of a charged disk of
radius a.
We calculate the total energy by bringing in each innitesimal ring of charge from innity and
adding up the energy for each ring. We assume that we have already built up the disc to radius
r. We now bring in a ring of width dr and stick it on the edge. Recall that the energy necessary
to bring in a test charge from innity to some point is just the potential at that point times the
charge. (This is more or less the denition of the potential.) The potential just outside the disc
where we are packing on the next ring is 4r. The energy necessary is then

dU = (r)dq = (4r)(2rdr) = 8 2 r2 dr.

To add up all the rings integrate from 0 to a.


 a  2
8 8 Q 8Q2
U = 8 2 r2 dr = a3 2 = a3 =
0 3 3 a2 3a

2 Purcell 2.29 Two nonconducting spherical shells of radius a carry charges of Q and Q
uniformly distributed over their surfaces. The spheres are brought together until they touch. What
does the electric eld look like, both outside and inside the shells? How much work is needed to
move them far apart?

The eld of a uniformly charged shell is zero inside the shell and that of a point charge outside.
Outside both shells, we have the eld of two point charges. Inside either shell, the eld is that of
a single point charge at the center of the other shell.
To nd the energy we use the following argument. Consider instead a uniform shell of charge
Q and a point charge Q a distance r from the center of the shell (but outside it). We know that
outside the shell, the potential due to the shell is just Q/r, so the energy needed to bring in the
point charge is Q2 /r and the energy needed to move it out is Q2 /r. However, this must be the
same energy as that required to move out the shell while keeping the point charge xed. So we nd
that the energy needed to move a shell out to innity in the eld of a point charge is Q2 /r. But
since the other shell creates the eld of a point charge outside of it, this is also the energy needed

1
to separate our two shells.
Q2
E=
2a
If you dont like this argument, you can integrate a shell distribution times the potential of a point
charge which isnt too hard and nd the same answer.

3 Purcell 2.30 Consider a cube with sides of length b and constant charge density . Denote by
o the potential at the center of the cube and 1 the potential at a corner, with zero potential at
ininity. Determine the ratio o /1 .

We imagine another cube with the same charge density but with twice the side length. Let the
potential at the center of this cube be o . The point at the center of this new cube lies at the
corner of each of eight cubes of the original size. Because the potential is additive, we have

o = 81 .

We can also use dimensional arguments to nd o . We can write

o = f (Q, s),

where Q is the total charge, s is the side length and the functional form of f depends on the shape
and nature of the distribution. We can now ask for whats called a scaling law which tells us what
happens if we multiply the variables Q and s by numerical factors while keeping all other details
of the distribution the same. Whatever the functional form of f is, we know it has units of charge
per length, the units of the potential. Fortunately, the only parameters carrying units which enter
into f are Q and s. The only way then to get the right units is if
Q
f (Q, s) .
s
The function f then satises the simple scaling

f (Q, s) = f (Q, s).

In our case s = 2s and because we are keeping the charge density constant, Q = s3 = (2s)3 =
8Q. Then
8
o = f (8Q, 2s) = f (Q, s) = 4o ,
2
4o = 81 ,

o
= 2.
1

2
4 Purcell 3.1 A spherical conductor A contains two spherical cavities. The total charge on the
conductor is zero. There are point charges qb and qc at the center of each cavity. A considerable
distance r away is another charge qd . What force acts on each of the four objects A, qb , qc , qd ?
Which answers, if any, are only approximate, and depend on r being relatively large?

The force on qb and qc is zero. The eld inside the spherical cavity is quite independent of
anything outside. A charge qb is uniformly distributed over the conducting surface to cancel the
eld from the point charge. The same happens with qc . This leave an excess charge of qc + qb
on the outside surface of the conductor. If qd were absent, the eld outside A would be the
symmetrical, radial eld E = |qb + qc |/r2 , the same as a point charge because the excess charge
would uniformly distribute itself over the spherical outer surface. The inuence of qd will slightly
alter the distribution of the charge on A, but without aecting the total amount. Hence for large
r, the force on qd will be approximately
qd (qb + qc )
Fd = r.
r2
The force on A must be precisely equal and opposite to the force on qd .

5 Purcell 3.9 Two charges q and two charges q lie at the corners of a square with like charges
opposite one another. Show that there are two equipotential surfaces that are planes. Obtain and
sketch qualitatively the eld of a single point charge located symmetrically in the inside corner
formed by bending a metal sheet through a right angle. Which congurations of conducting planes
and point charges can be solved this way and which cant?

The potential on each of the two lines A and B shown is zero because the contribution at each
point on either line from any charge is cancelled by the opposite charge directly across from it.
Therefore, the eld of a point charge in the corner of a bent conductor is the same as the eld
from these four point charges. You should be able to see by looking at the rst few cases that this
strategy will work any time we divide the space into an even number of wedges. This allows the
contributions to the potential to cancel pairwise. For example, in the picture at right the potential
is zero on lines A and B because all the charges come in equal and opposite pairs. The applicable
angles are n = 2/(2n) = /n, where n is an integer. This would not work for an angle of 120o .

3
6 Purcell 3.17 A spherical vacuum capacitor has radius a for the outer sphere. What radius b
should be chosen for the inner spherical conductor to store the greatest amount of electrical energy
subject to the constraint that the electric eld strength at the surface of the inner sphere may not
exceed Eo ? How much energy can be stored?
We rst need the capacitance of this capacitor. Assuming there is a charge Q on the inner shell
and a charge Q on the outer shell, the eld between the shells is
Q
E= r.
r2
The potential dierence is
 b  
Q ab
V = dr = Q ,
a r2 ab
and the capacitance
ab
C = Q/V = .
ab
The energy stored by this capacitor is
1 2 1ab 2
U= Q = Q .
2C 2 ab
The energy in the capacitor will depend on how much charge is on it. If we were allowed to put
arbitrary amounts on, the energy would have no maximum. However, for a given b, the maximum
eld near the inner sphere gives us the maximum allowed charge. This gives us the maximum
stored energy for a given capacitor.
Qmax
Eo =
b2

1 a b 2 4 1 ab3 b4 2
Umax = E b = Eo
2 ab o 2 a
Now we want to choose a b to make this as large as possible.
Umax 1 3ab2 4b3 2
(bmax ) = Eo = 0
b 2 a
3a 4bmax = 0
3
bmax = a
4
The energy is then
 
3  4
1a 4a 3 27 2 3
Umax =   Eo2 a = E a .
2 a 3 4 512 o
4a

7 Purcell 3.23 Find the capacitance of a capacitor that consists of two coaxial cylinder of radii a
and b and length L. Assume L  b a so that end corrections may be neglected. Check your result
in the limmit b a  a with the formula for the parallel-plate capacitor.

4
A cylinder of 2.00 in outer diameter hangs with its axis vertical from one arm of a beam balance.
The lower portion of the hanging cylinder is surrounded by a stationary cylinder with inner diameter
3.00 in. Calculate the magnitude of the force down when the potential dierence between the two
cylinders is 5 kV .
The eld between charged cylinders is
2 2Q
E= r = r,
r rL
assuming we have Q on the inside and Q on the outside. The potential dierence is
 b
2Q dr 2Q b
V = = ln .
a L r L a
Just arrange your signs so that the capacitance comes out positive.
L
C=
2 ln(b/a)
Let us now consider the general case where the potential dierence is being held constant by a
battery while the capacitance is changing. Initially we have charge and energy
1
Q = CV U = CV 2 .
2
After a change in capacitance C,
1
Q = (C + C)V = Q + V C U  = (C + C)V 2 .
2
The battery has done work on this system by moving this extra charge across the potential dier-
ence.

Wb = (C)V 2

If the change in capacitance is caused by movement of the components, the electric eld does work
on the plates or plate.

W = F (L)

From conservation of energy we have

U + Wb = U  + W,

1
(C)V 2 = (C)V 2 + W,
2
1
W = F (L) = (C)V 2 ,
2
1 C
F = V2 .
2 L
In our case we have
1 1 1 (16.7 statvolts)2
F = V2 = = 172 dynes.
2 2 ln(b/a) 2 2 ln(3/2)

5
8 Purcell 3.24 Two parallel plates are connected by a wire. Let one plate coincide with the xz plane
and the other with the plane y = s. The distance s is much smaller than the lateral dimensions of
the plates. A point charge Q is located between the plates at y = b. What is the magnitude of the
total surface charge on the inner surface of each plate?

The total induced charge is Q. We need to nd the fraction of induced charge on either
conductor. For this we may notice that the fraction of induced charge on both planes will be the
same for any distribution located at y = b because we may view it as the superposition of many
little point charges. So we want to consider the simplest possible case which is a uniformly charged
plane. (Once again, we are ignoring edge eects.) Using a Gaussian pillbox with its left face inside
the left plate and its right face at y, where 0 < y < b, the eld in the left region is

El = 41 y.

Similarly, the eld in the right region is

Er = 42 y.

Since the two conductors are connected by a wire, they are at the same potential so the line integral
from the middle to the left and right should be the same.

41 (b) = 42 (s b)

2 b
=
1 sb

Now switch back to the original problem.


Q2 b
= Q1 + Q2 = Q
Q1 sb

sb b
Q1 = Q Q2 = Q
s s

6
University of California, Berkeley
Physics H7B Spring 1999 (Strovink)
PROBLEM SET 7

1. Purcell problem 4.8


2. Purcell problem 4.20
3. Purcell problem 4.25
4. Purcell problem 4.26
5. Purcell problem 4.30
6. Purcell problem 4.31
7. Purcell problem 4.32
8. (Taylor & Wheeler problem 19)
(a.)
Two events P and Q have a spacelike separa-
tion. Show that an inertial frame can be found
in which the two events occur at the same time.
In this frame, find the distance between the two
events (this is called the proper distance). (Hint:
one method of proof is to assume that such an
inertial frame exists and then use the Lorentz
transformation equations to show that the veloc-
ity c of this inertial frame, relative to the frame
in which the events were initially described, is
such that < 1, thus justifying the assumption
made.)
(b.)
Two events P and R have a timelike separa-
tion. Show that an inertial frame can be found
in which the two events occur at the same place.
In this frame, find the time interval between the
two events (this is called the proper time).
University of California, Berkeley
Physics H7B Spring 1999 (Strovink)
SOLUTION TO PROBLEM SET 7
Solutions to Purcell problems by P. Pebler

1 Purcell 4.8 A copper wire 1 km long is connected across a 6 volt battery. The resistivity
of copper is 1.7 106 ohm cm, and the number of conduction electrons per cubic centimeter is
8 1022 . What is the drift velocity of the conduction electrons under these circumstances? How
long does it take an electron to drift once around the circuit?
The current will be
V VA
I= =
R L
and the current density
I V
J= = .
A L
The current density is the charge density times the drift speed and the charge density is ne where
n is the number density of electrons.
J V
v= =
ne Lne
6V
v= = 2.8 103 cm/s
(1.7 106 ohm cm)(105 cm)(8 1022 cm3 )(1.6 1019 C)

105 cm
t= = 3.6 107 s  1 year
2.8 103 cm/s

2 Purcell 4.20 A black box with three terminals a, b, and c contains nothing by three resistors
and connecting wire. Measuring the resistance between pairs of terminals, we nd Rab = 30 ohm,
Rac = 60 ohm, and Rbc = 70 ohm. Show that the contents cound be either of the following.

Is there any other possibility? Are the two boxes completely equivalent, or is there an external
measurement that would distinguish between them?
For the rst box, the resistance between any two terminals involves two of the resistors in series
with the third resistor extraneous. For example, Rab = 10 ohm + 20 ohm = 30 ohm.
For the second box, the resistance between any two terminals involves one resistor in parallel
with the other two in series. For example,
 1
1 1
Rab = + = 30 ohm.
34 ohm 85 ohm + 170 ohm

1
The other two are easily veried.
These are the only two ways to make these three resistances with only three resistors.
For the two arrangements to be electrically identical, they must both draw the same currents
given the same input voltages Va , Vb , and Vc . The details are somewhat messy. Ill leave it to you
to verify that given the input voltages Va , Vb , and Vc , both arrangements draw the currents
1
Ia = (7Va 5Vb 2Vc ),
170
1
Ib = (6Vb Vc 5Va ),
170
1
Ic = (3Vc Vb 2Va ).
170
Note that Ia + Ib + Ic = 0 as it must.

3 Purcell 4.25 A charged capacitor C discharges through a resistor R. Show that the total energy
dissipated in the resistor agrees with the energy originally stored in the capacitor. Suppose someone
objects that the capacitor is never really discharged because Q only becomes zero for t = . How
would you counter this objection?
Assume that the capacitor initially has charge Q on it. The current as a function of time is

i(t) = io et/

where = RC and io = Vo /R = Q/CR. The power dissipated in a resistor is P = i2 R, so the total


energy dissipated is
 
Q2 RC 1 2
E= P dt = R i2o exp (2t/ ) dt = R i2o =R 2 2 = Q .
0 0 2 C R 2 2C
This was the initial energy stored in the capacitor.
We can nd the time it takes for the charge left on the capacitor to be one electron. The charge
as a function of time is

q = Q exp (t/ ).

The time would be


Q
t = ln .
e
Because of the ln, even for macroscopic initial charges, the time wouldnt be that large.

2
4 Purcell 4.26 Two graphite rods are of equal length. One is a cylinder of radius a. The other is
conical, tapering linearly from radius a at one end to radius b at the other. Show that the end-to-end
resistance of the conical rod is a/b times that of the cylindrical rod.

We consider the conical rod to be the series combination of little cylindrical rods of length dx.
The radii of these little cylinders are
ba
r(x) = a + x.
L
We sum up the little resistances.
  L  L  
dx dx L a L
R= dR = = 2
= = 2
0 A 0 (a + (b a)x/L) ab b a
L/a2 is the resistance of the cylinder of radius a.

5 Purcell 4.30 Consider two electrodes 2 mm apart in vacuum connected by a short wire. An alpha
particle of charge 2e is emmitted by the left plate and travels directly towards the right plate with
constant speed 108 cm/s and stops in this plate. Make a quantitative graph of the current in the
connecting wire, plotting current against time. Do the same for an alpha particle that crosses the
gap moving with the same speed but at an angle of 45o . Suppose we had a cylindrical arrangement
of electrodes. Would the current pulse have the same shape?

The result of problem 3.24 tells us the induced charge on the electrodes for any given position
of the alpha particle. The current is just the time derivative of this charge.
 
sb
q1 = 2e
s
dq1 2e db 2ev
I= = = = 0.48 esu/s
dt s dt s
This pulse lasts for (0.2 cm)/(108 cm/s) o
= 2 ns. If the alpha particle travels at 45 , the normal
speed and the current just decrease by 2.
For concentric cylinders, the current pulse would have a dierent shape. If you work out the
details,
1
I
a + vt
during the time that the alpha particle is in motion.

3
6 Purcell 4.31 Suppose a cube has a resistor of resistance Ro along each edge. At each corner the
leads from three resistors are soldered together. Find the equivalent resistance between two nodes
that represent diagonally opposite corners of the cube. Now nd the equivalent resistance between
two nodes that correspond to diagonally opposite corners of one face of the cube.

A total current I enters one node. It then has a choice of three directions to go. Because of the
symmetry, each choice is identical to the others so the current must split up evenly so that I/3 goes
through each resistor. Likewise, the current reaching the other node comes through three resistors
each having current I/3. This leaves 6 resistors in the middle to share the current. Because each
one is identical due to the symmetry, they must each have current I/6. To nd the voltage drop
between the two nodes, follow a straight path from one to the other.
I I I 5
V = Ro + Ro + Ro = RI
3 6 3 6
5
Req = Ro
6
In the second situation, because of the symmetry, we notice that all the resistors on the top
square carry the same magnitude of current while all the resistors on the bottom square carry the
same magnitude also. This tells us that there is no current through the two resistors indicated
by arrows. We can therefore ignore them, because the circuit would behave the same without
them. It is then easy to combine the remaining resistors. The top and bottom squares are parallel
combinations of resistors 2Ro .
 1
1 1
R= + = Ro
2Ro 2Ro
This leaves us with one resistor Ro in parallel with three resistors Ro .
 1
1 1 3
Req = + = Ro
Ro 3Ro 4

4
7 Purcell 4.32 Find the input resistance (between terminals A and B) of the following innite
series.

Show that, if voltage Vo is applied at the input to such a chain, the voltage at successive nodes
decreases in a geometric series. What ratio is required for the resistors to make the ladder an
attenuator that halves the voltage at every step? Can you suggest a way to terminate the ladder
after a few sections without introducing any error in its attenuation?

If we put another link on the left of this innite chain, we get exactly the same conguration.
If this innite chain has equivalent resistance Req , the new chain with the extra link can be described
by the middle circuit. We can calculate the equivalent resistance of this circuit by considering Req
and R2 in parallel, in series with R1 . But since this circuit is the same as the original, this equivalent
resistance is again Req .
 1
1 1
Req = R1 + +
R2 Req

This leads to the equation


2
Req R1 Req R1 R2 = 0,

with positive solution



R1 + R12 + 4R1 R2
Req = .
2
Now consider an arbitrary link with voltage dierence Vi between top and bottom. To nd the
voltage Vi+1 , we can replace the rest of the series with the equivalent resistor. We could get Vi+1 if
we knew the current through R1 . For this purpose we can replace the ith link also with Req . The
current through this equivalent circuit will also be the current through R1 . This current is just
Vi /Req .
Vi Req R1
Vi+1 = Vi R1 = Vi
Req Req
If we wish to halve the voltage each step,
R1 1
= ,
Req 2

4R1 = R1 + R12 + 4R1 R2 ,

5
R2 = 2R1 .

If we wish to terminate the ladder without changing this property, we just replace the rest of the
chain at any point with a resistor with resistance Req .

8 Taylor & Wheeler 19 (a.) Two events P and Q have a spacelike separation. Show that an
inertial frame can be found in which the two events occur at the same time. In this frame, nd the
distance between the two events (this is called the proper distance). (Hint: one method of proof
is to assume that such an inertial frame exists and then use the Lorentz transformation equations
to show that the velocity c of this inertial frame, relative to the frame in which the events were
initially described, is such that < 1, thus justifying the assumption made.) (b.) Two events P
and R have a timelike separation. Show that an inertial frame can be found in which the two events
occur at the same place. In this frame, nd the time interval between the two events (this is called
the proper time).
Denote the frame in which P and Q were initially described by S, and the frame in which we
wish them to be simultaneous by S  . As usual, the origins of these frames coincide at t = t = 0.
In S, if P and Q have a spacelike separation, their spatial separation must be nonzero. Orient the
x and x axes along the direction of this separation, so that yQ = yP and zQ = zP but xQ = xP .
Applying the Lorentz transformation between S  and S,

ctP = ctP xP

ctQ = ctQ xQ

ctQ ctP = (ctQ ctP ) (xQ xP )

We wish the left-hand side to be zero. If it is, then

(ctQ ctP ) = (xQ xP )

(ctQ ctP )
=
(xQ xP )

Now, we are told that the separation between events Q and P is spacelike:

c2 (tQ tP )2 (xQ xP )2 < 0

This guarantees that || < 1. It is straightforward to calculate the spatial separation of the two
events in S  by using the inverse Lorentz transformation:

xP = xP + ctP

xQ = xQ + ctQ

xQ xP = (xQ xP ) + (ctQ ctP )

We have chosen so that the two events are simultaneous in S  ; this forces the last term to vanish.
Substituting the value that we found for ,

xQ xP = (xQ xP )

6

xQ xP = 1 2 (xQ xP )

 ct ct 2
Q P
xQ xP = 1 (xQ xP )
xQ xP

xQ xP = (xQ xP )2 (ctQ ctP )2

where the sign is chosen so that xQ xP has the same sign as xQ xP . This proper distance is
the smallest distance between the two events that can be reached in any reference frame.
Similarly, when the separation between events P and R is timelike,

xR xP = (xR xP ) (ctR ctP )

(xR xP ) = (ctR ctP )

(xR xP )
=
(ctR ctP )

ctR ctP = (ctR ctP ) + (xR xP )



ctR ctP = 1 2 (ctR ctP )

 x x 2
R P
ctR ctP = 1 (ctR ctP )
ctR ctP

ctR ctP = (ctR ctP )2 (xR xP )2

where the sign is chosen so that ctR ctP has the same sign as ctR ctP . This proper time is the
smallest time interval between the two events that can be reached in any reference frame.

7
University of California, Berkeley
Physics H7B Spring 1999 (Strovink)
PROBLEM SET 8

1. (Taylor and Wheeler problem 27) Using the following masses in AMU,
The clock paradox, version 1. (proton) p 1.007825
On their twenty-rst birthday, Peter leaves his
twin Paul behind on the earth and goes o in (neutron) n 1.008665
2
the x direction for seven years of his time at (deuteron) H 2.014102
24/25 the speed of light, then reverses direction (helium 3) He3
3.016030
and in another seven years of his time returns 3
at the same speed. [In this most elementary (triton) H 3.016050
version of the problem, we assume that the nec- (alpha particle) 4 He 4.002603 ,
essary periods of acceleration are innitesimal
calculate (to 5%) the kinetic energy released
in duration, requiring Peters acceleration to be
when one liter of heavy water (2 H)2 O undergoes
innite. Nonetheless, our plucky twin remains
deuterium-tritium fusion in an H-bomb. Express
uninjured.]
your answer in terms of tons of TNT (1 ton of
(a.)
TNT = 4.2109 J of explosive energy).
Make a spacetime diagram (ct vs. x) showing
Peters motion. Indicate on it the x and ct coor- 4. The universe is lled with old cold photons
dinates of the turn-around point and the point that are remnants of the big bang. Typically
of reunion. For simplicity idealize the earth as their energy is 6.6 104 eV.
an inertial frame, adopt this inertial frame in the A cosmonaut who is accelerated at 1 g for
construction of the diagram, and take the origin 10 years in her own rest frame attains a boost
to be the event of departure. (= arctanh ) of 10.34. As seen by her, what is
(b.) the typical energy of these photons?
How old is Paul at the moment of reunion?
5. Prove that an isolated photon (zero mass)
2. Prove that cannot split into two photons which do not both
continue in the original direction.
tanh 1 + tanh 2 6. The now retired Bevatron at Berkeley
tanh (1 + 2 ) = .
1 + tanh 1 tanh 2 Lab is famous for having produced the rst
observed antiprotons (you may have glimpsed
Using this relation, deduce Einsteins law for the white-maned Nobelist Owen Chamberlain, one
addition of velocities. of the rst observers, being helped to his seat at
Physics Department colloquia). An economical
3. The thermonuclear deuterium-tritium re- reaction for producing antiprotons is
actions are: p + p p + p + p + p ,

2
where the rst proton is part of a beam, the
H + 2 H 3 He + n second is at rest in a target, and p is an antipro-
2
H + 2H 3H + p ton. Because of the CP T theorem, both p and
2
H + 3 H 4 He + n . p must have the same mass (= 0.94 109 eV).
At threshold, all four nal state particles
have essentially zero velocity with respect to
These sum to
each other. What is the beam energy in that
case? (The actual Bevatron beam energy was
5(2 H) 3 He + 4 He + p + 2n. 6 109 eV).
7. Using Eqs. 1.33 in the lecture notes, prove
that E 2 B 2 , where E (B) is the magnitude of
the electric (magnetic) eld, is a Lorentz invari-
ant.
8. You shine a one-watt beam of photons on a
crow, who absorbs them. Calculate the force (in
N) on the crow.
1

University of California, Berkeley


Physics H7B Spring 1999 (Strovink)
SOLUTION TO PROBLEM SET 8

1. (Taylor and Wheeler problem 27) 2. Prove that


The clock paradox, version 1.
On their twenty-rst birthday, Peter leaves his tanh 1 + tanh 2
tanh (1 + 2 ) = .
twin Paul behind on the earth and goes o in 1 + tanh 1 tanh 2
the x direction for seven years of his time at
24/25 the speed of light, then reverses direction Using this relation, deduce Einsteins law for the
and in another seven years of his time returns addition of velocities.
at the same speed. [In this most elementary
version of the problem, we assume that the nec- Solution:
essary periods of acceleration are innitesimal
in duration, requiring Peters acceleration to be tanh =
innite. Nonetheless, our plucky twin remains exp (2) 1
uninjured.] =
exp (2) + 1
(a.)
tanh 1 + tanh 2 =
Make a spacetime diagram (ct vs. x) showing
Peters motion. Indicate on it the x and ct coor- exp (21 ) 1 exp (22 ) 1
= +
dinates of the turn-around point and the point exp (21 ) + 1 exp (22 ) + 1
of reunion. For simplicity idealize the earth as 2 exp (21 + 22 ) 2
an inertial frame, adopt this inertial frame in the =
exp (21 + 22 ) + 1 + exp (21 ) + exp (22 )
construction of the diagram, and take the origin
tanh 1 tanh 2 =
to be the event of departure.
(b.) exp (21 + 22 ) + 1 exp (21 ) exp (22 )
=
How old is Paul at the moment of reunion? exp (21 + 22 ) + 1 + exp (21 ) + exp (22 )
1 + tanh 1 tanh 2 =
Solution:
On a spacetime (ct vs. x) diagram in Pauls =
2 exp (21 + 22 ) + 2
(unprimed) frame, Peter begins at (0,0) and exp (21 + 22 ) + 1 + exp (21 ) + exp (22 )
proceeds with slope 1 = 25
24 for a time interval tanh 1 + tanh 2 2 exp (21 + 22 ) 2
=
1 + tanh 1 tanh 2 2 exp (21 + 22 ) + 2
ct = ct + (x = 0) = tanh (1 + 2 ) .
= ct
 Suppose that all velocities are in the x direc-
1  tion. Take the velocity of frame S1 with respect
=  24 2 ct
1 25 to frame S to be 1 c; of frame S2 with respect
25 to frame S1 to be 2 c; and of frame S2 with re-
= ct spect to frame S to be 3 c. 1,2,3 correspond to
7
= 25 lightyr . boost parameters (or rapidities) 1,2,3 according
to the relation
At Peters point of maximum excursion, (ct =
25, x = 24) lightyr. Peter then returns with 1,2,3 = tanh 1,2,3 .
slope 1 = 25
24 , reaching x = 0 at ct = 50
lightyr where he reunites with Paul. Peter has The boost parameters have the unique property
aged only 14 years, while Paul has aged 50 years that they are additive, i.e. a boost of 1 fol-
(and has reached the age of 71). lowed by a boost of 2 is equivalent to a boost
2

of 1 + 2 . So, with the above denitions, contains 55.5 NAvo = 55.5 6.023 1023 =
3.35 1025 molecules of heavy water. Each of
3 = 1 + 2 the summed reactions requires ve deuterium
3 = tanh 3 nuclei, or 2.5 molecules of heavy water, so 3.35
= tanh (1 + 2 ) 1025 /2.5 = 1.34 1025 summed reactions take
place. The mass energy in one amu is equivalent
tanh 1 + tanh 2 to mc2 = 0.9315 109 eV, or, with 1 eV =
=
1 + tanh 1 tanh 2 1.61019 J, 1.491010 J. Therefore the energy
1 + 2 released is 0.0267221.341025 1.491010 =
= .
1 + 1 2 5.33 1013 J. This is equivalent to the energy
released by the explosion of 12.7 kilotons of TNT.
This is Einsteins law for the addition of veloci-
ties. A corollary is that about 80 liters one
3. The thermonuclear deuterium-tritium re- Jeep gasoline tank of (2 H)2 O are needed to
actions are: make a one megaton H-bomb. This sets a lower
limit on the degree to which an H-bomb can be
2
H + 2 H 3 He + n miniaturized, irrespective of any espionage.
2
H + 2H 3H + p
2 4. The universe is lled with old cold photons
H + 3 H 4 He + n .
that are remnants of the big bang. Typically
their energy is 6.6 104 eV.
These sum to

5(2 H) 3 He + 4 He + p + 2n. A cosmonaut who is accelerated at 1 g for


10 years in her own rest frame attains a boost
Using the following masses in AMU, (= arctanh ) of 10.34. As seen by her, what is
the typical energy of these photons?
(proton) p 1.007825
Solution:
(neutron) n 1.008665
We know that the energy-momentum four-vector
(deuteron) 2 H 2.014102 (E/c, p) satises the same Lorentz transfor-
(helium 3) 3 He 3.016030 mation equations as the spacetime four-vector
3 (ct, r):
(triton) H 3.016050
4
(alpha particle) He 4.002603 , E  /c = E/c px
px = E/c + px
calculate (to 5%) the kinetic energy released
when one liter of heavy water (2 H)2 O undergoes py = py
deuterium-tritium fusion in an H-bomb. Express pz = pz ,
your answer in terms of tons of TNT (1 ton of
TNT = 4.2109 J of explosive energy). where the cosmonaut (in the primed frame) is
Solution: assumed to be travelling with respect to the
The energy released in the summed reaction big bangs (unprimed) frame with a velocity
corresponds to a mass decit equal to m = c = tanh 10.34 in the x direction. [With re-
5(2.014102) 3.016030 4.002603 1.007825 spect to this large velocity, here we are neglect-
2(1.008665) = 0.026722 amu. Heavy water ing the much smaller speed of 370 km/sec with
has a density about 20
18 times that of ordinary which the solar system moves with respect to
water, due to the extra two neutrons. Thus the big bang radiation; this was rst measured
one liter of heavy water weighs 1.11 kg and by a Berkeley group, including Profs. Smoot and
corresponds to 1.11 3
20 10 = 55.5 moles. It Muller, in the 1970s.]
3

On average, px  = 0 for the big bang pho- where m is the particles rest mass. This is
tons in the big bangs frame; thus the fundamental equation for solving relativistic
kinematics problems. The fundamental equation
E  /c = E/c tells us that p p = 0 and |p| = E/c for any

1 massless particle like the photon. Returning to
= E/c
1 2 the problem,

1
= E/c p p = pa pa + pb pb + 2pa pb
1 tanh2
= E/c cosh 0 = 0 + 0 + 2Ea Eb /c2 2pa pb

= 6.6 104 eV/c cosh 10.34 = 2Ea Eb /c2 (1 cos ab )


E  = 10.2 eV . 1 = cos ab ,

Therefore, while the cosmic background radia-


where ab is the opening angle between the two
tion is in the far infrared as seen in the solar
photons. The last equation tells us that photons
system, on average it is boosted to the ultraviolet
a and b must be travelling in the same direc-
as seen in the frame of the cosmonaut.
tion (they are collinear); by conservation of
5. Prove that an isolated photon (zero mass) momentum, that must be the direction of the
cannot split into two photons which do not both initial photon.
continue in the original direction.
[For the case in which photons a and b
Solution:
do travel in the direction of the initial pho-
Assume that a photon decays into two other
ton, which is allowed by the above kinematic
photons a and b. The photons have energy-
calculation, the decay nevertheless is prevented
momentum four-vectors denoted by (E/c, p),
by conservation of angular momentum. Angu-
(Ea /c, pa ), and (Eb /c, pb ), repectively. Both
lar momentum nonconservation in the collinear
energy and momentum must be conserved in the
decay arises from the photons internal angular
decay. We can express this requirement in a
momentum (spin).]
single four-component equation:
[Also, we note that the (electrically neutral)
(E/c, p) = (Ea /c, pa ) + (Eb /c, p)b ) .
photon couples to electric charge, so, to low-
To save writing we will use the shorthand no- est order, no electromagnetic interaction occurs
tation p (E/c, p); similarly for pa and pb . when three photons meet at a common vertex.
Rewriting the above equation in this shorthand This is not the case for the strong force carriers
notation, and taking the inner product of each (gluons), which also are massless; gluons both
side with itself, carry and couple to a dierent kind of charge
called color.]
p = pa + p b
p p = (pa + pb ) (pa + pb ) [How could we express the above solution
in words? If the two decay photons are not
= pa pa + pb pb + 2pa pb .
collinear, their combined invariant mass must
In the above, the symbol refers to the four- be greater than zero. Since the initial state
vector inner product, i.e. p p E 2 /c2 p p . has invariant mass equal to zero, this violates
Since the inner product of any two four-vectors energy-momentum conservation.]
has the same value in any Lorentz frame, it is
easiest to evaluate p p in the rest frame of the 6. The now retired Bevatron at Berkeley
particle; there one nds that Lab is famous for having produced the rst
observed antiprotons (you may have glimpsed
p p = E 2 /c2 |p|2 = m2 c2 , white-maned Nobelist Owen Chamberlain, one
4

of the rst observers, being helped to his seat at Heisenbergs uncertainty principle. This is called
Physics Department colloquia). An economical Fermi momentum. When the target protons
reaction for producing antiprotons is Fermi momentum is directed against the incom-
ing beam proton, the energy available for the
p + p p + p + p + p , interaction can be augmented up to 20%.
where the rst proton is part of a beam, the 7. Using Eqs. 1.33 in the lecture notes, prove
second is at rest in a target, and p is an antipro- that E 2 B 2 , where E (B) is the magnitude of
ton. Because of the CP T theorem, both p and the electric (magnetic) eld, is a Lorentz invari-
p must have the same mass (= 0.94 109 eV). ant.
At threshold, all four nal state particles Solution:
have essentially zero velocity with respect to The equations for Lorentz transformation of the
each other. What is the beam energy in that electric eld E and magnetic eld B may be
case? (The actual Bevatron beam energy was derived from three facts:
6 109 eV).
(, A) = a four vector
Solution: 1 A
We shall use the notation of the previous solu- E =
c t
tion. Denote by pa and pb the four-momenta of
B=A,
the incident and target protons, each of which
has mass m. At threshold, we are told that
where is the scalar potential and A is the vec-
the four nal-state particles are at rest with re-
tor potential. The result of the derivation is
spect to each other. Therefore, for kinematic
Eq. 1.33 in the distributed relativity notes:
purposes, they are equivalent to a single particle
of mass 4m. Denote by pc the four-momentum E = (E + B )
of this four-particle state. Energy-momentum
conservation demands B = (B E )
E = E
pa + pb = pc
B = B ,
(pa + pb ) (pa + pb ) = pc pc
pa pa + pb pb + 2pa pb = pc pc where c is the velocity of frame S  relative to
2 2 2 2 2 2 S, the subscript refers to the component per-
m c + m c + 2pa pb = (4m) c
pendicular to , and the subscript  refers to the
7m2 c2 = pa pb
component parallel to . Note that, in the rst
= (Ea , pa ) (m, 0) two equations, the subscript may be dropped
= Ea m from the last term, since taking the cross product
Ea = 7mc2 with automatically picks out the perpendicu-
lar part. Using the rst two equations,
= 7 0.94 109 eV
= 6.58 109 eV . 2 (E
 2 2
) = E + 2 B
2
+ 2E ( B )
2 (B
 2 2
) = B + 2 E
2
2B ( E ) .
This is 10% more proton beam energy than
the Bevatron (= 6109 GeV) was able to supply!
We rearrange the last line using the invariance
How then were Chamberlain, Segre, Wie- under cyclic permutation of the triple product:
gand, and Ypsilantis able to discover the an-
tiproton at the Berkeley Bevatron in 1956? They a (b c) = b (c a) = c (a b) ,
took advantage of the fact that protons con-
ned inside the atomic nucleus have a signicant a relation which may be found on the inside
( 200 MeV/c) rms momentum as a result of cover of Griths (distributed in class), or, more
5

physically, may be understood from the fact that


the triple product describes the (invariant) vol-
ume of a parallelopiped with sides a, b, and c.
Cyclically permuting the triple product in the
last line,

2 (B
 2 2
) = B + 2 E
2
2E (B )
2
= B + 2 E
2
+ 2E ( B )
2 2
E B 2
= E + 2 B2 2
B 2 E
2
2
= (1 2 )(E2
B2
)
2 2 2 2
E B = E B .

2 2
This demonstrates that E B is conserved;
2 2
E B is conserved automatically since E
and B are invariant under the transformation.
Finally, E 2 = E2
+ E2 , etc., because the dot
product in the cross term vanishes.
8. You shine a one-watt beam of photons on a
crow, who absorbs them. Calculate the force (in
N) on the crow.
Solution:
Suppose that a photon in the ashlight beam has
an energy E. Then it must have momentum p =
E/c. The beam power (= 1 W) is P = N E,
where N is the number of photons emitted per
second and E is their average energy. If the
photons are totally absorbed by the crow, the
momentum absorbed by the crow per second is

F = N p = N E/c = P/c .

Therefore the force F on the crow is


1W
F = = 3.3 109 N .
c
University of California, Berkeley
Physics H7B Spring 1999 (Strovink)
PROBLEM SET 9

1. Purcell problem 5.3


2. Purcell problem 5.7
3. Purcell problem 5.10
4. Purcell problem 5.17
5. Purcell problem 6.4
6. Purcell problem 6.14
7. Purcell problem 6.18
8. Purcell problem 6.22
University of California, Berkeley
Physics H7B Spring 1999 (Strovink)
SOLUTION TO PROBLEM SET 9
Solutions by P. Pebler

1 Purcell 5.3 A beam of 9.5 M eV electrons ( = 20) amounting as current to 0.05 A, is traveling
through vacuum. The transverse dimensions of the beam are less than 1 mm, and there are no
positive charges in or near it. In the lab frame, what is approximately the electric eld strength
1 cm away from the beam, and what is the average distance between the electrons, measured parallel
to the beam? Answer the same questions for the electron rest frame.
With = 20, the speed of the electrons will be essentially c. The number of electrons passing
per second is
0.05 106 A
e = = 3.1 1011 1/s .
1.6 1019 C
The mean distance between electrons is
c
d= = 1 mm ,
e
and the charge per unit length is
4.8 1010 esu
= = 4.8 109 esu/cm .
0.1 cm
If we think of this as a continuous line charge, the eld strength a distance 1 cm away is
2
E= = 9.6 109 statvolt/cm .
r
Since the electrons are moving, the distance between them will be contracted. Therefore, in the
electron rest frame, they will be more spread out so that
d = d = 2 cm .
We might nd a new eld strength by
E
E = = 4.8 1010 statvolt/cm ,

however, this will be only the average eld strength along a line parallel to the beam. Since the
electrons are so far apart in this frame, there will be big variation in the eld.

2 Purcell 5.7 A moving proton has = 1010 . How far away from such a proton would the eld
rise to 1 V /m as it passes?
The electric eld strength of a moving point charge is (in SI units)
Q 1 2
E = .
4o r2 (1 2 sin2  )3/2
The maximum eld strength is directly beside the particle where  = /2. Then
Q 1 Q
E = 2
 = .
4o r 1 2 4o r2

1
We want the distance where the eld is 1 V /m.
1010 (1.6 1019 C)
= 1 V /m
4(8.85 1012 C 2 /N m2 )r2

r = 3.8 m

3 Purcell 5.10 In the rest frame of a particle with charge q1 another particle with charge q2
is approaching, moving with velocity v not small compared with c. If it continues to move in a
straight line, it will pass a distance d from the position of the rst particle. It is so massive that
its displacement from the straight path during the encounter is small compared with d. Likewise,
the rst particle is so massive that its displacement from its initial position is small compared with
d. Show that the increment in momentum acquired by each particle as a result of the encounter is
perpendicular to v and has magnitude 2q1 q2 /vd. Expressed in terms of other quantities, how large
must the masses of the particles be to justify our assumptions?

In the rst approximation, we consider the trajectory to be a straight line. For this trajectory,
the net impulse in the x direction will be zero. Using cylindrical coordinates,
  
1 q2
pr = Fr dt = Fr dx = Er 2d dx .
v 2dv
The purpose of transforming this integral in this way is so that it becomes the electric ux through
an innite cylinder of radius d. We can evaluate it easily by using Gausss law.

q2 q2 2q1 q2
pr = E da = 4q1 =
2dv 2dv dv
In the rest frame of q2 , the situation is reversed, and q1 acquires the same y momentum in the op-
posite direction. This will be the same momentum in the rest frame of q1 because the perpendicular
component of the momentum is unchanged by a Lorentz transformation.
For this approximation to be good, the acquired y momentum must be much smaller than the
x momentum, so that
2q1 q2
mv  ,
dv
where m can represent either mass.

4 Purcell 5.17 Two protons are moving parallel to one another a distance r apart, with the same
velocity c in the lab frame. At the instantaneous position of one of the protons the electric eld
strength caused by the other is e/r2 . But the force on the proton measured in the lab frame is not
e2 /r2 . Verify that by nding the force in the proton rest frame and transforming that force back to
the lab frame. Show that the discrepancy can be accounted for if there is a magnetic eld times
as strong as the electric eld, accompanying this proton as it travels through the lab frame.

2
In the rest frame of the protons, the force is e2 /r2 . In the lab frame this force is
1 e2 e2
Fy = = + Fy(b) ,
r2 r2
where the rst term is the electric force in the lab frame. The extra term is
   
e2 1 2e
2 e v
Fy(b) = 2 2
1 = 2
= 2 e .
r r r c
From the Lorentz force law
e
F = eE + v B ,
c
we can account for this extra force with a magnetic eld out of the page
B = E z .

5 Purcell 6.4 A long wire is bent into the hairpin like shape shown. nd an exact expression for
the magnetic eld at the point P which lies at the center of the half-circle.

In the integral for the magnetic eld of an innite wire, each half of the wire contributes the
same amount in the same direction, so for each of the half wires, we can take half of the formula
for an innite wire. Also, the half circle will contribute half of the ring formula. All contributions
point out of the page so
1 2I 1 2I 1 2I (2 + )I
Bz = + + = .
2 cb 2 cb 2 cb bc

6 Purcell 6.14 A coil is wound evenly on a torus of rectangular cross section. There are N turns
of wire in all. Assume that the current on the surface of the torus ows exactly radially on the
annular end faces, and exactly longitudinally on the inner and outer cylindrical surfaces. Show that
the magnetic eld everywhere would be circumferential. Second, prove that the eld is zero at all
points outside the torus, including the interior of the central hole. Third, nd the magnitude of the
eld inside the torus as a function of radius.

We set a coordinate system so the point P under consideration is in the z x plane with
coordinates (xo , 0, zo ). Consider a current loop at an angle from the x axis. One small section of
the loop has coordinates (r cos , r sin , z) and
r = (xo r cos , r sin , zo z) .

3
The direction of this little current is arbitrary within the plane so the current direction vector can
be written

I = Ir r + Iz z = (Ir cos , Ir sin , Iz ) .

Then the contribution to the magnetic eld will be in the direction

I r = [sin (Ir (zo z) + rIz )] x + [Iz (xo r cos ) Ir cos (zo z)] y Ir xo sin z .

However, for each section of current, there is a similar section that is identical except that .
We see that the x and z components change sign. Therefore, these components cancel out and the
net eld at the point P is in the y direction. For our coordinate system, this in the circumferential
direction.
Once we have this information, it is easy to use Amperes law.

4
B dl = 2rB = NI
c
2N I
B= a<r<b
cr
If we are outside the solenoid, the enclosed current will be zero and so the eld is zero.

7 Purcell 6.18 Two long coaxial aluminum cylinders are charged to a potential dierence of
50 statvolts. The inner cylinder has an outer diameter of 6 cm, the outer cylinder an inner
diameter of 8 cm. With the outer cylinder stationary the inner cylinder is rotated around its axis
at a constant frequency of 30 Hz. Describe the magnetic eld this produces and determine its
intensity in gauss. What if both cylinders are rotated in the same direction at 30 Hz?
The capacitance for two coaxial cylinders is
L
C= .
2 ln(b/a)
Assuming we have equal amounts of positive and negative charge on the inside and outside respec-
tively, we can nd the charge per unit length.
Q CV V
= = = 87 esu/cm
L L 2 ln(b/a)
The spinning charged cylinders are essentially perfect solenoids. The eld of a long solenoid is
constant inside and practically zero outside. The solenoid formula is
4In
B= ,
c
where n is the number of turns per unit length. The quantity nI can be thought of as the charge
per unit time per unit length passing through a line running parallel to the cylinder. You should
be able to convince yourself that this quantity in our case is Q/L, where is the frequency of
revolution. With just the inner cylinder rotating with = 30 Hz,
4Q 4(87 esu/cm)(30 Hz)
B= = = 1.1 106 gauss r<a ,
cL 3 1010 cm/s
and the eld elsewhere is zero. With a clockwise rotation as shown, the eld is into the page.

4
If the outer cylinder rotates in the same direction, it will produce a eld with the same magnitude
but opposite direction. These two elds will cancel for r < a, but for a < r < b, the eld is
1.1 106 gauss out of the page. It is zero again for r > b.

8 Purcell 6.22 A constant B eld lies in the y z plane. An arbitrary current loop lies in the xy
plane. Show, by calculating the torque about the x axis, that the torque on the current loop can be
written N = m B, where the magnetic moment m of the loop is dened as a vector of magnitude
IA/c where I is the current in esu/s and a is the area of the loop in cm2 , and the direction of the
vector is normal to the loop with a right-hand relation to the current. What about the net force on
the loop?

The force on a small section of loop is


1
dF = Idl B .
c
Only the part of the force in the z direction will contribute to the torque about the x axis. We
split the magnetic eld into two parts in the z and y directions. dl (Bz z) is in the x y plane,
so we only need the y part of B.
1
dFz = IBy sin dl
c
From the gure we see that dl sin = dx so that dFz = 1c IBy dx and the torque is
 
1
Nx = y dFz = IBy y(x) dx .
c loop

This integral if we go forward along the top of the loop and back along the bottom is the area of
the loop and
1
Nx = IBy a .
c
With m = Ia/c and pointing in the z direction,
1
m B = m (By y) = mBy x = IBy a x .
c
For a constant eld there is no net force because
  
1 1
F= dF = I dl B = IB dl
c c

and dl around the loop is zero.

5
University of California, Berkeley
Physics H7B Spring 1999 (Strovink)
PROBLEM SET 10

1. Purcell problem 6.26


2. Purcell problem 6.28
3. Purcell problem 6.32
4. Purcell problem 7.4
5. Purcell problem 7.9
6. Purcell problem 7.11
7. Purcell problem 7.14
8. Purcell problem 7.16
University of California, Berkeley
Physics H7B Spring 1999 (Strovink)
SOLUTION TO PROBLEM SET 10
Solutions by P. Pebler

1 Purcell 6.26 A round wire of radius ro carries a current I distributed uniformly over the cross
section of the wire. Let the axis of the wire be the z axis, with z the direction of the current. Show
that a vector potential of the form A = Ao (x2 + y 2 )z will correctly give the magnetic eld B of this
current at all points inside the wire. What is the value of the constant Ao ?

The magnetic eld is the curl of the vector potential.


B = A = 2Ao y x 2Ao x y
If we use plane polar coordinates in the x y plane,
B = 2Ao r(sin x cos y) = 2Ao r .
We know that the magnetic eld circles in the counterclockwise direction for a current coming out
of the page. We can nd the magnitude from Amperes law.
4 r2
2rB = I
c ro2

2I
B= r
cro2

The vector potential A therefore works with the constant


I
Ao = .
cro2

2 Purcell 6.28 A proton with kinetic energy 1016 eV ( = 107 ) is moving perpendicular to the
interstellar magnetic eld which in that region of the galaxy has a strength 3 106 gauss. What
is the radius of curvature of its path and how long does it take to complete one revolution?
Magnetic forces do no work. They can only change the direction of the momentum. Because
the force is perpendicular to the velocity, we can instantaneously think about the motion as being
along a circle of some radius R. Because the eld and velocity are perpendicular, the magnitude
of the force is
evB
F = .
c
If this were a non-relativistic problem, we could nd the radius R by equating the force with mv 2 /R.
In the relativistic case, this formula turns out to be correct with the replacement m m, but

1
this is something that must be proved. If we wait a time t, the momentum will swing through
some angle , and |p| = p. (Please note that p is not the same thing as |p|.) This angle
will also be the angle of the circle we go through in this time. Therefore, in the innitesimal
limit d, v = r. Consequently,
 
 dp  d pv
 
 dt  = p dt = p = R .

Equating this with the force,


pc mc2 107 (1.5 103 ergs)
R=  = = 1 1019 cm .
eB eB (4.8 1010 esu)(3 106 gauss)
The period is
2 2R 2R
= =  = 2.1 109 s .
v c

3 Purcell 6.32 Two electrons move along parallel paths, side by side, with the same speed v. The
paths are a distance r apart. Find the force acting on one of them in two ways. First, nd the force
in the rest frame of the electrons and tranform this force back to the lab frame. Second, calculate
the force from the elds in the lab frame. What can be said about the force between them in the
limit v c?

In the particle rest frame, the eld is just the Coulomb eld and the force magnitude is e2 /r2 .
If we use the transformation formulas (14) in Purcell, the primed frame must be the particle rest
frame.
1 e2
F= y
r2
To nd the elds in the lab frame, it is easiest to transform them back from the rest frame
where
e
E = 2 y B = 0 .
r
Please note that most transformation formulas found in books assume that the primed frame is
moving in the positive x direction of the unprimed frame. If you wish to use these formulas verbatim,
you must choose your frames correctly. Here the particles are going to the right so the rest frame
is the primed frame. To switch back to the lab frame, we need the inverse of the equations (6.60)
in Purcell. We can accomplish this by simply switching the primes and the sign of . Then

E = E = 0 ,

e
E = (E B ) = E = y ,
r2

B = B = 0 ,

2
e
B = (B + E ) = z .
r2
The force is then
(e) e2 e2 e2 1 e2
F = (e)E + v B = 2 y 2 2 y = 2 (1 2 ) y = y .
c r r r r2
In the limit v c, we see that F 0.

4 Purcell 7.4 Calculate the electromotive force in the moving loop in the gure at the instant
when it is in the position there shown. Assume the resistance of the loop is so great that the eect
of the current in the loop itself is negligible. Estimate very roughly how large a resistance would be
safe, in this respect. Indicate the direction in which current would ow in the loop, at the instant
shown.

We rst calculate the ux. We will dene the positive direction to be into the page. The eld
is that of a wire. The current is given in SI, so we must use SI formulas.
  x+L
o I o Iw x + L
B da = w dr = ln
x 2r 2 x
 
d o Iw x v (x + L)v o Iw Lv
E = B = =
dt 2 x + L x x2 2 x(x + L)

E = 2.1 105 V

By choosing into the page as positive for ux, we have also dened clockwise as the positive way
to go around the loop. Since E is positive, the induced current will be clockwise.

5 Purcell 7.9 Derive an approximate formula for the mutual inductance of two circular rings of
the same radius a, arranged like wheels on the same axle with their centers a distance b apart. Use
an approximation good for b a.
From Purcell Eq. 6.41 (where a and b are interchanged relative to this problem) the eld along
the axis of a ring is
2a2 I
Bz = .
c(a2 + z 2 )3/2
We may use the information b a, and approximate the z component of the eld everywhere in
the second loop as
2a2 I 2a2 I
Bz =  ,
c(a2 + b2 )3/2 cb3

3
so the ux is
2a2 I 2 2 2 a4
B = a = I .
cb3 cb3
The induced emf is then
1 d 2 2 a4 dI
E = B = 2 3 ,
c dt c b dt
and the mutual induction is
2 2 a4
M= ,
c2 b3
in cgs units. If you use SI formulas this becomes
o 2 2 a4
M= .
4 b3

6 Purcell 7.11 Two coils with self-inductances L1 and L2 and mutual inductance M are shown
with the positive direction for current and electromotive force indicated. The equations relating
currents and emf s are
dI1 dI2 dI2 dI1
E1 = L1 M E2 = L2 M .
dt dt dt dt
Given that M is always to be taken as positive, how must the signs be chosen in these equations?
What if we had chosen the other direction for positive current and emf in the lower coil? Now
connect the two coils together as in b. What is the inductance L of this circuit? What is the
inductance L of the circuit formed as shown in c? Which circuit has the greater self-inductance?
Considering that the self-inductance of any circuit must be a positive quantity, see if you can deduce
anything concerning the relative magnitudes of L1 , L2 , and M .

Imagine rst that the current I2 is positive and increasing so that dI2 /dt > 0. In this case the
magnetic eld due to coil 2 will point up through coil 1. As the current I2 increases, the eld it
creates will increase and the ux up through coil 1 will increase. By using Lenzs law, we nd we
need an induced current that will create a magnetic eld that will oppose this change in the ux.
In this case, the eld should point down through coil 1. To do this the induced current must ow
in the negative direction as it is dened for coil 1. Thus, the induced emf must be negative and we
need the negative sign. The same argument will tell you to choose the negative sign in the second
equation also. (You should go through it yourself however.)

4
If the sign convention for coil 2 had been switched, the same argument would switch the sign
in both equations. (Do it yourself though.)
With the circuit in b, since both emf positive directions point in the same way, the total emf
across the new circuit is
dI1 dI2 dI2 dI1
E = E1 + E2 = L1 M L2 M .
dt dt dt dt
We also have I = I1 = I2 so that
dI
E = (L1 + L2 + 2M ) ,
dt
and the self inductance is

L = L1 + L2 + 2M .

With the circuit in c, the sign conventions conict so that


dI1 dI2 dI2 dI1
E = E1 E2 = L1 M + L2 +M ,
dt dt dt dt
but with I = I1 = I2 so that
dI
E = (L1 + L2 2M ) ,
dt
and the self inductance is

L = L1 + L2 2M .

If the self inductance of a coil were negative, the circuit would be unstable any change in current
would result in more current the same direction which would built indenitely. Therefore we must
have L > 0 and
L1 + L2
M .
2

5
7 Purcell 7.14 A metal crossbar of mass m slides without friction on two long parallel conducting
rails a distance b apart. A resistor R is connected across the rails at one end; compared with R, the
resistance of bar and rails is negligible. There is a uniform eld B perpendicular to the plane of the
gure. At time t = 0 the crossbar is given a velocity vo toward the right. What happens then? Does
the rod ever stop moving? If so, when? How far does it go? How about conservation of energy?

Let us assume the magnetic eld is into the page, and lets make that positive so that clockwise
is positive for the loop. The ux is then

B = bxB .

The emf (in SI) is


d
E = B = bvB = IR ,
dt
so the current is counterclockwise. The bar will feel a force due to the magnetic charges moving
through it. The force is

F = |I|L B = |I|bB x .

We can solve for the motion using F = ma.


dv b2 B 2
m = |I|bB = v
dt R
2 B 2 t/mR
v(t) = vo eb

The bar never stops moving. It will approach the distance



2 B 2 t/mR mRvo
d= vo eb dt = .
0 b2 B 2
The lost kinetic energy is dissipated by the resistor.
b
I= Rv
B
  2 2
b B 2 2b2 B 2 t/mR b2 B 2 2 mR 1
U= I 2 Rdt = vo e = vo 2 2 = mvo2
o o R R 2b B 2

6
8 Purcell 7.16 The shaded region represents the pole of an electromagnet where there is a strong
magnetic eld perpendicular to the plane of the paper. The rectangular frame is made of 5 mm
diameter aluminum. suppose that a steady force of 1 N can pull the frame out in 1 s. If the force is
doubled, how long does it take? If the frame is made of 5 mm brass, with about twice the resistivity,
what force is needed to pull it out in 1 s? If the frame were 1 cm diameter aluminum, what force
is needed to pull it out in 1 s? Neglect inertia of the frame and assume it moves with constant
velocity.

If we assume a constant velocity, the force necessary to pull out the loop will be equal in
magnitude to the magnetic force on the loop. We will ignore signs here. The net force will be on
the left wire of the frame.

F = |I|hB

The magnetic ux will be something like = LxB, where x is the length of loop in the eld. Then
the emf is
 
 d 
|E| =  = hvB .
dt 
The current is |I| = |E|/R and the resistance is R = L/A = L/r2 , where L is the total length
of the loop and r is the radius of the wire of which it is made.
A vr2
F = hBhBv
L
If the force is doubled, the speed doubles and it takes half the time or 0.5 s. If the resistivity
doubles with the same speed, the force is halved so that F = 1 N . If the radius doubles with the
same speed, the force is four times as great or 4 N .

7
University of California, Berkeley
Physics H7B Spring 1999 (Strovink)
PROBLEM SET 11

1. Purcell problem 7.21. Assume that b2  a2


and b2  b1 .
2. Purcell problem 7.22. Assume that the charge
is distributed uniformly around the ring.
3. Purcell problem 7.23
4. Purcell problem 7.29
5. Purcell problem 8.5. For simplicity, if you
wish, in parts (a) and (b) you may assume that
the battery is connected only until just before
the switch is closed.
6. Purcell problem 8.7
7. Purcell problem 8.11
8. Purcell problem 8.16. An infinite L-C ladder
of the type studied in this problem is called a
lumped-element delay line. It is of great prac-
tical importance when one needs to delay an ana-
log electrical pulse by a longer time than would
be conveniently achieved by using the finite
speed ( c) of propagation in a coaxial cable.
University of California, Berkeley
Physics H7B Spring 1999 (Strovink)
SOLUTION TO PROBLEM SET 11
Solutions by P. Pebler

1 Purcell 7.21 A solenoid of radius a1 and length b1 is located inside a longer solenoid of radius
a2 and length b2 . The total number of turns is N1 in the inner coil and N2 on the outer. Work out
a formula for the mutual inductance M .

The mutual inductances M12 and M21 are equal, so we are free to calculate the inductance with
whatever coil is more convenient. We nd the ux through the inner coil. If we assume b2  b1 ,
the eld through coil 1 will be fairly uniform and with the sign conventions shown,
o N2 I2
B2 = x ,
b2
along the common axis of both coils. Since there are N1 loops in this coil, the ux through all of
them is
o N2 I2
12 = a21 N1 ,
b2
and the induced emf is
d o a21 N1 N2 dI2
E12 = 12 = ,
dt b2 dt
and the mutual inductance is
o a21 N1 N2
M= .
b2

2 Purcell 7.22 A thin ring of radius a carries a static charge q. This ring is in a magnetic eld of
strength Bo , parallel to the rings axis, and is supported so that it is free to rotate about that axis.
If the eld is switched o, how much angular momentum will be added to the ring? If the ring has
mass m, show that it will acquire an angular velocity = qBo /2mc.
Well assume the charge is uniformly distributed around the ring, with linear density = q/2a.
Then the torque about the z axis is
  
q q
z = dz = af dl = E dl = E .
2 2

1
The emf is
1 d
E = ,
c dt
so that
dLz q d
z = = ,
dt 2c dt
and
q q
Lzf Lzi = = a2 Bo ,
2c(f i ) 2c

qa2 Bo
Lzf = .
2c

The moment of inertia of the ring is I = ma2 , and


qa2 Bo
Lzf = I = ma2 = ,
2c
qBo
= ,
2mc
equal to half the cyclotron frequency of a particle with mass m and charge q in a eld B0 .

3 Purcell 7.23 There is evidence that a magnetic eld exists in most of the interstellar space with
a strength between 106 and 105 gauss. Adopting 3 106 gauss as a typical value, nd the total
energy stored in the magnetic eld of the galaxy. Assume the galaxy is a disk roughly 1023 cm in
diameter and 1021 cm thick. Assuming stars radiate about 1044 ergs/s, how many years of starlight
is the magnetic energy worth?
The magnetic energy is

1 1
U= B 2 dV = (3 106 gauss)2 (1021 cm)(1023 /2 cm)2 = 3 1054 ergs ,
8 8
and this is
3 1054 ergs
= 3 1010 s = 900 yr
1044 ergs/s
of starlight.

4 Purcell 7.29 Consider the arrangement shown. The force between capacitor plates is balanced
against the force between parallel wires. An alternating voltage of frequency f is applied to the
capacitors C1 and C2 . The charge owing through C2 constitutes the current through the rings.
Suppose the time-average downward force on C2 exactly balances the time averaged force on the
wire loop. Show that under these conditions the constant c is
 1/2  
3/2 b C 2
c = (2) a f .
h C1
Assume h  b and ignore the self-inductance of the wire loop.

2
The electric eld in capacitor C1 is
1
E = Eo cos t ,
s
and the charge on it is
Q = C1 V = C1 Eo cos t .
The downward force will be the charge times the electric eld due to the bottom plate. This eld
will be half of the total eld.
1 E 2 C1
F1 = EQ = o cos2 t
2 2s
Because the capacitance is
a2 a2
C1 = = ,
4s 4s
and the time average of cos2 is 1/2, we may rewrite this as
Eo2 C12
F1 = .
a2
If we have h  b, we can use the force of two long wires. The eld due to a wire is
2|I|
B= ,
cr
and the total force on the wire will be
1 1 2I 2 4bI 2
F2 = |I|LB = 2b = 2 .
c c ch c h
The current is the time derivative of the charge on capacitor 2.
dQ2 d
I= = (C2 V ) = C2 Eo 2f sin t
dt dt
4b 2 2
F2 = C E (2f )2 sin2 t
c2 h 2 o

3
The time average of sin2 is also 1/2 so
8 3 bC22 f 2 Eo2 2 2
1 = C1 Eo ,
F2 = = F
c2 h a2
 1/2  
3/2 b C 2
c = (2) a f .
h C1

5 Purcell 8.5 The coil in the circuit shown in the diagram is known to have an inductance of
0.01 henry. when the switch is closed, the oscilloscope sweep is triggered. Determine the capacitance
C. Estimate the value of the resistance R of the coil. What is the magnitude of the voltage across
the oscilloscope input a long time, say 1 second after the switch has been closed?

Parts (a.) and (b.) of this problem may be approximated by assuming that the battery is
disconnected when the switch is closed. However, the problem actually is not too bad with the
battery connected, so we will solve the original problem. The answers are the same except for the
nal voltage.
If you work out the equation for the charge on the capacitor C, you will nd
   
d2 Q L dQ R1 + R2 Q V R2
L 2 + R2 + + = .
dt R1 C dt R1 C R1
If we assume that the resistance R2 of the inductor is much less that R1 , this becomes the LCR
circuit equation. From the trace we see that
2 4
= = 8 103 Hz .
103 s
For low damping the capacitance is approximately
1
C= = 1.6 107 F .
L 2
Also from the trace, the amplitude falls o by a factor of e in about 0.5 103 s.

eRt/2L = e1
2L
R= = 40 ohms
t

4
If we wait one second, a long time, things will settle so that a steady current passes through the
inductor and the voltage across it will be due to the resistance. If the current is I, the voltage is
V2 = IR2 and
 
5 105 + 40
20 V = I(10 ohm + R2 ) = V2 ,
40

V = 8 mV .

6 Purcell 8.7 A resonant cavity of the form illustrated is an essential part of many microwave
oscillators. It can be regarded as a simple LC circuit. The inductance is that of a toroid with
one turn. Find an expression for the resonant frequency of this circuit and show by a sketch the
conguration of the magnetic and electric elds.

The inner narrow circle will act as the capacitor while the outer ring is the solenoid. For a
single turn toroid, the inductance is
2h b
L= ln ,
c2 a
and for the parallel plate capacitor,
a2 a2
C= = ,
4s 4s
so that
 
1 c 2s
= = .
LC a h ln(b/a)

The electric eld is concentrated in the circular gap, where its direction is vertical; the magnetic
eld in the toroidal cavity is azimuthal in direction, with magnitude proportional to r1 .

5
7 Purcell 8.11 An alternating voltage Vo cos t is applied to the terminals at A. The terminals at
B are connected to an audio amplier of very high input impedance. Calculate the ratio |V1 |2 /Vo2 ,
where V1 is the complex voltage at terminals B. Choose values for R and C to make |V1 |2 /Vo2 = 0.1
for a 5000 hz signal. Show that for suciently high frequencies, the signal power is reduced by a
factor 1/4 for every doubling of the frequency.

Since the impedance on the right is very large, the impedance of the circuit is approximately
1
Z =R+ ,
iC
and the magnitude is

1
|Z| = R2 + .
2C 2
This gives us the magnitude of the complex current.
Vo Vo
Io = = 2
|Z| R + 1/ 2 C 2
The impedance of just the capacitor is
1 1
ZC = |ZC | = .
iC C
This gives us the magnitude of the voltage across C.
Vo
|V1 | = Io |ZC | =
R2 C 2
2 +1

|V1 |2 1
= 2 2 2
Vo2 R C +1

We would like an R and C such that


1
= 0.1 ,
[2(5000 hz)]2 R2 C 2 + 1
which can be done with many values of R and C, for example

R = 100 ohm C = 1 F .

For suciently high frequencies we have

P |V1 |2 2 .

A reduction of signal power by a factor 8 rather than 4 per octave can be achieved by substituting
an inductor L for the resistor R.

6
8 Purcell 8.16 An impedance Zo is to be connected to the terminals on the right. For given
frequency nd the value which Zo must have if the resulting impedance between the left terminals
. The required Zo is a pure resistance Ro provided 2 < 2/LC. What is Zo in the special case
is Zo
= 2/LC?

We combine the impedances like resistances so that the total impedance is


1
Z = ZL + 1 1 ,
ZC + ZL +Zo

with ZL = iL and ZC = 1/iC. We set this equal to Zo and simplify to obtain



Zo = 2 L2 + 2L/C .

This will be pure real and thus a pure resistance if


L
2 L2 + 2 >0 ,
C
2
2 < .
LC

In the special case = 2/LC, we have Zo = 0.

7
University of California, Berkeley
Physics H7B Spring 1999 (Strovink)
PROBLEM SET 12

1. Purcell problem 9.3


2. Purcell problem 9.5
3. Purcell problem 9.9
4. Purcell problem 9.10
5. Purcell problem 10.7
6. Purcell problem 10.17
7. Purcell problem 10.19
8. Purcell problem 10.21
University of California, Berkeley
Physics H7B Spring 1999 (Strovink)
SOLUTION TO PROBLEM SET 12
Solutions by P. Pebler

1 Purcell 9.3 A free proton was at rest at the origin before the wave
(5 statvolt/cm) y (5 gauss) z
E= B=
1 + [k(x + ct)]2 1 + [k(x + ct)]2
came past with k = 1 cm1 . Where would you expect to nd the proton after 1 s? The proton
mass is 1.6 1024 g.
To begin, we will neglect the magnetic force and see later if this is justied. In this case, the
impulse due to the electric force will be in the y direction. The pulse only has an appreciable
magnitude for a few nanoseconds, so we may extend the integral to innity.
 
dt (5 statvolt/cm)e
p = Fe dt = e (5 statvolt/cm)y = y
1 + (kct)2 kc

p = (2.5 1019 g cm/s) y

This corresponds to a speed of

v = 1.6 105 cm/s .

From the Lorentz force law


e
F = eE + v B ,
c
we see that because the electric and magnetic elds have the same strength, the magnetic force is
smaller by
v
FB  Fe  (5 104 )Fe ,
c
so our approximation is pretty good. The acceleration while the pulse is passing occurs for a very
small time, so the position of the proton after one microsecond is essentially

y = (1.6 105 cm/s)(1 106 s) = 0.16 cm .

2 Purcell 9.5 Consider the wave in free space

Ex = 0 Ey = Eo sin(kx t) Ez = 0

Bx = 0 By = 0 Bz = Eo sin(kx t) .

Show that this eld can satisfy Maxwells equations if and k are related in a certain way. Suppose
= 1010 hz and Eo = 0.05 statvolt/cm. What is the wavelength in cm? What is the energy density
in ergs/cm3 , averaged over a large region? From this calculate the power density, the energy ow
in ergs/cm2 s.

1
We see immediately that E = B = 0. Its also easy to calculate

E = kEo cos(kx + t) z ,

B = kEo cos(kx + t) y ,

1 B
= Eo cos(kx + t) z ,
c t c
1 E
= Eo cos(kx t) y .
c t c
The other two Maxwells equations will be satised if

c= .
k
In this case,
2 1010 1/s 1
k= = = ,
3 1010 cm/s 3 cm

= 6 cm = 18.8 cm .

The average energy density is


Eo2 (0.05 statvolt/cm)2
= = 9.95 105 erg/cm3 ,
8 8
and the average intensity (= power density) is
cEo2
= 3 106 erg/cm2 s .
8

3 Purcell 9.9 The cosmic microwave background radiation apparently lls all space with an energy
density of 4 1013 erg/cm3 . Calculate the rms electric eld strength in statvolt/cm and in V /m.
Roughly how far away from a 1 kW radio transmitter would you nd a comparable electromagnetic
wave intensity?
The average energy density is
2
Erms
= 4 1013 erg/cm3 ,
4
and

Erms = 2.2 106 statvolt/cm = 0.067 V /m .

Assuming the transmitter projects in all directions, a distance r away, the intensity of the trans-
mitter is
1 kW 2
Erms (0.067 V /m)2
=  = ,
4r2 o / o 376.73 ohms

r = 2584 m .

2
4 Purcell 9.10 Find the magnetic eld at a point P midway between the plates of capacitor a
distance r from the axis of symmetry. A current I is owing through the capacitor.
We assume that the magnetic eld circles the capacitor axis. If the capacitor spacing is small,
the electric eld will be fairly uniform and
V Q 4sQ 4Q
E= = = 2
= 2 ,
s sC sb b

4r2 Q
E = r2 E = ,
b2
and ignoring signs,
1 4r2 dQ
2rB = ,
c b2 dt
2rI
B= .
cb2
At the edge of the capacitor (r = b) this is the same as the magnetic eld around a long wire.

5 Purcell 10.7 A cell membrane typically has a capacitance around 1 F/cm2 . It is believed
the membrane consists of material having a dielectric constant of about 3. Find the thickness this
implies. Other electrical measurements have indicated that the resistance of 1 cm2 of cell membrane
is around 1000 ohms. Show that the time constant of such a leaky capacitor is independent of the
area of the capacitor. How large is it in this case? What is the resistivity?
The capacitance is given in Farads so we will use SI. The constant o appears in SI formulas.
To deal with a dielectric material, we make the replacement o . However, in SI, is not
dimensionless. For example, if = 3 in cgs, the value in SI is 3 o . The capacitance of a parallel
plate capacitor is
A 1 cm2
C= =3 = 1 106 F ,
s s
so

s = 2.66 109 m .

We may view the leaky capacitor as a simple RC circuit, where the resistor and the capacitor are
really the same element. The time constant is
s A
= RC = = ,
A s
which is independent of the area of the membrane. It is also independent of its thickness..

= (1000 ohms)(1 106 F ) = 1 103 s = 3(8.85 1012 C 2 /N m2 )

= 3.8 107 ohm m

= sec

3
6 Purcell 10.17

In the rst two cases, we assume the left dipole to be present and we bring in the right dipole
from innity. We would like to do this in such a way that the work required is zero. This will be
the case if the path of the dipole coming in is perpendicular to the force on it. We will bring in the
second dipole on a straight line from the right. The force on it is

F = (p )E ,

where E is the eld created by the other dipole. We wish to nd an orientation for the right dipole
so that this force is perpendicular to the path. We can do this if the dipole is pointed to the right.
In this case the force is
E
F=p .
x
The eld from the left dipole on the line of the path is
p p
E= 3
z = 3 z .
r x
The force is then
p2
F=3 z ,
x4
which is perpendicular to the path. Intuitively we can think of the dipole as two charges. The
positive charge feels a force down and the negative charge feels a force up. But the positive charge
is further away so the force on it is smaller. The net force is then up. But since this is perpendicular
to the path, it still requires no work to bring in the dipole.
The eld at the right dipole is
p
E= z ,
d3
and the torque exerted on this is

= p E = pE sin y .

The work we do in rotating the dipole is minus the work done by the eld. So

p2
W = pE sin d = pE = .
/2 d3
This is good since the dipoles dont like to point in the same direction in this orientation so it
should take positive work to arrange it.
In the second case, we rotate the dipole the same amount in the opposite direction and do the
opposite work.
p2
W = pE =
d3
In the third situation, the eld from the left dipole points to the right along the path, so we
cant bring in the second dipole pointing to the right. However, if we bring it in pointing up, the

4
force on it will be up, perpendicular to the path. Taking the derivative is a little messy in this case,
but we can nd the direction of the derivative intuitively. If you think of the dipole as two charges,
the force on the positive charge will be up to the right, and the force on the negative charge will
be up to the left. But since the eld is symmetric with respect to the x axis, the x components
cancel out leaving a force up.

The eld at the second dipole in this case has strength E = 2p/d3 . In analogy with the above
results, the work to rotate it to the right is
p2
W = 2 ,
d3
because the dipole wants to be in this orientation. The work for the nal situation is then
p2
W =2 .
d3

7 Purcell 10.19
If the ion is positive, the dipole will point away from it. The dipole eld at the ion location will
then point towards the dipole and the force will be attractive. If the ion is negative, the dipole will
point towards it and the dipole eld at the ion will be away from the dipole. The force again will
be attractive. The polarization is p = E = q/r2 where q is the ion charge. The force on the ion
is
2p 2q 2
F =q = .
r3 r5
To nd the potential energy, we bring in the ion from innity. The work required is
 r
2q 2 q 2
U = dr = .
r5 2r4

For sodium, = 27 1024 cm3 .


(4.8 1010 esu)2 (27 1024 cm3 )
4 1014 erg =
2r4

r = 9.4 108 cm

5
8 Purcell 10.21
The maximum eld strength is
14 103 V
Em = = 5.5 108 V /m .
0.0000254 m
The energy density is
1 2 1
E = (3.25)(8.85 1012 C 2 /N m2 )(5.5 108 V /m)2 = 4.35 106 J/m3 .
2 m 2
Remember that in SI, we need to insert the o. The energy per mass is then
4.35 106 J/m3
= 3100 J/kg .
1400 kg/m3
This can raise the capacitor to a height

mgh = (0.75)m(3107 J/kg) ,

h = 238 m .

6
University of California, Berkeley
Physics H7B Spring 1999 (Strovink)
PROBLEM SET 13

1. Purcell problem 10.13


2. Purcell problem 10.16
3. Purcell problem 11.2
4. Purcell problem 11.4
5. Purcell problem 11.7
6. Purcell problem 11.9
7. Purcell problem 11.12
8. Purcell problem 11.16
University of California, Berkeley
Physics H7B Spring 1999 (Strovink)
SOLUTION TO PROBLEM SET 13
Solutions by P. Pebler

1 Purcell 10.13
Consider a parallel plate capacitor. The energy required to charge it to a potential dierence
V is E = CV 2 /2. The capacitance increases with a dielectric to C = Co = A/4s. The potential
dierence is Es. Then
1 AE 2 s2  2
E = CV 2 = = E (As) ,
2 8s 8
and the energy density is
E2
 .
8

For a wave in a dielectric B = E and the energy density in the magnetic eld is
B2 E2
= .
8 8

2 Purcell 10.16
We use Gausss law inside the uniform spherical charge distribution.
4 3
4r2 Er = 4Qenc = 4 r
3
4
E= r
3
Let the sphere of density be centered at the origin, and the sphere of density be centered at
the location s. The total eld is
4 4 4
E= r + ()(r s) = s .
3 3 3
In the middle of a long cylinder, we can nd the eld from Gausss law.

2rLEr = 4(r2 L)

E = 2rr

We are using cylindrical coordinates here so r points away from the axis. The total eld of two
cylinders with their axes displaced by s is

E = 2rr + 2()(rr s) = 2s .

1
3 Purcell 11.2
The magnetic eld of a current loop with its axis on the z axis has only a z component with
2b2 I 2m
Bz = = 2 .
c(b2 + z 2 )3/2 (b + z 2 )3/2
The dipole eld on this axis is all radial, which here is the z direction.
2m 2m
Bz = Br = = 3
r3 z
So
z3
Bz = B
(b2 + z 2 )3/2 z
and the loop eld approaches the dipole eld when z  b. There is a 1% dierence when
z3
= 0.99 ,
(b2 + z 2 )3/2
z = 12.2 b .

4 Purcell 11.4
The earths radius is about 6 108 cm so
2m
0.62 gauss = ,
(6 108 cm)3

m = 6.7 1025 erg/gauss = 6.7 1022 J/T .

If we have a current loop of radius 3 108 cm, we need a current I where


2(3 108 cm)2 I
0.62 gauss = ,
c[(3 108 cm)2 + (6 106 cm)2 ]3/2

I = 9.9 1018 esu/s = 3.3 109 A .

5 Purcell 11.7

We will use polar coordinates for the integration. We divide the surface into little strips sub-
tended by the small change in polar angle d. The surface area of one of these strips is
da = 2(R sin )(Rd) .

2
The amount of charge on this strip is
Q 1
dq = da = 2
2R2 sin d = Q sin d .
4R 2
This charge revolves around with a frequency f = /2, so it represents a little current
Q
dI = f dq = sin d .
4
Each strip contributes a moment dm = A dI/c.
  /2  /2
1 2 Q QR2 QR2
m= A dI = (R sin )2 sin d = sin3 d =
c c 0 4 2c 0 3c

6 Purcell 11.9

From Chapter 6, the eld from a nite solenoid is


2In
Bz = (cos 1 cos 2 ) .
c
For a semi-innite solenoid, 2 = and with z measuring the distance of the point outside the top
of the solenoid,
 
2In z
Bz = 1  2 .
c z + ro2

We want to maximize Bz (dBz /dz).


   
dBz 2In 1 z2 2In ro2
=  =
dz c z 2 + ro2 (z 2 + ro2 )3/2 c (z 2 + ro2 )3/2
 
dBz 1 z
Bz 2 1  2
dz (z + ro2 )3/2 z + ro2

Taking a derivative and setting to zero yields the equation



3z 2 ro2 = 3z z 2 + ro2 .

Squaring and solving the quadratic equation gives z 2 = ro2 /15. Only the negative root solves the
original equation so

1
z = ro .
15
This is slightly inside the solenoid.

3
7 Purcell 11.12

The potential of a single dipole in a magnetic eld can be chosen to be

U = m B .

This does not have the zero where we want our zero to be. However, for the purposes of nding
work done in rotating the dipoles we may use

W = Uf U i .

In the initial conguration, the eld due to dipole 2 at m1 is as shown above with
m2
B2 = .
r3
The work required to rotate m1 is
m1 m2
W1 = Uf Ui = m1 B2 cos(90 + 1 ) 0 = m1 B2 sin 1 = sin 1 .
r3
To rotate m2 , we break up the eld from m1 into two parts with
2m1 cos 1 m1 sin 1
B1 = B1 = .
r3 r3
The work to rotate m2 is then

W2 = Uf Ui = [m2 B1 cos 2 m2 B1 cos(90 + 2 )] [m2 B1 cos ]


= m2 B1 cos 2 + m2 B1 sin 2 m2 B1
2m1 m2 m1 m2 m1 m2
= 3
cos 1 cos 2 + 3
sin 1 sin 2 sin 1 .
r r r3
the total work is
m1 m2
W = W1 + W2 = (sin 1 sin 2 2 cos 1 cos 2 ) .
r3

4
8 Purcell 11.16

The exterior eld of a uniformly magnetized sphere turns out to be that of a magnetic dipole
with dipole moment
4 3
m= r M .
3
This is something that needs to be proved, however. One can prove this by nding the eld from
the bound current. The bound current density is

Jb = c M = 0 ,

and the bound surface current is

Kb = c M n = M sin .

This is identical to the surface current of a rotating sphere with uniform surface charge. One can
integrate to nd the vector potential which is that of a magnetic dipole at the center. We leave
this to you as an exercise.
The eld at the pole is
2m 8
B= 3
= (750 erg/gauss cm3 ) = 6280 gauss .
r 3
At the equator
m
B= = 3140 gauss .
r3
To nd the force, we need to know the force on a uniformly magnetized sphere in the eld of a
dipole. Fortunately, this is simple due to the following argument. The force on the sphere on the
right must be the same if we replace the sphere on the left with a dipole at its center. This force
must be equal and opposite to the force on the imaginary dipole. But the eld from the sphere
on the right at the dipole is that of a dipole, so the force between spheres is the same as the force
between two dipoles. (This is not obvious without the argument just given.)
    
 db1z   
F = m2   = m2  d 2m1  = 6 m1 m2 = 3 m1 m2
dz   dz z3  z4 8 r4
 2
3 4
F = M r2 = 3.7 106 dynes = 37 N
8 3

5
University of California, Berkeley
Physics H7B Spring 1999 (Strovink)
EXAMINATION 1

Directions. Do all four problems (weights are indicated). This is a closed-book closed-note exam
except for one 8 12 11 inch sheet containing any information you wish on both sides. You are free to
approach the proctor to ask questions but he or she will not give hints and will be obliged to write
your question and its answer on the board. Dont use a calculator, which you dont need roots,
circular functions, etc., may be left unevaluated if you do not know them. Use a bluebook. Do not
use scratch paper otherwise you risk losing part credit. Cross out rather than erase any work that
you wish the grader to ignore. Justify what you do. Box or circle your answer.

1. (25 points) A heat engine for which the work- c. (8 points) Deduce the value of the mechan-
ing material is an ideal monatomic gas moves ical work 
slowly enough that all parts of it are always in p dV
mutual equilibrium. It is described by a rect- 12341
angular path on the T (absolute temperature)
done by the gas on the rest of the universe
S (entropy) plane, as in the gure. While on
over one complete cycle of the engine.
the path 1 2, the gas in the engine takes heat
from a bath at high temperature T1 ; on the path d. (7 points) In one cycle, what fraction of the
3 4, it returns heat to bath at lower tempera- heat withdrawn from the hot reservoir is
ture T3 . On the paths 2 3 and 4 1, the en- converted to mechanical work done by the
tropy has constant values S3 and S1 , respectively. gas on the rest of the universe?
Hint: Keep in mind that the only parameters
given in this problem are T1 , T3 , S1 , and S3 ;
your answers, if nontrivial, must be expressed in
terms of these parameters.

2. (25 points) In a hypothetical one-dimensional


system, thermal motion of atoms in the y and
z directions is frozen out, so, eectively, the
atoms are able to move only in the x direc-
tion. In that direction, an atom has velocity v
( < v < ). The fraction dF of atoms with
velocity between v and v + dv is
a. (5 points) Write down the net change
 2
(U23 + U41 ) exp mv
2kT dv
dF fv (v) dv =   2 ,

exp mv
2kT dv
in internal energy for the sum of the two
paths 2 3 and 4 1.
where fv (v) is the probability density (RHK:
b. (5 points) Compute the net change relative probability) of the value v, m is the
 atomic mass, k is Boltzmanns constant, and T
T dS is the absolute temperature.
12341
a. (10 points) Calculate the mean value of the
over one complete cycle of the engine. square of v, i.e. v 2 . If you wish, you may
leave your answer in the form of a ratio of 4. (25 points) The innite plane z = 0 car-
denite integrals. Do not merely guess the ries a uniform surface charge density esu/cm2 .
answer. There are no other charges in the problem.
b. (15 points) Dene E to be the kinetic energy
1 2
2 mv of an atom. The fraction dF of atoms
with kinetic energy between E and E +dE is

dF fE (E) dE ,

where fE (E) is the probability density of


the value E. One might imagine fE (E) to
take the possible forms:
 E 
fE (E) E 1/2 exp ?
kT
 E 
exp ?
kT
 E 
E 1/2 exp ?
kT
 E  a. (5 points) Find the magnitude and direc-
E exp ? tion of the electric eld E+ everywhere in
kT
the region z > 0.
Which one form is correct, and why?
b. (5 points) Find the magnitude and direc-
tion of the electric eld E everywhere in
3. (25 points) the region z < 0.
A xed line charge of + esu/cm on the x c. (8 points) Consider a spherical surface of
axis extends from x = D to x = 2D, and a xed radius R centered at the origin. Find the
line charge of esu/cm on the y axis extends electric ux 
from y = D to y = 2D. E da

through the top half (top hemisphere) of


this surface.
d. (7 points) Consider a second spherical sur-
face, again of radius R, but now centered
at the point (0, 0, 2R), so that it does not
enclose any charge. Find the electric ux

E da

a. (10 points) Find the work required to bring through the bottom half (bottom hemi-
a test point charge q from innity to the ori- sphere) of this surface.
gin. Does your answer depend on the path
you chose? If so, specify the path.
b. (15 points) Calculate the mechanical force
(magnitude and direction) that is required
to keep the test charge at the origin.
1

University of California, Berkeley


Physics H7B Spring 1999 (Strovink)
SOLUTION TO EXAMINATION 1

Directions. Do all four problems (weights are indicated). This is a closed-book closed-note exam
except for one 8 12 11 inch sheet containing any information you wish on both sides. You are free to
approach the proctor to ask questions but he or she will not give hints and will be obliged to write
your question and its answer on the board. Dont use a calculator, which you dont need roots,
circular functions, etc., may be left unevaluated if you do not know them. Use a bluebook. Do not
use scratch paper otherwise you risk losing part credit. Cross out rather than erase any work that
you wish the grader to ignore. Justify what you do. Box or circle your answer.

1. (25 points) A heat engine for which the work- c. (8 points) Deduce the value of the mechan-
ing material is an ideal monatomic gas moves ical work
slowly enough that all parts of it are always in 
mutual equilibrium. It is described by a rect- p dV
12341
angular path on the T (absolute temperature)
S (entropy) plane, as in the gure. While on
done by the gas on the rest of the universe
the path 1 2, the gas in the engine takes heat
over one complete cycle of the engine.
from a bath at high temperature T1 ; on the path
3 4, it returns heat to bath at lower tempera- d. (7 points) In one cycle, what fraction of the
ture T3 . On the paths 2 3 and 4 1, the en- heat withdrawn from the hot reservoir is
tropy has constant values S3 and S1 , respectively. converted to mechanical work done by the
gas on the rest of the universe?

Hint: Keep in mind that the only parameters


given in this problem are T1 , T3 , S1 , and S3 ;
your answers, if nontrivial, must be expressed in
terms of these parameters.

Solution:
(a.)
U of an ideal gas is a function only of T , so the
isothermal segments 1 2 and 3 4 cause no
change in U . Therefore

a. (5 points) Write down the net change 


(U23 + U41 ) = dU = 0
(U23 + U41 ) 12341

in internal energy for the sum of the two


because U is a state function.
paths 2 3 and 4 1.
(b.)
b. (5 points) Compute the net change
 
T dS T dS = (T1 T3 )(S3 S1 ) ,
12341
12341

over one complete cycle of the engine. the area of the rectangle in the gure.
2

(c.) where fE (E) is the probability density of


  the value E. One might imagine fE (E) to
p dV = W take the possible forms:
12341
12341 
 E 
= dU + Q fE (E) E 1/2 exp ?
12341 12341
  E 
kT
=0+ T dS exp ?
12341 kT
 E 
= (T1 T3 )(S3 S1 ) . E 1/2 exp ?
kT
(d.)  E 
E exp ?
  kT
12341
p dV 12341
p dV
= 2
Q2 T dS Which one form is correct, and why?
1
(T1 T3 )(S3 S1 )
= Solution:
T1 (S3 S1 )
(a.)
T3 From the denition of it that is given, this prob-
=1 .
T1 ability density is explicitly normalized:
It is also acceptable to state that this is a Carnot
engine and quote this standard result for its ef- 
ciency. fv (v) dv 1 .

2. (25 points) In a hypothetical one-dimensional Using the standard method for taking the aver-
system, thermal motion of atoms in the y and age, when fv is normalized,
z directions is frozen out, so, eectively, the
atoms are able to move only in the x direc- 
2
tion. In that direction, an atom has velocity v v  = v 2 fv dv

( < v < ). The fraction dF of atoms with   2
velocity between v and v + dv is
v 2 exp mv
2kT dv
=   2 .
 2 exp mv
exp mv dv 2kT dv
dF fv (v) dv =  2kTmv2  ,

exp 2kT dv
This answer is enough to earn full credit. For
where fv (v) is the probability density (RHK: completeness, dening 1/kT and u 12 mv 2 ,
relative probability) of the value v, m is the we can rewrite this quotient as
atomic mass, k is Boltzmanns constant, and T
is the absolute temperature. 2

u1/2 exp (u) du
a. (10 points) Calculate the mean value of the v 2  =  1/2
m


u exp (u) du
square of v, i.e. v 2 . If you wish, you may   
2
leave your answer in the form of a ratio of = ln u1/2 exp (u) du
denite integrals. Do not merely guess the m
answer. 2
= ln (C 1/2 )
b. (15 points) Dene E to be the kinetic energy m
1 2 1
2 mv of an atom. The fraction dF of atoms =
with kinetic energy between E and E +dE is m
kT
dF fE (E) dE , = ,
m
3

where, in the above, C is a constant whose value tain distance from the origin, there is a corre-
is immaterial here. sponding negative charge element located at the
same distance from the origin (but in an orthog-
onal direction). Therefore, by symmetry, the
(b.)
electrostatic potential vanishes at the origin,
dF fE (E) dE
as does the work W required to bring the charge
dF in from innity:
fE =
dE
dF dv  
= W = q (0) () = 0 .
dv dE
dv
fv (v)
dE
fv (v) (b.)
= d 1 2 From the positive part of the charge distribution,
dv ( 2 mv )
the electric eld at the origin is
fv (v)
=
mv   2D

exp kT E
E+ = x dx
. x2
E 1/2 D
 
= x
D 2D

3. (25 points) = x .
2D
A xed line charge of + esu/cm on the x
axis extends from x = D to x = 2D, and a xed Likewise, from the negative part of the charge
line charge of esu/cm on the y axis extends distribution,
from y = D to y = 2D.

E = +y .
2D

The total electric eld is

y x
E = E+ + E = .
2 D 2

The mechanical force F required to keep the test


charge at the origin must oppose qE:
a. (10 points) Find the work required to bring
x y q
a test point charge q from innity to the ori- F= ,
gin. Does your answer depend on the path 2 D 2
you chose? If so, specify the path.
b. (15 points) Calculate the mechanical force where the rst factor is its direction (at 45 to
(magnitude and direction) that is required the x axis), and the second is its magnitude.
to keep the test charge at the origin.
Solution: 4. (25 points) The innite plane z = 0 car-
(a.) ries a uniform surface charge density esu/cm2 .
For every positive charge element that is a cer- There are no other charges in the problem.
4

A parallel to the z = 0 plane,



E da = 4Qencl

((E+ )z (E )z )A = 4A
((E+ )z + (E+ )z )A = 4A
(E+ )z = (E )z = 2
E+ = E = z 2 .

It is acceptable simply to recall that the electric


eld on either side of an innite sheet of charge
has this value, in the absence of other charges.
(c.)
Again because the charge distribution is sym-
metric about the plane z = 0, substituting a
a. (5 points) Find the magnitude and direc- sphere of radius R centered at the origin for the
tion of the electric eld E+ everywhere in Gaussian pillbox used in the solution of part (a.),
the region z > 0.
 top  bot
b. (5 points) Find the magnitude and direc- 1
E da = E da = 4Qencl
tion of the electric eld E everywhere in hemi hemi 2
the region z < 0.  top
1
E da = 4R2
c. (8 points) Consider a spherical surface of hemi 2
radius R centered at the origin. Find the = 2 2 R2 .
electric ux 
E da
(d.)
Because E is constant throughout the semi-
through the top half (top hemisphere) of
innite region z > 0, the ux of E through the
this surface.
top of the hemisphere centered at (0, 0, 2R) is
d. (7 points) Consider a second spherical sur- the same as the ux in part (c.) through the
face, again of radius R, but now centered top of the hemisphere centered at the origin.
at the point (0, 0, 2R), so that it does not Since the hemisphere centered at (0, 0, 2R) con-
enclose any charge. Find the electric ux tains no charge, the ux of E through its bottom
 half must cancel the ux through its top half.
E da Therefore
 bot

through the bottom half (bottom hemi- E da = 2 2 R2 .


hemi
sphere) of this surface.
Solution:
(a.) (b.)
The charge distribution is symmetric about the
plane z = 0, so

E+ = E ,

and both elds are normal to the z = 0 plane.


Using a Gaussian pillbox with at surface area
1

University of California, Berkeley


Physics H7B Spring 1999 (Strovink)
FINAL EXAMINATION

Directions. Do all six problems (weights are indicated). This is a closed-book closed-note exam
except for three 8 12 11 inch sheets containing any information you wish on both sides. You are
free to approach the proctor to ask questions but he or she will not give hints and will be obliged
to write your question and its answer on the board. Use a bluebook. Do not use scratch paper
otherwise you risk losing part credit. Cross out rather than erase any work that you wish the grader
to ignore. Justify what you do. Box or circle your answer.

Problem 1. (35 points)


Four straight stainless steel wires of length x,
cross-sectional area A, and resistivity are
welded together so that they lie along four of the
six edges of a regular tetrahedron, as shown.

Is there a value of for which the voltmeter will


measure Eout (t) = 0? Explain.

Problem 2. (30 points)


Experiments using heavy electrons (muons)
have exploited the fact that, when a pi meson
( + ) decays at rest into a muon (+ ) plus a
The remaining two sides ab and cd are empty. neutrino (), the muon has a unique momen-
Consider a and b to be electrical input terminals, tum with which its spin angular momentum is
and c and d to be electrical output terminals. fully aligned. The reaction is

+ + + .
a. (15 points)
When c is shorted to d, what resistance is mea- A beam of such muons is called a surface muon
sured between a and b? beam (because the pion is stopped near the sur-
face of a solid target where it was produced by
protons from a cyclotron). The muons in a sur-
b. (20 points)
face beam are so well dened that, if they were
Suppose that wires ac and bd are replaced by two
allowed to impinge normally on a book, nearly
identical inductors L, and wires bc and ad are
all would stop in the same page.
replaced by two identical capacitors C. Across
terminals a and b is placed a source of input Given that the muon mass is 3/4 of the
EMF Ein (t) = E0 cos t where E0 and are real pion mass, while the neutrino mass is negligi-
constants. Across terminals c and d, an output bly small, compute the velocity of the muons in
EMF Eout (t) is measured by an ideal voltmeter a surface beam, expressed as a fraction of the
which draws no current. speed of light.
2

Problem 3. (35 points) c. (15 points)


A Panofsky quadrupole magnet consists of four Suppose that a dierent region of space has
long thin copper bars, pointing in the z direction
(out of the page), arranged so that their inside B(x, y) = A0 (xy yx) ,
surfaces form a square box of side 2b. The bars
at y = b carry a uniform current density in the where A0 is a constant (a bullseye magnetic
+z direction while the bars at x = b carry the eld). Show that the current density along z
same current in the z direction. Within the must be nonzero everywhere in the region; give
box enclosed by the bars, the magnetic vector its magnitude and any dependence that it may
potential is have on x and y. (This magnetic eld acts as a
converging lens in both the x and y projections.

A= z(y 2 x2 ) , However, the beam particles are required to pass
2 through the magnets current-carrying element,
which needs to be made as light as possible,
where is a constant.
e.g. of molten lithium.)

Problem 4. (35 points)


Consider a uniform region of space containing an
insulating material with xed dielectric constant
 and magnetic permeability .
a. (3 points)
Write Faradays law in dierential form.
b. (3 points)
For this material there are two dierential ver-
sions of Amperes Law, as modied by Maxwell
a. (10 points) one version uses the free current density Jfree ,
Suppose that a particle of charge +e travels the other uses the total current density. Write
along z at position (x, y) = (0, y). Show that down the version that uses Jfree (which is zero
the particle is deected toward (0,0) with a force for this insulating material).
that is proportional to y. (This means that, in c. (3 points)
the y projection, the Panofsky quadrupole acts Expressing H in terms of B and , and D
as a converging lens. However, it acts as a di- in terms of E and , taking advantage of the
verging lens in the x projection. Fortunately, fact that  and are constant in this material,
the combination of a converging and a diverging rewrite equation (b.) in terms of B and E.
lens of equal strength remains slightly converg-
ing, if the two lenses are separated along their d. (3 points)
axis; this allows a pair of quadrupole magnets Take 1c t

of both sides of equation (c.). On the
to focus a particle beam weakly in both the x left-hand side, interchange the order of dieren-
and y projections. One of the rst experiments tiation, i.e. apply 1c t

to B before taking its curl.
to use this fact discovered the antiproton at the e. (3 points)
Berkeley Bevatron in 1956.) Use equation (a.) to substitute for 1c t

B. Now
b. (10 points) you should have an equation in which E is the
Prove that the current density J within the box only vector eld that appears.
enclosed by the bars (|x| < b and |y| < b) must f. (3 points)
be zero. (This allows the box to be evacuated Use the identity
so that a particle beam can travel unimpeded
within it.) ( E) = ( E) 2 E
3

to eliminate ( E) from the left-hand side Problem 6. (35 points)


of equation (e.). Consider a hollow cubical box containing parti-
cles which make elastic collisions with its walls.
g. (6 points)
Give an argument, based on the absence of free a. (10 points)
charges in this insulator, and the strict propor- Suppose that the particles are molecules of a
tionality of E to D, which allows you to ignore perfect gas. Using standard arguments about
one of the terms on the left-hand side of equation the number of molecules bouncing o an area of
(f.). wall per unit time, and the momentum per col-
lision that is imparted to the wall, prove that
h. (6 points) the pressure p (N/m2 ) and the kinetic energy
Your result should be a wave equation for E. density u (J/m3 ) of the gas are related by
Show that any function of (kx t), where k
and are constants, solves this equation. 2
p= u.
i. (5 points) 3
Calculate /k, the phase velocity of the solution
(h.). Evaluate it in terms of c, , and . b. (15 points)
Suppose that the box is lled not with molecules,
but with electromagnetic radiation, which is
Problem 5. (30 points) quantized into photons. These photons can be
An (insulating) hollow spherical shell of dielec- considered to be massless particles which, like
tric with inner (outer) radius a (b) has frozen- the perfect gas molecules, do not interact with
in polarization each other and bounce elastically o the walls.
Deduce the relationship between the pressure p
q0
P = r , and the energy density u of the electromagnetic
2(a + b)r radiation.
where r is the radius measured from its center b. (10 points)
and q0 is a constant. The dielectric encloses a At suciently high temperature, the electromag-
conducting sphere of radius a which holds total netic radiation pressure inside the box would be
free charge q0 (see the gure). sucient to balance the ambient pressure (106
dynes/cm2 ) of the earths atmosphere at sea
level. If this were to occur, what would be the
root mean square magnetic eld (in gauss) inside
the cavity?

At what values of r does E vanish? [Your an-


swer may include particular values of r (includ-
ing those which are not nite), and/or ranges of
r.] Note that this (nonlinear) dielectrics elec-
tric susceptibility is not dened or supplied here,
and it should not appear in your answer.
University of California, Berkeley
Physics H7C, Fall 1999 (Strovink)

General Information (27 Aug 99)

Web site for this class: First link on http://d0lbln.lbl.gov .

Instructors: Prof. Mark Strovink, 437 LeConte; (LBL) 486-7087; (home, before 10) 486-8079; (UC) 642-
9685. Email: strovink@lbl.gov . Web: http://d0lbln.lbl.gov . Office hours: M 3:15-
4:15, 5:30-6:30. Mr. Derek Kimball, (UC research lab) 221 Birge, 643-1829; (home, before 11) 548-3115.
Email: dfk@uclink4.berkeley.edu . Office hours (to be held in 211 LeConte): Tu 2-4. You may
also get help in the 7C Course Center, 262 LeConte.

Lectures: Tu-Th 11:10-12:30, 3 LeConte. Lecture attendance is essential, since not all of the course content
can be found in the course text or handouts. On one or two occasions it is possible that the lecture normally
held on Thursday will be given instead on Wednesday, at 4:30-5:50 PM, in 343 LeConte.

Labs: Begin in the third week, in 278 LeConte. As soon as possible, please enroll in any one of the 7C lab
sections that fits your schedule.

Discussion Sections: Begin in the second week. Section 1: W 4-5, 343 LeConte; Section 2: W 5:30-6:30,
also in 343 LeConte. You are welcome at either or both sections. You are especially encouraged to attend
discussion section regularly. There you will learn techniques of problem solving, with particular application to
the assigned exercises.

Texts (required): Fowles, Introduction to Modern Optics, Second Edition (Dover paperback, 1989). Rohlf,
Modern Physics from to Z0 (Wiley, 1994). Supplementary text (recommended): Hecht, Schaums
Outline of Theory and Problems of Optics (McGraw-Hill paperback, 1975). (Dont confuse this with
Hechts hardbound books on optics.)

Problem Sets: Twelve problem sets are assigned and graded. Solutions will be available. Problem sets are
due on Thursday at 5 PM on weeks in which there is no exam, beginning in week 2. Deposit problem sets in
the box labeled "H7C" outside 211 LeConte. You are encouraged to attempt all the problems. Students who
do not do them find it almost impossible to learn the material and to succeed on the examinations. Discuss
these problems with your classmates as well as with the teaching staff; however, when the time comes to write
up your solutions, work independently. Credit for collective writeups, which are easy to identify, will be
divided among the collectivists. Late papers will not be graded. Your lowest problem set score will be
dropped, in lieu of due date extensions for any reason.

Syllabus: H7C has one mandatory syllabus card. It will be collected when the first midterm examination is
handed back in lecture. This card pays for the 7C laboratory experiment descriptions and instructions. Copies
of solutions to each problem set will also be available for separate purchase at Copy Central.

Exams: There will be two 80-minute midterm examinations and one 3-hour final examination. Before
confirming your enrollment in this class, please check that its final Exam Group 9 does not conflict with the
Exam Group for any other class in which you are enrolled. Please verify that you will be available for the
midterm examinations (Th 7 Oct and Th 11 Nov, 11:10-12:30), and for the final examination, F 10 Dec, 5-8
PM. Except for unforeseeable emergencies, it will not be possible for the midterm or final exams to be
rescheduled. Passing H7C requires passing the final exam.

Grading: 30% midterms; 25% problem sets; 40% final exam; 5% lab. Grading is not "curved" -- it does not
depend on your performance relative to that of your H7C classmates. Rather it is based on comparing your
work to that of a generation of earlier lower division Berkeley physics students, with due allowance for
educational trends.
Physics H7C Course Outline Fall 1999 (Strovink)

Week Week Reading Topic Problem Due 5 PM 7C lab


No. of... chapter (Pu=Purcell, Fo=Fowles, Ro=Rohlf) Set No. on...

1 23-Aug Ro 4, Pu A, Review of special relativity; NONE


Fo App. I relativistic transformation of EM fields

2 30-Aug Pu 9 Review of Maxwell's equations and EM waves NONE


Pu 10,11 in vacuum and in material; boundary conditions 1 2-Sep

3 6-Sep LABOR DAY


Pu B Radiation by an accelerated charge reflect/
Fo 2 Polarization 2 9-Sep refract

4 13-Sep Fo 2 Plane reflection/refraction geom


Fo 3 Interference 3 16-Sep optics

5 20-Sep Fo 3 Coherence michelson


Fo 4 Multiple beams 4 23-Sep interferom

6 27-Sep Fo 4 Multiple beams diffract/


Fo 5 Diffraction 5 30-Sep interfer

7 4-Oct Fo 5 Diffraction NONE


Fo 6 Optics of solids
7-Oct MIDTERM 1 (covers PS 1-5)

8 11-Oct Ro 2 Optics of solids, Maxwell-Boltzmann distribution polarI-


Ro 3 Planck's constant 6 14-Oct zation

9 18-Oct Ro 3 Planck's constant


Ro 5 Wave properties of matter 7 21-Oct NONE

10 25-Oct Ro 5 Uncertainty principle photo-


Ro 6 Probing the structure of matter 8 28-Oct electric

11 1-Nov Ro 7 Schroedinger equation NONE


Ro 8 Hydrogen atom 9 4-Nov

12 8-Nov Ro 9 Periodic table atomic


11-Nov MIDTERM 2 (covers PS 1-9) spectra

13 15-Nov Ro 12 Quantum statistics radio


Ro 17 Quarks and leptons 10 18-Nov half-life

14 22-Nov Ro 18 Unification of the forces 11 24-Nov NONE


25-Nov THANKSGIVING

15 29-Nov Ro 19 Cosmology makeups


LAST LECTURE (review) 12 2-Dec

16 6-Dec
8-Dec Final exams begin
10-Dec 5-8 PM H7C FINAL EXAM (Group 9) (covers PS 1-12)

updated 12/31/99
http://d0lbln.lbl.gov/h7cf99/h7cf99-texts.txt

NOTES ON H7C TEXTS:

RELATIVITY TEXTS (to supplement distributed handwritten notes):

Taylor and Wheeler, Spacetime Physics:


Good for spacetime, Lorentz transformations as spacetime
rotations, the boost parameter, and rocket problems.

Rohlf, Modern Physics from alpha to Z0 (H7C required), chapter 4:


Fair for relativistic kinematics -- a Compton scattering
derivation is given, though it is not the most elegant.

Purcell, Electricity and Magnetism (H7B text), appendix A and chapter 9:


Good for relativistic transformation of electric and magnetic
fields.

OPTICS TEXTS:

Fowles, Introduction to Modern Optics (H7C required): Terse.

Hecht, Schaum's Outline of Theory and Problems of Optics (H7C


recommended): Students have found this to be a useful summary
of formulae and a place to find simple practice problems.

OPTICS TEXTS THAT PREVIOUSLY HAVE BEEN USED IN H7C:

Pedrotti & Pedrotti, Introduction to Optics, 2nd ed. (Prentice-Hall)

Hecht, Optics, 3rd ed. (Addison Wesley)

MODERN PHYSICS TEXT:

Rohlf, Modern Physics from alpha to Z0 (H7C required)

MODERN PHYSICS TEXTS THAT PREVIOUSLY HAVE BEEN USED IN H7C:


These are listed in descending (subjective) order of
usefulness:

Blatt, Modern Physics (McGraw-Hill)

Beiser, Concepts of Modern Physics, 5th ed. (McGraw-Hill)

Ohanian, Modern Physics, 2nd ed. (Prentice-Hall)

Eisberg & Resnick, Quantum Physics of Atoms, Molecules, Solids,


Nuclei, and Particles, 2nd ed. (Wiley)

Serway, Modern Physics, 2nd ed. (Saunders)

Tipler, Physics for Scientists and Engineers, Vol II, 3rd ed.
(Worth)

http://d0lbln.lbl.gov/h7cf99/h7cf99-texts.txt [1/5/2000 9:52:25 AM]


Errata
Fowles, Introduction to Modern Optics, Second Edition
M. Strovink
This book was chosen for the optics portion of H7C because the selection and relative coverage of
topics is excellent. As now reprinted by Dover, it is also bargain priced. For the most part Fowles tells
you what you want to know, with brevity and insight. Perhaps more important for a general course like
H7C, Fowles usually does not tell you what you do not want to know. This is in contrast to the pricey
and encyclopdic Hecht, in which fundamental concept and arcane detail are mixed together without any
obvious dierentiation.
The main problem with the book is that Fowles, while a good physicist, seems as an author not to be
a suciently concerned with correctness. The text is full of annoying lapses in accuracy of the drawings.
Despite the existence of two editions, the book seems never to have been adequately proof-read or revised.
Therefore, to benet from this otherwise good book, you should correct the many errors in your
copy, using this guide.
Page 23, Eq. 2.16 should read
nE 0
H=
Z0
Page 25, Eq. 2.23 should read
1 n 0
I= E0 H 0 = |E0 |2
2 2Z0
Page 41, Eqs. 2.49, 2.50, and 2.51: these equations assume that is the same in both media.
Page 43 (an omission, not an error). To Eqs. 2.54 and 2.55 should be added:

2 cos
ts =
cos + n cos

2 cos
tp =
n cos + cos
Page 45. The details in Fig. 2.12 are reliable only to a factor of 2, due to bad registration of the curves
(printed in color in the hardcover version) with respect to the axes.
Page 56, Problem 2.23: Brewster window should be Brewster interface, i.e. one interface between n = 1
and n = n.
Page 64, last sentence, beginning In this case the central fringe..., and continuing onto page 65, should be
ignored. This sentence would be correct only if plate A were not (even half) silvered.
Page 75, Fig. 3.13: For your own increased comprehension, indicate the following on the gure: s is the
distance between the sources Sa and Sb ; l is the distance between the receivers P1 and P2 ; r is the distance
between the average of Sa and Sb , and the average of P1 and P2 .
Page 76, sentence following Eq. 3.39 should read: ... small in comparison with 0 .
Page 76, sentence preceding Eq. 3.40 should read: We then have b a = (r2b r2a )/c, or approximately
Page 76, Eq. 3.40 should read:
sl
b a
cr
Page 77, Eq. 3.41 should read:
slt
(b a ) = =
cr
Page 77, Eq. 3.42 should read:
r
lt =
2s
Page 77, Eq. 3.43 should read:

lt =
2s

1
Page 81, Eq. 3.46 should read: 
1
G(k) = W (x)eikx dx
2

The factor 2 should appear to the power 1 rather than 12 because there is no corresponding factor of

1/ 2 in Eq. 3.45. Conversely, in Eq. 3.28 on page 71 Fowles instead has adopted the neater convention of
inserting a factor 1/ 2 in the denitions of each of the Fourier transform pairs.
Page 97, Fig. 4.7: For consistency with Eqs. 4.23, 4.27, and 4.32, in the [central] region with index n1 shown
in the gure, k1 and k1 should be replaced by k and k  .
Page 98, last sentence: add at the end of this sentence , with R + nT T /n0 = 1.
Page 137, Fig. 5.28; page 139, Fig. 5.29; and page 143, Fig. 5.32: The patterns should be centered on y = 0
or = 0 (again a regisration problem).
Page 150, Problem 5.16: The last parenthesis should be (Assume the trailing edge of the moon to be
eectively straight.)
Page 220, 4th sentence should read: For left circularly polarized light, the direction of spin of the photon
is parallel to the direction of propagation, whereas for right circularly polarized light, it is antiparallel to
the direction of propagation. [This is a convention that depends on the charge of the particle (electron or
positron) to which the photon couples; our (more usual) convention takes that charge to be positive.]
Page 297, Eq. 10.11 should read:

1 1 1 (n 1)t 
= (n 1) +
f r1 r2 nr1 r2

Page 297, Eq. 10.12 should read:


1 n
d2 = f t
r1
assuming that d1 and d2 are positive distances as drawn in Fig. 10.3.
Page 298, Fig. 10.3. The rays are so badly drawn as to be meaningless. The top ray should emanate from
the object in a direction parallel to the axis, and it should be bent only at principal plane H  . The bottom
ray should emanate from the image in a direction parallel to the axis, and it should be bent only at principal
plane H.
Page 307, 1st complete sentence should read: The interference pattern can be made to shift by one fringe
i.e. from a bright fringe to the adjacent dark fringe, or vice versa) by displacing either of the two mirrors
M1 or M2 a distance of 14 wavelength.

2
1

University of California, Berkeley


Physics H7C Fall 1999 (Strovink)
PROBLEM SET 1

1. Two supernov are observed on earth in the with respect to S. Find L for this case. (Hint.
direction of the north star, separated by 10 years. Rotate to a system in which V is along the x
From the theory of supernov these are known to axis, transform using your answer for part (a.),
have identical (standard candle) light output, and then rotate back. Check that your result is
yet the rst is observed to have four times the symmetric under interchange of x and y, as is V,
light intensity of the second because it is closer. and that it reduces to the unit matrix as 0.)
(a.)
An astronomer theorizes that the two stars were 3. Work out the Lorentz transformation ma-
at rest with respect to the earth, and that the trix L for the general case in which of frame
rst supernova triggered the second. What is S  is directed along an arbitrary unit vector
the maximum distance between the earth and n = (nx , ny , nz ) as seen in frame S, e.g.
the rst supernova under this hypothesis?
(b.) r = Lr, L = ?
A physicist theorizes that the two stars were
traveling with the same (unspecied) velocity
away from the earth, and that, in their common
4. In a straight channel oriented along the z
rest frame, the two supernov occurred at the
axis there are two opposing beams:
same proper time. What is the minimum dis-
tance between the earth and the rst supernova a beam of positrons (charge +e) with veloc-
under this hypothesis? ity +zc.
a beam of electrons (charge e) with veloc-
2. Inertial reference frames S  and S coincide
ity zc.
at t = t = 0. You may ignore the z dimension,
so that a point in spacetime is determined by Each beam is conned to a small cylindrical vol-
only three quantities r (ct, x, y). The Lorentz ume of cross sectional area A centered on the
transformation between S and S  is given by z axis. Within that volume, there is a uni-
 form number density = n positrons/m3 and n
ct ct electrons/m3 .
x = L x , (a.)
y y In terms of n, A, e, and , calculate the total
current I in the channel due to the sum of both
where L is a 33 matrix. beams (note I = 0).
(a.) (b.)
Assume for this part that S  moves with velocity Use Amperes Law to calculate the azimuthal
magnetic eld B outside the channel a distance
V = cx r from the z axis.

with respect to S. Using your knowledge of Consider now a Lorentz frame S  travelling in
Lorentz transformations (no derivation neces- the z direction with velocity c relative to the
sary), write L for this case. lab frame described above. (This is the same
(b.) as above.)
Assume for this part that S  moves with velocity (c.)
As seen in S  , calculate the number density
x + y n+ of positrons within the cylindrical volume.
V = c
2 (You may use elementary arguments involving
2

space contraction, or you may use the fact that the (momentary) inertial frame of reference rel-
(c, j) is a 4-vector, where is the charge den- ative to which the acceleration is g.
sity (Coul/m3 ) and j is the current density (b.)
(amps/m2 ).) How much velocity does the spaceship have after
(d.) a given time? This is the moment to object to the
As seen in S  , calculate the number density n question and to rephrase it. Velocity c is not
of electrons within the cylindrical volume. the simple quantity to analyze. The simple quan-
(e.) tity is the boost parameter . This parameter is
Calculate the radial electric eld Er seen in S  . simple because it is additive in this sense: Let the
Do this both boost parameter of the spaceship with respect to
the imaginary instantaneously comoving inertial
by using the results of (c.) and (d.) plus frame change from 0 to d in an astronaut time
Gausss law, and d . Then the boost parameter of the spaceship
by using the results of (b.) plus the rules for with respect to the laboratory frame changes in
relativistic E and B eld transformations. the same astronaut time from its initial value
to the subsequent value + d. Now relate d to
5. (Taylor and Wheeler problem 51) the acceleration g in the instantaneously comov-
The clock paradox, version 3. ing inertial frame. In this frame g d = c d =
Can one go to a point 7000 light years away c d(tanh ) = c tanh (d) c d so that
and return without aging more than 40 years?
Yes is the conclusion reached by an engineer c d = g d
on the sta of a large aviation rm in a recent
report. In his analysis the traveler experiences a Each lapse of time d on the astronauts watch is
constant 1-g acceleration (or deceleration, de- accompanied by an additional increase d = gc d
pending on the stage reached in her journey). in the boost parameter of the spaceship. In
Assuming this limitation, is the engineer right in the laboratory frame the total boost parame-
his conclusion? (For simplicity, limit attention ter of the spaceship is simply the sum of these
to the rst phase of the motion, during which additional increases in the boost parameter. As-
the astronaut accelerates for 10 years then sume that the spaceship starts from rest. Then
double the distance covered in that time to nd its boost parameter will increase linearly with
how far it is to the most remote point reached astronaut time according to the equation
in the course of the journey.)
(a.) c = g
The acceleration is not g = 9.8 meters per sec-
ond per second relative to the laboratory frame. This expression gives the boost parameter of
If it were, how many times faster than light the spaceship in the laboratory frame at any
would the spaceship be moving at the end of time in the astronauts frame.
ten years (1 year = 31.6 106 seconds)? If the (c.)
acceleration is not specied with respect to the What laboratory distance x does the spaceship
laboratory, then with respect to what is it spec- cover in a given astronaut time ? At any
ied? Discussion: Look at the bathroom scales instant the velocity of the spaceship in the lab-
on which one is standing! The rocket jet is al- oratory frame is related to its boost parameter
ways turned up to the point where these scales by the equation dx/dt = c tanh so that the
read ones correct weight. Under these condi- distance dx covered in laboratory time dt is
tions one is being accelerated at 9.8 meters per
second per second with respect to a spaceship dx = c tanh dt
that (1) instantaneously happens to be riding
alongside with identical velocity, but (2) is not Remember that the time between ticks of the
being accelerated, and, therefore (3) provides astronauts watch d appear to have the larger
3

value dt in the laboratory frame (time dilation) 7. (Taylor and Wheeler problem 75)
given by the expression Doppler equations.
A photon moves in the xy laboratory plane in
dt = cosh d a direction that makes an angle with the x
axis, so that its components of momentum are
px = p cos , py = p sin , and pz = 0.
Hence the laboratory distance dx covered in as- (a.)
tronaut time d is Use the Lorentz transformation equations for
the momentum-energy 4-vector and the relation
dx = c tanh cosh d = c sinh d E 2 /c2 p2 = 0 for a photon to show that, in the
rocket frame S  (moving with velocity r c along
Use the expression c = g from part b to obtain the x, x direction, and coinciding with the lab-
oratory frame at t = t = 0), the photon has an
 g  energy E  given by the equation
dx = c sinh d
c
E  = E cosh r (1 r cos )
Sum (integrate) all these small displacements dx
and moves in a direction that makes an angle 
from zero astronaut time to a nal astronaut
with the x axis given by the equation
time to nd
cos r
c2   g   cos  =
1 r cos
x= cosh 1
g c

(b.)
This expression gives the laboratory distance x
Derive the inverse equations for E and cos as
covered by the spaceship at any time in the
functions of E  , cos  , and r .
astronauts frame.
(c.)
(d.)
If the frequency of light in the laboratory is ,
Plugging in the appropriate numerical values,
what is the frequency  of light in the rocket
determine whether the engineer is correct in his
frame? This dierence in frequency due to rel-
conclusion reported at the beginning of this ex-
ative motion is called the relativistic Doppler
ercise.
shift. Do these equations enable one to tell in
what frame the source of the photons is at rest?
6. Electrons (mc2 = 0.5 106 eV) are acceler-
ated over a distance of 3.2 km from rest to a
total energy of 5 1010 eV at SLAC (Stanford).
(a.)
To what boost are the electrons ultimately
brought?
(b.)
Assuming that the electrons are subjected to a
uniform acceleration as observed in their comov-
ing inertial frame, how many gs of acceleration
do they feel?
(c.)
As observed in the lab, for what time interval is
each electron in ight? What is the correspond-
ing proper time interval? Evaluate the ratio of
the two intervals (a sort of average factor).
4

8. Consider the following situation. A star is


known, by means of external data, to be located
instantaneously a distance D from an observer
on earth. The external data do not tell us the
rate of change of D with time.
In her measurements, the observer corrects for
aberration caused by the local velocity of the
earths surface, due both to its daily rotation
and its yearly orbit. Therefore we do not need
to take into account these boring local phenom-
ena in what follows.
After making these corrections, the observer sees
that the star is undergoing angular motion d/dt
across the sky, such that D d/dt = c, where c
is the speed of light.
Finally, the observer measures the wavelength
spectrum of light from this star, and nds its
features not to be redshifted or blueshifted at all
they are exactly where they would be if the
star were perfectly at rest with respect to the
observer.
Is it possible that this situation is physically rea-
sonable? If so, what might be the true motion
of the star with respect to the observer? If not,
why not?
Physics H7C Fall 1999 Solutions to Problem Set 1 Derek Kimball
Lets get something straight here... e is real, 10 is just the number of ngers we From Eq. (2), we see that if the distance between SN2 and SN1 is x, the
have. distance between SN1 and earth is also x. Then the elapsed time tearth between
detection of the two supernovae on earth, taking into account the propagation time
- Prof. Nima Arkani-Hamed, UC Berkeley
of the light to the earth, is given by:
If you have any questions, suggestions or corrections to the solutions, dont hesitate    
to e-mail me at dfk@uclink4.berkeley.edu! 2x x
ctearth = ct + c c = ct + x. (4)
If you liked problem 1 and youre interested in astrophysics, general relativity, and c c
cosmology, you should check out a paper by Saul Perlmutter, Michael S. Turner,
and Martin White (Physical Review Letters, July 26, 1999, Volume 83, Issue 4, From Eq. (3), we know that ct x, so we nd that:
pp. 670-673). This article and references therein describe an ongoing study of ctearth
type Ia supernovae which have standard candle light output and have enabled x , (5)
2
these scientists to measure large-scale cosmological parameters. One of the most
interesting results is that their data is consistent with a universe that is expanding or that xmax = 5 light years.
at an accelerating rate! Saul Perlmutters group is here at Berkeley and works at
LBL. (b)
A physicist theorizes that the two supernovae were traveling away from the earth
Problem 1 at some velocity and occurred at the same proper time. In this case the two events
have a spacelike or lightlike separation:
First, lets consider the implications of the dierence in light intensity of the two
supernovae (SN1 and SN2). These particular supernovae are known to have iden- c2 t2 x2 0. (6)
tical standard candle light output, i.e. the total light power P emitted is the
same for SN1 and SN2. A small solid angle d of the total light is detected on Thus, in the earth frame there is an observed time dierence tobs between SN1
earth, so the intensity of light I detected is given by: and SN2, which from Eq. (6) must satisfy:

d P ctobs xobs , (7)


I= , (1)
4R2
where xobs is the distance between SN1 and SN2 as observed in the earth frame.
where R is the distance from a supernova to the earth at the time the light is As in part (a) we include the light propagation time, and nd that:
emitted. Therefore the ratio of light intensities tells us the ratio of distances:
ctearth = ctobs + xobs 2xobs . (8)
I1 R22
= 2 = 4. (2)
I2 R1 So in this case we nd that xobs,min = 5 light years.

(a) Problem 2

An astronomer theorizes that SN1 causes SN2, and that they are both at rest with (a)
respect to the earth. Since the two events SN1 = (ct1 , x1 ) and SN2 = (ct2 , x2 )
This is just the traditional Lorentz matrix, only in 3D, so it is similar to the
are causally related, there must be a timelike or lightlike separation between the
expression (1.12) in Prof. Strovinks notes on relativity,
events:

2 2 2 ct 0 ct
c t x 0, (3) x = 0 x . (9)

where t = t t and x = x x . y 0 0 1 y
2 1 2 1

September 2, 1999
Physics H7C Fall 1999 Solutions to Problem Set 1 Derek Kimball
(b)
The general idea is to rotate to a system where we know the correct transform
(from part (a)), and then rotate back. So we begin with:

r  = L r. (10)

 is along
Then we rotate the coordinate system with a rotation matrix R so that
x:

Rr  = RL r = RLR1 (Rr). (11)

In this frame we know the Lorentz transform from part (a), so we nd that:

RLR1 = . (12)

In other words,
Figure 1: Relationship between n and the angles and employed in problem (3)
1
L=R R. (13) in the rst approach.

The math can be made a little easier in these cases because rotations are described
by orthogonal matrices which satisfy R1 = RT where RT is the transpose of R. As you can see by inspection, this matrix is symmetric under interchange of x and
y and reduces to the identity matrix as 0.
Now for the actual math. From (a) and Eq. (13) we nd:

1 0 0 0 1 0 0 Problem 3
L = 0 cos sin 0 0 cos sin ,
0 sin cos 0 0 1 0 sin cos Here are two common approaches to this problem. The rst method involves
(14) matrix multiplication in a manner similar to that employed in problem 2. The
second involves determining a general vector formula for the Lorentz transform.
where we use straightforward 3D extensions of the usual rotation matrices. Mul-
tiplying these matrices gives us: Approach 1
First, well determine the rotation matrix R which will take us into the frame
cos sin  is along x. For me, its easier to think of this in terms of the angles and
where
L = cos 2 2
cos + sin cossin + cossin . (15)
as dened in Fig. 1.
sin cossin + cossin cos2 + sin2
From Fig. 1, we notice that there is a natural correspondence between (nx , ny , nz )
We are given that: and , given by
x + y
V = c , (16) nx coscos
2 ny = cossin . (18)
so = /4. Then L is given by: n z sin
Later, these relations will be used to express L in terms of nx , ny and nz .

2 2
1+ 1
1) R is given by the multiplication of two rotation matrices A and B,
L= 2 (
(17)
2 2
1 1+

2 2 ( 1) 2 R = B A, (19)

September 2, 1999
Physics H7C Fall 1999 Solutions to Problem Set 1 Derek Kimball
where A rotates the axes about z by example, n is along x, from Eq. (1.12) in Strovinks notes is applicable and we
nd:
1 0 0 0 
0 cos sin 0 ct (ct x)
A= 0 sin
, (20) x x + ( 1)x ct
cos 0  = . (25)
y y
0 0 0 1
z z
and B rotates the axes about a new y  (the y-axis after rotation by A) by so
Let r (x, y, z), then by analogy with Eq. (25) we nd that:
that x is along n:
ct = (ct r n) (26)
1 0 0 0
0 cos 0 sin and
B= 0
. (21)
0 1 0
0 sin 0 cos r = r + n(( 1)r n ct). (27)

We can then express these equations in terms of nx , ny and nz :


Applying Eq. (13), we can now solve for L: 
ct ct (nx x + ny y + nz z)
L = A1 B 1 B A = AT B T B A, x  x + nx ( 1)(nx x + ny y + nz z) ct

(22)  = . (28)
y y + ny ( 1)(nx x + ny y + nz z) ct
where is given by Eq. (1.12) in Prof. Strovinks notes on relativity. z z + nz ( 1)(nx x + ny y + nz z) ct

The result of this rather tedious matrix multiplication, after some simplication These expressions can be re-written in matrix form, yielding L from Eq. (10) to
using basic trigonometric identities, is given by L = be:

coscos sincos sin
coscos 1 + ( 1)cos2 cos2 ( 1)sincoscos2 ( 1)cossincos

sincos ( 1)sincoscos2 1 + ( 1)sin2 cos2 ( 1)sinsincos .
sin ( 1)cossincos ( 1)sinsincos 1 + ( 1)sin2
(23)

nx ny nz
If we then use the relations given in Eq. (18) to re-express Eq. (23) in terms of nx 1 + ( 1)n2x ( 1)nx ny ( 1)nx nz
nx , ny and nz we nd that: L= ny ( 1)ny nx 1 + ( 1)n2y ( 1)ny nz ,
(29)
nz ( 1)nz nx ( 1)nz ny 1 + ( 1)n2z
nx ny nz
nx 1 + ( 1)n2x ( 1)nx ny ( 1)nx nz which you will notice is the same result as the one obtained in approach 1.
L= ny ( 1)ny nx 1 + ( 1)n2y ( 1)ny nz .
(24)
nz ( 1)nz nx ( 1)nz ny 1 + ( 1)n2z Problem 4

(a) The current I is the charge per second traveling through the channel, given
Approach 2
by:
In this approach, we work out a vector formula for the Lorentz transformation
 If, for
using the fact that length contraction occurs only in the direction of . I = nA(+e)(+c) + nA(e)(c) = 2nAec. (30)

September 2, 1999
Physics H7C Fall 1999 Solutions to Problem Set 1 Derek Kimball
(b) as above.
Amperes law (in SI units, feel free to use whatever units you like of course) is:
(d)

B d = 0 Ienclosed . (31) Here well just stick to the four-vector method. The relationship between the
charge density of electrons seen in S  and S is given by:
      
So in our case, assuming an innitely long channel and using I from Eq. (30): c c
= , (39)
jz jz
B 2r = 0 2nAec, (32)
from which we nd the charge density:
therefore
0 nAec c = c jz . (40)
B = . (33)
r The electron charge density in S satises c = n (e)c and the current density in
S is given by jz = n (e)()c. Plugging these into Eq. (40) allows us to solve
(c) for n :

Lets solve this both suggested ways... rst using length contraction. The density n = 1 + 2 n . (41)
of positrons n+ is given by:
N+ (e)
n+ = , (34)
Ad First we solve the problem using Gausss law:
where d is a unit length of the channel in the lab frame S and N+ is the total  

number of positrons contained in this volume. This new frame S  is the rest frame Er dA = dV. (42)

of the positrons, so d = d (sort of length un-contraction). Therefore the observed "0

positron density in S  is given by: Choosing a cylindrical Gaussian surface centered on the z-axis with radius r and
N+ n+ length d, we nd:
n+ = = . (35)
Ad Er 2rd = Ad + +  /"0 . (43)

Using the relations for the charge density of positrons and electrons in S  from
We can arrive at the same conclusion using the fact that c, j is a four-vector.
Eqs. (38) and (41), we nd that:
Considering only the z-direction, we have the relation:
ne
       + +  = 1 2 (1 + 2 ) = 2 2 ne. (44)
c c
= , (36)
jz jz
Combining these results, we nd for the radial electric eld Er seen in S  :
where the charge density in the lab frame S satises c = n+ ec and the current
2 neA
density in S is given by jz = n+ ec. Thus from Eq. (36) we nd that: Er = r . (45)
"0 r
c = n+ ec = n+ ec 2 n+ ec . (37)
We can also solve this problem using the relativistic eld transformations for E
Consequently, and B given in Prof. Strovinks notes (Eq. (1.33)), in particular:
n+

n+ = (38) E
= E + c B  . (46)

September 2, 1999
Physics H7C Fall 1999 Solutions to Problem Set 1 Derek Kimball
Employing B from Eq. (33) and noting that Er = 0 in S, we nd that: Finally arriving at the solution:

2 neA c2
Er = cB r = r . (47) xf = (cosh(gf /c) 1). (53)
"0 r g

Where we use the fact that "0 0 = 1/c2 . This, of course, agrees with our result
from Eq. (45) using Gausss law. (d)
If we plug in the numbers we nd that xf 1020 meters or 104 light years. This
Problem 5 is just the rst leg of the journey, so the furthest distance the astronaut can reach
is twice this, or 20,000 light years away! So the engineer was right...
(a)
Note the interesting fact that the number of seconds in a year is approximately Problem 6
107 , a useful fact at cocktail parties and for back-of-the-envelope calculations.
If you naively multiply the acceleration by the time, you nd: (a)
The relativistic expression for energy E of particles with non-zero mass is given
v = gt 10c. (48) by Eq. (1.23) in Strovinks notes:
So, if youre a believer in relativity, this cant be right...
E = mc2 , (54)
(b) where m is the rest mass of the particles. Since = cosh(), the boost is given
This part is basically worked out in the text of the problem, so theres nothing to by:
say...  
E
= cosh1 (55)
(c) mc2

We start with Knowing from the problem that mc2 = 0.5 106 eV and Ef inal = 5 1010 eV, we

g can solve for :
dx = c sinh()d = c sinh d, (49)
c
= 12.2 (56)
where we use the expression

c = g. (50) (b)
Next we integrate the small displacements from 0 f where f is the nal We can use the result obtained in problem 5, namely Eq. (53), replacing g with
astronaut time. some constant acceleration a. We also replace cosh(gf /c) with f , which from
 xf  f
g part (a) we nd is f 105 . This gives us:
dx = c sinh d (51)
0 0 c c2
xf f (57)
a
We can make a straightforward change of variable = g /c:
 Solving for a and making the appropriate substitutions yields:
c2 gf /c
xf = sinh()d. (52)
g 0 a 3 1017 g (58)

September 2, 1999
Physics H7C Fall 1999 Solutions to Problem Set 1 Derek Kimball
(c) (b)
We can use the relation between proper time d and time in the laboratory frame Now we use the inverse Lorentz transform:
dt from problem 5:
E/c cosh() sinh() 0 E  /c
dt = cosh()d. (59) (E/c)cos = sinh() cosh() 0 (E  /c)cos . (66)
(E/c)sin 0 0 1 (E  /c)sin
If we apply the relation c = a , then integrating this expression yields:
If we perform calculations similar to those in part (a), we nd:
c c
tlab = sinh(f ) = f f 105 s (60)
a a E = E  (cosh() + sinh()cos ) = E  cosh()(1 + cos ). (67)
where tlab is the time interval in the lab frame.
and
From c = a we can quickly calculate the proper time interval:
E cos +
c cos = (cosh()cos + sinh()) = . (68)
= f 109 s (61) E 1 + cos
a

So, taking the ratio gives an average factor of 104 . (c)


Here, we can employ the relationship between energy and frequency of a photon,
Problem 7 namely:

(a) E = h, (69)
We know that photons satisfy E 2 p2 c2 = 0. Then, if we substitute the appropriate where h is Plancks constant. Thus from Eq. (63) we solve for  , nding the
values from the problem into the equation describing the Lorentz transformation relativistic Doppler shift formula:
for the four-momentum (ignoring the z-direction), we nd:
 = cosh()(1 cos). (70)
E  /c cosh() sinh() 0 E/c
(E  /c)cos = sinh() cosh() 0 (E/c)cos . (62)
(E  /c)sin 0 0 1 (E/c)sin If an observer knows only the frequency as observed in a given frame, one cannot
gure out what the frequency of light was in the rest frame of the source. Thus
Solving for E  gives us: a measurement of light frequency in a particular frame does not directly tell us
about the velocity of the source. However, if we have prior knowledge of what the
E  = E(cosh() sinh()cos) = Ecosh()(1 cos). (63) frequency of light at rest should be (for example, well-known atomic transitions in
hydrogen or helium), we can tell something about the motion of the source.
If we then nd the equation for px we can solve for cos :

E Problem 8
cos = (cosh()cos sinh()). (64)
E
This situation is physically reasonable, here is one example of how it could hap-
Substituting in the expression for E  from Eq. (63) yields: pen...
Could the star be moving toward or away from us, even though the spectral features
cos
cos = . (65) are not redshifted or blueshifted? The answer is yes, as can be seen from the
1 cos relativistic Doppler shift given in Eq. (70). We demand that  = , and then

September 2, 1999
Physics H7C Fall 1999 Solutions to Problem Set 1 Derek Kimball
 and the
nd a condition on the velocity of the source c and the angle between
direction to earth :
1
cos = . (71)

So long as this condition is satised, there is no restriction on the motion of the
source (save that the source, if massive, cannot move at the speed of light!).
Next we consider if some par-
ticular type of motion could
increase the apparent veloc-
ity of the star across the
sky. Once again the answer
is yes. Consider the situation
depicted in the gure to the
right. Of course, the draw-
ing is greatly exaggerated in
dimensions since z
D and
t2 t1 is dierentially small,
but hopefully it will give you
the basic idea. Suppose the
astronomer makes two mea-
surements with which she de-
termines the motion of the
star across the sky. The star
is moving toward the earth in this case, so it takes the light detected in the rst
measurement longer to get to the earth. Suppose that the star gets closer to the
earth by z between the times it emits the detected light. Then the time between
the two light measurements on earth is:
t2 t1 = t z/c. (72)
where t is the time it takes the star to move to the new location in the earth
frame. Then the apparent angular motion is given by:
d D
D = . (73)
dt t z/c

So in fact (which is clear if you try some reasonable numbers), this apparent
velocity can exceed the real velocity of the source by quite a bit, enough to make
the star look like its going c or faster. There are real cases of this in astronomy...
for example at the center of the galaxy there are stars whose apparent velocity
greatly exceeds c (of course theyre redshifted and blueshifted all over the place,
but you get the idea...)!

September 2, 1999
1

University of California, Berkeley


Physics H7C Fall 1999 (Strovink)
PROBLEM SET 2

1. 3.
A closely spaced circular parallel plate capacitor (based on Purcell 10.23.)
with long axial leads has a small (temporarily Consider an oscillating electric eld, E0 cos t,
constant) current I passing through it (because inside a dielectric medium that is not a perfect
the voltage across it is changing very slowly). insulator. The medium has dielectric constant
The plates are perfectly conducting and have ra-
and conductivity . This could be the elec-
dius b. tric eld of some leaky capacitor which is part
(a.) of a resonant circuit, or it could be the electric
Use Gausss law to nd the time rate of change eld at a particular location in an electromag-
dE/dt of the electric eld within the plates. You netic wave. Work in SI units. Show that the Q
may assume that the charge densities on the factor, dened by
inside surfaces of the plates do not vary appre-
ciably across those surfaces. energy stored
Q= ,
(b.) average power dissipated
Use the Ampere-Maxwell equation to nd the
magnitude and direction of the magnetic eld is
/ for this system, and evaluate it for
halfway between the plates, at a radius r < b seawater at a frequency of 1000 MHz. The
from the axis. conductivity is 4 (ohm-m)1 , and the dielectric
(c.) constant may be assumed to be the same as that
Use Amperes law to evaluate the magnetic eld of pure water at the same frequency,
in the vicinity of one of the long axial leads, far
from the capacitor. Compare it to the answer

78 .
for (b.).
0
(d.)
Suppose instead that I varies slowly. Far from What does your result suggest about the propa-
the fringe of the capacitor, would you expect the gation of decimeter waves through seawater?
electric eld to vary slightly with r? Explain.
4.
2. (based on Purcell 10.24.) 
(based on Purcell 10.14.) A block of glass, refractive index n =
/
0 , lls
Consider three closely spaced parallel plate ca- the space y > 0, its surface being the xz plane.
pacitors of the same square area and plate sep- A plane wave traveling in the positive y direc-
aration. The rst capacitor C1 consists only of tion through the empty space y < 0 is incident
those plates and vacuum. Both C2 and C3 are upon this surface. The electric eld in this wave
half-lled with an insulating material having di- is z Ei sin (ky t). There is a wave inside the
electric constant
, but the dielectric is arranged glass block, described exactly by
in dierent ways: C2 s dielectric extends over the
full plate area, but lls only the half gap closest E = z E0 sin (ky t)
to one of the plates; C3 s dielectric extends from B = x B0 sin (ky t) .
one plate to the other, but covers only half of the
gap area. (The dielectric boundaries are always
There is also a reected wave in the space y < 0,
either parallel or perpendicular to the plates.)
traveling away from the glass in the negative y
Calculate the capacitances C2 and C3 , expressed direction. Its electric eld is z Er sin (ky + t).
as a ratio to C1 . Of course, each wave has its magnetic eld of
2

amplitude, respectively, Bi , B0 , and Br . The to-


tal magnetic eld must be continuous at y = 0,
and the total electric eld, being parallel to the
surface, must be continuous also. Show that this
requirement, and the relation of B0 to E0 given
in the equation

B0 =
0 E0 ,

suce to determine the ratio of Er to Ei . When


a light wave is incident normally on a vacuum-
glass interface, what fraction of the energy is
reected if the index n is 1.6?

5.
(based on Purcell 11.11.)
Write out Maxwells equations as they would
appear if we had magnetic charge and magnetic
charge currents as well as electric charge and
electric currents. Invent any new symbols you
need and dene carefully what they stand for.
Be particularly careful about + and signs.
Work in SI units.

6.
(based on Purcell 11.17.)
An iron plate 0.2 m thick is magnetized to satu-
ration in a direction parallel to the surface of the
plate. A 10 GeV/c muon having momentum
p, with pc = 1010 eV, moving perpendicular to
the plates surface, enters the plate and passes
through it with relatively little loss of energy.
(This is possible because the muon, of mass
m with mc2 108 eV, is 200 times heavier
than an electron, so it radiates 2002 times fewer
photons.) Calculate approximately the angular
deection of the muons trajectory. Take the sat-
uration magnetization of iron to be equivalent
to 1.5 1029 electron magnetic moments per m3
(the electron magnetic moment is B 6 105
eV per Tesla).

7.
Fowles 1.4.

8.
Fowles 1.6.
Physics H7C Fall 1999 Solutions to Problem Set 2 Derek Kimball
The purpose of physics is to understand the universe... the purpose of mathemat- (b)
ics is, well, obscure to me...
The Ampere-Maxwell equation, since there is no real (conduction) current J be-
- Prof. Seamus Davis, UC Berkeley tween the plates of the capacitor, reduces to:

If you have any questions, suggestions or corrections to the solutions, dont hesitate 
E
 = 0 0
B (5)
to e-mail me at dfk@uclink4.berkeley.edu! t
If youre interested in the possibility of magnetic monopoles, you might want to Then using our result from part (a) and integrating (we choose an Amperian loop
look up a paper by Blas Cabrera (Physical Review Letters, vol. 48, no. 20, centered on the z-axis of radius r), we nd:
1982 pp. 1378-81), where the possible detection of a single magnetic monopole  2
is discussed. There have been no further monopoles detected since that time, so  d = 2rB = 0 Ir
B (6)
this report remains unconrmed. There is also an excellent discussion of magnetic b 2

monopoles in J.D. Jacksons Classical Electrodynamics.


Thus we nd the magnetic eld in the direction to be:
A discussion of the additional problem presented in discussion section this week
can be found in a paper by Robert Romer (American Journal of Physics vol. 50, 0 Ir
B = . (7)
no. 12, 1982 pp. 1089-93). 2b2

Problem 1
(c)
(a) Far from the capacitor, there is no changing electric eld and therefore only con-
We use Gausss law and choose a cylindrical surface of radius r centered on the axis duction current, so this is the familiar Amperes law:
(well call it z) of the parallel plate capacitor, far from the edges of the capacitor 
(r  b). Then:  d = 0 Iencl ,
B (8)

E  = Qencl = r2 / 0 ,
 dA (1) from which we nd a magnetic eld in the direction:
0
where is the surface charge density of the capacitor. We nd directly from Eq. 0 I
B = . (9)
(1) that: 2r

 = (/ 0 )z. which you will note is equivalent to Eq. (7) when r b. Also note that inside the
E (2) capacitor, the magnetic eld grows with r while far from the capacitor the eld
falls as 1/r.
Since there is a current I, the surface charge density changes with time by an
amount:
(d)
d I
= 2, (3)  far
Lets consider the electric eld in two dierent regions. First, well consider E
dt b
from the capacitor in the vicinity of one of the long axial leads (as in part (c)).
where we assume the current is owing in the z direction. So from Eqs. (2) and The changing current produces a changing magnetic eld, and from Maxwells
(3), we nd that: equations we know this creates an electric eld:

   
dE I  d = B 
= z. (4) E dA. (10)
dt 0 b2 t

September 2, 1999
Physics H7C Fall 1999 Solutions to Problem Set 2 Derek Kimball

B
From Eq. (9), we see that t is given by: Problem 2


B 0 I We can simplify the problem by thinking of C2 and C3 as two capacitors in series
= . (11)
t 2r t or in parallel, respectively (Fig. 1). The capacitance C of a parallel plate capacitor
is given by:
We choose an Amperian loop as indicated in Fig. 1. There is no electric eld
perpendicular to the wire (along r). This can be deduced from symmetry consid- A
erations. Suppose there was an electric eld in the r direction. How does it know C= (15)
d
whether to point in the +r or r direction? That has to be decided by either the
direction of the current or the change in current. If we reverse these quantities, where A is the area of the plates and d is the plate separation. So for C1 :
the electric eld in the r direction should reverse. But on the opposite sides of the
0 A
wire, these quantities have opposite signs! The only way this can be true is if the C1 = (16)
electric eld in the r direction is zero. d

Furthermore, we know that


C2a
the electric eld must go to
zero
 as r . But since Axial Lead
 d = 0, it must be the
E C2b
r1 C2
case that we have an electric
eld in the z direction which z
varies with r. In other words,
r
it is apparent that the electric
eld is larger closer to the wire
(z-axis). r2
Amperian Loop
This can be done explicitly, of C3
course, from Eqs. (10) and Figure 1
C3a C3b
(11):
0 I
E(r2 ) E(r1 ) = ln r1 /r2 . (12)
2 t
Figure 2
Lets now consider the electric eld inside the capacitor, far from the fringe (as in
part (b)). Once again we apply Eq. (10), but in this case:
For C2 we break up the problem into two parts, solving for C2a and C2b (shown

B 0 r I
= , (13) in Fig. 1), then determining C2 using:
t 2b2 t
 1
We see that there is also a component of the electric eld in the z direction which 1 1
C2 = + . (17)
varies with r by utilizing similar arguments as those presented above: C2a C2b

0 r2 I From Eq. (15) we can nd C2a and C2b , where:


E(r) = . (14)
4b2 t
0 A
C2a = = 2C1 (18)
d/2

September 2, 1999
Physics H7C Fall 1999 Solutions to Problem Set 2 Derek Kimball
and where the current density is given by Ohms law:
A
C2b = = 2 C1 . (19) J = E. (28)
d/2 0
So with a wee bit of algebra, we nd that: Once again taking the time average, we nd:

2C1 E02
C2 = . (20) P
= . (29)
0 / + 1 2

Similarly for C3 , we break up the capacitor into two parts C3a and C3b , and then The Q-factor is the ratio of these two quantities, U
and P
, multiplied by the
solve for C3 using: frequency:
C3 = C3a + C3b . (21)
Q= . (30)

We use Eq. (15) to solve for C3a and C3b , nding:
0 A/2 1 If we plug in the numbers for seawater, we nd that Q 1.1. This suggests that
C3a = = C1 (22) decimeter waves cannot propagate very far in seawater, since the energy in the
d 2
wave falls to 1/e its initial value in about one decimeter!
and
A/2 Problem 4
C3b = = C1 . (23)
d 2 0
So here the overall capacitance is given by: First, we can write down the the electric and magnetic elds of the incident,
transmitted and reected waves:
C1
C3 = ( / 0 + 1). (24)
2 zEi sin (ky t)
xBi sin (ky t)
Problem 3
zE0 sin (k0 y t)
The energy per unit volume U stored in an electromagnetic wave is given by: xB0 sin (k0 y t)
 
1 2 1 2
U= E + B = E 2 . (25)
2 zEr sin (ky + t)
If we then time average the energy, we nd that the average energy stored is: xBr sin (ky + t) (31)

E02 We note that k0 = nk since the transmitted wave is in glass. Then we can impose
U
= E0 cos2 (t)dt =
2
. (26) the condition
2

The average power P dissipated per unit volume is given by the relation: |B| = | E| (32)

J2 = 1E
on each of the waves, and demand that the Poynting vector, S  B,
 is along
P = , (27) 0
the direction of propagation of the waves. This xes the amplitudes and signs of

September 2, 1999
Physics H7C Fall 1999 Solutions to Problem Set 2 Derek Kimball
the magnetic elds with respect to the electric elds: Problem 5

zEi sin (ky t)


If there were magnetic charges, a magnetic charge density m and a magnetic
x 0 0 Ei sin (ky t)
current density Jm would appear in Maxwells equations. To avoid confusion, lets
denote the traditional electric charge density e and electric current density Je .
zE0 sin (k0 y t) We can place both of these, with some constants c1 and c2 which will be dened

x 0 E0 sin (k0 y t) later, in Maxwells equations to make them nice and symmetric:

 E
 = e / 0
zEr sin (ky + t)
 B
 = c1 m
x 0 0 Er sin (ky + t) (33)

Now we consider the elds at y = 0, the interface between the block of glass and  E
 = B + c2 Jm
t
vacuum. We require that the electric and magnetic elds parallel to the surface of 
the glass satisfy:  B
 = 0 0 E + 0 Je (38)
t
E = E
B B
=  . (34) We can go further and work out a relationship between magnetic charge density
and current density. We begin by demanding that magnetic charges and currents
After substitution, this leaves us with two equations: satisfy the continuity equation, namely:

Ei + Er = E0  Jm + m = 0.
(39)
 t

Ei + E r = E0 . (35)
0 Then if we take the divergence of the new third Maxwells equation, we get:
We can then eliminate E0 from these equations yielding the ratio of Er to Ei :

 
 E  B + c2
 =  Jm . (40)
Er / 0 1 t
= . (36)
Ei / 0 + 1
There is a vector derivative rule that states for any vector eld A,    A  = 0.
So the left-hand side of (40) is 0. The derivatives on the right hand side, 
and
The energy is proportional to E 2 (as can be readily seen by 
considering the Poynt-
 and in this case the index of refraction n = / 0 . Thus the ratio t , can be swapped and we get:
ing vector S),
of reected to incident energy Ur /Ui is given by:
   Jm = 0.
 2  2 B + c2 (41)
Ur Er n1 t
= = . (37)
Ui Ei n+1
From the second Maxwell equation we know that  B  = c1 m , so we nd:
For n = 1.6, 5% of the energy is reected.
m  Jm = 0.
c1 + c2 (42)
t

If we then apply the continuity equation, Eq. (39), we nd that c1 = c2 c.

September 2, 1999
Physics H7C Fall 1999 Solutions to Problem Set 2 Derek Kimball
Thus the nal form of Maxwells equations is: mean that H = 0 everywhere. The fundamental reason for this is that in order to
 E = e / 0 completely determine a vector eld you must know both its curl and divergence.

Only in cases where we have planar, cylindrical, toroidal or solenoidal symmetry
 B = cm  H
can we conclude that  = 0, and get H
 quickly. This is dierent from Amperes
 where we always know
law with B  B = 0. So, be careful when using H! 

 E
 = B cJm
t Anyhow, in this case its no problem, we nd that:

E
 B
 = 0 0 + 0 Je , (43) B .
 = 0 M (46)
t
where c is a constant of proportionality between the magnetic charge unit and the inside the iron plate and zero outside the plate.
magnetic eld it produces (the equivalent of 1/ 0 for electric elds). This problem now reduces to the tra-
ditional problem of solving for the
Problem 6 cyclotron orbit of a moving charged
particle in a magnetic eld. An im-
The rst part of this problem is to portant dierence, as pointed out by Iron Plate
calculate the magnetic eld B  inside 
M
Paul Wright in section (thanks!), is
the magnetized iron. We can use the Iron Plate
that in this case we need to be care-
auxiliary eld H  to make our job a
ful about relativistic corrections to
little easier. We know that: the radius of the cyclotron orbit.

 = B M
H . (44) To nd the radius of the cyclotron
0 d
orbit R, we balance the Lorentz force
Also, we have the relation: qvB with the relativistic centrifugal
 force mv 2 /R. This tells us:
 d = If ree ,
H (45) mv
R= . (47)
qB
where in our problem If ree = 0 ev- Amperian Loop
erywhere. We choose an Amperian where R is the radius of the circular
loop as pictured in Fig. 3 (M  is in orbit. If you take a look at Fig. 4,
the z direction), taking advantage of hopefully the simple geometric argu-
the planar symmetry of the problem ments suggested convince you that
(we can assume the iron plate is in- in fact:
 z R
nite). Since the component of H per- dqBc dqBc
pendicular to the surface of the iron sin = d/R = = , R
mvc pc
plate must be zero based on symme- (48)
try, and outside the iron plate H  0 y
as y , we conclude that in fact where is the deection angle and
H = 0 everywhere. x d is the thickness of the plate. The Figure 4
rest of the problem is working out
I would like to pause here and point
the correct units...
out that this conclusion is not en- Figure 3
tirely trivial. If there is no free cur- First lets get B in SI units. B = 0 M = 4 107 N/A2 1.5 1029 electron
rent If ree , that does not necessarily magnetic moments per m3 9 1024 J/T, or about 1.7 T. Then dqcB = 108 eV,

September 2, 1999
Physics H7C Fall 1999 Solutions to Problem Set 2 Derek Kimball
2
so sin = dqBc
pc = 10 rad. Thats only about half a degree, so not too big of where k = 2/ is the wave vector. We can also express in terms of u and :
a deection... 2u
= ku = . (55)

Problem 7 If you take the derivative of with respect to :
d 2u 2 du
Fowles 1.4 = 2 + . (56)
d d
The 3D wave equation is: Now we calculate d
dk :

1 2f d 2
2 f = (49) = . (57)
u2 t2 dk 2
We employ spherical coordinates, and since our wavefunction is a function only of If we then substitute the expressions in Eqs. (56) and (57) into Eq. (54), we arrive
r, 2 is also a function only of r: at our result:
  u
1 f ug = u . (58)
2 f = 2 r2 . (50)
r r r
(b)
Plugging in the spherical harmonic wavefunction f = 1r e(krt) , we get:
We use similar tricks to derive the result:
2 1 1 0 dn
k
2 f = e(krt) = k 2 f. (51) = . (59)
r u g u c d0
If we evaluate the right-hand side of Eq. (49), and use the fact that k = /u, we We begin by noting that
nd that: 1 dk dk d0
= = (60)
1 2f ug d d0 d
= k 2 f. (52)
u2 t2 Lets write the wave vector in terms of 0 and n:
which veries that f is a solution to the 3D wave equation. 2n
k= (61)
0
Problem 8 We can then take some derivatives, and nd that:
dk 2n 2 dn
Fowles 1.6 = 2 + (62)
d0 0 0 d0
(a) and
d0 2
Lets begin by deriving = 0 . (63)
d 2c
u
ug = u . (53) Substituting these results back into Eq. (60) gives us the answer we were looking
for:
We can begin by using Fowles (1.33), the denition of the group velocity: 1 1 0 dn
= . (64)
ug u c d0
d d d
ug = = , (54)
dk d dk Thats all folks!

September 2, 1999
1

University of California, Berkeley


Physics H7C Fall 1999 (Strovink)
PROBLEM SET 3

1. the dimensions of area and is called a scattering


(based on Purcell B.1.) total cross section. The energy radiated, or scat-
An electron of rest mass me and charge e, mov- tered, by the electron, and thus lost from the
ing initially at a constant velocity v, is brought plane wave, is equivalent to the energy falling on
to rest with a uniform deceleration a that lasts an area . (The case considered here, involving a
for a time t = v/a. Compare the electromagnetic free electron moving nonrelativistically, is often
energy radiated during the deceleration with the called Thomson scattering, after J.J. Thomson,
electrons initial kinetic energy. Express this ra- the discoverer of the electron, who rst calcu-
tio in terms of two lengths: the distance that lated it.)
light travels in time t, and the classical electron
radius r0 , dened as 3.
e 2 (based on Purcell B.4.)
r0 .
4 0 me c2 The master formula
To carry out this calculation, you need a formula
like Purcell Eq. (B.6) that relates the instanta- 1 2 e2 a2
Prad = .
neous radiated power Prad to the instantaneous 4 0 3 c3
acceleration a. In SI units, this formula is
1 2 e2 a2 is useful for particles moving relativistically, even
Prad = . though v  c was assumed in Purcells deriva-
4 0 3 c3
tion of it. To apply it to a relativistic situation,
(Note that, as far as one has been able to tell ex-
all we have to do is (i) transform to a comoving
perimentally, the electron actually is consistent
inertial frame F  in which the particle in ques-
with having zero radius, and it must have a ra-
tion is, at least temporarily, moving slowly; (ii)
dius at least several orders of magnitude smaller
apply the master formula in that frame; and (iii)
than the classical radius r0 .)
transform back to any frame we choose.
2. Consider a highly relativistic electron (  1)
(based on Purcell B.3.) moving perpendicular to a magnetic eld B. It
A plane electromagnetic wave with angular fre- is continually accelerated (in a direction perpen-
quency and electric eld amplitude E0 is dicular both to its velocity and to the eld), so it
incident on an isolated electron. In the resulting must radiate. At what rate does it lose energy?
sinusoidal oscillation of the electron, the max- To answer this, transform to a frame F  moving
imum acceleration is |e|E0 /m, where e is the momentarily along with the electron, nd E  in
electrons charge. that frame, and thereby nd Prad 
.
Averaged over many cycles, how much power
is radiated by this oscillating charge? (Note Now show that, because power is energy/time,

that, when the maximum acceleration of the Prad must be equal to Prad .
electron rather than its maximum amplitude is This radiation generally is called synchrotron ra-
held xed, the power radiated is independent of diation. It is both a blessing and a curse. The
the frequency .) blessing is that intense beams of UV and X-ray
Divide this average radiated power by 0 E02 c/2, photons are created at synchrotrons designed for
the average power density (per unit area of wave- that purpose, such as Berkeley Labs Advanced
front) in the incident wave. The quotient has Light Source. These beams are essential for
2

many studies and uses such as semiconductor Here nx > ny if the x direction is slow, is the
lithography. The curse is that synchrotron ra- (xed) angular frequency of the light, and D is
diation prevents circular electron accelerators of the thickness of the plate. Because the absolute
practical size (up to tens of km in circumference) phase of the light is of no experimental inter-
from exceeding about 1011 eV in energy, much est, the eect of the wave plate is equivalent to
weaker than the 1012 eV proton beams that have multiplying the x component of (complex) E1 by
been available at Fermilab for a decade. exp (i/2) and the y component by exp (i/2).
(a.)
4. Write the Jones matrix for the ideal wave plate
Fowles 2.4. just described.
The solution to this problem was sketched in (b.)
lecture on 7 Sep. In Fowles notation, E0 and B0 Calculate the Jones matrix for the general case
are the same as the E1 and B1 discussed in class. in which the slow plate axis lies at an angle
from the x axis, where, as in Problem 7, is an
5. angle in the xy plane.
Fowles 2.7. (c.)
For the wave plate in (b), show that its Jones
6. matrix M is unitary, i.e. Mij = M1 ji . This
Fowles 2.10. means that the action of the wave plate is time-
reversal invariant. For any polarization, the irra-
7. diance of a light beam is unaected by traversing
(a.) the plate (though its polarization may change
For an ideal linear polarizer with its transmis- dramatically).
sion axis at an arbitrary angle with respect
to the x axis, calculate the Jones matrix. (As
usual, the beam direction is z, is an angle in
the xy plane, and is positive as one rotates
from x toward y.)
(b.)
For a linear polarizer, show that its Jones
 matrix
M is not unitary, i.e. Mij = M1 ji . This
means that the action of the wave plate violates
time-reversal invariance. This makes sense be-
cause, for general polarization, the irradiance of
a light beam is reduced after passing through
the plate.

8.
A wave plate is made out of a birefringent crystal
whose lattice constants are dierent in the fast
and slow directions of polarization. This leads
to dierent indices of refraction for the two po-
larizations. If the x axis is along the slow
direction of the plate, x polarized light accu-
mulates a phase shift with respect to light
polarized in the fast or y direction, with

D
= (nx ny ) .
c
Physics H7C Fall 1999 Solutions to Problem Set 3 Derek Kimball
Niels Bohr (not for the rst time) was ready to abandon the law of conservation I believe that the answer is frame dependent (as is the case for most relativity
of energy. It is interesting to note that Bohr was an outspoken critic of Einsteins paradoxes). If youre free-falling with the particle, you dont see any radiation.
light quantum (prior to 1924), that he discouraged Diracs work on the relativis- If the charge is accelerating with respect to you, then you see radiation. This
tic electron theory (telling him, incorrectly, that Klein and Gordon had already can be shown with the general relativistic eld transformations. What about
succeeded), that he opposed Paulis introduction of the neutrino, that he ridiculed energy conservation? Well, Im no Niels Bohr, so I think that if you change your
Yukawas theory of the meson, and that he disparaged Feynmans approach to quan- acceleration into the frame of the particle, everything works out... but to prove
tum electrodynamics. this seems a bit complicated... anyhow, good stu to think about, keep up the
great work! Thanks!
- Prof. David Griths, Reed College, excerpted from Introduction to Elementary
Particles
Problem 1
If you have any questions, suggestions or corrections to the solutions, dont hesitate
to e-mail me at dfk@uclink4.berkeley.edu! First well calculate the electromagnetic energy radiated Prad t during the decel-
An interesting (to me) point concerning the physical meaning of orthogonal po- eration lasting for t = v/a, which is given by:
larization was raised after discussion section the other day. Fowles says that two
1 2 e2 av
waves E1 and E2 whose complex electric eld amplitudes satisfy: Prad t = . (2)
4 0 3 c3
E1 E2 = 0 (1)
The electrons initial kinetic energy K is just:
are orthogonally polarized. 1
K = mv 2 , (3)
For linear polarization, there is a simple geometric analogy. Linearly polarized 2
light (e.g., in the x direction) is orthogonal to light with a perpendicular linear
polarization (e.g., in the y direction). So no light will get through two linear and the ratio Prad t/K is given by:
 
1 e2  a 
polarizers which are orthogonal in the Euclidean geometry sense. However,
Prad t 4
this picture breaks down for more complicated polarization states, e.g. circular = . (4)
polarizations. For example, two circular polarizations whose electric elds are K 3 4 0 mc2 cv
always at right angles to each other are not orthogonal! The distance d traveled by light in t is cv/a, and the classical radius
  of the elec-
Orthogonality for polarization states can be understood using notions from lin- tron r0 is given by the formula in the problem set: r0 e2 / 4 0 mc2 . Therefore:
ear algebra. In this sense the complex vector space of polarization states can be
spanned by two linearly independent, complex vectors - any two linearly indepen- Prad t 4  r0 
= . (5)
dent complex vectors are said to be orthogonal. This is what Fowles means when K 3 d
he says two polarization states are orthogonal.
Heres an interesting fact that might help you remember some important lengths
Another interesting question raised after discussion section, although it is a bit
in physics:
beyond the scope of the course, was whether or not a free-falling (in a gravitational
eld) charged particle radiates. You would expect that it might not based on the One of the most important constants in physics is the ne-structure constant ,
equivalence principle, which basically states that a free-falling frame is equivalent which sets the strength scale for the electromagnetic force:
to an inertial frame. However, if you observe the charge from the surface of some
planet, you would see a charged particle undergoing acceleration. This is a little e2 1
= , (6)
funny, since you would expect that the charge either loses energy or it doesnt... c 137
After thinking about it a little and consulting some wise general relativity texts where e is in CGS units. You may recognize the upper division physics course,
(e.g., Walds General Relativity or Misner, Thorne and Wheelers Gravitation), Physics 1 .
September 16, 1999
Physics H7C Fall 1999 Solutions to Problem Set 3 Derek Kimball
You can guess the the classical radius of the electron by setting the rest energy of
the electron equal to the potential energy stored in a spherical shell of radius r0 Problem 3
with charge e on the surface.
We transform to a comoving inertial frame F  in which the electron is temporarily
2 e2 
mc = . (7) at rest. The electric and magnetic elds in F are given by:
r0
 
  + cB
E = E
In a few weeks, youll learn about Compton scattering (photon-electron scattering).  
The Compton wavelength of the electron is given by r0 /. The Bohr radius, the  = B
B  1 E


radius of an electrons orbit in the hydrogen atom, is given by r0 /2 . c
  
E = E
Problem 2  = B
B 

. (12)
The electron oscillates sinusoidally with the acceleration given by:

|e|E0 The force on the electron is the Lorentz force given by:
a= sin t. (8)
m  
F = e E + v B
 , (13)
The power Prad radiated is given by:

1 2 e4 E02 but in the comoving frame F  the electrons velocity is zero. Since in the lab frame
Prad = sin2 t, (9) F the electric eld is zero, the force acting on the electron in F  is:
4 0 3 c3 m2

and of course if we average over many cycles... F = eE = ecB. (14)

1 1 e4 E02 Thus, the acceleration a is:


Prad  = . (10)
4 0 3 c3 m2
ecB
a= (15)
m
Next if we divide this result by the average power density U  in the incident wave
we get the scattering cross section : This acceleration can be used in our old pal which describes the power radiated:

1 e4  1 2 e2 a2
= 2 . (11) Prad = , (16)
6 0 m2 c4 4 0 3 c3

You might notice that = (8/3)r02 , r0 being the classical electron radius from which gives us:
problem 1...
 1 2 e4 2 2 B 2
Prad = . (17)
4 0 3 m2 c

When we transform back to frame F , the energy transforms as E  E


and the time transforms as t t. Therefore, since power is just E/t,

Prad = Prad .

September 16, 1999


Physics H7C Fall 1999 Solutions to Problem Set 3 Derek Kimball
Problem 4 This is pretty straightforward. Heres the prescription:
 in the form:
Given the electric eld of the wave, E,
Fowles 2.4  
E = E0 i + jbei ei(krt) , (24)
For this problem we employ the complex exponential form of the wave functions
 and H:
for E 
  the Jones vector is given by:
 
E = Re E0 e i(krt)    
  E x 1
= E0 . (25)
 = Re H
H 0 ei(krt) . (18) Ey bei

You can normalize the Jones vector if you want, but if you didnt feel like doing
The Poynting vector is given by: that in this problem thats okay too. So pretty much we can just write down the
answers, here they are:
=E
S  H.
 (19)
(a)
Using the expressions from Eq. (18), we nd the Poynting vector is:
   
 = Re E0 ei(krt) Re H
S 0 ei(krt) (20)  

Ex 1
= 2E0 2 . (26)
Ey 1
We can expand the exponentials in terms of sines and cosines and nd the real 2
parts:
 
 = Re(E0 ) cos (k r t) Im(E0 ) sin (k r t)
S (b)
 
Re(H 0 ) cos (k r t) Im(E0 ) sin (k r t) . (21)
 

Ex 1
= 5E0 5 . (27)
If we expand this expression and time average (i.e. we set sin2 (k r t) and Ey 2
5
cos2 (k r t) equal to 12 and sin (k r t) cos (k r t) equal to 0), then we
get:
1  (c)
 =
S Re(E0 ) Re(H
0 ) + Im(E0 ) Im(H
0 ) . (22)
2
 

This expression, by inspection, is equivalent to: Ex 1


= 2E0 2 . (28)
i
  Ey
 = 1 Re E0 H ,
S (23)
2
0
2
which veries the claim. (d)

Problem 5
  
2

Ex 2
= 2E0 1+i . (29)
Fowles 2.7 Ey 2

September 16, 1999


Physics H7C Fall 1999 Solutions to Problem Set 3 Derek Kimball
(b)
Problem 6  
A quick way to check if the matrix is unitary is to multiply M by MT and see
if it equals the identity matrix:
Fowles 2.10   
  cos2 sin cos cos2 sin cos
M MT =
Well start with an arbitrary polarization: sin cos sin2 sin cos sin2
     
Ex a cos2 sin cos
= . (30) = . (36)
Ey bei sin cos sin2
Now well send it through a linear polarizer. Lets orient the linear polarizer at Well thats not the identity matrix, so the Jones matrix of a linear polarizer is not
45o , so our resultant polarization is given by: unitary.
        
Ex 1 1 1 a 1 a + bei a + bei 1
= i = i = (31) Problem 8
Ey 2 1 1 be 2 a + be 2 1
We can ignore the amplitude out front. Now well sent it through a quarter-wave
plate with the fast axis horizontal. (a)
       The Jones matrix for the ideal wave plate is:
Ex 1 0 1 1
= = , (32)  i/2 
Ey 0 i 1 i
e 0
M= . (37)
which is indeed circular polarization! What happens if we change the order? 0 ei/2
       
Ex 1 1 1 1 0 a a + bei 1
= = , (33) (b)
Ey 2 1 1 0 i bei 2 1
which is linear polarization. So circular polarized light is created only by placing For the general case we just do the matrix multiplication as in problem 7 (a):
the optical elements in the proper order.    i/2  
 cos sin e 0 cos sin
M =
sin cos 0 ei/2 sin cos
Problem 7  2 2
  
i/2 e cos  sin
i i
1 + e sin cos
=e . (38)
1 + ei sin cos cos2 ei sin2
(a)
For this problem, we can use the technique for changing the basis of a matrix (c)
operator that we employed in the rst problem set to nd Lorentz transforms in
Now we check for unitarity just as in problem 7 (b):
rotated frames (see PS1 solutions, problem 2). Namely,
   
M = R1 MR. T ei cos2  sin2 1 + ei sin cos
(34) M (M ) = ei/2
1 + ei sin cos cos2 ei sin2
where M is the Jones matrix and R is the appropriate rotation matrix. For a linear      
i/2 ei cos2  sin2 1 + ei sin cos 1 0
polarizer with transmission axis at an arbitrary angle : e = (39)
1 + ei sin cos cos2 ei sin2 0 1
     2

cos sin 1 0 cos sin cos sin cos
M= = . So the Jones vector for the waveplate is a unitary operator!
sin cos 0 0 sin cos sin cos sin2
(35) Bye for now!

September 16, 1999


1

University of California, Berkeley


Physics H7C Fall 1999 (Strovink)
PROBLEM SET 4

1. electrically insulating material which has  = 0


A plane wave polarized in the x direction is nor- but = m 0 . That is, the material has triv-
mally incident upon an ideal quarter-wave plate ial dielectric but nontrivial magnetic properties.
which has its slow axis oriented at +45 with Consider the ferromagnetic limit m .
respect to the x axis. Next it is reected at nor- In that limit, what is the ratio of peak elec-
mal incidence by a perfectly conducting mirror.
tric eld |E |max in the transmitted wave to the
Finally the wave passes back through the same same quantity |E|max in the incident wave?
quarter-wave plate. After completing this jour-
ney...
5.
(a.)
Plane waves propagating in the z directions
What is the nal waves irradiance, relative to
bounce between two semi-innite nonconducting
the incident wave?
materials. The rst material occupies the region
(b.)
z < 0, and the second occupies z > L, where L
What is the nal waves state of polarization? (If
is a positive xed distance. These (hypothetical)
linearly polarized, specify the polarization direc-
materials have no unusual magnetic properties
tion; if circularly polarized, state whether right-
( = 0 ), but they have infinite dielectric con-
or left-handed.)
stant, /0 = .
Both of your answers should be justied.
(a.)
What components of E and H vanish, and where
2. do they vanish? Justify each answer that you
A ber-optic cable consists of a cylindrical core give.
with refractive index n1 surrounded by a sheath (b.)
with refractive index n2 , where 1 < n2 < n1 . What angular frequencies for the light are pos-
The ends of the cable are cut perpendicular to sible?
the cable axis and polished. A point source of
light is placed on the cable axis a negligible dis- 6.
tance away from one end. What fraction of the A nonrelativistic particle of mass m in one (x)
total light emitted by the point source is trans- dimension can be represented by a (complex)
mitted by the core to the (distant) end of the wavefunction
cable? (You may ignore losses due to reection
at the cable ends.) (x, t) exp (i(kx t))

3. where
A vector eld F(r) is equal to the curl of a vec- h2 k 2 (x) = 2m(E V (x)),
tor potential A which is a continuous function
E = h is the total and V (x) is the potential
of r: F(r) = A(r).
energy, and 2h is Plancks constant. If V (x)
Prove that F is continuous across any surface, is piecewise at, at the discontinuities in V the
where refers to the component of F which Schrodinger equation demands that both and
is perpendicular to the surface. /x remain continuous.
Consider the reection of a particle of initial ki-
4. netic energy T by a potential barrier of height
A transverse electromagnetic wave in vacuum V < T . Show that the ratio of reected to in-
is normally incident on a semi-innite slab of cident amplitudes is given by the same formula
2

as for the normal reection of an electromag-


netic wave at a dielectric interface with = 0
everywhere, provided that the refractive index
in the quantum mechanical case is taken to be
proportional to k.

7.
Fowles 3.1.

8.
Fowles 3.6.
Physics H7C Fall 1999 Solutions to Problem Set 4 Derek Kimball
Physicists are like 3% of rats. This result can also be arrived at using Fowless reection matrix (page 52).
-Max Zolotorev, Lawrence Berkeley National Laboratory The beam now passes back through the quarter waveplate, but now the wave sees
the fast-axis oriented at +45o . So we nd that:
If you have any questions, suggestions or corrections to the solutions, dont hesitate         
to e-mail me at dfk@uclink4.berkeley.edu! Ex 1 1 i E0 1 0
= = iE0 (4)
Ey 2 i 1 2 i 1
Problem 1 In other words the resultant light is linearly polarized in the y direction.

(a) Problem 2

As demonstrated in last weeks problem set (problem 8), an ideal quarter-wave First lets derive Fowles result regarding the acceptance angle for a ber-optic
plate is described by a unitary Jones matrix. This means that the irradiance of cable (pages 46-47).
the light beam is unaected by traversing the plate. Also, the mirror is a perfect At the rst phase
conductor, so 100% of the light is reected. So the nal waves irradiance must be transition as the light
the same as the incident waves. enters the ber-optic
cable, we have from n2
(b) Snells law:
The wave, initially polarized along the x direction, rst passes through the quarter
sin = n1 sin . (5)
wave plate, whose fast axis is oriented at -45o with respect to the initial light n1
polarization: We want = /2
       to be greater than
Ex 1 1 i E0 E0 1
= = (1) or equal to the crit-
Ey 2 i 1 0 2 i n2
ical angle, sin1 n
Then the light bounces o a perfectly conducting mirror. This reverses the sign of for total internal re- Figure 1
the Poynting vector, which in turn changes the sign of the B-eld relative to the ectance, where n =
E-eld, since (as can be shown from Maxwells equations): n2 /n1 . With a little
trigonometry it can be shown that for these conditions,
 i(
k
r t)
E = E0 e 
1 n21 n22

B = k E,  (2) sin = . (6)
v n1

where v = c/n is the phase velocity of light in a medium and is the direction of Combining Eqs. (5) and (6) yields:
the lights electric eld. The light electric eld undergoes a phase shift of upon 
relection (as can be deduced from the boundary conditions), but for circularly sin = n21 n22 , (7)
polarized light the direction of rotation (clockwise or counter-clockwise) of the
electric eld with respect to a xed coordinate system is preserved. However, we which proves Fowless assertion.
are now viewing it from the opposite direction (since k changed sign). Therefore The next step is to compute the solid angle of light accepted by the ber-optic
the handedness of polarization has changed upon reection: cable, given by:
      2 
Ex E0 1
= (3) = sin dd = 2(1 cos ). (8)
Ey 2 i 0 0

September 23, 1999


Physics H7C Fall 1999 Solutions to Problem Set 4 Derek Kimball
Divided by the total solid angle (4), this yields the fraction T of the total light where Er , Ei , Et are the reected, incident and transmitted electric eld ampli-
transmitted by the core to the end of the cable: tudes, respectively, and

(1 cos ) 1,2
T = . (9) Z1,2 = . (16)
2 1,2
In particular, for the ferromagnetic material described, Z2 while Z1 is nite,
Problem 3 so:
A vector eld F(r) is equal to the curl of a vector potential A(r), so we know that Et 2Z2
= 2. (17)
the divergence of F(r) is zero: Ei Z1 + Z2
F(r) = ( A(r)) = 0. (10)
Then we know that: Problem 5
 
F(r)dV = =0
F(r) dA (11)
(a)
Choose a volume of vanishing thickness  about a surface of area A (where  is Plane waves propagating in the z directions must satisfy Eqs. (15) at the inter-
always normal to the surface), then from Eq. (11) we have that: faces (z = 0 and z = L). At either interface we have, since Z2 0,
 
F r +  A F (r)A + O() = 0 (12) Et 2Z2
= 0. (18)
Ei Z1 + Z2
where O() indicates a term of order , which arises from some nite amount of
Therefore E vanishes in the material, i.e. E=0 for z < 0 and z > L. So at the
ux of F(r) out the sides of the volume.
interfaces, E=0.
The conditionfor continuity
 of F (r) is that for each  > 0, there exists a > 0
The magnetic elds of the plane waves propagating in the z directions between

such that |F r + F (r)| < . From Eq. (12) we know that: the materials must satisfy E+ H+ = -E H (i.e. the Poynting vectors of
  O()
right- and left-traveling waves must be oriented in opposite directions). So while

|F r + F (r)| = . (13) the electric elds cancel (E+ = -E ) at the interfaces the magnetic elds must
A add (H+ = H )! Also we have the boundary condition H = H , so that if H is
Given any  > 0, clearly we can choose to make: nite on one side of the interface, it must also exist on the other side. So there
O() can be components of H everywhere.
< . (14)
A
(b)
Therefore F (r) is continuous.
Our requirements from part (a) set up a standing wave, where the components of
E and H are out of phase. The wave is time independent in order to assure that
Problem 4
E=0 at the interfaces for all times t, so we can postulate:
From Strovinks treatment of reection/refraction at a plane interface between
E = E0 sin kz, (19)
insulators, we have for normal incidence:
Er Z2 Z1 which works so long as k = N /L where N is an integer. So we get the condition
= on angular frequency from k = /c, which implies
Ei Z +Z
1 2
Et 2Z2 N c
= , (15) = . (20)
Ei Z1 + Z2 L

September 23, 1999


Physics H7C Fall 1999 Solutions to Problem Set 4 Derek Kimball

Problem 6
P
We have two regions as shown in Figure 2, with k1 and k2 in each (dened as in
the problem, where they are dependent on the potential V and the particles total r
energy U, which is conserved, U = T + V). In region 1 we have the wavefunctions:

Aei(k1 xt) + Bei(k1 x+t) , (21) a


r+asin
and in region 2 we have a transmitted wavefunction: r+2asin
a
Cei(k2 xt) . (22)

Continuity of the wavefunc-


tions across the boundary
(x=0) demands: V (x)

A + B = C. (23)
U=T+V Figure 3

Since x is also continuous:
The sum of the amplitudes of the waves A coming from the slits at point P (see
k1 (A B) = k2 C. (24) Figure 3) are given by the proportionality:
Region 1 V Region 2
Substituting Eq. (23) into x A eikr + eik(r+a sin ) + eik(r+2a sin ) (27)
(24), we get:
or,
B k1 k2

= (25) Figure 2 A eikr 1 + eika sin + e2ika sin (28)
A k1 + k2

If we assume n k, then we
8
get the formula for normal re-
ection of an EM wave at a
dielectric interface with = 0 : 6

B n1 n2
= (26) 4
A n1 + n2

Problem 7
-0.001 -0.0005 0.0005 0.001

Fowles 3.1
Figure 4

September 23, 1999


Physics H7C Fall 1999 Solutions to Problem Set 4 Derek Kimball
The interference pattern is given by the norm square of the amplitude:
2
I() |A|2 (1 + cos(ka sin )) (29)

The pattern that you get when you plot this function depends on what value you
choose for ka. Lets take a = 1 mm and k = 12,000 mm1 , then our pattern is
shown in Figure 4 as a function of .

Problem 8

Fowles 3.6
Light passes through the gas cell twice, so the optical path dierence dop is given
by:
2l 2l
dop = ct = = 2l(n 1) (30)
c/n c

n changes as the gas lls the cell, and since I 1 + cos (2dop /), a new fringe
appears every time dop = 1/2. Thus the total number of fringes N is given by:

2l(n 1) 4l(n 1)
N =2 = . (31)

Plugging in the suggested values gives us N = 203 fringes.

September 23, 1999


1

University of California, Berkeley


Physics H7C Fall 1999 (Strovink)
PROBLEM SET 5

1. 6.
It is known that, in Region 1 (y > 0, 1 = 40 , Fowles 4.4.
1 = 0 ), there exists a plane wave propagating
in the x direction. 7.
a. A camera lens is purplish because it is optically
What is 2 in Region 2 (y < 0, 2 = 0 )? coated to minimize reection at the center of
b. the visible spectrum. (The coating parameters
What state(s) of polarization (E along y, z, or therefore are not optimized for red or blue light.)
both) can the plane wave be in? Why?
Consider a plane EM wave in vacuum with wave-
length normally incident on a semi-innite
2. piece of glass with refractive index n > 1 and
Fowles 2.23. Substitute interface for window unit permeability /0 = 1. Choose the thick-
that is, consider just one glass-air interface, not
ness and the refractive index of a coating on
two. The degree of polarization is dened by the glass in order to force the reected wave to
Fowles Eq. (2.26). vanish.
3. 8.
A beam of light travelling in the plane y = 0 is Show that the matrix equation (Fowles 4.24) for
refracted by the interface z = 0 between two in- a single-layer lm in fact is an equation that
sulators: vacuum (z < 0) and a material (z > 0) merely transforms the total (complex) ET and
with = 0 and with dielectric constant , where HT just to the right of the right hand interface
 > 0 . In the semi-innite region z < 0, the an- to the total (complex) E0 and H0 just to the left
gles of incidence and reection, with respect to
of the left hand interface.
the z axis, are 60 . In the semi-innite region
z > 0, the angle of refraction, with respect to Show, therefore, that, for a multilayer lm, the
the same axis, is 30 . overall transfer matrix is equal to the product of
(a.) the transfer matrices for the individual lms, as
Taking the reected and refracted angles to be (Fowles 4.28) asserts without proof.
as given, calculate /0 for the material.
(b.)
The incident beam is right-hand circularly po-
larized. What is the state of polarization of the
reected beam? Explain your answer.
(c.)
Calculate the ratio R of irradiances
Ireected
R= .
Iincident

4.
Fowles 3.11.

5.
Fowles 3.13.
Physics H7C Fall 1999 Solutions to Problem Set 5 Derek Kimball
Guys, dont worry about midterms. Theyre not the best measure of your worth We have the following boundary conditions at the interface:
as a physicist. In fact, I did rather poorly on my rst physics midterm, I got
(1) (2)
something like 3 out of 40. Of course, everyone else got 1 out of 40, but thats not 1 E = 2 E
really the point... (1) (2)
1 H = 2 H
- Prof. Nima Arkani-Hamed, UC Berkeley (1) (2)
E =E  
(1) (2)
If you have any questions, suggestions or corrections to the solutions, dont hesitate H = H (5)
to e-mail me at dfk@uclink4.berkeley.edu!
can be along y. In this case, from Eqs. (5), we have that:
First lets see if E
Problem 1
1 Ey(1) = 2 Ey(2)
4Ey(1) = Ey(2) (6)
(a)
and we also know that for the wave in Region 1:
Boundary conditions at the interface (y = 0) between Region 1 and Region 2 imply 
1 (1)
there will be EM waves propagating in the x-direction in both regions. Without Hz(1) = E
knowing their polarizations just yet, let the electric elds of the waves be given 1 y

by: 0 (1)
Hz(1) = 2 E , (7)
i(k1 xt)
0 y
E1 e
E2 ei(k2 xt) . (1) and for the wave in Region 2:

2 (2)
At the interface between Region 1 and Region 2, since the electric and magnetic Hz(2) = E
2 y
eld amplitudes in both regions are independent of x and t, the only way to satisfy  
boundary conditions at all positions and times is if: 1 0 (2) 0 (1)
Hz(2) = Ey = 2 E (8)
2 0 0 y
ei(k1 xt) = ei(k2 xt) (2)
(1) (2)
We see that these relations verify that Hz = Hz as demanded by the boundary
For example, at t = 0, this requires that: conditions in Eqs. (5). The relations are consistent, so therefore such a polarization
is allowed.
k1 = k2 (3)
be along z? We have from Eqs. (5) that
Can E
which means the indices of refraction in the two regions must be the same. Thus
we have, Ez(1) = Ez(2) (9)
  For the wave in Region 1:
1 1 2 2
= (4) 
0 0 0 0 1 (1)
Hy(1) = E
which gives us 2 = 40 . 1 y

0 (1)
Hy(1) =2 E . (10)
(b) 0 z

September 30, 1999


Physics H7C Fall 1999 Solutions to Problem Set 5 Derek Kimball
In Region 2: transmission of TM and TE waves, using the denitions on page 43 of Fowles:
  
2 (2) E
Hy(2) = E rs =
2 y E
   T E
1 0 (1) E
Hy(2) = E . (11) ts =
2 0 y E
   TE
E
We see that this veries the boundary condition on Hy from Eqs. (5): rp =
E
  T M
E
1 Hy(1) = 2 0 0 Ey(1) = tp = (15)
E TM
2 Hy(2) = 2 0 0 Ez(1) . (12)
From the boundary conditions for E and H at the interface (Eqs. (5)), we nd
some basic relations between the quantities in Eqs. (15):
So in fact the wave can be in either polarization state.
ts = rs + 1
Problem 2 ntp + rp = 1 (16)

At the Brewster angle rp = 0, so tp = 1/n. Employing the fact that I nE 2


Reection at the Brewster angle transmits all TM light and reects part of the TE (thanks to Paul Wright!), we have for the partial polarization:
light. The degree of polarization P (for linear polarized light) is given by Fowles
Eq. (2.27): 1 rs2 1 (ts 1)2
P = = (17)
n t2s + t2p n t2s + 1/n2
Imax Imin
P = (13)
Imax + Imin
Fowles works out the general formulae (Fresnels equations) for reec-
tion/refraction at a plane interface, and in particular for ts we have from Fowles
The light is initially unpolar-
(2.56):
ized, so it is 50% TE and TM+TE
50% TM. The light transmit- TE
2 cos sin
ted has reduced TE intensity ts = . (18)
= sin ( + )
(since some is reected at the B
interface), but the TM inten- If we combine Fowles Eq. (2.64)
sity remains the same. Conse-
quently, the transmitted light tan B = n (19)
is partially polarized:
with Snells Law
IT M IT E  IT E 
P = = . sin
IT M + IT E  IT M + IT E  n= , (20)
(14) sin
we have the Brewster condition:

Now we can apply the Fowles TM+TE
Figure 1 + = /2, (21)
formalism for reection and

September 30, 1999


Physics H7C Fall 1999 Solutions to Problem Set 5 Derek Kimball
which follows from the fact that sin = cos . Knowing also then from Eq. (21) From which we have the condition:
that sin ( + ) = 1, we nd that:
Ei + Er = 3Et . (27)
ts = 2 cos2 (22)
Using the trigonometric identity sec2 = tan2 +1 and Eq. (19) we nd that cos2 = Adding Eq. (25) to Eq. (27) gives us (Er /Ei = 1/2)T E , or (Ir /Ii = 1/4)T E . The
1/(n2 + 1) so intensity in the TE component is half the initial intensity, so in total Ir /Ii = 1/8.

2
ts = (23) Problem 4
n2 + 1
Plugging Eq. (23) into Eq. (17), with a little algebra, gives us: Here we treat the tungsten lament as a relatively long straight wire of thickness
2 2 s = 0.1 mm. The distance between the lament and an aperture is r. We want
n[(n 1)]
P = . (24) a transverse coherence width lt 1 mm. Then from Fowles Section 3.7, and in
1 + 6n2 + n4 particular Eq. (3.42), we nd that:
For glass, where n = 1.5, we have that P 12%. r
lt = 1mm (28)
s
Problem 3
If we assume that the tungsten lamp has a central wavelength of 5000 A, then Eq.
(a) (28) demands that r 200 mm.

Note that the Brewster If a double-slit aperture is used, the slits should be oriented parallel to the lamp
condition (Eq. (21)) is met. Thus from Eq. (19) we have
that n = tan 60o = 3. Since = 0 , we nd that  = 30 . lament, otherwise the thickness of the wire s would have to be replaced with the
length of the wire, which is naturally much greater than s. This would force r to
(b) be much greater.
Since the Brewster condition is met, the reected light is 100% TE. Therefore
reected light is linearly polarized along y. Problem 5
(c) The power spectrum of the Gaussian pulse f (t) is given by G() = |g()|2 , where
The light is initially circularly polarized, so it is an equal superposition of linear in our case g() is:
polarizations (TE and TM). Since the TM component has 100% transmission, it 
A  
suces to consider the transverse electric case (E along y) where all reection g() = dt exp at2 + i( 0 )t (29)
occurs. We have the boundary condition that E is continuous, so 2

Ei + E r = E t . (25)
It
 is rst
ax2
useful to derive a result about Gaussian integrals. It turns out that
Also H is continuous, and we have for the incident, reected and transmitted e dx converges, so lets set it equal to some constant c. Now consider the
waves the following components of H in the x direction: integral over the entire plane:
  
0
Hx(i) = Ei cos 60o ax2 2
e dx eay dy = c2 . (30)
0

0
Hx(r) = Er cos 60o Next we convert the integral into polar coordinates (r2 = x2 + y 2 ).
0
   2
30 ar 2
(t)
Hx = Et cos 30 o
(26) e rdr d = c2 . (31)
0 0 0

September 30, 1999


Physics H7C Fall 1999 Solutions to Problem Set 5 Derek Kimball
2

Which is relatively straightforward with the substitution u = ar , du = 2ardr. So N . Since the radius of the fringes r is proportional to , it follows
From this integral we nd that c2 = /a. immediately that r N .
Thus all we need to do is to convert the integral in Eq. (29) to a form resembling
a Gaussian. This can be done by completing the square in the exponent: Problem 7
 
0 ( 0 )2 ( 0 )2 This solution follows directly from the discussion of antireecting lms in Fowles
at2 + i( 0 )t = a t2 2i t + (page 99). We want to choose the thickness of the lm to be 4 . Then the reectance
2a 4a2 4a2

 2 is zero if the index of refraction of the coating n = 0 .


0 ( 0 )2
= a t i (32)
2a 4a
Problem 8
Now the integral in Eq. (29) is just a Gaussian integral, which is no longer a
problem... Fowles Eq. (4.24) states:
     
Working through the constants gives us: 1 1 1
+ r=M t (38)
 2
 n0 n0 nT
A ( 0 )
g() = exp . (33)
2a 4a which is equivalent to:
     
G() = |g()|2 is clearly of the same form, and so G() is a Gaussian function 1 1 1
E0 + E0 = M ET (39)
centered at . n0 n0 nT
0

The total E and H just to the right of the right-hand interface are:
Problem 6
ERH = ET
The condition for a fringe maximum to occur is given by Fowles Eq. (4.10): HRH = nT ET . (40)

4 The total E and H just to the left of the left-hand interface are:
2N = nd cos + r . (34)
ELH = E0 + E0
Use the small angle approximation: HLH = n0 E0 n0 E0 . (41)

4 2 When the relations in Eqs. (40) and (41) are substituted into Eq. (39), we nd
2N = nd 1 + r . (35)
2 that:
   
ELH ERH
We are told that the zeroth order fringe (N = 0) has zero radius ( = 0), so we =M (42)
HLH HRH
have:
4 Therefore the overall transfer matrix Mtot is merely the product of transfer matri-
r = nd (36) ces for the individual lms Mi . This follows from induction. Suppose there are n

lms with transfer matrices M1 , M2 , ..., Mn . Let the elds to the right of the last
and subsequently: lm be En and Hn . The elds just to the left of the nth lm are:
   
4 2 En1
= Mn
En
(43)
2N = nd . (37) Hn1 Hn
2

September 30, 1999


Physics H7C Fall 1999 Solutions to Problem Set 5 Derek Kimball
th
the elds just to the left of the (n-1) lm are:
     
En2 En1 En
= Mn1 = Mn1 Mn (44)
Hn2 Hn1 Hn

and so on...
This argument leads to Fowles Eq. (4.28) as stated.

September 30, 1999


1

University of California, Berkeley


Physics H7C Fall 1999 (Strovink)
PROBLEM SET 6

Problems 1-4 are suitable review problems for (i.e. closer to the half-silvered mirror). Show
Midterm 1. Problems 5-8 involve new material your calculation for V.
beyond the range covered by Midterm 1.
5.
1. Two identical horizontal thin slits in a black
Model an animals eye as a sphere composed of plate are centered at y = h2 , where y is the
vitreous humor, backed by a retina. If the eye is vertical coordinate. A screen with vertical coor-
focused at innity, what is the refractive index dinate Y is located a distance D downstream. If
of the humor? an analyzer is present, it is located just upstream
of the screen. Fraunhofer conditions apply, i.e.
2. A point source of isotropic light is located at kh2  D, and small-angle approximations can
the center of a small hemispherical hole in the be made, i.e. |y|  D, |Y |  D. Plane wave A is
plane end face of a cylindrical light guide with normally incident on the top slit and plane wave
refractive index n = 2, permeability = 0 . B is normally incident on the bottom slit, with
EA [(x + iy)ei(kzt) ]
What fraction of the light emitted can be trans-
mitted a long distance (relative to its radius) by and
the light guide? (A number is required.) EB [(x iy)ei(kzt) ] ,
with  denoting the real part. When either slit
3. is blocked and no analyzer is in place, the inten-
Using a combination of optical devices (polar- sity I(Y = 0) I0 . When neither slit is blocked,
izers, wave plates...), design an optical system nd I(Y )/I0 , where I0 is dened above, for the
that will pass right-hand circularly polarized following cases:
light without changing its polarization, but will (a.)
completely block left-hand circularly polarized No analyzer is in place.
light. This system is called a right-hand circu- (b.)
lar analyzer. Use Jones matrices to prove that The analyzer accepts only y polarized light.
your design will work. (c.)
The analyzer accepts only right-hand circularly
4. polarized light E [(x iy)ei(kzt) ].
A Michelson interferometer produces fringes on
its screen (which is not quite perfectly aligned). 6.
It is fed by laser light polarized out of the inter- Prove that
ferometer plane along z. With equal path lengths N
 sin N /2
the fringe visibility V (Imax Imin )/(Imax + exp (in ) = exp (i),
Imin ) is unity. n=1
sin /2
(a.)
An ideal linear polarizer, with transmission axis where
oriented at 90 to z, is placed in one leg of the n+1 n ,
interferometer. What is V? Explain. and is the average of the n .
(b.)
Same as (a.) except that a second linear polar-
izer, with transmission axis oriented at 45 to z, 7.
is added in the same leg upstream of the rst A plane wave of wavelength is incident on (A)
2

no screen, i.e. all the light passes through; (B)


a black disk of radius R; (C) a black screen
with a circular hole of the same radius R. The
relative intensities seen by an observer on the
axis at distance D downstream are in the ratio
IA : IB : IC = 1 : 1 : 0. Fraunhofer conditions do
not apply to this geometry, although the obliq-
uity and 1/r factors do not vary appreciably
across the screen.
(a.)
Find the smallest possible value of R that is con-
sistent with the above conditions, expressed in
terms of D and .
(b.)
In this problem the screen aperture functions gB
for case (B) and gC for case (C) sum together to
give the aperture function gA for case (A). For
the particular R that you obtained for part (a),
the intensities IB and IC also sum together to
give IA . For what other choices of R would that
be true? Explain.

8.
A plane electromagnetic wave propagates in the
z direction within a good conductor (EM wave
(= skin depth) plasma ). Evaluate the total
power lost per square meter due to Joule (ohmic)
heating in the region 0 < z < . Show that this
is equal to the average value of |S| at z = 0. You
may take = 0 for this conductor.
Physics H7C Fall 1999 Solutions to Problem Set 6 Derek Kimball
At the question period after a Dirac lecture at the University of Toronto, somebody where n = 2 is the refractive index. The percent of light transmitted (intensity) is
in the audience remarked: Professor Dirac, I do not understand how you derived T = |t|2 = 4/9 in our case.
the formula on the top left side of the blackboard.
This is not a question, snapped Dirac, it is a statement. Next question, please. n1
- George Gamow, excerpted from Thirty Years that Shook Physics, a very fun book
on the people involved in the early development of quantum mechanics.

If you have any questions, suggestions or corrections to the solutions, dont hesitate
to e-mail me at dfk@uclink4.berkeley.edu! n2

Problem 1

The eye is focused at innity, so we assume the rays incident on the eye are all
parallel (Fig. 1). From the geometry of the diagram in Fig. 1 it is clear that
= 2. Snells law demands that sin = n sin where n is the refractive index
of the humor. We can make the small angle approximation (making the realistic Figure 2
assumption that light passes only through a small iris in the center of the front of
the eye) and just say that = n, which gives us n = 2. Next consider the diagram in Fig. 2. We require that sin1 (1/n) for total
internal reection. From geometry this demands that cos1 (1/n). We can
now integrate to nd the total solid angle of light accepted into the light guide:
eye
 2 
= sin dd = 2(1 cos ) = 2(1 1/n) (3)
= 2 0 0

The percent of light accepted is then /4, or 1/4. So then the fraction of light
that travels an appreciable distance is given by the fraction of light transmitted
Figure 1 through the interface in the correct direction which is 1/9.

Problem 3
Problem 2
We start out with right-hand circularly polarized light and send it through a
All light coming from the point source is normally incident on the surface of the quarter-wave plate with the fast axis vertical:
hemispherical hole in the end of the light guide. Thus the amplitude of the electric
     
eld of transmitted light Et is given by (from Strovink and/or Fowles): 1 0 1 1
= . (4)
Et 2Z2 0 i i 1
= , (1)
E0 Z2 + Z1
 Next, we send the light through a linear polarizer with the transmission axis at
Et
where Z1,2 = 1,2 / 1,2 . Thus the transmission coecient t = E 0
is given by: 45o :
     
2 1 1 1 1 1
t= , (2) = . (5)
1+n 2 1 1 1 1

October 14, 1999


Physics H7C Fall 1999 Solutions to Problem Set 6 Derek Kimball
So we have 100% transmission for right-circularly polarized light. So now at the output we have two waves from the dierent arms of the interfer-
ometer (supposing light is propagating in the x direction):
For left circularly polarized light, no light is transmitted:
         1 = E0 z + E0 y
E
1 1 1 1 0 1 0 4 4
= . (6)
2 1 1 0 i i 0 E 2 = E0 ei z, (9)
So the right hand circular analyzer works as claimed. where is the phase dierence induced by the diering path lengths for the arms
of the interferometer.
Problem 4 Thus the intensity of light at the output is given by:
 2
E0 E2
I=2 + E02 + 2 0 cos (10)
(a) 4 4

If a linear polarizer at 90o to z (the direction of light polarization) is placed in a leg so we see that for Imax and Imin :
of the Michelson interferometer, no light travels in one leg of the interferometer. 9 E2
Then there will be no fringes and no interference, so V (Imax Imin )/(Imax + Imax,min = E02 0 . (11)
8 2
Imin ) = 0. This is clear since Imin = Imax if there are no fringes.
Using these results in our equation for fringe visibility we nd that V (Imax
(b) Imin )/(Imax + Imin ) = 4/9
Now, with a linear polarizer at 45o to z upstream of the rst linear polarizer,
there is light transmitted in both legs of the interferometer. Interference will not Problem 5
occur for light of orthogonal polarizations, so only light polarized in the z direction
contributes to the fringes.
(a)
The linear polarizer at 45o transmits:
We have two light beams, whose electric eld amplitudes are given by:
     
1 1 1 1 E 0 1
E0 = . (7) EA Re[(x + iy)ei(kzt) ]
2 1 1 0 2 1
and
The next polarizer only transmits light in the orthogonal direction, so the trans-
EB Re[(x iy)ei(kzt) ] ,
mitted light is given by
 
E0 0 with Re denoting the real part. We are not given the slit widths, so lets assume
. that they are small enough to be ignored in our analysis...
2 1
The light bounces o the mirror, which preserves polarization, passes through With no analyzer in place, the two beams are orthogonally polarized so there is
the second polarizer with no loss of amplitude, then passes through the polarizer no interference. Thus the intensity at the screen is simply the sum of the two
at 45o (which now appears to be at -45o with respect to the direction of light individual intensities, which with small angle approximations is roughly I 2I0 .
propagation):
(b)
     
1 1 1 E0 0 E0 1
= . (8) With an analyzer that accepts only y polarized light, the two light beams have the
2 1 1 2 1 4 1 same polarization after the analyzer and then can interfere. The interference is

October 14, 1999


Physics H7C Fall 1999 Solutions to Problem Set 6 Derek Kimball
that of a typical double slit experiment (Youngs experiment), as solved in Fowles as the dierence of two innite geometric series:
pp. 59-61. Of course, half the amplitude of each wave has been removed by the
N N
analyzer, so the resulting interference pattern is given by:
exp (in ) n exp (i1 ) exp (in)
   n=1 n=0
I0 hY
I(Y ) = 1 + cos . (12)  
4 D = n+1 exp (i1 ) exp (in) N +n+1 exp (i1 + N ) exp (in)
n=0 n=0
(1 x) exp (i1 ) (1 x)(N +1) exp (i1 + N )
=
1 (1 x) exp (i) 1 (1 x) exp (i)
(c)
(13)
The analyzer blocks out left-hand circularly polarized light so there is contribution
only from EB . Therefore the intensity at the screen is I I0 . We now let x go to 0, and we then have:
N
 exp (i1 ) exp (i1 + N )
exp (in ) =
Problem 6 1 exp (i) 1 exp (i)
n=1
  
N 1 exp (iN /2) exp (+iN /2)
= exp 1 + (14)
We want to prove that 2 exp (i/2) exp (+i/2)

N from which we deduce that:


 sin N /2
exp (in ) = exp (i), N
sin /2  sin N /2
n=1 exp (in ) = exp (i) (15)
n=1
sin /2
where
proving the original conjecture.
n+1 n ,

and is the average of the n . Problem 7


 a
Recall that for a geometric series 0 arn = 1r , where |r| < 1. Let us rewrite the As discussed in Fowles pp. 126-128, this problem can be solved using Fresnel zones.
sum above as the sum of two innite series. In order to use the geometric series The radius of the N th Fresnel zone in this case is given by:
formula, lets multiply each term in the series by an amplitude n = (1 x)n
which ensures that |n exp (in )| < 1. We can require that the n s are so close R = N D (16)
to unity that they are very well approximated by 1 for the rst N terms. We are
only worried about the convergence of the tail of the series, which is taken care of since the factor L = (1/h + 1/h )1 = D in our case (see Fowles Eq. 5.36).
with this postulate. We could even take the limit as x 0 to make this argument
more mathematically sound. (a)

Now lets write the series (C) The smallest radius for which the light from Fresnel zones cancel is that which
N includes the rst two. In this case the optical disturbance is given by

exp (in )
n=1
Up = |U1 | |U2 | 0.

October 14, 1999


Physics H7C Fall 1999 Solutions to Problem Set 6 Derek Kimball
(A) With no screen, the optical disturbance is half that due to the rst Fresnel The power lost per square meter due to ohmic heating is given by the relation:
zone, 
1
Up = |U1 |. P  = E 2 dV
2

2 2
(B) If we block out the rst two Fresnel zones, the optical disturbance is approxi- = |E0 | exp ( z) = E02 (21)
mately half that due to the third zone, or 2 2 2

1 1  is given by  1 E
Up = |U3 | |U1 |. The average value of |S|
 B.
 Plugging in values from above, we
2 2
nd
So as you can see, the choice of
 =
|S|
2
E2 (22)
R= 2D 2 0
satises all the required conditions. which conrms the conjecture stated in the problem.

(b)
This will hold whenever a similar situation occurs, i.e. an even number of Fresnel
zones are blocked by the black disk. The optical disturbances UA and UB will
always sum to equal the optical disturbance without the screen U0 because they
are complementary apertures. However, only when either UA or UB is zero can
the squares ( I) be equal.

So whenever R = 2nD where n is an integer, this is the case.

Problem 8

From Maxwells equations in conducting media, we get the wave equations:

2 E(z,
 t) = E0 ei(zt)
2 B(z,
 t) = B0 ei(zt) (17)

where

2 = 2 + i. (18)

For a good conductor,




= (1 + i). (19)
2
Also from Maxwells equations for a conducting medium, we nd that:
||
B0 = . (20)

October 14, 1999
1

University of California, Berkeley


Physics H7C Fall 1999 (Strovink)
PROBLEM SET 7

1. blackbody radiation from the dirt is totally ab-


Fowles 6.1. sorbed by the glass and reradiated (half in and
half out). Assuming that the temperature out-
2. side the greenhouse is 300 K, estimate the tem-
Fowles 6.2. perature inside it.

3. 8.
Fowles 6.6. Nuclear winter (inverse Greenhouse eect).
According to some experts (though this is con-
4. troversial), after nuclear war a thin layer of dust
Rohlf 2.13. would remain in the upper atmosphere of the
Note that this problem is more dicult than it earth. To a rst approximation, the dust ab-
rst appears to be. You may not assume that sorbs all light from the sun, which is near visible
the average number of unexpected events found, in wavelength. The dust then reradiates that en-
per 106 interactions, is equal to unity! ergy in the infrared, to which it is transparent:
half in toward the earth, half out to space. The
5. dust is nearly transparent to the earths outward
Rohlf 2.38. radiation, also in the infrared.
If the peacetime surface temperature of the earth
6. is 300 K, what would that temperature become
Assume that the sun is a blackbody of tem- after nuclear war? To relate to it physiologically,
perature 5800 K and radius 7 108 m, located express this latter temperature in F.
1.5 1011 m from the earth. Assume further
that the earth is a gray body which absorbs part
of the radiation incident upon it from the sun,
and then reradiates it isotropically. Neglect any
other eects which could heat the earth. Calcu-
late the surface temperature of the earth under
these assumptions.

7.
Greenhouse eect.
Take the suns blackbody spectrum to have
its peak in the yellow ( = 0.58 m). Take
Tsun = 5800 K and Tearth = 300 K.
(a.)
Use Wiens Law (max T 1 ) to estimate the
wavelength at the peak of the earths blackbody
spectrum.
(b.)
Imagine that the (visible) suns rays pass through
the clear glass of a greenhouse. On the oor of
the greenhouse is black dirt with  1, which
absorbs these rays. In contrast, the (infrared)
Physics H7C Fall 1999 Solutions to Problem Set 7 Derek Kimball
Above the front door of Niels Bohrs cottage was nailed a horseshoe. A visitor who equations break down in the vicinity of the resonance, which is where we must
saw it exclaimed: Being as great a scientist as you are, do you really believe that work to solve this problem. So here we bring back the 2 2 terms, but continue
a horseshoe above the entrance to a home brings good luck? to assume there are few electrons. In this case our formulas for and n are given
No, answered Bohr, I certainly do not believe in this superstition. But you by:
know, he added with a smile, they say that it does bring luck even if you dont
believe in it! N e2 02 2
n1+ (4)
2m0 (02 2 )2 + 2 2
- George Gamow, excerpted from Thirty Years that Shook Physics.
and
If you have any questions, suggestions or corrections to the solutions, dont hesitate
N e2
to e-mail me at dfk@uclink4.berkeley.edu! . (5)
2m0 (02 2 )2 + 2 2

Problem 1 If we take the derivative of n with respect to and set it equal to zero, we nd
two positive roots yielding the values for the max and min of the the function n,
This problem makes sense only if you make some rather poorly motivated ap- namely

proximations. In particular, we must assume that we are far away from resonance = 1 /0 .
0
(namely that 02 2  ). We also can assume that there are very few electrons
e2 It is safe to assume, since damping is small, that this value can be approximated
(namely that N m0  1, which is well-motivated by the fact that is much less than
n, i.e. few absorbers). If we make these approximations, the results follow almost by the rst order Taylor expansion:
immediately. If you dont make these assumptions, then the results are clearly = 0 /2.
incorrect (see Figure 6.1 in Fowles, which is nothing like the equations Fowles asks
us to derive). Thus, well make these assumptions! If we plug these values into our expression for (Eq. 5), we see that these values
are those where attains half its maximum value.
Then we can apply these approximations to equations 6.34 and 6.35 in Fowles. We
nd that:
Problem 3
2
N e 1
n2 2 n2 1 + (1)
m0 02 2 We are given that = 6.8 107 mho/m and that Ne = 1.5 1028 electrons/m3 .
Using these values in the appropriate Fowles formulas gives us the desired an-
Using a rst order Taylor expansion, we then nd that: swers...
N e2 1
n1+ (2) (a)
2m0 02 2
Plasma frequency
Since n is approximately 1, is given by: 
N e2
Ne 2
p = = 6.9 1015 s1
. (3) m0
2m0 (02 2 )2

(b)
Problem 2 Relaxation time

Once again Fowles attempts to confuse us by implying the above results can be 0 c2
= = 1.6 1013 s
applied in the solution to this problem when they cant. This is because the above p2
October 21, 1999
Physics H7C Fall 1999 Solutions to Problem Set 7 Derek Kimball
(c) our condence in the value of a we have measured and then convolve it with the
probability for seeing a second event.
Our frequency with a wavelength of 106 m is given by
2c The probability density function, in this case that for Poisson statistics, is given
= = 1.9 1015 s1 . by:

Real and imaginary parts of the index of refraction can be derived from from ea ax
fp (x) = (6)
Fowles Eqs. 6.55 and 6.56: x!

p2 where x is a non-negative integer and a is the average value of x. The probability


n2 2 = 1 density function (with known parameter a) allows us to predict the frequency with
2 + 2
which random data x will take on some particular value.
1 p2
2n =
2 + 2 We rst want to calculate a value for a = np (where n is the number of interactions
and p is the probability for an event), a distribution function based on the most
Clearly, p ,  1 , so we get likely values for a and our condence in those values. The most likely value of p
 2 from the data is 106 .
p
n2 2 1 = 12.2 For a conservative upper limit (without assuming very much about the prior prob-
ability distribution), we can estimate that the lower limit of p (at a 95% condence
1 p2 level) must be that for which the probability of seeing one event in 106 interactions
2n = 0.044. is at least 5%:
2
We can then solve these equations for n and , and with a little algebra we get: fp (1) = enp np 0.05, (7)

n = 0.006 which tells us that np 0.05, or p 5 108 . Furthermore, we know that the
upper limit at 95% condence level on p can be found from:
= 3.5.
fp (1) = enp np 0.95, (8)
(d)
which tells us that p 5.14 106 .
The reectance is given by the Hagen-Rubens formula,
 We want to be 90% sure well see a second event. We could guess that if were
80 95% sure p is bigger than pmin = 5 108 and 95% sure that well see at least one
R=1 1.
more event after n2 interactions using this value for p, well be 90% sure to see a
second event. Were 95% sure well see at least one more event if the probability
to see zero events is less than 0.05:
Problem 4
fp (0) = en2 pmin 0.05,
This problem, as Prof. Strovink pointed out, is a little bit tricky. Our experimenter
nds 1 event in 106 interactions. Now we want to be 90% sure we nd a second which gives us
event. How many interactions do we need? The basic problem is that we dont n2 6 107 interactions.
really know the average number of events we should see per 106 interactions, which
is needed to calculate how many more interactions are necessary to be 90% sure This is a conservative upper limit on the number of interactions we need before
well see a second event. So well have to try to gure out some function describing well see another event.

October 21, 1999


Physics H7C Fall 1999 Solutions to Problem Set 7 Derek Kimball
The above analysis gives us some idea of what our likelihood distribution L We can take the derivative of N with respect to time to obtain:
for p looks like... it is peaked at 1 106 and nears zero at both 5 108 and
5.14 106 . It is well described by the function dN P dV
= . (10)
dt kT dt
p p/p0
L(p) = e We know from the statement of the problem that the number of CO2 molecules is
p0
proportional to the surface area of the beer bubbles,
where p0 = 106 is the most likely value for p. It is not an accident that this
looks exactly like fp (1). This, as Prof. Strovink explained to me, is just Bayes dN
= Cr2 .
assumption of a uniform prior probability distribution meaning that we assume dt
we found the most likely value of p0 in our experiment and the distribution of Also, assuming a spherical bubble, we know that
probabilities is that from, in our case, Poisson statistics.
dV dr
A little more mathematical rigor can be applied if we take our likelihood distri- = 4r2 .
dt dt
bution L for p and convolve it with the restriction that the probability for zero
events must be less than 10%. This approach yields more or less the same result Plugging these into Eq. (10), we nd that:
as Rohlfs answer, which is reasonable since in the Bayesian approach we assume
a prior probability distribution with p0 = 106 as the central value. Rohlf just dr
=
CkT
, (11)
assumed that a priori we knew the probability for an event to occur would be dt 4P
p0 = 106 . If we try this approach, we nd that: which indicates the radius of the bubble increases linearly with time.

L(p)en2 p dp
0
 0.1 Problem 6
0
L(p)dp
Plugging in our assumed prior probability distribution or likelihood distribution The power given o by the sun is S (power radiated per unit area) times the
and using the substitution u = (1/p0 + n2 )p in the numerators integral, we nd: surface area of the sun, which is:

1 p p/p0 n2 p
e e dp = 2
1
0.1. PS = (TS4 ) (4RS2 ) (12)
2
p0 0 p0 p0 (1/p0 + n2 )
The portion of this power received by the earth is scaled down by the emissivity
From which we can calculate n2 = 2.16 106 for 90% CL that we will see a second factor  (earth is treated as a gray body) and the cross-sectional area of the earth
event. This is far smaller than our original rough estimate, but assumes a prior over the surface area of a sphere with a radius equal to the distance between the
probability distribution. This is probably the more correct approach. earth and the sun:
Well, as you can probably tell, this problem was quite dicult for me, so dont 2
RE
(in)
feel too bad if you had some trouble as well... PE = (TS4 ) (4RS2 ) 2 (13)
4RES

Problem 5 The power re-radiated by the earth is given by:


(out)
As evidenced by the sampling of problems from Rohlf, we can guess his two main PE = (TS4 )(4RE
2
). (14)
interests are particle physics and beer.
(in) (out)
In equilibrium, PE = PE . If we equate these expressions, we nd that:
The expression for the number of particles from the ideal gas law is:
PV TE4 1 RS2
N= . (9) 4 = 2 , (15)
kT TS 4 RES

October 21, 1999


Physics H7C Fall 1999 Solutions to Problem Set 7 Derek Kimball
from which we deduce that Since we only get half the light power from the sun that we used to down on the
TE = 290 K. earth, the new temperature on earth TE is given simply by:
 1/4
1
Problem 7 TE = TE = 252 K = 5.3 o F.
2

Brrrr......
(a)
We apply Wiens law to get the peak of the earths blackbody spectrum. First,
the constant can be derived from plugging in the known parameters for the sun:
C
max =
T
C = max,sun TS = (0.58 m)(5800 K) = 3364m K.

Applying Wiens law to the earth, we get:


C
max,earth = = 11.2 m.
TE

(b)
If half the power re-radiated from the dirt is radiated back into the greenhouse at
every interface with the walls, in equilibrium the power radiated by the dirt should
be twice that incident from the sun. The power from the sun hitting the dirt of
the greenhouse is:

(dirt) Area of dirt


PS = (TS4 )(4RS2 ) 2 , (16)
4RES

The power re-radiated by the dirt is:


(dirt)
Pout 4
= Tdirt (Area of dirt) (17)
(dirt) (dirt)
Setting 2Pout = PS , we nd that:

4 TS4 RS2
Tdirt = 2
2 RES

Tdirt = 333 K.

Problem 8
October 21, 1999
1

University of California, Berkeley


Physics H7C Fall 1999 (Strovink)
PROBLEM SET 8

1. to the correspondence principle, when n is very


(a.) large this should agree with a proper quantum
Consider a distant star with the same luminosity mechanical calculation.)
and surface temperature as the sun. A per- (b.)
son (who is as ecient as one of Rutherfords The decay rate for an electron in an orbit may
graduate students) can see the star if 250 visi- be dened to be the power radiated, P , divided
ble photons per second pass through her pupil, by the energy emitted in the decay. (The de-
which has a radius of 2 mm when nearly fully di- cay rate is the inverse of the lifetime). Use the
lated. What is the maximum distance at which Bohr theory expression for the energy radiated,
the star is visible to the naked eye? and the expression for P from part (a.) to cal-
(b.) culate the correspondence value of the decay
How many cosmic photons per second per square rate when the electron makes a transition from
cm were incident on the Nobel-Prizewinning mi- orbit n to orbit n 1. What is the value of this
crowave antenna of Penzias and Wilson? decay rate when n = 2? (This will not agree ex-
actly with the true quantum theory, since the
2. correspondence principle will not hold when n
(a.) is not  1.) What is the decay rate when the
The maximum energy of photoelectrons from transition is from an orbit n to an orbit n m?
aluminum is 2.3 eV for incident radiation of 0.2 (c.)
m and 0.9 eV for radiation of 0.313 m. Use Use the value of the lifetime of an electron
these data to calculate Plancks constant and in an n = 2 Bohr orbit, calculated in part (b.),
the work function of aluminum. to estimate the uncertainty in the energy of the
(b.) n = 2 energy level. How does it compare with
An aluminum photocathode receives incident ra- the energy of that level?
diation of 0.313 m. When the intensity of this
radiation is 1 mW, a current of 1 A is observed 5.
in a circuit that detects the photoelectrons that Rohlf 3.42.
are liberated. Estimate the quantum eciency Do the rst part of the problem. Then answer
of the photocathode. the nal question posed in this problem for two
extreme cases:
3. (a.)
Rohlf 3.20. The muon capture probability is of the same or-
der of magnitude as the decay probability.
4. (b.)
(a.) The muon capture probability is many orders of
The power radiated by an accelerated charge e magnitude smaller than the decay probability.
is given in classical physics by the formula

1 2e2 2 6.
P = a (SI units), There exists a fundamental constant of nature O
40 3c3
whose value is 25,813 ohms. Calculate the ratio
where a is the acceleration. Using this formula, between the numerical value of O and the numer-
calculate the power radiated by an electron in a ical value of Z0 , the characteristic impedance of
Bohr orbit characterized by the quantum num- free space. Using any hint that you can obtain
ber n. No numbers are required. (According from this ratio, determine the algebraic value
2

of O, expressed in terms of other fundamental


constants.

7.
Rohlf 3.56.

8.
Rohlf 4.54.
Physics H7C Fall 1999 Solutions to Problem Set 8 Derek Kimball
Stop talking and write down the Hamiltonian! (b)
- I. B. Khriplovich, Novosibirsk State University (Russia), during a seminar at The cosmic background radiation lls the universe roughly isotropically (there is
Berkeley. no solid angle suppression), and the temperature of the radiation is T = 2.74 K.
The incident number of photons per square cm on Penzias and Wilsons antenna
If you have any questions, suggestions or corrections to the solutions, dont hesitate is given by an equation similar to Eq. (3):
to e-mail me at dfk@uclink4.berkeley.edu! 
2c
n= d 4 hc/kT = 2.6 1012 photons s1 cm2 . (4)
0 (e 1)
Problem 1

Problem 2
(a)
First we determine how many visible photons are emitted from the surface of the
sun-like star. The radiated power per unit area per unit wavelength dR (a)
d is given
by the Planck distribution: The maximum energy of a electron ejected by the photoelectric eect is given by:
dR 2hc2 hc
= 5 hc/kT . (1) , (5)
d (e 1)

We want to convert this quantity into the number of photons per second per unit where is the work function. We have two data points to use in this relation,
area per unit wavelength dn dR with which we can determine h:
d , which can be done by dividing d by the energy per
photon: hc
= 2.3 eV
hc 0.2m
Ephoton =  = . (2)
hc
= 0.9 eV
We integrate over the visible spectrum to get photons per second per unit area, 0.313m
using T = 5800 K for the temperature of the sun-like star:
Subtracting these equations and solving for h gives us
 700 nm
2c
n= d = 6.3 1025 photons s1 m2 . (3) h 2.6 1021 MeV s.
400 nm 4 (ehc/kT 1)
Substituting this value of h back into one of the photoelectric eect equations gives
The total number of photons given o by the sun per second N is n times the us the work function
surface area of the star, which is roughly 4RS2 = 6 1018 m2 . This gives us = 1.58 eV.
3.81044 photons per second from the star! We scale this by the solid angle
subtended by the observers eye,
(b)
2
reye
2 ,
Quantum eciency of the photocathode is just the ratio of emitted electrons to
4RSE incident photons. The number of photons is the power of light over the average
energy per photon at this wavelength:
and solve for RSE such that the eye receives at least 250 photons. This gives us
P
max
RSE 1019 m 1300 light years. N = = 1.6 1015 photons/sec.
hc
October 28, 1999
Physics H7C Fall 1999 Solutions to Problem Set 8 Derek Kimball
The number of electrons is just the current over the charge per electron: which gives us:
   
I 8hc d
Ne = = 6.2 1012 electrons/sec. n= d = 8 . (11)
qe 0 hc 5 (ehc/kT 1) 0 4 (ehc/kT 1)
Taking the ratio gives us the quantum eciency: Making the appropriate substitution x hc/kT and dx = kT hc d
2 , we get the
desired result:
QE = 0.0039 = 0.39%.
 3 
kT x2
This was using the correct value for h. If you used the value of h you obtained n = 8 dx x (3.17 1019 eV3 m3 )(kT)3 . (12)
from part (a) of this problem, you would nd hc 0 e 1

QE = 0.0025 = 0.25%.
(c)
The average energy per photon is u/n. u = 4R/c can be found from the Stefan-
Problem 3 Boltzmann law (the factor of 4 comes from averaging over all angles, Rohlf Eqs.
(3.17) and (3.18)),

(a) 4  (kT )4
u = 4R/c = . (13)
c
For thermal radiation, the average energy per photon is given by:
 Therefore, the average energy per photon is found from the ratio of Eqs. (13) and
1 dn
E = dE E (6) (12) to be:
n 0 dE
u 4  (kT )
The energy per unit volume u is given by: E = = . (14)
n c (3.17 1019 eV3 m3 )

dn
u = nE = dE E (7) Plugging in the constants gives us:
0 dE
du dn E 2.7 kT. (15)
We can employ the fact that from Eq. (7), dE = E dE , which gives us

du dE dn dE
=E (8) (d)
dE d dE d
For the number density of photons, we obtain from Eq. (12):
This reduces to our desired result, simply that:
du dn hc dn n = (3.17 1019 eV3 m3 )[(8.62 105 eV/K)(2.74 K)]3 4 108 m3 .
=E = . (9)
d d d For the energy density, we from Eq. (15) we nd that

(b) E 0.64meV.

The total photon density n is given by:


 
dn du
n= d = d , (10)
0 d 0 hc d

October 28, 1999


Physics H7C Fall 1999 Solutions to Problem Set 8 Derek Kimball
The decay rate is then the ratio of the radiated power (we assume that the
Problem 4 electron is in the nth orbit until the moment it decays) to the energy dierence
between the levels:
P
(a) = . (24)
E
Acceleration a for a circular orbit is given by:
If we plug in our result from part (a) and use n = 2, we get
v2
a= .
r
(16) = 108 s1 .

Angular momentum L, applying the Bohr quantization condition, is given by:


If the decay is from the nth level to the (n m)th level, we merely adjust the
L = mvr = n. (17) energy dierence in Eq. (23):
 
Solving for v from Eq. (17) and substituting into Eq. (16), we obtain: 1 1
E = 13.6 eV 2 . (25)
2 2 2
n (n m)2
L n 
a = 2 3 = 2 3. (18)
m r m r and employ this equation in Eq. (24). Qualitatively, we see that if the energy dif-
ference is greater and the power radiated is the same, the decay rate will decrease.
The Bohr radius r is given by:
This is an example of the limitations of the Bohr model, since although it correctly
n2 2 predicts the order of magnitude of the transition rates it does not correctly predict
r = (40 ) 2
. (19) the dependence of transition rates on the energy dierence between levels, which
me
actually scales as 3 .
Using Eq. (19) in conjunction with Eq. (18), we nd an expression for a in terms
of fundamental constants: (c)
2 2
 3 6
n 
(20) There is an energy time uncertainty principle, which can be derived from xp
1 me
a= 2 3 = .
m r 40 n4 4 /2 in the following hand-waving fashion:
Using Eq. (20) in the classical expression for radiated power gives us: p  m 
Et = p x = xp.
 7 m p
1 2e2 2 1 2 m2 e14
P = a = . (21)
40 3c3 40 3 c3 n8 8 We use the lifetime (1/) as the uncertainty in time, and then nd for the uncer-
tainty in energy:
(b) 
E = (26)
The energy En in the nth level of the Bohr atom is given by: 2

2 mc2 13.6 eV The value of E is (108 s1 ) (197.3 MeV fm(3 1023 fm/s)1 ), or 6 108 eV.
En = 2
= , (22) The energy dierence between the rst and second levels in hydrogen is 10 eV, so
2n n2
the linewidth is smaller than the energy dierence by nine orders of magnitude!
so the energy radiated in a n n 1 transition is:
 
1 1
E = 13.6 eV 2 . (23)
n (n 1)2

October 28, 1999


Physics H7C Fall 1999 Solutions to Problem Set 8 Derek Kimball

Problem 5 Problem 7

In calculations involving the Bohr model, the electron mass is replaced by the
reduced mass of the muon-proton system, which is near the mass of the muon: (a)
The typical electron velocity in the Bohr model is v = c, for the deuteron we
m mp
m= = 95 MeV. replace by s . So we have v = s c = 3 107 m/s for both the proton and
m + m p
neutron.
Energy levels in the Bohr model are linear with respect to the electron mass, and
(b)
are given by
The reduced mass in the deuteron is roughly mp /2, so the nuclear Bohr radius
13.6 eV 95 MeV
En = (27) r is given by:
n2 0.5 MeV 2c 2 197.3 MeV fm
r= = 4 fm.
mp c2 s 0.1 938 MeV
for the muon-proton system.

(a) (c)
1 2 2
A free muon decays with a characteristic lifetime 2.2 106 s, primarily in the The binding energy of the deuteron is roughly 2 s mc = 2 MeV.
mode:
e + e + . Problem 8
If the capture probability was large, it would make the lifetime shorter compared First, we equate the relativistic centripetal force to the electrostatic force acting
to the lifetime of a muon (at rest), since the two rates would add. on the electron:
ke2 mv 2
(b) = , (28)
r2 r
If the capture probability was small enough to be neglected, then the eect of
and proceed to solve for the radius:
time dilation would lengthen the lifetime of the muon (since in the muon-proton
system, the muon has a characteristic velocity c), compared to a muon at rest. ke2
r= . (29)
mv 2
Problem 6
We can also use the Bohr quantization condition
If O is divided by the characteristic impedance of free space Z0 , we get (2)1 . pr = n
So
Z0 in conjunction with the relativistic expression for the momentum
O= ,
2
p = mv
probably.
to nd the radius:

r= . (30)
mv
Setting Eqs. (29) and (30) equal, we can solve for the velocity, and we obtain the
desired result v = c.

October 28, 1999


1

University of California, Berkeley


Physics H7C Fall 1999 (Strovink)
PROBLEM SET 9

1. What is the expectation value H of the par-


Rohlf 5.31. ticles kinetic energy? (Cogent arguments can
substitute for some algebra here, and are encour-
2. aged.)
Rohlf 6.12. (d.)
At what time t0 is the probability for the par-
3. ticle to be located on the right-hand side of
Rohlf 6.29. the well (L/2 < x < L) a maximum? Reason-
ing rather than detailed calculation is needed.
4. Please supply it.
Rohlf 6.32.

5.
Rohlf 7.4.

6.
Rohlf 7.5.

7.
Show that the conservation of probability law

 
+ j = 0, where
t x
h  
j,
2mi x x
and j is the probability current in one dimension
x, holds if (x, t) is a solution of the Schrodinger
equation with a potential V (x), provided that
V (x) is real. Therefore absorptive potentials are
imaginary.

8. A particle of mass m is bound in an innitely


deep one-dimensional potential well extending
from x = 0 to x = L. At t = 0 it is described by
a wavefunction of the form

u(x) sin (x/L) + sin (2x/L).

(a.)
Normalize u(x).
(b.)
When the particles energy E is measured for
the rst time, what value(s) could be obtained?
(c.)
Physics H7C Fall 1999 Solutions to Problem Set 9 Derek Kimball
When the rules of quantum mechanics were formulated in the 1920s they rep- The strength of the electric force is given by
resented a revolutionary break with the past, and an enormous extrapolation from
experience. Since they were something very new, they could not be derived from ke2 1.44 MeV fm
Fe = 2 = = 0.36 MeV/fm.
something old and incorrect, that is, classical physics. Instead they had to be for- x0 (2 fm)2
mulated by guessing, intuition, and inspiration. Their ultimate justication was,
and is, logical consistency and agreement with experiment. (c)
- Prof. Eugene D. Commins, U.C. Berkeley. The maximum acceleration of the proton is given by:
F kx0 c2
If you have any questions, suggestions or corrections to the solutions, dont hesitate amax = = = 1.3 1029 m/s2 1028 g. (4)
to e-mail me at dfk@uclink4.berkeley.edu! m mc2

Problem 1 Problem 2
d d
It is convenient to write the dierential cross section as d cos instead of d be-
(a) cause it makes integration over solid angles a little easier, since the integral always
involves cos and the dierential solid angle contains the term sin d = d(cos ).
The uncertainty principle yields an estimate for the minimum momentum of a
It is relatively straightforward to show that either method of solving for the total
proton trapped in the nucleus:
cross section gives the same result, since

p . (1) d
=
d

d cos
= sin
d
.
2r d d cos d d cos
For the kinetic energy of the proton, we obtain: If we integrate over all angles, we obtain for the total cross section:
 +1 
(p)2 2 2 c2 197.3 MeV fm d 1 d
Ek = = 2
= = , (2) = d(cos ) = d sin .
2m 8m(r) 8mc2 (r)2 8(938 MeV)(2 fm)2 1 d cos 0 sin d

from which we nd
Ek = 1.4 MeV Problem 3

(b) (a)
Well suppose, for the sake of this back of the envelope calculation, that the ki- The maximum kinetic energy that can be transferred to a gold nucleus in a collision
netic energy found in part (a) can be set equal to the potential energy at maximum with a 6 MeV -particle would be when the collision is head-on and the -particle
displacement in a classical harmonic oscillator: bounces straight back. Because the gold nucleus is very massive compared to the
-particle, the amount of kinetic energy transferred to the gold nucleus should be
1
Ek = kx20 (3) small, so roughly vi = vf where vi and vf are the initial and nal velocities of the
2 -particle. Thus, Mgold V = 2m vi . Using this result in the equation for kinetic
The magnitude of the restoring force at maximum displacement is given by kx0 . energy, we nd:
So we nd:  2  
2Ek 1 1 2m vi 4m 1 44
F = = 1.4 MeV/fm.
2
Mgold V = Mgold = ( m vi2 ) = 6 MeV 0.49 MeV
x0 2 2 M gold M gold 2 197
November 4, 1999
Physics H7C Fall 1999 Solutions to Problem Set 9 Derek Kimball
 2  cos 1
(b) kq1 q2 1
= 2 d(cos )
If we go into the rest frame of the -particle (S  ), we nd that because the - mv 2
cos 2 (1 cos )2
particle is very massive compared to the electron, the energy transferred to the -  2  
kq1 q2 cos 1 cos 2
particle (to the electron in the lab frame) is small. Therefore we can approximate = 2
mv 2 (1 cos 1 )(1 cos 2 )
that in S  , for a head-on collision m V  = 2me vi , where vi is the speed of the
-particle in the lab frame and V  is the recoil speed of the -particle in S  .
Transforming into the lab frame, we nd that the recoil speed of the electron Problem 5
ve = 2vi . So the kinetic energy transferred to the electron is
  A particle is conned to the region L/2 < x < L/2. As discussed in section,
1 4me m vi2 4 (0.511 MeV)
Ek = me (2vi ) = 2
= 6 MeV 3.3 keV. this means that any state (wavefunction) of the particle can be described as a
2 m 2 3730 MeV superposition of eigenfunctions of the energy operator (the Hamiltonian). These
eigenfunctions span the Hilbert space corresponding to our system (a Hilbert
space is an innite dimensional vector space which is a subspace of the vector
Problem 4 space of all continuous complex functions). Since inside the innite potential well
the particle is free, our Hamiltonian H is given by:

(a) 2 2
H= .
2m x2
The relationship between dierential scattering cross section d and the impact
parameter b is given by Rohlf (6.18): Eigenfunctions of H are 
2  nx 
d = 2bdb (5) n = cos
L L
 
The total scattering cross section is derived from this expression: 2 mx 
m = sin
 L L
b1
= 2 bdb = (b21 b22 ). (6) where n = 1, 3, 5... and m = 2, 4, 6.... They have the eigenvalues
b2
n2 2 2
Using Rohlf (6.40) En =
 2   2mL2
kq1 q2 1 + cos
= b2 where n = 1, 2, 3.... These eigenfunctions are orthonormal, meaning that
mv 2 1 cos
 L/2
we nd that the total scattering cross section is given by:
i (x)j (x)dx = 0
 2   L/2
kq1 q2 cos 1 cos 2
= 2 . (7)
mv 2 (1 cos 1 )(1 cos 2 ) if i = j, and
 L/2
i (x)i (x)dx = 1.
L/2
(b)
Integrating explicitly gives us the same result: (a)
 cos 1
d Assume the particle is in an eigenstate of energy. The probability that the par-
= d(cos )
cos 2 d cos ticle is found in the region 0 < x < L/2 is 1/2 by symmetry. This is because

November 4, 1999
Physics H7C Fall 1999 Solutions to Problem Set 9 Derek Kimball
 m/2
the potential is symmetric about x = 0, so every eigenfunction is symmetric or 2L2 u3 u2 sin (2u) sin (2u) 4 cos (2u)
antisymmetic about x = 0. The square of any eigenfunction is symmetric about
x2 = + .
m3 3 6 4 8 4 m/2
x = 0.
L2 L2
It is clear that probability does not depend on n because all of the eigenfunctions
x2 = 2 2.
12 2n
are symmetric or antisymmetric.
For odd n, the procedure is pretty much the same... you even end up with the
(b) same result. 
2 L/2 2
The probability Pc that a particle in the ground state is in the central half of the
x 2
= x cos2 (nx/L)dx.
L L/2
box is given by the integral:
Making the u-substitution u = nx/L, we obtain:
 L/4  L/4
2  m/2
Pc dx|1 |2 = dx cos2 (x/L). (8) 2L2
L/4 L L/4
x = 3 3
2
u2 cos2 udu
m m/2
From which we nd:  m/2
 x=L/4 2L2 u3 u2 sin (2u) sin (2u) 4 cos (2u)
2 x sin (2x/L)
x2
= + .
Pc = + = 0.82. m3 3 6 4 8 4 m/2
L 2 (4/L) x=L/4
L2 L2

x2 = 2 2.
The probability decreases with n, and at large n approaches the classical limit 12 2n
Pc = 0.5.
Taking the limit as n , we see that the rms value of x approaches L/ 12.

Problem 6
Problem 7

We intend to prove the conservation of probability law:


(a)


The average value (expectation value) of x2 as a function of n is given by: + j = 0, where
t x
 L/2   

x2 = dxn x2 n . (9) j,
L/2
2mi x x
and j is the probability current in one dimension x, where (x, t) is a solution of
We use the eigenfunctions discussed in problem (5), solving rst for even n. In the Schrodinger equation with a real potential V (x).
this case
x2 is given by:
We can start with the time-dependent Schrodinger equation:
 L/2  
2 2 2

x2 = x2 sin2 (nx/L)dx. + V (x) (x, t) = i (x, t). (10)
L L/2 2m x 2 t

Making the u-substitution u = nx/L, we obtain: Then take the complex conjugate of (10):
  
2L2 m/2 2 2

x2 = u2 sin2 udu + V (x) (x, t) = i (x, t). (11)
m3 3 2m x2 t
m/2

November 4, 1999
Physics H7C Fall 1999 Solutions to Problem Set 9 Derek Kimball

Now multiply (10) by (x, t) and (11) by (x, t), then subtract the equations. which correspond to the energy eigenvalues:
We obtain:
      n2 2 2
 2
2
 2
2
E n = . (18)

= i
+
. (12) 2mL2
2m x2 2m x2 t t
From which we can deduce: From the postulates of quantum mechanics (discussed in section), we know that
  any state of an isolated system corresponds to a function in the corresponding

2
2 ( ) Hilbert space. This Hilbert space is spanned by the eigenfunctions of a Hermitian
= . (13)
2mi x2 x2 t operator (which corresponds to an observable, in this case energy). Therefore, if
u (x) were a state in our system, it could be represented as a superposition of
Now consider dierent eigenfunctions of energy:
x j:
  


j= (14) u (x) = cn n (x). (19)
x x 2mi x x n=1

  From equation (17), it is clear that all of the n s vanish at zero, whereas u (x)

2
2
j= 2 (15) does not. Thus u (x) is a wavefunction that extends beyond our Hilbert space, or
x 2mi x2 x in other words is a particle not conned in the innite square well, which creates
If we use Eq. (15) in Eq. (13), we obtain the conservation of probability law: a dilemma... one which is easily solved by use of u(x) as the initial state of the
particle. u(x), by the way, is the wavefunction that would be obtained if the

potential suddenly (which can be quantitatively dened) sprung up from nowhere
+ j = 0. (16)
t x and captured a particle formerly in u (x).

(a)
Problem 8
The normalization condition is:
There was a correction to this problem, specically that the initial wavefunction  L
of the particle in the box is supposed to be dx|u(x)|2 = 1. (20)
0
u(x) sin (x/L) + sin (2x/L) Let
instead of u(x) = C(sin (x/L) + sin (2x/L)),

u (x) exp (ix/L) + exp (i2x/L). then normalization implies:

It is useful to consider the problem with u (x). If we solve for the energy eigen-  L

functions of the Hamiltonian for this problem C2 dx sin2 (x/L) + 2 sin (x/L) sin (2x/L) + sin2 (2x/L) = 1
0
2 2
H= , Note that u(x) is a superposition of the rst two energy eigenfunctions (given by
2m x2
Eq. (17)). Since eigenfunctions are orthonormal (discussed in problem (5)), the
we obtain: normalization condition reduces to:
  nx   L
n (x) =
2
sin , (17) L

L L C 2
dx |1 |2 + |2 |2
0 2
November 4, 1999
Physics H7C Fall 1999 Solutions to Problem Set 9 Derek Kimball
from which we conclude  If we include the time evolution of u(x) as described by the time-dependent
2 Schrodinger equation, the constants in front of 1 and 2 acquire a time-
C= .
L dependence:
The same result can be obtained through explicit integration.
u(x, t) = c1 (t)1 (x) + c2 (t)2 (x). (23)
(b) The phase between the two wavefunctions 1 and 2 oscillates at a frequency:
Since u(x) is a superposition of the rst two energy eigenfunctions, a measurement
E
of the energy of the particle will yield either E1 or E2 (given in Eq. (18)), each = , (24)
with a 50% probability. A measurement of an observable will always yield an 
eigenvalue of the corresponding Hermitian operator. Prof. Strovink mentions that where E = E2 E1 . Since u(x, 0) has maximum probability to be found on the
this is the rst measurement. This is important, since from another postulate of left-hand side, when
quantum theory we know that after measuring the energy, the wavefunction of the E t0
= n
particle is subsequently described by the energy eigenfunction corresponding to 
the eigenvalue of energy obtained in the measurement. (n = 1, 3, 5...) the probability to be found on the right-hand side is a maximum.
So
(c) n
t0 = .
The expectation value of the energy is E

E1 + E2

E = .
2
This follows from the fact that u(x) is a superposition of the rst two energy
eigenfunctions with equal probability to be found in either state. Thus repeated
measurements on identical systems will yield E1 half the time and E2 the other
half.

(d)
There are some subtle and important points in this part of the problem. As you
saw in problem (5), a particle in an energy eigenstate of a symmetric potential
always has an equal probability to be found on either the left- or right-hand side
of the potential. This is not true for a superposition of energy eigenstates. This is
readily seen by evaluating the expectation value of x for u(x):
 L

x = x|u(x)|2 dx (21)
0

 L       x   
2 2 x 2x 2x

x = x sin + x sin2
+ 2x sin sin dx (22)
L 0 L L L L

L 16L

x = .
2 9 2
November 4, 1999
1

University of California, Berkeley


Physics H7C Fall 1999 (Strovink)
PROBLEM SET 10

1. Prove that
A particle of mass m in a harmonic oscil-
[Lx , Ly ] = ihLz
lator potential V (x) = 12 m02 x2 has possible
denite-energy wavefunctions un (x) with ener- [Ly , Lz ] = ihLx
gies En = h0 (n + 12 ), where n is zero or a pos- [Lz , Lx ] = ihLy
itive integer. The particle is in thermal equilib- [L+ , L ] = 2hLz
rium with a bath at temperature T . Its probabil-
[L , Lz ] = hL
ity of having total energy E (relative to the bot-
tom of the well) is proportional to the Boltzmann [L+ , Lz ] = hL+
factor eE/kT where k is Boltzmanns constant. [L2 , Lz ] = 0
Calculate the average energy E of the particle, [L2 , L ] = 0
true for any temperature. Then take the high L2 = L L+ + L2z + hLz
temperature limit kT >> h0 and show that E
L2 = L+ L + L2z hLz
reduces to kT , the classical result.
This is the formal basis for the assertion, proved
2. in lecture, that angular momentum is quantized
Consider the potential 12 m 2 x2 of the harmonic in integral or (for intrinsic [spin] angular mo-
oscillator, where is a constant.
 Dene the op- mentum) half-integral units of h.
A y + iq, where y x m/2 and q
erator
p/ 2m. Use the fact that x and p are phys- 4.
ical observables so that x = x and p = p, where The spherical harmonic Ylm (, ) is an eigenfunc-

denotes the Hermitian conjugate. Remember tion of L2 with eigenvalue h2 l(l + 1) and also of
that c = c when c is a constant. H = H is Lz with eigenvalue hm. It is normalized so that
the Hamiltonian. For this potential, prove that 

d Ylm (, )Ylm (, ) = 1.
A = y iq
[A, A ] = h (a.)
Show
 formally that L+ Yll = 0. (Hint: evaluate
H = (q 2 + y 2 ) d(L+ Yll ) (L+ Yll ). )
H = (A A + h/2) (b.)
In lecture it was proved that L is a lowering
H = (AA h/2)
operator:
[H, A ] = hA
[H, A] = hA L Ylm = C (l, m)Yl,m1 ,

where C (l, m) is a constant depending on l and


(This is the formal basis for the assertion, proved m. Using the normality of the Ylm s, derive the
in lecture, that a harmonic oscillator has energy value of |C (l, m)|2 .
levels En = h(n+ 12 ), where n is an integer 0.)
5.
3. Consider the problem of an electron bound to an
Consider the angular momentum operator L innitely heavy nucleus, here using our modern
r p = (h/i)r .  For example, Lx understanding of orbital and spin angular mo-
(h/i) y(/z)z(/y) . Dene L Lx iLy . menta. Suppose that the nucleus is spinless, and
2

that the atom is in a state of denite orbital an-


gular momentum l = 2. Moreover, the atom is
in a state of denite projection mj = 52 of its to-
tal (spin + orbital) angular momentum on the z
axis.
(a.)
What value(s) of total angular momentum quan-
tum number j is (are) possible? Why?
(b.)
Dene the expectation value of the cosine of
the angle between the atoms orbital and spin
angular momenta as follows:

L S
cos   .
L2 S 2 

Evaluate cos  for this problem. (Hint. Con-


sider J 2 , where J = L + S.)

6.
Rohlf 7.27.

7.
Rohlf 8.21.

8.
Rohlf 8.25.
Physics H7C Fall 1999 Solutions to Problem Set 10 Derek Kimball
Its really quite straightforward, theres nothing mystical about it. Next we use the fact that for a geometric series
- Prof. Eugene D. Commins, U.C. Berkeley, on the subject of the Einstein-
 a
Podolsky-Rosen paradox. arn = .
n=0
1r
If you have any questions, suggestions or corrections to the solutions, dont hesitate Thus for the average energy, we have:
to e-mail me at dfk@uclink4.berkeley.edu!
 
1
1e0 1
Problem 1 E = 1 + 0 . (6)
1e0
2

The average energy of a particle E in a harmonic oscillator potential V (x) = From which we nd:
1 2 2
2 m0 x is given by:
0 e0 1 0 1
E = 
+ 0 =  /kT + 0 . (7)
U 1e 0 2 e 0 1 2
E = , (1)
N
This is an expression for the average energy of the particle at any temperature. If
where U is the total energy of N particles in identical potentials (this is the usual we take the high temperature limit (kT  0 ), e0 1 + 0 , so
imaginary ensemble of identical quantum mechanical systems used to calculate
expectation values, etc.). The total energy is given by: 1
E = kT.
   
1 n0
U= N0 En eEn = N0 0 n + e , (2)
n=0 n=0
2
Problem 2
where = 1/(kT ) and the total number of particles is:

 Here, we consider the potential 12 m 2 x2 of the harmonic oscillator, where is
n0 
N= N0 e . (3) a constant.
We dene the operator A y + iq, where y x m/2 and q
n=0
p/ 2m. Since x and p are physical observables, x = x and p = p and
subsequently y = y and q = q.
The average energy is then given by the expression:
(a)

U N  nen0 1 A = (y + iq) = y iq = y iq
= n=0 0 0n + 0 . (4)
N n=0 N0 e
0 2
(b)
Cancelling out common factors of N0 and noting that 
A, A = (y + iq)(y iq) (y iq)(y + iq) = [y, y] + [y, iq] + [iq, y] + [iq, iq]
n0
(n0 )en0 = e , We know that y and q commute with themselves, and that

we can simplify Eq. (4) to: [y, q] = [q, y].


 n0 Employing these results, we nd that
U n=0 e 1
=  n + 0 . (5) 
N n=0 e 0 2 A, A = 2i[y, q].

November 18, 1999


Physics H7C Fall 1999 Solutions to Problem Set 10 Derek Kimball


From the denition of y and q, (f) Since constants commute with anything /2, A = 0. Thus we get:

   
m p 1 H, A = AA , A = AA A A AA = A, A A = A .
[y, q] = x , = [x, p] = i/2.
2 2m 2
(g) Similarly,
From which we conclude:   
 [H, A] = AA , A = AA A AAA = A A , A = A.
A, A = 2i[y, q] = .

(c) Problem 3
p2 1
H= + m 2
2m 2 Consider the
angular momentum operator L r p = (/i)r . For example,
Since p2 = 2mq 2 and x2 = 2y 2 /(m), we nd Lx (/i) y(/z) z(/y) . Dene L Lx iLy .
(a)
H = (q 2 + y 2 ).
[Lx , Ly ] = [ypz zpy , zpx xpz ] = [ypz , zpx ] + [zpy , xpz ] [zpy , zpx ] [ypz , xpz ]
(d),(e) Note that from the denitions of A and A , we have (8)

A + A Recall that in guring out these commutation relations, it often helps to think of
y= the commutators as operators acting on functions. This is especially helpful in
2
dealing with commutators involving derivatives. Well look at each of the terms in
and the above expression individually:
A A
q= .
2i
[ypz , zpx ] = 2 y z + 2 z y
Thus we nd that z x x z
1 2 
y2 = A + A2 + AA + A A 2 2
4 [ypz , zpx ] = 2 yz 2 y + 2 yz = 2 y
and zx x xz x
1 
q2 = A2 A2 + AA + A A
4 [ypz , zpx ] = 2 y
x
From (c) and the above considerations we have that

H = (q 2 + y 2 ) = AA + A A . [zpy , xpz ] = 2 z x + 2 x z
2 y z z y
By adding and subtracting AA or A A where appropriate, 2 2
[zpy , xpz ] = 2 zx + 2 x + 2 xz
 yz y zy
AA = AA A A + A A = A, A + A A =  + A A

and also [zpy , xpz ] = 2 x
y

A A = A A AA + AA = A , A + AA =  + AA .
[zpy , zpx ] = 2 z z + 2 z z
y x x y
These expressions can be used in our above expression for H, and from them we
nd 2 2
   [zpy , zpx ] = 2 z 2 + 2 z 2
H= AA + A A = A A + /2 = AA /2 . yx xy
2
November 18, 1999
Physics H7C Fall 1999 Solutions to Problem Set 10 Derek Kimball
[zpy , zpx ] = 0 [L+ , L ] = 2Lz


[ypz , xpz ] = 2 y x + 2 x y (e)
z z z z
[L , Lz ] = [Lx iLy , Lz ] = [Lx , Lz ] i[Ly , Lz ]
2 2
[ypz , xpz ] = 2 yx 2 + 2 xy 2
z z [L , Lz ] = iLy i(iLx ) = (Lx iLy )
[ypz , xpz ] = 0
[L , Lz ] = L
Now we can put these simplied expressions into Eq. (8), and we nd:
       (f)
 
[Lx , Ly ] = 2 x y = i x y [L+ , Lz ] = [Lx + iLy , Lz ] = [Lx , Lz ] + i[Ly , Lz ]
y x i y i x
[Lx , Ly ] = i(xpy ypx ) [L+ , Lz ] = iLy + i(iLx ) = (Lx + iLy )

[Lx , Ly ] = iLz [L+ , Lz ] = L+

(b),(c) The arguments used in (a) can be basically repeated, just changing the (g)
identities of some of the variables. Or you can argue that since space is rotationally  2     
L , Lz = L2x , Lz + L2y , Lz + L2z , Lz = L2x , Lz + L2y , Lz
invariant, if we rotate our coordinate system in such a way that x y, y z
 2
and z x, the same commutation relation holds with the appropriate change of L , Lz = Lx Lx Lz Lz Lx Lx + Ly Ly Lz Lz Ly Ly
coordinate names. The basic principle is that for any (i, j, k) which are a cyclic
permutation of (x, y, z), we have: Now we employ a common trick in calculating commutation relations. We add
and subtract terms which allow us to substitute commutators we know into our
[Li , Lj ] = iLk . expression.
We know that in general if we interchange two operators in a commutator, the L2 , L = L L L L L L +L L L L L L +L L L L L L +L L L L L L
z x x z x z x x z x z x x y y z y z y y z y z y y
result of the commutator acquires a negative sign. In other words, for any two
operators A, B:  2
L , Lz = Lx [Lx , Lz ] + [Lx , Lz ]Lx + Ly [Ly , Lz ] + [Ly , Lz ]Ly
[A, B] = [B, A].
Thus if (i, j, k) are an anti-cyclic permutation of (x, y, z) (e.g., (y, x, z)), we have: Maybe, if you are like me, when you rst see this trick it seems quite clever. This
trick is used very often because of the potential non-commutativity of operators.
[Li , Lj ] = iLk . Suppose we have two operators A, B which dont commute. If we have AB and
want to get BA for some reason, we can use
(d)
AB = AB BA + BA = [A, B] + BA.
[L+ , L ] = [Lx + iLy , Lx iLy ] = [Lx , Lx ] + i[Ly , Lx ] i[Lx , Ly ] + [Ly , Ly ]
This is a very useful operator identity! Anyhow, continuing on with the math by
An operator always commutes with itself, so substituting in results from (a),(b) and (c) of this problem:
[Lx , Lx ] = 0, [Ly , Ly ] = 0.  2
L , Lz = iLx Ly iLy Lx + iLy Lx + iLx Ly
Thus we have  2
[L+ , L ] = 2i[Lx , Ly ] = 2i(iLz ) L , Lz = 0

November 18, 1999


Physics H7C Fall 1999 Solutions to Problem Set 10 Derek Kimball
This relation proves it is possible to nd simultaneous eigenfunctions of both op-
erators. This is an important result, so it probably wont hurt to see why this is Problem 4
the case once more. Suppose we have operators A, B where [A, B] = 0. Consider
an eigenfunction of B, b , with eigenvalue b . Is Ab an eigenfunction of B? It The spherical harmonic Ylm (, ) is an eigenfunction of L2 with eigenvalue 2 l(l+1)
is, since and also of Lz with eigenvalue m. It is normalized so that
BAb = ABb = A(b b ) = b (Ab ). 

d Ylm (, )Ylm (, ) = 1.
As you can see, this result relies on the fact that A and B commute. Thus mea-
surement of one observable does not aect measurement of the other observable.
This shows that there is no fundamental quantum uncertainty in measurement of (a)
observables which correspond to commuting operators. So we can nd a function Consider the integral:
a b which is is an eigenfunction of both A and B with eigenvalues a and b , as  
we do for L2 and Lz . d(L+ Yll ) (L+ Yll ) = dYll (L L+ )Yll
(h) We can use arguments analagous to those above to show that:
where we make use of the fact that:
 2  2

L , Lx = 0, L , Ly = 0. (L+ Yll ) = Yll L+ = Yll L
If you want to avoid the math, you can just use the isotropy of space to claim From problem 3 parts (i),(j) we have:
that L2 should not preferentially commute with a particular direction in space. It
L L+ = L2 L2z Lz .
immediately follows that
 2 Conveniently, Yll is an eigenfunction of L2 and Lz with eigenvalues 2 l(l + 1) and
L , L = 0 l respectively. Thus we obtain
 

d(L+ Yll ) (L+ Yll ) = dYll 2 l(l + 1) l2 l Yll = 0.

(i),(j) First, lets consider L+ L and L L+ :
Therefore, it must be that
L+ L = (Lx + iLy )(Lx iLy ) = L2x + iLy Lx iLx Ly + L2y
L+ Yll = 0
L+ L = L2x + L2y i[Lx , Ly ] = L2x + L2y + Lz
To go from L+ L to L L+ we use the identity (b)
We can use quite similar methods to nd C (l, m). Consider the integral:
L+ L = [L+ , L ] + L L+ = L L+ + 2Lz .  

d(L Ylm ) (L Ylm ) = dYlm (L+ L )Ylm
Since
L2 = L2x + L2y + L2z
From problem 3 parts (i),(j) we have:
and
L+ L = L2 L2z + Lz .
L+ L = L2x + L2y + Lz , L L+ = L2x + L2y Lz
Ylm is an eigenfunction of L2 and Lz with eigenvalues 2 l(l+1) and m respectively.
we have:
Thus we have the integral
 
L2 = L L+ + L2z + Lz 
d(L Ylm ) (L Ylm ) = dYlm 2
 l(l + 1) m2 + m Ylm

L2 = L+ L + L2z Lz Because the Ylm s are orthonormal, we have that

November 18, 1999


Physics H7C Fall 1999 Solutions to Problem Set 10 Derek Kimball

|C (l, m)|2 =  l(l + 1) m2 + m
2 n1 n2 n3 Energy
1 1 1 6
1 2 1 9
2 1 1 9
Problem 5
2 2 1 12
1 3 1 14
(a) 3 1 1 14
2 3 1 17
Since the atom is in a state of denite projection mj = 5/2 of its total (spin + 3 2 1 17
orbital) angular momentum on the z axis, j 5/2. This follows from the fact that
possible values for mj range between +j and j. What values of j are possible
for a one electron atom with orbital angular momentum l = 2? The values of j
range between l + s and |l s| where s is the electron spin. Therefore, in general Problem 7 Rohlf 8.21
j = 5/2, 3/2 are possible, but since the atom is in the mj = 5/2 state we know
j = 5/2.
(a)
(b) Of course, rst we specify that we know l and s. The possible values for j are
Consider j = l + s, l + s 1, ..., |l s|. We choose one particular value of j. Then there are
2
J 2 = (L + S) = L2 + S 2 + 2L S. 2j + 1 states with total angular momentum j. There are many ways to show this
result. You could start with a stretched state (mj = j) and use the raising or
Since the atom is in an eigenstate of J 2 , L2 and S 2 , we have that: lowering operator, for example. But if you simply note that mj can take on the
2 possible values
LS= (j(j + 1) l(l + 1) s(s + 1)). mj = j, j 1, ..., j + 1, j
2
and count these states, we see immediately that the number of states is 2j + 1.
With j = 5/2, l = 2, and s = 1/2, we obtain L S = 2 . Also,
  3 (b)
L2 S 2  = 2 l(l + 1) s(s + 1) = 2 .
2 The quantum numbers of the n = 2 states of hydrogen in terms n, l, ml and ms
So we nd, are shown in the following table:
n l ml ms
2
cos  = 2 1 1 1/2
3
2 1 1 -1/2
2 1 0 1/2
2 1 0 -1/2
Problem 6 Rohlf 7.27
2 1 -1 1/2
The energies for the particle in the 3D box are given by: 2 1 -1 -1/2
2 0 0 1/2
2 2 2  2 0 0 -1/2
En1 ,n2 ,n3 = 2
n1 + n22 + 4n23 .
2mL
Note there are 8 states in total in the table.
2 2
The following table shows the rst 5 unique energies (in units of 2mL2 ) and the The quantum numbers of the n = 2 states of hydrogen in terms n, l, j and mj are
quantum numbers of the states that possess them. shown in the following table:

November 18, 1999


Physics H7C Fall 1999 Solutions to Problem Set 10 Derek Kimball
n l j mj So we get energy shifts proportional to ml + 2ms .
2 1 3/2 3/2 )
For hydrogen in the n=3 state with a strong B-eld, we have the following possible
2 1 3/2 1/2 values for the angular momentum quantum numbers and energy shifts in units of
2 1 3/2 -1/2 eB/(2m):
2 1 3/2 -3/2
2 1 1/2 1/2 ml ms E
2 1 1/2 -1/2 2 1/2 3
2 0 1/2 1/2 2 -1/2 1
2 0 1/2 -1/2 1 1/2 2
1 -1/2 0
Here there are also 8 states. The system we are considering is described by an 8D 0 1/2 1
Hilbert space, so any complete, orthonormal set of eigenfunctions which span the 0 -1/2 -1
space must consist of 8 states. -1 1/2 0
-1 -1/2 -2
(c) -2 1/2 -1
Since Jz = Lz +Sz , for the state mj = 3/2, l = 1, ml = 1 and ms = 1/2. This is the -2 -1/2 -3
stretched state, and we can readily convert from the n, l, j and mj basis to the
n, l, ml and ms basis by making the correspondence between the stretched states
and employing the raising and lowering operators. Note also that J = L + S . (b)
Such transformations are used quite often and are tabulated (these are the famed In the absence of a magnetic eld the energy separation E0 of the 3p and 1s
Clebsch-Gordan coecients). states is
13.6
E0 = 13.6 eV eV = 12.1 eV.
(d) 9
If mj = 1/2, then we can have The electric dipole transition selection rules demand that the dierence in the
projection of the orbital angular momentum on the z-axis bewteen the initial and
(l, ml , ms ) = (0, 0, 1/2) nal states of an atomic transition must obey

(l, ml , ms ) = (1, 0, 1/2) ml = 1, 0, 1.


(l, ml , ms ) = (1, 1, 1/2).
So the energies of the photons E can be

eB
Problem 8 Rohlf 8.25 E = 12.1 eV , 12.1 eV.
2m

(a)
The magnetic dipole moment )
) of a hydrogen atom, in the limit of a strong B-eld,
is given by:
e
) =
(L + 2S)
2m
and the energy shift E due to the external eld is given by

E = ) )
B.

November 18, 1999


1

University of California, Berkeley


Physics H7C Fall 1999 (Strovink)
PROBLEM SET 11

1. nonrelativistic fermions in a gas at nite tem-


Rohlf 8.41. perature T which have energy above the Fermi
energy EF . The density of states is propor-
2. tional to E 1/2 and the probability that a state
Rohlf 9.3. is occupied is

1
3.  
Rohlf 9.10. exp (E EF ) + 1

4. where = (kT )1 . You dont need to perform


Rohlf 9.25. the integration, but you should set up the inte-
gral so that doing it would yield the correct an-
5. swer without any additional physical reasoning.
Rohlf 9.31.

For those of you itching to do more, here is a


preview of the rst three problems in Problem
Set 12:

1.
Rohlf 12.5.

2.
N electrons each of mass m are conned within
a (formerly) cubic innite potential well that has
been squashed almost at: V = 0 for (0 < x <
L and 0 < y < L and 0 < z < L), V = oth-
erwise. Here  1 (cube is squashed in the
z direction) and N  1. The electrons do not
interact with each other and are at very low tem-
perature so that they ll up the available states
in order of increasing energy. Take N  1, so
that the z part of each electrons wavefunction
may be assumed to be the same (lowest possi-
ble kz ). Thus the problem is reduced to two
dimensions. Calculate the dierence between
the energy of the most energetic electron (Fermi
energy) and the energy of a ground state elec-
tron, using the approximation N  1. should
depend on m, N , and L, but not .

3.
Write an integral equation for the fraction F of
Physics H7C Fall 1999 Solutions to Problem Set 11 Derek Kimball
Civilization as we know it is based on ten ideas. These are Newtons three laws,
the three laws of thermodynamics and Maxwells equations. Everything that you Problem 3 Rohlf 9.10
see around you which dierentiates modern times from the past is based on these
concepts. Soon, Schrodingers equation will join these ten ideas. People will tell you An atom has two electrons in the d-subshell. What are the possible values of total
that civilization is about art, or literature, or architecture... but these components z-angular momentum?
of civilization have more or less been the same for thousands of years. The only
Well, we know that the total orbital angular momentum ltot can range from l1 + l2
real dierence between today and thousands of years ago is physics. If civilization
to |l1 l2 |. Thus ltot = 4, 3, 2, 1, 0. If ltot is even, then the spatial part of the
collapsed tomorrow, we could rebuild it in the same fashion armed with these ideas.
wavefunction spatial is symmetric, and if ltot is odd, then spatial is antisymmetric.
- Prof. Seamus C. Davis, U.C. Berkeley.
With two electrons, the total spin can be stot = s1 + s2 = 1, 0. If stot = 1 then the
spin function spin is symmetric. The total wavefunction total = spatial spin
If you have any questions, suggestions or corrections to the solutions, dont hesitate
must be antisymmetric since we are dealing with identical fermions. Therefore if
to e-mail me at dfk@uclink4.berkeley.edu!
stot = 1 then spatial must be antisymmetric, meaning that ltot is odd (1 or 3). In
this case the total angular momentum j can take on the values j = 4, 3, 2, 1, 0.
Problem 1 Rohlf 8.41
If stot = 0 then the spin function spin is antisymmetric, and spatial must be
Here we wish to show that the average value of 1/r is independent of the orbital symmetric. In this case ltot is even (0, 2 or 4). The possible values of j are 0, 2 or
angular momentum for the hydrogen atom. This result is straightforward if we 4.
introduce the virial theorem (see, e.g., B.H. Bransden and C.J. Joachain, Intro- The largest j value possible is 4, so the possible mj values are:
duction to Quantum Mechanics, pgs. 227-228), which states (in one particular
form) that for a spherically symmetric potential V (r) rn one has for a stationary mj = 4, 3, 2, 1, 0, 1, 2, 3, 4
state:
2T  = nV (r),
where T is the kinetic energy. This is analagous to a classical result of the same Problem 4 Rohlf 9.25
name. The average potential energy for hydrogen is proportional to 1/r, as is
the kinetic energy and hence the total energy from the virial theorem. We know
(a)
that the total energy for the hydrogen atom (from the Bohr model) is independent
of l and depends only on the principal quantum number n. Consequently, 1/r is Sodium atoms are placed in a magnetic eld of 1.5 T. The Zeeman splitting of the
also independent of the orbital angular momentum. ground state (n = 0, l = 0) is given by the shift of energy due to the dierent spin
states the single valence electron can have. Energy shifts are
Problem 2 Rohlf 9.3
E = B B
We seek a totally antisymmetric wavefunction for 3 electrons in terms of a (r1 ), where B is the Bohr magneton. The numerical value for the splitting is given by
b (r2 ) and c (r3 ). The wavefunction must be totally antisymmetric because we
have three identical fermions. Such a wavefunction is given below. It is easily E = 2B B = 2(6 105 ev/T)(1.5 T) = 1.8 104 eV.
veried that under particle interchange it ips sign.
(b)
1 If 1/3 of the sodium atoms are in the higher energy state, then the Boltzmann
= [a (r1 )b (r2 )c (r3 ) a (r1 )c (r2 )b (r3 ) + b (r1 )c (r2 )a (r3 ) factor
3! 1/3
b (r1 )a (r2 )c (r3 ) + c (r1 )a (r2 )b (r3 ) c (r1 )b (r2 )a (r3 )] (1) eE/(kT ) = = 1/2.
2/3
November 23, 1999
Physics H7C Fall 1999 Solutions to Problem Set 11 Derek Kimball
From which we calculate
E
kT = = 2.5 104 eV.
ln 2
Hence,
T 2.9 K

(c)
Same as above:

eE/(kT ) = 49/51.
From which we calculate
kT = 4.4 103 eV
and
T 51 K

Problem 5 Rohlf 9.31

A sample of Na atoms are placed in a 1.0 T magnetic eld. We calculate the energy
shifts for the 3s1/2 , 3p1/2 and 3p3/2 states. Note that in this case B B = 6 105
eV/T.
For the 3s1/2 state (s = 1/2, l = 0, and j = 1/2), the Lande factor is given by:
j(j + 1) + s(s + 1) l(l + 1)
gL = 1 + .
2j(j + 1) = 2
Therefore the energy shifts, given by
E = z Bz = gL B Bmj
are
E = B B.

Similarly, for the 3p1/2 state, gL = 2/3 so


1
E = B B.
3
For the 3p3/2 states gL = 4/3 so
2
E = B B, 2B B.
3

November 23, 1999


1

University of California, Berkeley


Physics H7C Fall 1999 (Strovink)
PROBLEM SET 12

1. 6.
Rohlf 12.5. Rohlf 18.11.

2. 7.
N electrons each of mass m are conned within Rohlf 19.18.
a (formerly) cubic innite potential well that has
been squashed almost at: V = 0 for (0 < x < 8.
L and 0 < y < L and 0 < z < L), V = oth- Rohlf 19.29.
erwise. Here  1 (cube is squashed in the
z direction) and N  1. The electrons do not
interact with each other and are at very low tem-
perature so that they ll up the available states
in order of increasing energy. Take N  1, so
that the z part of each electrons wavefunction
may be assumed to be the same (lowest possi-
ble kz ). Thus the problem is reduced to two
dimensions. Calculate the dierence between
the energy of the most energetic electron (Fermi
energy) and the energy of a ground state elec-
tron, using the approximation N  1. should
depend on m, N , and L, but not .

3.
Write an integral equation for the fraction F of
nonrelativistic fermions in a gas at nite tem-
perature T which have energy above the Fermi
energy EF . The density of states is propor-
tional to E 1/2 and the probability that a state
is occupied is

1
 
exp (E EF ) + 1

where = (kT )1 . You dont need to perform


the integration, but you should set up the inte-
gral so that doing it would yield the correct an-
swer without any additional physical reasoning.

4.
Rohlf 12.16.

5.
Rohlf 17.27.
Physics H7C Fall 1999 Solutions to Problem Set 12 Derek Kimball
Entropy in the universe is always increasing. At some point, the universe will
reach its maximum state of entropy and then no work can be done. The universe f (0) = 1
will become a cold, lifeless place. This is known as the heat death of the universe.
f (1) = 0.79
Get ready, its coming...
f (2) = 0.62
- Prof. Seamus C. Davis, U.C. Berkeley.
f (3) = 0.44
If you have any questions, suggestions or corrections to the solutions, dont hesitate f (4) = 0.32
to e-mail me at dfk@uclink4.berkeley.edu!
f (5) = 0.12
Problem 1 Rohlf 12.5 f (6) = 0.6
f (E > 6) = 0
Consider a system of 6 spin-1/2 fermions having a total energy of 10 units. The
fermions are in a quantum mechanical system where the ground state has 0 energy The Fermi energy EF is where this distribution has f (EF ) 1/2. This is seen
units, the rst excited state has 1 energy unit, the second excited state has 2 energy from the Fermi-Dirac distribution:
units, etc. Determine the energy distribution function df /dE. Make an estimate 1
of the Fermi energy. fF D (E) =
e(EEF )/(kT ) + 1
We make a table of the possible distributions and the spin degeneracy of each state
(i.e., if there is an isolated electron, it can be either spin up or spin down). We fF D (EF ) = 1/2.
then total up number of times an electron is found in a state, and from this total
This is around E = 3 for this system. It doesnt work out exactly because this is
we arrive at our distribution function f , which describes the probability to nd an
a discrete system with a small number of possible distributions. If the number of
electron in a particular energy level. The total number of times we nd an electron
particles was greatly increased, the distribution would become increasingly well-
in an energy level for a distinct distribution (including spin degeneracy) is shown
described by fF D , which is derived in the large N limit.
on the bottom line.

Degen. Energy Problem 2


0 1 2 3 4 5 6
4 2 2 1 0 0 0 1 The energy of the 3D innite square well in this case is given by:
4 2 2 0 1 0 1 0  
2 2 2 2 n2z
1 2 2 0 0 2 0 0 En = n + n + .
16 2 1 1 1 1 0 0 2mL2 x y
2
4 2 1 2 0 0 1 0 is small, so since the number of particles is chosen to be small ( N  1), the
1 2 0 2 2 0 0 0 temperature must be large in order to excite states with nz = 1. We consider
4 1 2 1 2 0 0 0 the case of low temperature, where always nz = 1. In this case, the problem is
4 1 2 2 0 1 0 0 eectively 2D, with the energies:
TOTAL 68 54 42 30 22 8 4  
2 2 2 2 1
f , which in this case is discrete, is just the ratio of the various total number of En = nx + ny + 2 .
2mL2
times a particle is found in an energy level in one of the distributions to the ground
state (E = 0) number. So f (E) is a discrete function described by the following We seek the dierence between the ground state energy and the Fermi energy,
relations: both of which have a term
2 2
2mL2 2
December 3, 1999
Physics H7C Fall 1999 Solutions to Problem Set 12 Derek Kimball
which will cancel out.
First, lets calculate the Fermi energy. The Fermi-Dirac distribution at T = 0 is Problem 3
given by
fF D = 1 E < EF We start with the knowledge that the density of states is proportional to E 1/2 and
fF D = 0 E > EF . the probability of occupation is

The electrons will try to achieve the lowest energy possible, lling up states in 1
P (E) = .
accordance with the Pauli exclusion principle. The total number of particles is e(EEF ) + 1
given by: 
dN Let
N= 2 fF D (E)dE dN
0 dE = cE 1/2 .
dE
where the 2 is for spin degeneracy of the electrons. At T = 0, this integral becomes: where c is a constant. The fraction of nonrelativistic fermions in a gas of nite
 EF temperature T above the Fermi energy is given by the integral:
dN
N= 2 dE. 
0 dE P (E) dN
dE dE
F= EF
dN
.
Now we must determine the density of states for 2D. The energy of a state (ne- 0
P (E) dE dE
2 2 2
glecting the common factor 2mL 2 2 ) is N where N 2 = n2x + n2y and is the square The denominator of the above equation is just the total number of particles, which
of the total number of available states. The derivative of N 2 with respect to energy is easiest to evaluate at T = 0 where, since P (E) is just the Fermi-Dirac distribu-
is tion function, we have:
d 2 1 dN
N = 2N P (E) = 1 E < EF
dE 4 dE
where the factor of 1/4 arises because we consider only positive nx , ny and the 2 P (E) = 0 E > EF .
is from the integration about a ring of thickness dN in n-space. From this we can
So the denominator is simply
explicitly solve for dN
dE :
 EF  EF
dN 2 3/2
dN 4mL 1 2
2 2mL 1/2 dE = cE 1/2 dE = cEF .
= 2 3 = E . 0 dE 0 3
dE  N  2
So employing all of our information, the simplest expression we can get for F is:
Now we employ this expression for dN dE in our integral for the total number of
particles: 
 EF 3 3/2 E 1/2 dE
2 2mL 1/2 2mL 1/2 F = EF
N= 2 E dE = 8 EF . 2 (EE F) + 1
 2  2 EF e
0
Solving for EF yields

2 4 N 2 Problem 4 Rohlf 12.16


EF =
128mL2
We want to deduce the expression for density of states of a relativistic electron
gas. We rst get the density of states with respect to k (where k is the electron
wave vector) in the usual manner for a 3D particle in a box problem. The volume
of a shell of thickness dk in k-space (considering only positive kx , ky , and kz ) is
(4k 2 dk)/8. From the boundary conditions ki = (/L)ni (where i = x, y, z and

December 3, 1999
Physics H7C Fall 1999 Solutions to Problem Set 12 Derek Kimball
the box is length L on a side), we know the number of states per unit volume of where s1 = 1/2 and s2 = 1/2 are the spins of the quarks. The orbital angular
k-space is (L/)3 . So momentum l of charmonium can be l = 1, 0 in the n = 2 state. The total (spin +
dN k2 V orbital) angular momentum is given by
=
dk 2 2
where V is the volume of the box. Dividing by volume, to get the density of states J = L + S = l + s, ... , |l s|.
per unit volume, and converting to momentum p using the deBroglie relation
So we have the following possible states:
p = k, we have:
dn 4p2 j = 2, 1, 0 for s = 1, l = 1
= 3 .
dp h
To convert this expression into density of states per unit energy (E), we use: j=1 for s = 0, l = 1
j=1 for s = 1, l = 0
dn dn dp
(E) = = .
dE dp dE j=0 for s = 0, l = 0

The relativistic expression for momentum in terms of energy is: which total six.

1 2 (b)
p= E m2 c4 .
c
The unobserved state has s = 0, l = 1 and j = 1 by inspection.
Therefore
dp E
= 2 . (c)
dE c p
By analogy to similar states in the chart of charmonium, the energy dierence
We now solve for (E):
4pE between states with aligned vs. anti-aligned spins of the quarks is 100 MeV
(E) = 2 (the energy dierence of the (2s) and c (2s) states and (1s) and c (1s) states,
c2 h3
see Rohlf pg. 494). One could imagine that the energy splitting arises because of
where the factor of 2 is for spin degeneracy.
some spin-spin interaction between the quarks, so the s = 0, l = 1, j = 1 state of
The relativistic momentum is p = mv and v c. The relativistic energy is
charmonium (called the or smiley particle) should be split in energy from the
E = mc2 . Substituting these expressions in,
c1 (2p) state by 100 MeV. So the mass of should be roughly 3400 MeV.
 
4m2 c 2
(E) = 2 (d)
h3
The particle is not observed because the method by which all the states of
charmonium were observed involved creation of a (2s) particle and subsequent
electromagnetic decay. The energy of emitted photons was measured and the
Problem 5 Rohlf 17.27 spectrum of charmonium particles was established. Note that decays of (2s)
change both s and l by 1. Such a decay would involve interaction with both the
electric (to change l) and magnetic (to change s) components of the photon and
(a)
is highly suppressed.
There are six states of charmonium with n = 2. Charmonium is a bound state of
a charm and anti-charm quark. We dont have to worry about symmetrization of
the wavefunction because these are not identical fermions. The total spin s of the
system can be:
s = s1 + s2 = 1, 0

December 3, 1999
Physics H7C Fall 1999 Solutions to Problem Set 12 Derek Kimball
(b)
Problem 6 Rohlf 18.11 The tau particle has ve times as many decay channels as the muon, so the phase
space is increased by a factor of 5. So the tau lifetime is given, from the above
arguments, by:
(a)   5
1 105.7 MeV
= = 0.3 ps.
Consider this problem in the context of the theory of weak interactions as it existed 5 1777 MeV
before Glashow, Weinberg, Salam discovered how to unify it with the theory of
electromagnetic interactions. In this context (Rohlf p. 509), the coupling constant
for weak interactions is the Fermi constant GF . According to Rohlfs Eq. (18.28), Problem 7 Rohlf 19.18
3
GF has dimensions GeV fm . In a system of natural units in which  = c = 1,
we can transform a length (fm) into an inverse energy (inverse GeV) using the fact We have redshift parameter z = 2. We can employ the formula (19.15) on pg. 539
that c 0.2 GeV fm. In natural units, GF therefore has dimensions GeV2 . of Rohlf:
1+
Since GF is a coupling constant, like the ne structure constant , it describes (1 + z)2 =
1
the strength of a quantum mechanical amplitude. The rate is proportional to the
square of the modulus of this amplitude. Thus the decay rate W (which is inversely where = v/c as usual. We can solve for :
proportional to the muon lifetime ) is proportional to the square of GF .
(1 + z)2 1
2 = = 0.8
Now we have a dilemma. If |GF | were the only dimensionful component of W , (1 + z)2 + 1
W would have units of GeV4 . However, using the fact that  = 6.6 1025 GeV
sec, in natural units we know that W must have units of sec1 or GeV. So far we and then use in Hubbles law to determine the distance to the galaxy. Hubbles
are o by ve powers of energy! law is
c
d=
The solution is to bring in the only other relevant dimensionful quantity around, H0
the muon mass m . Remembering that mc2 is the same as m in natural units, where H is the Hubble constant. We nd:
0
we nd that we need ve powers of m in the numerator of W to make its units
correct. Therefore its inverse, the muon lifetime, must have ve powers of m in c 0.8 (3 108 m/s)
d= = = 3400 Mpc.
its denominator. H0 7 104 m/s Mpc 1

Alternatively, we can approach this problem a bit more formally. Fermis Golden
rule says that the decay rate W is given by:
Problem 8 Rohlf 19.29
2
W = |M|2 (phase space)
 As a rough estimate, we simply set the thermal energy of particles in the early
universe kT equal to the mass of 2 bottom quarks (actually a bottom and anti-
where M is a transition amplitude obtained from perturbation theory (youll learn bottom, which have the same mass). The bottom quarks must be produced in pairs
2
all about this in 137B). In this case, all we need to know is that M 1/MW . By so that beauty is conserved, since the b and b have equal and opposite beauties.
dimensionality, we need to cancel the mass of the W boson with something, the best You begin to wonder where this stu comes from. Anyhow, the mass of 2 bottom
guess is the mass of the muon. The phase space available to the decay products quarks is 10 GeV, which implies T = 1014 K. The characteristic expansion time
in this case is proportional to the available energy, in other words the muon mass texp comes from
again. So
|M|2 (phase space) m5  2   2
1 2.7 K 3c2 2.7 K
texp = = = 5 1019 s .
which again implies m5
. H(t) T 8G T
December 3, 1999
Physics H7C Fall 1999 Solutions to Problem Set 12 Derek Kimball
You should probably check out the discussion on pp. 558-559 of Rohlf. So we can
estimate:
4 1020 s K2
texp = 4 108 s.
T2
And thats all folks!
Good luck on your nals! Merry winter break!

December 3, 1999
University of California, Berkeley
Physics H7C Fall 1999 (Strovink)
EXAMINATION 1

Directions. Do both problems (weights are indicated). This is a closed-book closed-note exam
except for one 8 12 11 inch sheet containing any information you wish on both sides. You are free to
approach the proctor to ask questions but he or she will not give hints and will be obliged to write
your question and its answer on the board. Dont use a calculator, which you dont need roots,
circular functions, etc., may be left unevaluated if you do not know them. Use a bluebook. Do not
use scratch paper otherwise you risk losing part credit. Cross out rather than erase any work that
you wish the grader to ignore. Justify what you do. Box or circle your answer.

1. (58 points) where t is the time as observed at the origin


In a free-electron laser, a beam of relativistic of S  , compute 0 in terms of the constants
electrons is subjected to a transverse magnetic previously given.
eld that varies sinusoidally with lab coordinate
z, the (average) beam direction: c. (8 points) Consider an electron of charge e
and mass m whose average position is
2z
B = xB0 cos
0 x , y  , z   = (0, 0, 0)

where B0 and 0 are constants. In the lab, the


as observed in S  . In this frame, its veloc-
z component of the electrons velocity is
ity is so small that you may ignore v B
with respect to E . In frame S  , making this
vz = 0 c
approximation, compute the electrons mo-
tion y  (t ). (In case you didnt get part b.
where 0 is a constant.
exactly right, leave your answer in terms of
a. (8 points) Consider a Lorentz frame S  mov- 0 .)
ing with velocity
d. (8 points) The electric dipole moment p
0 c = z0 c of a distribution of N point charges qi at
positions ri is dened as
with respect to the lab. The Lorentz trans-
formation for electromagnetic elds is N

p= ri qi .
E = E i=1

B = B
The power P (t) radiated by a charge dis-
E = 0 (E + 0 cB ) tribution with time-varying dipole moment
cB = 0 (cB 0 E ) , p(t) is

where 0 (1 02 )1/2 . Calculate the elec- 1 2 (d2 p/dt2 )2


tric eld E seen in S  ; continue to express P = .
40 3 c3
it in terms of 2z/0 .
b. (8 points) Dening As seen in S  , calculate P  , the time-
averaged power radiated by a single electron
2z/0 0 t , in the free-electron laser.
e. (8 points) Energy and time both transform e. (8 points) Can a linear combination of a
as the 0th component of a four-vector. Cal- right-hand and a left-hand circularly polar-
culate P , the time-averaged power radi- ized plane wave in the region L < y < L
ated by a single electron as observed in the propagate in the z direction? If not, why
lab. You may leave your answer in terms of not? If so, what combination(s) would be
P  . possible? Explain fully.
f. (10 points) For light, the relativistic Doppler
shift is


= .
0 (1 0 cos )

Calculate the ratio 0 /, where 0 , as be-


fore, is the characteristic length describing
the spatial variation in the lab of the free-
electron lasers magnetic eld, and is the
wavelength of the light that its electrons ra-
diate in the forward direction, as observed
in the lab. Express this ratio in terms of 0 ,
in the limit 0 1.
g. (8 points) What is the state of polarization
of the free-electron lasers light? Explain.

2. (42 points)
Semi-innite regions y > L and y < L are lled
by perfect conductor, while the intervening slab
L < y < L is lled by dielectric with constant
dielectric constant  and permeability .
a. (8 points) In SI units, write Maxwells equa-
tions for E and H inside the dielectric. Do
not write any terms involving free charges
or free currents, which both vanish there.
b. (8 points) Prove that Ex and Ez both must
vanish at y = L.

For parts c. and d. only, assume, for


L < y < L, that the elds are given by
  
Ephysical = yE2 exp i(kz t)
  
Hphysical = (xH1 + zH3 ) exp i(kz t) ,

where E2 , H1 , and H3 are unknown complex con-


stants, and k and are unknown real constants.
c. (8 points) Prove that H3 = 0.
d. (10 points) Calculate the ratio H1 /E2 in
terms of known quantities.
University of California, Berkeley
Physics H7C Fall 1999 (Strovink)
SOLUTION TO EXAMINATION 1

Directions. Do both problems (weights are indicated). This is a closed-book closed-note exam
except for one 8 12 11 inch sheet containing any information you wish on both sides. You are free to
approach the proctor to ask questions but he or she will not give hints and will be obliged to write
your question and its answer on the board. Dont use a calculator, which you dont need roots,
circular functions, etc., may be left unevaluated if you do not know them. Use a bluebook. Do not
use scratch paper otherwise you risk losing part credit. Cross out rather than erase any work that
you wish the grader to ignore. Justify what you do. Box or circle your answer.

1. (58 points) gives us the total electric eld seen in S  :


In a free-electron laser, a beam of relativistic
electrons is subjected to a transverse magnetic E = 0 cB
 2z 
eld that varies sinusoidally with lab coordinate = 0 0 cB0 cos (z x)
z, the (average) beam direction: 0
 2z 
= 0 0 cB0 cos y .
2z 0
B = xB0 cos
0
b. (8 points) Dening
where B0 and 0 are constants. In the lab, the
z component of the electrons velocity is 2z/0 0 t ,
where t is the time as observed at the origin
vz = 0 c of S  , compute 0 in terms of the constants
previously given.
where 0 is a constant.
Solution. We need a Lorentz transformation
a. (8 points) Consider a Lorentz frame S  mov- to relate (ct, z) to (ct , z  ). Taking advantage
ing with velocity of the fact that z  = 0, we minimize algebra by
choosing the inverse transformation:
0 c = z0 c
z = 0 z  + 0 0 ct
with respect to the lab. The Lorentz trans- = 0 + 0 0 ct
formation for electromagnetic elds is = 0 0 ct
2 2
z= 0 0 ct
E = E 0 0
B = B 0 t
E = 0 (E + 0 cB ) 20 0 c
0 = .
0
cB = 0 (cB 0 E ) ,
c. (8 points) Consider an electron of charge e
where 0 (1 02 )1/2 . Calculate the elec- and mass m whose average position is
tric eld E seen in S  ; continue to express
it in terms of 2z/0 . x , y  , z   = (0, 0, 0)
Solution. There are no electric or parallel mag- as observed in S  . In this frame, its veloc-
netic elds in the lab frame so the third equation ity is so small that you may ignore v B
with respect to E . In frame S  , making this Plugging this result into the formula given above
approximation, compute the electrons mo- for radiated power P  ,
tion y  (t ). (In case you didnt get part b.
exactly right, leave your answer in terms of 1 2 e4 02 02 c2 B02
P = cos2 0 t .
0 .) 40 3 m2 c3
Solution. The Lorentz force on the electron in
S  is given by: Since cos2 0 t  = 1/2, the time average of P  is
given by:
F = eE ev B ,
1 1 e4 02 02 B02
   P   = .
but we can ignore the v B term since v is 40 3 m2 c
always small. So then we have an equation for
the acceleration: e. (8 points) Energy and time both transform
as the 0th component of a four-vector. Cal-
me a = eE .
culate P , the time-averaged power radi-
ated by a single electron as observed in the
Plugging in E from (a.) and integrating twice
lab. You may leave your answer in terms of
with respect to t ,
P  .
e cos 0 t
y  (t ) = 0 0 cB0 . Solution. Since both energy and time trans-
m (0 )2 form as the 0th component of a four-vector, if
we have measured a change in energy of the
d. (8 points) The electric dipole moment p electron E  and a change in time t in the
of a distribution of N point charges qi at electrons rest frame S  , then the same quanti-
positions ri is dened as ties in the lab frame are given by E = 0 E 
and t = 0 t . So
N

p= ri qi . P   = E  /t
i=1
= E/t
The power P (t) radiated by a charge dis- = P  .
tribution with time-varying dipole moment
p(t) is This was also solved in problem set 3, problem 3!

1 2 (d2 p/dt2 )2 f. (10 points) For light, the relativistic Doppler


P = . shift is
40 3 c3

As seen in S  , calculate P  , the time- 


= .
averaged power radiated by a single electron 0 (1 0 cos )
in the free-electron laser.
Calculate the ratio 0 /, where 0 , as be-
Solution. The second derivative of the time- fore, is the characteristic length describing
varying electric dipole moment for a single elec- the spatial variation in the lab of the free-
tron in the free electron laser is given by: electron lasers magnetic eld, and is the
wavelength of the light that its electrons ra-
d2 p
= ea , diate in the forward direction, as observed
dt2
in the lab. Express this ratio in terms of 0 ,
where a is the acceleration found in (c.): in the limit 0 1.
e Solution. In S  , we know from (c.) that the
a = 0 0 cB0 cos 0 t .

electron is oscillating with angular frequency 0 .
m
Then, in S  , the EM radiation produced by that not write any terms involving free charges
electron has the same angular frequency. A or free currents, which both vanish there.
forward observer in S, upon whom the beam
Solution. Within the dielectric, D = E and
impinges with = 0, sees this radiation with a
B = H, where  and are constants. The
Doppler shifted angular frequency 0 that, by
source-free equations are
the above Doppler formula, is equal to

0 B=0
0 = .
0 (1 0 ) B = H
H=0
Substituting 0 = 2c/, and plugging in the
value of 0 from (b.),
and
B
2c 20 0 c 1 E=
= t
0 0 (1 0 )
B = H
0 0
= H
1 0 E =
t
1 + 0 0
=
1 + 0 1 0 The useful source-dependent equations are the
= 02 0 (1 + 0 ) . variety that depend on free rather than total
charges and currents, because free charges and
This reduces to 202 in the relativistic limit, a fa- currents are zero in the dielectric. These are
mous (and simple) result. If, say, 0 is 0.1 m and
the electron energy is 500 MeV ( 2 106 ), the D = free = 0
FEL or wiggler can be made to radiate in the far D = E
UV ( 0.05 m), where no conventional laser E=0
is available.
g. (8 points) What is the state of polarization and
D D
of the free-electron lasers light? Explain. H = Jfree + =
t t
Solution. From (c.) we know that the elec- D = E
tron oscillates in the y direction. The on-axis
E
radiation from the dipole is propagating in the H=
z direction, so light is polarized orthogonal to t
z. From the formula derived in class describing
dipole radiation, we know the radiation is polar- b. (8 points) Prove that Ex and Ez both must
ized in the direction where the vertical axis is vanish at y = L.
dened by the direction the dipole oscillates in. Solution. Electric elds vanish in perfect
Hence we can conclude that the light is linearly conductors, because the innitely mobile free
polarized in the y direction. charges instantaneously rearrange themselves to
shield out any externally applied electric eld.
Consider a rectangular loop with long side S and
2. (42 points)
short side s. One long side lies in the conduc-
Semi-innite regions y > L and y < L are lled
tor, parallel to the plane y = L; the other long
by perfect conductor, while the intervening slab
side lies in the dielectric. In the limit s 0, the
L < y < L is lled by dielectric with constant
right-hand side of Faradays law,
dielectric constant  and permeability .
 
a. (8 points) In SI units, write Maxwells equa- d
tions for E and H inside the dielectric. Do E dl = B da ,
dt
vanishes because the area vanishes, and the con- Setting the two values of H1 /E2 equal,
tributions of the short parts to the rectangular
loop on the left-hand side also vanish. The only k 
=
nonvanishing contribution to the left-hand side k
is E S in the dielectric. This proves that E k
in the dielectric must vanish at |y| = L. This in- =  .

cludes E in either the x or z directions, which
both are parallel to the interface. Plugging this value for k/ into either of the
equations for H1 /E2 ,
For parts c. and d. only, assume, for 
L < y < L, that the elds are given by H1
=

.
   E2
Ephysical =  yE2 exp i(kz t)
  
Hphysical =  (xH1 + zH3 ) exp i(kz t) ,
e. (8 points) Can a linear combination of a
where E2 , H1 , and H3 are unknown complex con-
right-hand and a left-hand circularly polar-
stants, and k and are unknown real constants.
ized plane wave in the region L < y < L
c. (8 points) Prove that H3 = 0. propagate in the z direction? If not, why
Solution. As usual we require the complex not? If so, what combination(s) would be
electromagnetic elds to satisfy Maxwells equa- possible? Explain fully.
tions (not just their (physical) real part). When Solution. From (b.) we know that Ex = 0
their dependence on r and t is of the form at |y| = L. In this part (only!) we are asked
exp i(k r t) , the operators and re- to assume that a plane wave is propagating in
duce to ik and ik, respectively, while the the z direction within the dielectric. This means
operator /t reduces to i. Using the rst that E must be to z and it cannot depend on
Maxwell equation in (a.), x and y. So, if Ex vanishes at the boundaries
|y| = L, it must vanish throughout the dielec-
H=0
tric. Thus the only nonzero component of E
ik (xH1 + zH3 ) = 0 lies in the y direction. We then ask, what com-
k = zk bination of RH polarization ( x iy) and LH
ikH3 = 0 . polarization ( x + iy) add to pure y polariza-
tion? Evidently, the RH and LH waves must
d. (10 points) Calculate the ratio H1 /E2 in have equal amplitude and opposite sign.
terms of known quantities.
Solution. Using the methods of (c.), the second
Maxwell equation in (a.) requires
ik(z y)E2 = +ixH1
ikxE2 = +ixH1
k H1
=
E2
while the fourth Maxwell equation in (a.) re-
quires
ik(z x)H1 = iyE2
ikyH1 = iyE2
H1 
= .
E2 k
University of California, Berkeley
Physics H7C Fall 1999 (Strovink)
EXAMINATION 2

Directions. Do all four problems (weights are indicated). This is a closed-book closed-note exam
except for two 8 12 11 inch sheets containing any information you wish on both sides. You are free
to approach the proctor to ask questions but he or she will not give hints and will be obliged to
write your question and its answer on the board. Calculators are allowed but not essential roots,
circular functions, etc., may be left unevaluated if you do not know them. Use a bluebook. Do not
use scratch paper otherwise you risk losing part credit. Cross out rather than erase any work that
you wish the grader to ignore. Justify what you do. Box or circle your answer.

1. (25 points) a. 
(10 points) Let be the maximum value of
The basis of scalar diraction theory is the x2 + y 2 on the aperture plane for which
Fresnel-Kircho integral formula. In Fowles no- the aperture is not opaque. Thus, for
tation (Eq. 5.11), this formula states this part of the problem, there are three
characteristic lengths: , , and D. By
ikU0 exp (it) moving around in the plane z = D, re-
Up =
4
  stricting
her own coordinates X, Y such
 
exp ik(r + r )  
 that X 2 + Y 2  D, the observer nds
n r n r dA that the optical disturbance there is propor-
rr
tional to the Fourier transform of g(x, y).
where r is a vector from the (point) source to As someone who understands the physics of
a point on the aperture, r is a vector from the diraction, you realize that this information
observer to the same point on the aperture, implies that a single strong condition must
be satised which relates , , and D. Write
 
exp i(kr t) down this condition. (You neednt prove it,
U0 and you may omit factors of order unity.)
r
b. (15 points) For this part of the problem,
is the optical disturbance at a point on the
take the aperture function to be
aperture, Up is the optical disturbance at the
observer, = ck = 2c/ is the angular fre-
quency of the light, dA is an element of aperture g(x, y) = 0, x < 0
area, and n is the normal to dA. [Note that, g(x, y) = 1, x > 0 .
in a typical geometry (source on the left, aper-
ture in the middle, observer on the right, and n
This describes a knife edge at x = 0 ex-
pointing to the left), n r is positive while n r
tending from y = to y = . Therefore,
is negative, so that both terms in the square
in this part of the problem, = : the
bracket are positive.]
strong condition of part a. cannot be sat-
Consider this simple geometry: Let z be ised. In this part of the problem, the ob-
the axis pointing from left to right. Place the server is xed at (0, 0, D), i.e. at X = Y = 0.
source at (x, y, z) = (0, 0, D), the observer at With this aperture in place, the observer
(X, Y, D), and the aperture in the plane z = 0. records an irradiance Ia . With the aperture
The aperture is characterized by an aperture completely removed (g 1), the observer
function g(x, y) such that g = 1 where the aper- records an irradiance I0 . Give the ratio
ture is open, and g = 0 where the aperture is Ia /I0 . To receive credit you must explain
opaque. why this ratio is correct.
2. (25 points) section
James Rainwater was awarded the Nobel Prize d
,
in the 1980s for experiments done at the Nevis d
(Columbia) cyclotron in the 1950s. He mea- where d = sin d d is an element of solid
sured the sizes of nuclei using their interactions angle. (When you integrate your result over
with muons (heavy electrons) which were in orbit the full solid angle, do you conrm your
about them. answer to a.?)
In the following, use the Bohr picture to de-
scribe the muon orbit. For ease of numerical 4. (25 points)
computation, you may take the natural length A nonrelativistic particle of mass m is conned
unit h/me c to be 400 fm; the ratio m /me of to a one-dimensional box extending from x = 0
muon to electron masses to be 200; and the ne to x = L. Here a box is a square potential
structure constant to be 1/150. You may ne- well with innite sides.
glect the dierence between the muons actual
a. (10 points) In terms of n and other con-
and reduced mass.
stants, write down the energies En , 1 n <
A muon in n = 1 Bohr orbit reacts with (is cap- , measured with respect to the bottom
tured by) a Z = 50 nucleus before it decays: of the potential well, that the particle is
allowed by Schrodingers equation to have.
+ (A, Z) (A, Z 1) + , b. (15 points) Dene N (E) to be the total
number of allowed states with energy E.
where the neutrino has negligible rest mass. Taking n 1, so that the distribution of E
Assuming that the initial and nal nuclei have is approximately continuous, calculate the
the same innitely large rest mass and therefore density of states
a negligible kinetic energy, what is the neutrino
energy expressed in units of me c2 ? (1% accuracy dN
(E) .
is sucient.) dE

3. (25 points)
Consider the elastic scattering of a photon from
an innitely massive, perfectly reective, spher-
ical target of nite radius R (like a bowling ball
polished to a mirror nish). The bowling ball is
centered on the origin. The photon is incident
along the z direction and scatters (reects) into
the direction (, ), where and are the usual
spherical polar angles. Note that = 0 means
that the photon remains undeected. For this
problem, ignore diraction and any other eects
which arise from the wavelike properties of the
photon.
a. (10 points) What is the total scattering
cross section T , corresponding to any de-
ection of the photon? (You dont need a
calculation here, just a correct answer and
a convincing explanation for it.)
b. (15 points) Calculate the dierential cross
University of California, Berkeley
Physics H7C Fall 1999 (Strovink)
SOLUTION TO EXAMINATION 2

Directions. Do all four problems (weights are indicated). This is a closed-book closed-note exam
except for two 8 12 11 inch sheets containing any information you wish on both sides. You are free
to approach the proctor to ask questions but he or she will not give hints and will be obliged to
write your question and its answer on the board. Calculators are allowed but not essential roots,
circular functions, etc., may be left unevaluated if you do not know them. Use a bluebook. Do not
use scratch paper otherwise you risk losing part credit. Cross out rather than erase any work that
you wish the grader to ignore. Justify what you do. Box or circle your answer.

1. (25 points) a. 
(10 points) Let be the maximum value of
The basis of scalar diraction theory is the x2 + y 2 on the aperture plane for which
Fresnel-Kircho integral formula. In Fowles no- the aperture is not opaque. Thus, for
tation (Eq. 5.11), this formula states this part of the problem, there are three
characteristic lengths: , , and D. By
ikU0 exp (it) moving around in the plane z = D, re-
Up =
4
  stricting
her own coordinates X, Y such
 
exp ik(r + r )  
 that X 2 + Y 2  D, the observer nds
n r n r dA that the optical disturbance there is propor-
rr
tional to the Fourier transform of g(x, y).
where r is a vector from the (point) source to As someone who understands the physics of
a point on the aperture, r is a vector from the diraction, you realize that this information
observer to the same point on the aperture, implies that a single strong condition must
be satised which relates , , and D. Write
 
exp i(kr t) down this condition. (You neednt prove it,
U0 and you may omit factors of order unity.)
r
Solution. In order for the optical disturbance
is the optical disturbance at a point on the Up (X, Y ) to be the Fourier transform of g(x, y),
aperture, Up is the optical disturbance at the our system must satisfy the Fraunhofer condi-
observer, = ck = 2c/ is the angular fre- tion (see discussion in Fowles Section 5.6). The
quency of the light, dA is an element of aperture basic idea of this condition is that the spheri-
area, and n is the normal to dA. [Note that, cal curvature of the wavefront at the aperture
in a typical geometry (source on the left, aper- must be small compared to the wavelength of
ture in the middle, observer on the right, and n the light, allowing us to treat the light at the
pointing to the left), n r is positive while n r aperture as a plane wave. Omitting factors of
is negative, so that both terms in the square order unity, the Fraunhofer condition is
bracket are positive.]
2  D .
Consider this simple geometry: Let z be
the axis pointing from left to right. Place the This condition implies
 that the obliquity fac-
source at (x, y, z) = (0, 0, D), the observer at tor n r n r is constant over the aperture,

(X, Y, D), and the aperture in the plane z = 0. the quantity eikr /r is nearly constant, and the
The aperture is characterized by an aperture quantity eikr /r eikr . With these approxima-
function g(x, y) such that g = 1 where the aper- tions, the optical disturbance
 
ture is open, and g = 0 where the aperture is
opaque. Up (X, Y ) eikr dA.
b. (15 points) For this part of the problem, A muon in n = 1 Bohr orbit reacts with (is cap-
take the aperture function to be tured by) a Z = 50 nucleus before it decays:

g(x, y) = 0, x < 0 + (A, Z) (A, Z 1) + ,


g(x, y) = 1, x > 0 .
where the neutrino has negligible rest mass.
This describes a knife edge at x = 0 ex-
Assuming that the initial and nal nuclei have
tending from y = to y = . Therefore,
the same innitely large rest mass and therefore
in this part of the problem, = : the
a negligible kinetic energy, what is the neutrino
strong condition of part a. cannot be sat-
energy expressed in units of me c2 ? (1% accuracy
ised. In this part of the problem, the ob-
is sucient.)
server is xed at (0, 0, D), i.e. at X = Y = 0.
With this aperture in place, the observer Solution. The binding energy of the muon in
records an irradiance Ia . With the aperture the Bohr model is given by:
completely removed (g 1), the observer
records an irradiance I0 . Give the ratio 1
BE = m c2 (Z)2
Ia /I0 . To receive credit you must explain 2
why this ratio is correct.
and in our case Z 1/3 and m c2 200me c2 .
Solution. According to the Fresnel-Kircho in-
So
tegral, the optical disturbance Up arises from a
superposition of secondary waves which originate BE 11me c2 .
at the z = 0 plane. When the semi-innite screen Since the rest energies of the initial and nal nu-
is in place, due to the symmetry of the system ex- clei are taken to be the same, the kinetic energy
actly half the secondary waves are blocked. Thus of the neutrino must be equal to the rest mass
1 of the muon minus the binding energy, or
Up = U0 ,
2
KE( ) 200me c2 11me c2 189me c2 .
or, in terms of intensity I |U |2 ,

I 1
= .
I0 4 3. (25 points)
Consider the elastic scattering of a photon from
an innitely massive, perfectly reective, spher-
2. (25 points) ical target of nite radius R (like a bowling ball
James Rainwater was awarded the Nobel Prize polished to a mirror nish). The bowling ball is
in the 1980s for experiments done at the Nevis centered on the origin. The photon is incident
(Columbia) cyclotron in the 1950s. He mea- along the z direction and scatters (reects) into
sured the sizes of nuclei using their interactions the direction (, ), where and are the usual
with muons (heavy electrons) which were in orbit spherical polar angles. Note that = 0 means
about them. that the photon remains undeected. For this
problem, ignore diraction and any other eects
In the following, use the Bohr picture to de-
which arise from the wavelike properties of the
scribe the muon orbit. For ease of numerical
photon.
computation, you may take the natural length
unit h/me c to be 400 fm; the ratio m /me of a. (10 points) What is the total scattering
muon to electron masses to be 200; and the ne cross section T , corresponding to any de-
structure constant to be 1/150. You may ne- ection of the photon? (You dont need a
glect the dierence between the muons actual calculation here, just a correct answer and
and reduced mass. a convincing explanation for it.)
Solution. The total cross section T is the cross An element d of beam cross section is equal to
sectional area of the photon beam that suers |b db d|. Substituting from above,
any deection as a result of interaction with the
target. Neglecting diractive eects, the only
d = |b db d|
photons scattered are those which intercept the
area of a hemisphere of radius R, projected into R2
= cos sin d d
the z = 0 plane. This is a circle of area R2 . 2 2 2
Thus R2
= sin d d
T = R2 . 4
R2
b. (15 points) Calculate the dierential cross =
4
d
section
d d R2
, = .
d d 4
where d = sin d d is an element of solid
angle. (When you integrate your result over This is an isotropic (constant) dierential cross
the full solid angle, do you conrm your section. Integrated over = 4, it yields
answer to a.?) T = R2 as in (a.). Note that the isotropy of
the dierential cross section doesnt follow au-
Solution.
 A photon with impact parameter
tomatically from the spherical symmetry of the
b = x + y 2 intercepts the sphere at a point
2
potential (an innite wall at r = R). A dier-
on the sphere described by
ent spherically symmetric potential, for example
b the Coulomb potential, yields the dramatically
s = arcsin .
R dierent Rutherford result
Just before it hits the sphere, it is travelling in
the direction d 1
.
0 = 0 . d sin4
2
Before impact, the angle that the photon makes
with the normal to the sphere is . Since the an-
gle of incidence is equal to the angle of reection,
its direction changes by 4. (25 points)
A nonrelativistic particle of mass m is conned
= 2 .
to a one-dimensional box extending from x = 0
Therefore the nal angle of the photon is to x = L. Here a box is a square potential
well with innite sides.
= 0 +
= 0 + 2 a. (10 points) In terms of n and other con-
b stants, write down the energies En , 1 n <
= 2 arcsin . , measured with respect to the bottom
R
of the potential well, that the particle is
Rearranging and dierentiating, allowed by Schrodingers equation to have.
b
arcsin = Solution. We measure the energy E of the
R 2 2
  particle with respect to the bottom of the well,
b = R sin where V 0. We seek solutions of the time-
2 2
independent Schrodinger equation

= R cos
2
R  h2 2 
db = sin d . + V (x) uE (x) = EuE (x) ,
2 2 2m x2
with the boundary condition (because of the Expressing (E) in terms of E and other con-
innite potential wall) stants, we substitute

uE (0) = uE (L) = 0 . 2mL2 E


n2 = 2
2 h
The solutions are of the form
mL2 2 h2
(E) = 2 2
uE (x) sin kn x h 2mL2 E

L m
with kn = n/L. Therefore, from the time- = .
h 2E
independent Schrodinger equation,

h2 kn2 n2 2 h2
En = = ,
2m 2mL2

with 1 n . [As posed, the problem doesnt


require a proof like the above; you just need to
write down the correct values of En .]
b. (15 points) Dene N (E) to be the total
number of allowed states with energy E.
Taking n  1, so that the distribution of E
is approximately continuous, calculate the
density of states

dN
(E) .
dE

Solution. The dierence in energy between two


adjacent states is

E En En1
  2 h2
= n2 (n 1)2
2mL2
2 2
h
= (2n 1) .
2mL2

So when we increase the number of states by


N = 1 we increase the maximum energy by
E. The density of states is just the ratio:

dN
(E)
dE
N
as n
E
2mL2
=
(2n 1) 2 h2
mL2
as n .
n 2 h2
University of California, Berkeley
Physics H7C Fall 1999 (Strovink)
FINAL EXAMINATION

Directions. Do all six problems (weights are indicated). This is a closed-book closed-note exam
except for three 8 12 11 inch sheets containing any information you wish on both sides. You are free
to approach the proctor to ask questions but he or she will not give hints and will be obliged to
write your question and its answer on the board. Calculators are allowed but not essential roots,
circular functions, etc., may be left unevaluated if you do not know them. Use a bluebook. Do not
use scratch paper otherwise you risk losing part credit. Cross out rather than erase any work that
you wish the grader to ignore. Justify what you do. Box or circle your answer.

1. (35 points) The diracted image is viewed on a screen lo-


One circularly polarized photon is trapped be- cated in the plane z = L, where L  d; also
tween two parallel perfectly conducting plates, L  d2 , where is the EM wavelength.
separated by a distance L, which are parallel to
Quarter-wave plates are placed in each slit. They
the photons electric and magnetic elds.
are identical, except that the topplates slow
a. (10 points) If the photon has the smallest (high-index) axis is along (x+ y)/ 2 (+45 with
denite energy possible under these circum- respect to the x axis), while
the bottom plates

stances, at a point halfway between the slow axis is along (x y)/ 2 (45 with respect
plates describe a possible motion of its elec- to the x axis).
tric eld vector. a. (15 points) What is the state of polarization
b. (10 points) Same for its magnetic eld vec- of the diracted light that hits the center of
tor. the screen, at x = y = 0? Explain.

c. (15 points) For this plate conguration, b. (20 points) At what diracted angle x does
making no restriction on the photon en- the rst minimum of the irradiance occur?
ergy, evaluate the density of photon states 3. (35 points)
A lens has an f -number (ratio of focal length to
d2 N diameter) equal to F . The lens is used to concen-
dL d trate sunlight on a ball whose diameter is equal
to the diameter of the suns image. The ball is
where N is the number of states and is the convectively and conductively insulated, but it
photon wavelength  L. Take into account freely radiates energy outward so that its tem-
the possible states of circular polarization. perature can approach an equilibrium value Tb .
a. (10 points) The sun subtends a half-angle
of 0.005 radians. Is the size of its image
2. (35 points)
diraction-limited, i.e. determined largely
A linearly (x) polarized plane EM wave trav-
by the eects of diraction? Make an order-
elling along z is incident on an opaque bae
of-magnitude argument assuming that the
located in the plane z = 0. The bae has two
lens is a typical camera lens, with a radius
slits cut in it, which are of innite extent in the
of order 102 m.
y direction. In the x direction, the slit widths
are each a and their center-to-center distance is b. (25 points) Assuming the sun to be a black-
d. (Obviously d > a, but you may not assume body of temperature T , calculate the balls
that d  a.) The top and bottom slits are each temperature Tb . Neglect reection by the
an equal distance from x = 0. lens. (Hint: your answer should depend
only on T and F .) b. (10 points) The N particles are identical
bosons.
4. (30 points) c. (15 points) The N particles are identical
You are given a Hamiltonian spin 12 fermions.

1
H= (LR + RL) , 6. (30 points)
2
In the rest frame S  of a star, ignoring the grav-
where R and L are two operators such that itational redshift, some of the photons emitted
by the star arise from a particular atomic tran-
[L, R] = E0 sition with an unshifted wavelength  . When
these photons are observed on earth, they are
with E0 a constant. You are also given an shifted to longer wavelength =  + be-
eigenfunction uE (x) of H, such that cause the star is receding from the earth with
velocity 0 c due to the Hubble expansion of the
HuE = EuE , universe. Astronomers measure this redshift by
means of the parameter z, dened by
where E, another constant, is the energy eigen-

value. z .

Prove that
For light, the relativistic Doppler shift is
H(RuE ) = (E + E0 )(RuE ) , 
= .
0 (1 0 cos )
i.e. R is a raising operator.
a. (10 points) In the neighboring star limit
5. (35 points). 0  1, show that 0 is approximately equal
Consider a harmonic oscillator potential to the measured z.
b. (10 points) In the distant star limit 0 
1
V (x) = m02 x2 1, derive an expression for 0 in terms of
2 the measured z.
in one dimension. An even number N of parti- c. (10 points) The observation of Supernova
cles of mass m are placed in this potential. There 1987A marked the dawn of a new astron-
are no special interactions between the particles omy, in which humans are able to detect
no signicant mutual electrostatic repulsion, fermions (neutrinos) as well as bosons (pho-
gravitational attraction, etc., compared to the tons) from (spatially or temporally) resolved
strength of their interaction with the harmonic sources outside the solar system. About a
potential itself. dozen such neutrinos were detected in each
of two huge underground water tanks. The
You may use what you already know about the
photons from Supernova 1987A were red-
levels of a harmonic oscillator.
shifted by
The system is in its ground state, i.e. T = 0 z 105 .
Kelvin.
Taking the Hubble constant to be
Calculate the total energy E of the N -particle
system, relative to the bottom of the well, for H0 0.7 1010 yr1 ,
the cases for how many years did the neutrinos from
a. (10 points) The N particles are distinguish- SN1987A travel before humans observed
able. them?
University of California, Berkeley
Physics H7C Fall 1999 (Strovink)
SOLUTION TO FINAL EXAMINATION

Directions. Do all six problems (weights are indicated). This is a closed-book closed-note exam
except for three 8 12 11 inch sheets containing any information you wish on both sides. You are free
to approach the proctor to ask questions but he or she will not give hints and will be obliged to
write your question and its answer on the board. Calculators are allowed but not essential roots,
circular functions, etc., may be left unevaluated if you do not know them. Use a bluebook. Do not
use scratch paper otherwise you risk losing part credit. Cross out rather than erase any work that
you wish the grader to ignore. Justify what you do. Box or circle your answer.

L
1. (35 points) Halfway between the plates (z = 2 ), we nd that
One circularly polarized photon is trapped be-
tween two parallel perfectly conducting plates,   
 L , t) = E0 exp i ct x iy .
E(
separated by a distance L, which are parallel to 2 L
the photons electric and magnetic elds.
 moves in a circle with angular frequency
So E
a. (10 points) If the photon has the smallest
denite energy possible under these circum- = c/L.
stances, at a point halfway between the b. (10 points) Same for its magnetic eld vec-
plates describe a possible motion of its elec- tor.
tric eld vector.
Solution. From Maxwells equations,
 is par-
Solution. The electric eld of the light E
 must vanish
allel to the conducting plates, so E 
at the plates. Therefore we have  = B .
E
t
 t) = E(L,
E(0,  t) = 0 . Therefore we have

 t) is a solution to the wave equa-


Of course, E(z, By Ex
=
tion: t z
 2 1 2   Bx Ey
E(z, t) = 0 . = .
z 2 c2 t2 t z
In order to satisfy both the wave equation and
From the above relations, it follows that the spa-
the boundary conditions, we choose  t) is given by cos (z/L).
tial dependence of B(z,
  Halfway between the plates B  = 0.
 t) = E0 sin (kz) exp (it) x iy ,
E(z,
c. (15 points) For this plate conguration,
where making no restriction on the photon en-
n
k= where n = 1, 2, ... ergy, evaluate the density of photon states
L
d2 N
The smallest denite energy is achieved when
k = /L (consequently = c/L), in which case dL d
we have for the electric eld:
where N is the number of states and is the
     photon wavelength  L. Take into account
 t) = E0 sin z exp i ct x iy .
E(z,
L L the possible states of circular polarization.
Solution. We have from part (a.) that k = or left-hand circular polarization; light that ex-
n/L, so in terms of wavelength (recalling its the bottom slit (slow axis at 45 ) is in a
k = 2/): state of polarization
2L
n= .     
1 1 i 1 1 1
= ,
Since we have 2 possible polarization states, the 2 i 1 0 2 i
total number of states N as a function of wave-
length is or right-hand circular polarization. At the cen-
4L ter of the screen, light from each slit contributes
N= .
equally; the state of polarization is proportional
The derivative with respect to wavelength is to
dN 4L      
= 2 1 1 1 1 1
d|| + = 2 ;
2 i 2 i 0
and so we obtain for the density of photon states
it is x polarized like the incident beam. (See
d2 N 4 Fowles page 34 and Table 2.1 for the Jones
= 2 .
d||dL vectors and matrices.)
b. (20 points) At what diracted angle x does
the rst minimum of the irradiance occur?
2. (35 points)
A linearly (x) polarized plane EM wave trav- Solution. Right- and left-hand polarized states
elling along z is incident on an opaque bae are orthogonal; they do not interfere. To see
located in the plane z = 0. The bae has two this formally (though this is not required as part
slits cut in it, which are of innite extent in the of the solution), consult Fowles Eq. 3.11; the
y direction. In the x direction, the slit widths interference term there is proportional to
are each a and their center-to-center distance is
 
d. (Obviously d > a, but you may not assume 1
that d  a.) The top and bottom slits are each E2
E1 ( 1 i ) = 1 + i2 = 0 .
i
an equal distance from x = 0.
The diracted image is viewed on a screen lo- Since there is no interference between the light
cated in the plane z = L, where L  d; also from the top and bottom slit, the resulting irra-
L  d2 , where is the EM wavelength. diance is just twice that expected from a single
slit of width a. According to Fowles Eq. 5.18,
Quarter-wave plates are placed in each slit. They
this pattern is proportional to
are identical, except that the topplates slow
(high-index) axis is along (x+ y)/ 2 (+45 with  sin 2
respect to the x axis), while
the bottom plates ,
slow axis is along (x y)/ 2 (45 with respect
to the x axis).
where in this problems notation
a. (15 points) What is the state of polarization
of the diracted light that hits the center of 1
the screen, at x = y = 0? Explain. = ka sin x
2
Solution. Light that exits the top slit (slow
axis at +45 ) is in a state and k = 2/. The rst minimum occurs at
     = , or
1 1 i 1 1 1
= , sin x = .
2 i 1 0 2 i a
3. (35 points) Solution. The sun radiates total power
A lens has an f -number (ratio of focal length to
diameter) equal to F . The lens is used to concen- PS = TS4 4RS2 ,
trate sunlight on a ball whose diameter is equal
to the diameter of the suns image. The ball is where TS is the suns surface temperature and
convectively and conductively insulated, but it RS is the suns radius. The lens collects a
freely radiates energy outward so that its tem- fraction of this light power given by
perature can approach an equilibrium value Tb .
(D/2)2
a. (10 points) The sun subtends a half-angle = 2 ,
of 0.005 radians. Is the size of its image 4RES
diraction-limited, i.e. determined largely
where RES is the distance from the earth to the
by the eects of diraction? Make an order-
sun. The entirety of this light is focused on the
of-magnitude argument assuming that the
ball. The ball re-radiates power
lens is a typical camera lens, with a radius
of order 102 m.
Pb = Tb4 4Rb2 .
Solution. The image of a distant point source
formed at the focal plane of a lens is actually In equilibrium we have the light power absorbed
a Fraunhofer diraction pattern where the aper- by the ball equal to the light power radiated by
ture is the lens opening. The image becomes the ball. Setting the two equal, we obtain
diraction limited when the size of the image
is near the size of an Airy disk. From this D2 RS2
condition, we have the Rayleigh criterion: TS4 2 = Tb2 rb2 .
4 RES
1.22
2 > , To relate this result to F , we note that 2rb /f =
D 1/F where f is the focal length. Also we have
where is the half-angle subtended by the sun rb /f RS /RES , where is the half-angle
( 0.005 rad), is the wavelength of the light, subtended by the sun. Employing the above
and D is the diameter of the lens. We can as- relations in the equation relating the suns tem-
sume 600 nm for the sun, D = 2 102 m, perature to the balls temperature:
so the image is not diraction limited if
d2 2 1
Tb4 = TS4 = T4 .
> 2.5 105 rad , 4rb2 16F 2 S

which is clearly satised in this problem. Thus So we nd that


the image is not diraction limited. 
TS 1
(Note that this question requires only an order- Tb = .
2 F
of-magnitude analysis. So you dont need to
know anything about the details of Rayleighs
criterion, or Airy disks, to get full credit; all that
you need to say is that the diraction angle is of 4. (30 points)
order /D, which here is much smaller than the You are given a Hamiltonian
suns angular width.)
1
b. (25 points) Assuming the sun to be a black- H= (LR + RL) ,
2
body of temperature T , calculate the balls
temperature Tb . Neglect reection by the where R and L are two operators such that
lens. (Hint: your answer should depend
only on T and F .) [L, R] = E0
with E0 a constant. You are also given an The system is in its ground state, i.e. T = 0
eigenfunction uE (x) of H, such that Kelvin.
Calculate the total energy E of the N -particle
HuE = EuE ,
system, relative to the bottom of the well, for
the cases
where E, another constant, is the energy eigen-
value. a. (10 points) The N particles are distinguish-
able.
Prove that
Solution. The energy levels of a harmonic
H(RuE ) = (E + E0 )(RuE ) , oscillator are
 1
i.e. R is a raising operator. En = n + h0 ,
2
Solution.
where n = 0, 1, 2 . . . . Nothing prevents mutually
1  noninteracting distinguishable particles from oc-
H(Ru) = LRR + RLR u cupying the same spatial wavefunction. At T = 0
2
1  this will be the ground state n = 0. Then
= LRR RLR + 2RLR u
2 N
1  E= h0 .
= [L, R]R + 2RLR u 2
2
1 
= E0 R + 2RLR u b. (10 points) The N particles are identical
2
1  bosons.
= E0 R + RLR RRL + RRL + RLR u
2 Solution. Same as (a.). All the identical bosons
1  are in the ground state at T = 0.
= E0 R + R[L, R] + RRL + RLR u
2 c. (15 points) The N particles are identical
1 
spin 12 fermions.
= 2E0 R + R(RL + LR) u
2
1  Solution. Each spatial wavefunction can ac-
= 2E0 R + R(2H) u
2 commodate two identical spin 12 fermions, one
= (E + E0 )Ru . with spin up, one with spin down. At T = 0 the
lowest occupied state is the ground state, with
energy
1
E0 = h0 ,
5. (35 points). 2
Consider a harmonic oscillator potential while the highest-energy occupied state has
(Fermi) energy equal to
1
V (x) = m02 x2 N 
2
EF = E0 + 1 h0 .
2
in one dimension. An even number N of parti-
cles of mass m are placed in this potential. There The total energy is N times the average energy,
are no special interactions between the particles which is the mean of E0 and EF :
no signicant mutual electrostatic repulsion,
E0 + EF
gravitational attraction, etc., compared to the E=N
2
N N  
strength of their interaction with the harmonic
potential itself. = 2E0 + 1 h0
2 2
You may use what you already know about the N2
levels of a harmonic oscillator. = h0 .
4
6. (30 points) Solution. When 0  1, 0 1. Then, from
In the rest frame S  of a star, ignoring the grav- the solution to (a.),
itational redshift, some of the photons emitted
by the star arise from a particular atomic tran- z 20 1
sition with an unshifted wavelength  . When 0
1+z
.
these photons are observed on earth, they are 2
shifted to longer wavelength =  + be- Full credit is given with or without the 1 term.
cause the star is receding from the earth with
velocity 0 c due to the Hubble expansion of the c. (10 points) The observation of Supernova
universe. Astronomers measure this redshift by 1987A marked the dawn of a new astron-
means of the parameter z, dened by omy, in which humans are able to detect
fermions (neutrinos) as well as bosons (pho-
tons) from (spatially or temporally) resolved
z . sources outside the solar system. About a

dozen such neutrinos were detected in each
For light, the relativistic Doppler shift is of two huge underground water tanks. The
photons from Supernova 1987A were red-
 shifted by
= .
0 (1 0 cos ) z 105 .
Taking the Hubble constant to be
a. (10 points) In the neighboring star limit
0  1, show that 0 is approximately equal H0 0.7 1010 yr1 ,
to the measured z. for how many years did the neutrinos from
Solution. SN1987A travel before humans observed
them?

= Solution. From Rohlf Eq. (19.17) (necessary
0 (1 0 cos )
for solving assigned problem 19.18), the velocity
2c
= v with which SN1987A is receding from Earth is

1/ v = H0 d ,
1/ =
0 (1 0 cos )
where d is its present distance from Earth. Neu-

= 0 (1 0 cos ) trinos are nearly massless and travel essentially
 at the speed of light c. Using the result of part

= 0 (1 0 cos ) 1 (a.), the travel time T of the neutrinos from
 SN1987A was
z = 0 (1 0 cos ) 1 .
d
T =
For a receding star, cos = 1, and so c
v
=
H0 c
z = 0 (1 + 0 ) 1 .
0
=
When 0  1, 0 1 to second order in 0 . H0
z
Then
z 1 + 0 1 = 0 . H0
1 105

b. (10 points) In the distant star limit 0  0.7 1010 yr1
1, derive an expression for 0 in terms of 1.4 105 yr .
the measured z.

You might also like